You are on page 1of 212

Updated by K Pascual. 2014. 4C. Bill of Rights Poli Digests. Atty. Jack Jimenez. 4C.

1


CONTENTS

Section 13. Right to Bail ............................................................................................ 153
102. CAMARA VS. ENAGE .................................................................................... 153
103. OBOSA VS. CA .................................................................................................. 154
Section 14 (1). Criminal Due Process ................................................................. 156
104. SCOTYS DEPARTMENT STORE VS. MICALER ............................... 156
105. Almeda v. Villaluz ......................................................................................... 156
105.5. MATEO VS. VILLALUZ............................................................................. 158
106. PEOPLE VS. TEEHANKEE ......................................................................... 160
Section 14 (2). Presumption of Innocence ...................................................... 161
107. PEOPLE VS. MINGOA................................................................................... 161
Section 14 (2). Right to Counsel............................................................................ 161
108. PEOPLE VS. FRISCO HOLGADO.............................................................. 161
109. PEOPLE VS. SIM BEN................................................................................... 162
110. DELGADO VS. CA ........................................................................................... 163
111. PEOPLE VS. MALUNSING .......................................................................... 163
Section 14 (2). Right to be Informed .................................................................. 164
112. PEOPLE VS. REGALA ................................................................................... 164
113. PEOPLE VS. ORTEGA ................................................................................... 165
Section 14 (2). Right to Speedy Trial.................................................................. 166
114. CONDE VS. RIVERA ...................................................................................... 166
Section 14 (2). Right to Public Trial .................................................................... 166
115. GARCIA VS. DOMINGO ................................................................................ 166
Section 14 (2). Right to Confrontation of Witness ...................................... 167
116. PEOPLE VS. ORTIZ-MIYAKE .................................................................... 167
117. PEOPLE VS. SENERIS .................................................................................. 169
118. PEOPLE VS. NARCA ...................................................................................... 170
Section. 16 ........................................................................................................................ 171
119. TATAD VS. SB .................................................................................................. 171
120. GONZALES VS. SB.......................................................................................... 173
121. US VS. NAVARRO ........................................................................................... 174
122. CABAL VS. KAPUNAN ................................................................................. 176
123. PASCUAL VS. MEDICAL BOARD ............................................................ 178
124. ALMONTE VS. VASQUEZ ........................................................................... 179
125. PEOPLE VS. MALIMIT ................................................................................. 180
126. US. vs. TAN TENG .......................................................................................... 181
127. BELTRAN VS. SAMSON .............................................................................. 182
128. PEOPLE VS. ECHEGARAY .......................................................................... 183
129. LOZANO VS. MARTINEZ ............................................................................ 186
130. CUISON VS. CA ................................................................................................ 186
131. PEOPLE VS. OBSANIA ................................................................................. 188
132. CUDIA VS. CA ................................................................................................... 190
133. GUERRERO VS. CA ........................................................................................ 192
134. TUPAS VS ULEP. ............................................................................................ 193
135. PEOPLE VS. VELASCO ................................................................................. 194
136. GALMAN VS. SB .............................................................................................. 195
137. MELO VS. PEOPLE ........................................................................................ 196
Updated by K Pascual. 2014. 4C. Bill of Rights Poli Digests. Atty. Jack Jimenez. 4C. 2


138. PEOPLE VS. CITY COURT MANILA ....................................................... 197
139. PEOPLE VS. RELOVA ................................................................................... 198
140. PEOPLE VS. JABINAL................................................................................... 199
ARTICLE IV AND V CITIZENSHIP AND SUFFRAGE....................................... 201
141. TECSON VS. COMLEC .................................................................................. 201
142. MOY YAO VS. COMMISSION ON IMMIGRATION ........................... 203
143. CO VS. HRET .................................................................................................... 205
144. IN RE CHING. ................................................................................................... 206
145. BENGSON VS. HRET ..................................................................................... 207
146. MERCADO VS. MANZANO ......................................................................... 208
147. MAKALINTAL VS. COMELEC ................................................................... 209


Updated by K Pascual. 2014. 4C. Bill of Rights Poli Digests. Atty. Jack Jimenez. 4C. 3



6. RURAL BANK OF BUHI VS. CA
procedural due process

Buhi Bank was a rural bank. Its books were examined by the Rural Banks
division of the Central Bank
However, it refused to be examined. As a consequence, its financial
assistance was suspended
Later, a general examination of the banks affairs and operations were again
conducted.
The rural banks division found out massive irregularities in the operations,
giving out loans to unknown and fictitious borrowers, and sums amounting
to millions past due to the Central Bank. There were also promissory notes
rediscounted with the Central Bank for cash.
As a result, the Buhi Bank became insolvent.
The division chief, Odra, recommended that Buhi be placed under
receivership.
Thus, the Monetary Board adopted a Resolution # 583, placing the bank
under receivership. Odra, the division chief, was made the receiver.
Odra thus implemented the resolution, authorizing deputies to take control
and possession of Buhis assets and liabilities.
Del Rosario, the Buhi Bank Manager, filed an injunction against the receiver,
arguing that the resolution violated the Rural Banks Act and constitutes
gadalej. The bank claims that there was a violation of due process. They
claim that the bank was not given the chance to deny and disprove the
claim of insolvency or the other grounds and that it was hastily put under
receivership.
Later on, the Central Bank Monetary Board ordered the liquidation of the
Bank.
The judge ruled in favor of the Bank and issued a writ of execution.
The CA however restrained the enforcement of execution, citing that the
Judge did not follow the orders, and thus required the Bank to yield to the
CB.

ISSUE: Was due process observed?
SC: YES. CLOSURE VALID.

Under Sec 29 of the RA 265, on proceedings regarding insolvency, there is NO
REQUIREMENT that a hearing be first conducted before a bank may be placed
under receivership. The law explicitly provides that the Monetary Board can
IMMEDIATELY forbid a banking institution from doing business and
IMMEDIATELY appoint a receiver when: 1) there has been an examination by
CB, b) a report to the CB, and c) prima facie showing that the bank is insolvent.

As to the claim that the RA 265 violates due process, the claim is untenable. The
law could not have intended to disregard the constitutional requirement of due
process when it conferred power to place rural banks under receivership.

The closure and liquidation of the bank is considered an exercise of POLICE
POWER. It maybe subject to judicial inquiry and could be set aside if found to
be capricious, discriminatory, whimsical, arbitrary, etc. The appointment of a
receiver may be made by the Monetary Board, WITHOUT NOTICE AND
HEARING, but subject to the JUDICIAL INQUIRY, to insure protection of the
banking institution.

Updated by K Pascual. 2014. 4C. Bill of Rights Poli Digests. Atty. Jack Jimenez. 4C. 4


Due process does NOT necessarily require a PRIOR HEARING. A hearing or an
OPPORTUNITY TO BE HEARD may be made SUBSEQUENT to the closure. One
could just imagine the dire consequences of a prior hearing: bank runs would
happen, resulting in panic and hysteria. In that way, fortunes will be wiped out,
and disillusionment will run the gamut of the entire banking industry.

There is no question that the action of the MB may be subject to judicial review.
Courts may interfere with the MBs exercise of discretion. Here, the RTC has
jurisdiction to adjudicate the question of whether the MB acted in bad faith
when it directed the dissolution of Buhi Bank.
7. POLLUTION BOARD VS. CA
procedural due process

The board issued an EX PARTE ORDER directed against Solar Textile to
immediately cease and desist from utilizing its waste water pollution source
installations. The installations were allegedly discharging untreated waste
water directly into a canal leading to the adjacent Tullahan Tinejeros River.
The ex parte order was signed by Factoran, the Boards Chair.
The order was based on the findings made after inspection of Solars Plant
by the National Pollution Control Commission, and by the DENR. They
found out that the installation generated 30 gallons per minute of
wastewater pollutants, in excess of that allowed under PD 984.
The order was received by Solar. A writ of execution was issued.
Solar assailed the order, contending that the same was issued without due
process
The Board claims that it has authority to issue ex parte orders to suspend
operations, under PD 984, when there is prima facie evidence of waste
water discharge beyond the allowable limits. According to the investigators
reports, there was prima facie evidence.
Solar insists that the order may issue only when there is immediate threat
to life, public health, safety and welfare. It contends that there was no such
finding.

ISSUE: Order valid?
SC: YES.
It is clear to this Court that, based on the numerous reports, THERE WAS AT
LEAST PRIMA FACIE EVIDENCE before the board that the effluents emanating
from the plant exceeded the maximum allowable lmits of chemical substances,
and that accordingly there was adequate basis supporting the ex parte order to
cease and desist. (it will be noted that the previous owner of the plant had
earlier been issued a similar cease and desist order way back in 1985).
It will also be noted that Solar was earlier summoned by NPCC for a hearing in
1986, yet the Board refrained from issuing an ex parte order until after 1986,
and 1988 when they conducted re-inspections. Thus, the Board appears to have
been forbearing in its efforts to enforce the applicable standards against Solar.
Solor however, remain casual about its continued discharge of untreated
wastewater. Here, the order was issued not by a local government, but directly
by the Pollution Adjudication Board, the very agency tasked to determine
whether the emissions of a particular industrial establishment comply with the
anti-pollution law.

Ex parte cease and desist orders are permitted by law and regulations in
situations like these since stopping the continuous discharge of pollutants into
the river cannot be made to wait until protracted litigation over the ultimate
correctness or propriety of such orders has run its full course, including
multiple and sequential appeals (which Solar has taken), which may take
several years.

The relevant pollution control statute and implementing regulations were
enacted and promulgated in the exercise of that pervasive, sovereign power to
protect the safety, health, and general welfare and comfort of the public, as well
Updated by K Pascual. 2014. 4C. Bill of Rights Poli Digests. Atty. Jack Jimenez. 4C. 5


as the protection of animal and plant life, commonly known as POLICE POWER.
The ordinary requirements of procedural due process may yield to the
necessities of protecting vital public interests, through the exercise of police
powers.

The court is not saying that the ex parte order could no longer be contested by
Solar. It may still do so, in a hearing before the Board itself. Where the
establishment affected by an ex parte case and desist order contests the
correctness of the prima facie findings of the Board, the Board must hold a
public hearing where such establishment will have an opportunity to controvert
the basis of the order. That such opportunity is SUBSSEQUENTLY AVAILABLE, is
really all that is required by the due process clause. Thereafter, the Boards
decision may again be tested by an appeal to the CA.

What Solar should have done is to contest the order in a public hearing rather
than going to court to have the order nullified.
8. CASTILLO VS. JUAN
due process right to be heard fair and impartial judge

another case by Justice Fernando kaya wordy

Castillo, et al are 2 young maidens who are offended parties in 2 rape cases.
They ask for the disqualification of the judge on the ground of bias and
prejudice.
Judge Juan allegedly approached them, and in the secrecy of his chambers,
he informed them of the weakness of their case, the likelihood of an
acquittal, and impressed on them that it would be to their advantage to just
settle and make the accused indemnify them.
Judge Juan also said that settling can spare them from embarrassment
occasion by suits of this character.
These conversations took place even before the prosecution had finished
presenting its evidence and one of the victims have not yet testified.
Judge Juans defense was that he was merely acting out of charity, and as a
clear attempt to humanize justice.

ISSUE: Should the judge be disqualified?
SC: YES.

Due process cannot be satisfied in the absence of that degree of objectivity on
the part of the judge sufficient to reassure the litigants of his being fair and
being just. There is the legitimate expectation that the decision arrived at would
be the application of the law to the facts as found by the judge who does not
play favorites. Justice Dizon once said that in fact, due process of law requires a
hearing before an impartial. And disinterested tribunal, and that every litigant is
entitled to nothing less than the COLD NEUTRALITY OF AN IMPARTIAL JUDGE.

A Judge should strive at all times to be wholly free, disinterested and
independent. Elementary due process requires hearing before an impartial and
disinterested tribunal. A judge has both the duty of rendering a just decisions,
and the duty of doing it in a manner completely free from suspicion as to its
fairness and as to his integrity.

In every litigation, perhaps much more so in criminal cases, the manner and
attitude of a trial judge are crucial to everyone concerned, the offended party,
no less than the accused. It is not for him to indulge or even to give the
appearance of catering to the attimes human failing of yielding to first
impressions. He is to refrain from reaching hasty conclusions or prejudging
matters. It would be deplorable if he lays himself open to the suspicion of
Updated by K Pascual. 2014. 4C. Bill of Rights Poli Digests. Atty. Jack Jimenez. 4C. 6


reacting to feelings rather than to facts, of being imprisoned in the net of his
own sympathies and predilections. It must be obvious to the parties as well as
the public that he follows the traditional mode of adjudication requiring that he
hear both sides with patience and understanding to keep the risk of reaching an
unjust decision at a minimum. It is not necessary that he should possess marked
proficiency in law, but it is essential that he is to hold the balance true.

This is not to discount in its entirety the submission of respondent Judge, who
argued on his own behalf, that his final decision would be dependent on the
evidence that could be presented by petitioners. What cannot be denied,
however, is that after such conferences, they could no longer be expected to
have faith in his impartiality. Even before they had been fully heard, they were
told that their cases were weak. They could very well conclude then that there
was a prejudgment. Under the circumstances, the fact that he acted as he did
because any monetary settlement would benefit petitioners, considering their
straitened financial circumstances, was of no moment. Even if it be admitted
that, according to his beat lights, respondent Judge acted from a sense of
sympathy or "charity", his conduct cannot be said to be consonant with the
exacting standard of the cold neutrality of an impartial judge. The
administration of justice would thus be subject to a reproach if there be a
rejection of the plea for disqualification.

NOTE: cold neutrality of an impartial judge is an element of due process. Due
process does not only mean a CHANCE TO BE HEARD, but also refers to the
QUALITY OF THE HEARER.
Last saved by CHAMP 8/16/2005 11:09 PM
9. WEBB VS. PEOPLE
due process right to be heard fair and impartial judge

This is the same Hubert Webb- Vizconde Massacre case.
Webb sought the disqualification of judge Amelita Tolentino on the ground
that the judge allegedly told allegedly told the media that "failure of the
accused to surrender following the issuance of the warrant of arrest is an
indication of guilt." Respondent judge denied the motion.
Later, Webb filed a second motion to disqualify respondent judge as the
latter allegedly told the media that the accused "should not expect the
comforts of home," pending the resolution of his motion to be committed to
Bicutan, Paranaque. Respondent judge again denied the motion to inhibit.
Again, Biong filed another motion to disqualify respondent judge on the
ground of bias and partiality. This was likewise denied by respondent judge.
After arraignment, Webb filed an Urgent Motion for Hospitalization. He
alleged that he was sick of dermatitis or asthma of the skin which
aggravated due to his continuous commitment at the Paranaque Municipal
Jail. The motion was denied by respondent judge.
During trial, there were other incidents when during cross examination
(regarding Jessica Alfaros affidavit, and another regarding departure for
the US), whenever the prosecution objects, Judge Tolentino sustains the
objection.
Webb thus filed another motion to inhibit on ground of bias and prejudice
of Judge. As usual, Judge Tolentino denied it.
Also, Judge denied the taking of deposition of Webbs witness who was in
the US.
Later, during formal offer of evidence, Judge admitted only 10 out of 142
exhibits offered by Webb.
According to Webb, all these sets irreversibly, the eventual conviction of the
accused.


ISSUE: Should Judge Tolentino inhibit herself on the ground of bias and
prejudice?
Updated by K Pascual. 2014. 4C. Bill of Rights Poli Digests. Atty. Jack Jimenez. 4C. 7


SC: NO.

The Bill of Rights guarantees that "(n)o person shall be held to answer for a
criminal offense without due process of law."19 A critical component of due
process is a hearing before an impartial and disinterested tribunal. We have
ingrained the jurisprudence that every litigant is entitled to nothing: less than
the cold neutrality of an impartial judge for all the other elements of due
process, like notice and hearing, would be meaningless if the ultimate decision
would come from a partial and biased judge.20 Hence, the Rules of Court allows
a judge to voluntarily inhibit himself from hearing a case for "just or valid
reasons" other than those referring to his pecuniary interest, relation, previous
connection, or previous rulings or decisions.

A party has the right to seek the inhibition or disqualification of a judge who
does not appear to be wholly free, disinterested, impartial and independent in
handling the case. This right must be weighed with the duty of a judge to decide
cases without fear of repression. Hence, to disqualify a judge on the ground of
bias and prejudice the movant must prove the same by clear and convincing
evidence. This is a heavy burden and petitioners failed to discharge their
burden of proof.

he alleged adverse and erroneous rulings of respondent judge on their various
motions. By themselves, however, they do not sufficieritly prove bias and
prejudice to disqualify respondent judge. To be disqualifying, the bias and
prejudice must be shown to have stemmed from an extrajudicial source and
result in an opinion on the merits on some basis other than what the judge
learned from his participation in the case. Opinions formed in the course of
judicial proceedings, although erroneous, as long as they are based on the
evidence presented and conduct observed by the judge, do not prove personal
bias or prejudice on the part of the judge. As a general rule, repeated rulings
against a litigant, no matter how erroneous and vigorously and consistently
expressed, are not a basis for disqualification of a judge on grounds of bias and
prejudice. Extrinsic evidence is required to establish bias, bad faith, malice or
corrupt purpose, in addition to the palpable error which may be inferred from
the decision or order itself. Although the decision may seem so erroneous as to
raise doubts concerning a judge's integrity, absent extrinsic evidence, the
decision itself would be insufficient to establish a case against the judge.

The only exception to the rule is when the error is so gross and patent as to
produce an ineluctable inference of bad, faith or malice.

Petitioners simply lean on the alleged series of adverse rulings of the
respondent judge which they characterized as palpable errors. This is not
enough. We note that respondent judge's rulings resolving the various motions
filed by petitioners were all made after considering the arguments raised by all
the parties. It is true that the respondent judge erred in some of her rulings such
as her rejection of petitioners' one hundred thirty two ( 132) pieces of evidence.
It appears, however, that respondent judge reversed this erroneous ruling and
already admitted these 132 pieces of evidence after finding that "the defects in
(their) admissibility have been cured through the introduction of additional
evidence during the trial on the merits."

There is still another reason why we should! observe caution in disqualifying
respondent judge. The trial of the petitioners is about to end and to assign a new
judge to determine the guilt or innocence of petitioners will not be for the best
interest of justice. The records of the case at bar run into volumes. These
voluminous records cannot capture in print the complete credibility of
witnesses when they testified in court. As the respondent judge observed the
demeanor of witnesses while in the witness chair, she is in the best position to
calibrate their credibility.





Updated by K Pascual. 2014. 4C. Bill of Rights Poli Digests. Atty. Jack Jimenez. 4C. 8

















10. ANG TIBAY VS. CIR
administrative due process

Ang Tibay was a manufacturer of rubber slippers.
There was a shortage of leather soles, and it was necessary to temporarily
lay off members of the National Labor Union.
According to the Union however, this was merely a scheme to
systematically terminate the employees from work, and that the shortage of
soles is unsupported. It claims that Ang Tibay is guilty of ULP because the
owner, Teodoro, is discriminating against the National Labor Union, and
unjustly favoring the National Workers Brotherhood, which was allegedly
sympathetic to the employer.
NLU filed for new trial, but Ang Tibay opposed.
Ang Tibay, filed an opposition

SC: We have re-examined the entire record of the proceedings had before the
Court of Industrial Relations in this case, and we have found no substantial
evidence to indicate that the exclusion of the 89 laborers here was due to their
union affiliation or activity.

However, we deem it necessary, in the interest of orderly procedure in cases of
this nature, to make several observations regarding the nature of the powers of
the CIR and emphasize certain guiding principles which should be observed in
the trial of cases brought before it.

The Court of Industrial Relations is a special court whose functions are
specifically stated in the law of its creation (Commonwealth Act No. 103). It is
more an administrative board than a part of the integrated judicial system of
the nation. It is not intended to be a mere receptive organ of the Government.
Unlike a court of justice which is essentially passive, acting only when its
jurisdiction is invoked and deciding only cases that are presented to it by the
parties litigant, the function of the Court of Industrial Relations, as will appear
from perusal of its organic law, is more active, affirmative and dynamic. It not
only exercises judicial or quasi-judicial functions in the determination of
disputes between employers and employees but its functions are far more
comprehensive and extensive.

Updated by K Pascual. 2014. 4C. Bill of Rights Poli Digests. Atty. Jack Jimenez. 4C. 9


The fact, however, that the Court of Industrial Relations may be said to be free
from the rigidity of certain procedural requirements does not mean that it can,
in justiciable cases coming before it, entirely ignore or disregard the
fundamental and essential requirements of due process in trials and
investigations of an administrative character.
There are cardinal primary rights which must be respected even in proceedings
of this character:
(1) The first of these rights is the right to a hearing, which includes the right of
the party interested or affected to present his own case and submit evidence in
support thereof.
(2) Not only must the party be given an opportunity to present his case and to
adduce evidence tending to establish the rights which he asserts but the
tribunal must consider the evidence presented.
(3) "While the duty to deliberate does not impose the obligation to decide right,
it does imply a necessity which cannot be disregarded, namely, that of having
something to support its decision. A decision with absolutely nothing to support
it is a nullity.
(4) Not only must there be some evidence to support a finding or conclusion but
the evidence must be "substantial. "Substantial evidence is more than a mere
scintilla. It means such relevant evidence as a reasonable mind might accept as
adequate to support a conclusion."
(5) The decision must be rendered on the evidence presented at the hearing, or
at least contained in the record and disclosed to the parties affected.
(6) The Court of Industrial Relations or any of its judges, therefore, must act on
its or his own independent consideration of the law and facts of the controversy,
and not simply accept the views of a subordinate in arriving at a decision.
(7) The Court of Industrial Relations should, in all controversial questions,
render its decision in such a manner that the parties to the proceeding can
know the various issues involved, and the reasons for the decisions rendered.



Last saved by CHAMP 8/16/2005 11:40:21 PM
Important Points for Adminstrative Due Process
1. public hearing only for administrative agencies exercising quasi-judicial
functions, and NOT rule making powers. (but prior hearing still required when
the rule is changed as to increase the burden of the public).

2. administrative due process does NOT require that the one who heard the
evidence is also the one who renders the decision (as long as the one who
decides familiarized himself with the evidence)

3. If a decision is appealed, the person deciding the appealed decision should
not decide on the appeal. (otherwise, due process is violated).,






11. CORONA VS. UNITED HARBOR PILOTS ASSOC.
substantive vs. procedural due process

The Phil. Ports Authority (PPA), was created to control, regulate, and
supervise pilots and the pilotage profession.
Updated by K Pascual. 2014. 4C. Bill of Rights Poli Digests. Atty. Jack Jimenez. 4C. 10


It promulgated AO 03-85 providing that : that aspiring pilots must be
holders of pilot licenses and must train as probationary pilots in outports
for three months and in the Port of Manila for four months. It is only after
they have achieved satisfactory performance that they are given permanent
and regular appointments by the PPA itself to exercise harbor pilotage until
they reach the age of 70, unless sooner removed by reason of mental or
physical unfitness by the PPA General Manager.
Later however, another AO 04-92 was issued: all existing regular
appointments which have been previously issued either by the Bureau of
Customs or the PPA shall remain valid up to 31 December 1992 only" and
that "all appointments to harbor pilot positions in all pilotage districts shall,
henceforth, be only for a term of one (1) year from date of effectivity subject
to yearly renewal or cancellation by the Authority after conduct of a rigid
evaluation of performance."
In the meantime, PPA issued a Memorandum laying down the criteria or
factors to be considered for the re-appointment of harbor pilots.
The United Harbor Pilots Association questioned this AO 04-92. They
requested for the suspension of implementation of AO 04-92 before DOTC
Secretary Jesus B. Garcia.
Sec. Garcia however said that the matter is within the jurisdiction of the
Board of Directors of the PPA.
The Association thus appealed this ruling to the Office of the President (OP).
The OP gave due course to the appeal and directed the PPA to hold in
abeyance the implementation of the questioned AO 04-92.
Now, the OP, through then Assistant Secretary for Legal Affairs Renato
Corona, dismissed the appeal and lifted the restraining order issued earlier.
Secretary Corona opined that:
"The exercise of one's profession falls within the constitutional guarantee
against wrongful deprivation of or interference with, property rights
without due process. In the limited context of this case, PPA-A0 04-92 does
not constitute a wrongful interference with, let alone a wrongful
deprivation of the property rights of those affected thereby. As may be
noted, the issuance aims no more than to improve pilotage services by
limiting the appointment to harbor pilot positions to one year, subject to
renewal or cancellation after a rigid evaluation of the appointee's
performance. PPA-AO 04-92 does not forbid, but merely regulates, the
exercise by harbor pilots of their profession in PPA's jurisdictional area."
As to the claim by the Association that there was absence of prior
consultation before the issuance of the AO, Secretary Corona likewise ruled
that the law has been sufficiently complied with by PPA. (the law merely
requires PPA to consult with relevant government agencies. Since the PPA
itself is already composed of representatives from the DOTC, DENR, DPWH,
DOF, NEDA, secretary Corona deemed this sufficient compliance with
consultation).

ISSUES: Was procedural due process satisfied? Was substantive due process
satisfied?
SC: Procedural YES. Substantive NO.

No person shall be deprived of life, liberty, or property without due process of
law, x x x."
In order to fall within the aegis of this provision, two conditions must concur,
namely, that there is a deprivation and that such deprivation is done without
proper observance of due process. When one speaks of due process of law,
however, a distinction must be made between matters of procedure and
matters of substance. In essence, procedural due process "refers to the method
or manner by which the law is enforced," while substantive due process
"requires that the law itself, not merely the procedures by which the law would
be enforced, is fair, reasonable, and just."

PROCEDURAL:
Respondents argue that due process was not observed in the adoption of
PPAAO No. 04-92 allegedly because no hearing was conducted whereby
Updated by K Pascual. 2014. 4C. Bill of Rights Poli Digests. Atty. Jack Jimenez. 4C. 11


"relevant government agencies" and the pilots themselves could ventilate their
views. They are obviously referring to the procedural aspect of the enactment.

SC: As long as a party was given the opportunity to defend his interests in due
course, he cannot be said to have been denied due process of law, for this
opportunity to be heard is the very essence of due process. Moreover, this
constitutional mandate is deemed satisfied if a person is granted an opportunity
to seek reconsideration of the action or ruling complained of."
Here, the Pilots Association, questioned PPA-AO No. 04-92 no less than four
times before the matter was finally elevated to this Tribunal. Their arguments
on this score, however, fail to persuade. While respondents emphasize that the
Philippine Coast Guard, "which issues the licenses of pilots after administering
the pilots' examinations," was not consulted, the facts show that the MARINA,
which took over the licensing function of the Philippine Coast Guard, was duly
represented in the Board of Directors of the PPA. Thus, there being no matters
of naval defense involved in the issuance of the administrative order, the
Philippine Coast Guard need not be consulted.
Neither does the fact that the pilots themselves were not consulted in any way
taint the validity of the administrative order. As a general rule, notice and
hearing, as the fundamental requirements of procedural due process, are
essential only when an administrative body exercises its quasi-judicial function.
In the performance of its executive or legislative functions, such as issuing rules
and regulations, an administrative body need not comply with the requirements
of notice and hearing.

SUBSTANTIVE:
There is no dispute that pilotage as a profession has taken on the nature of a
property right. The exercise of one's profession falls within the constitutional
guarantee against wrongful deprivation of, or interference with, property rights
without due process.

Pilotage, just like other professions, may be practiced only by duly licensed
individuals. Licensure is "the granting of license especially to practice a
profession." It is also "the system of granting licenses (as for professional
practice) in accordance with established standards."21 A license is a right or
permission granted by some competent authority to carry on a business or do
an act which, without such license, would be illegal.22
Before harbor pilots can earn a license to practice their profession, they literally
have to pass through the proverbial eye of a needle by taking, not one but five
examinations, each followed by actual training and practice.

Their license is granted in the form of an appointment which allows them to
engage in pilotage until they retire at the age 70 years. This is a vested right.
It is readily apparent that PPA-AO No. 04-92 unduly restricts the right of harbor
pilots to enjoy their profession before their compulsory retirement. In the past,
they enjoyed a measure of security knowing that after passing five examinations
and undergoing years of on-the-job training, they would have a license which
they could use until their retirement, unless sooner revoked by the PPA for
mental or physical unfitness. Under the new issuance, they have to contend with
an annual cancellation of their license which can be temporary or permanent
depending on the outcome of their performance evaluation. Veteran pilots and
neophytes alike are suddenly confronted with one-year terms which ipso facto
expire at the end of that period. Renewal of their license is now dependent on a
"rigid evaluation of performance" which is conducted only after the license has
already been cancelled. Hence, the use of the term "renewal." It is this
preevaluation cancellation which primarily makes PPA-AO No. 04-92
unreasonable and constitutionally infirm. In a real sense, it is a deprivation of
property without due process of law.
12. US. VS. TORIBIO
substantive due process

Toribio slaughtered or caused to be slaughtered for human consumption, a
carabao, without permit from the municipal treasurer where it was
Updated by K Pascual. 2014. 4C. Bill of Rights Poli Digests. Atty. Jack Jimenez. 4C. 12


slaughtered. He was charged for violation of Act # 1147, an Act Regulating
the Registration, Branding, and Slaughter of Large Cattle.
Toribio argues that in his place, Carmen, Bohol, there is no municipal
slaughterhouse, and thus, under those circumstances, the law does not
prohibit slaughter without permit. He argued that under Sec 30, no large
cattle shall be slaughtered or killed for food at the municipal slaughterhouse
except upon permit from the municipal treasurer.
In short, his argument is that the prohibition is applicable only to slaughter
of large cattle for food in a municipal slaughterhouse without a permit.

ISSUE: Can he be prosecuted?
SC: YES.

The prohibition covers 1) slaughter of large cattle for human consumption,
ANYWHERE, as long as it is without a permit from the municipal treasurer, and
2) expressly and specifically to the killing of large cattle for food at a municipal
slaughterhouse without such permit. Thus, the penalty would apply generally to
the slaughter of large cattle for human consumption, ANYWHERE, without a
permit from the treasurer, and specifically to the killing for food of large cattle
at a municipal slaughterhouse without such permit.

The law primarily seeks to protect the large cattle of the Philippines against
theft and to make easy the recovery and return of such cattle to their proper
owners. Where the language of the statute is fairly susceptible of 2 or more
interpretation, that construction should be adopted as will most tend to give
effect to the manifest intent of the lawmakers, and promote the objective for
which the statute was enacted. Here, the Act prohibits and penalized the
slaughtering or causing to be slaughtered for human consumption, large cattle,
at any place, without the required permit.

If we were to adopted the construction which would limit the prohibition and
penalty to only the killing of such animals in municipal slaughterhouses, then it
will leave unprohibited and unpenalized the slaughter of the same outside such
establishments. This manifestly tends to defeat the purpose and objective of the
legislator.

ISSUE: It appears that the Toribio earlier applied for permit to slaughter his
carabao, but was denied because his carabao was not unfit for agricultural work
or draft purposes. Toribio argues that the statute, in so far as it penalizes the
slaughter of carabaos without obtaining the permit, is unconstitutional because
that permit will never really be procured in the event that the animal is not unfit
for agricultural work or draft purposes. He contends that the law violates the
constitution that no law shall be enacted which shall deprive of any person of
life, liberty, or property without due process of law.

SC: There is no violation of due process.
The law is not an interference with the right and title of owners, as is involved
in the exercise by the State of the right of eminent domain which would have
entitled the owners to compensation, because the law is no more than a JUST
RESTRAINT OF AN INJURIOUS PRIVATE USE OF THE PROPERTY WHICH THE
LEGISLATURE HAD AUTHORITY TO IMPOSE.

Rights of property, are subject to such reasonable limitations in their enjoyment
as shall prevent them from being injurious, and to such reasonable restraints
and regulations established by the legislature as it may think necessary and
expedient. The power involved in this case is not eminent domain but rather
police power, the power vested in the legislature by the constitution, to make,
ordain and establish all manner of wholesome and reasonable laws, with or
without penalties, as they shall judge to be for the good and welfare of the
commonwealth.

Updated by K Pascual. 2014. 4C. Bill of Rights Poli Digests. Atty. Jack Jimenez. 4C. 13


The restrain placed by law on the slaughter for human consumption of carabaos
fit for agricultural work is not an appropriation of property for public use, and
thus not under the power of eminent domain. It is in fact a mere restriction or
limitation upon private use, which the legislature deemed to be detrimental to
the public welfare. The limitations and restraints imposed upon the exercise of
rights of ownership by the statute were imposed not for private pruposes but
strictly in the promotion of GENERAL WELFARE, and PUBLIC INTEREST, in the
exercise of the sovereign police power which every state possesses.

Here, the crime of cattle stealing has become extremely prevalent necessitating
the enactment of a special law penalizing the theft of carabaos by roving bands.
Thus, the statute was enacted in the due and proper exercise of police power,
justified by exigent necessities of existing conditions, and the right of the State
to protect itself against the overwhelming disasters incident to the further
reduction of the supply of animals fit for agricultural work. (there was
apparently a disease that ravaged the animals during the years prior to the
enactment of the law), It thus threatened not only the work animals, but the
very life and existence of the inhabitants who would be imperiled by the
continued destruction of large cattle by disease or otherwise.

Thus, the Legislature had to adopt reasonable measures for the preservation of
work animals, even to the extent of prohibiting and penalizing would ordinarily
would be legitimate and proper exercise of rights of ownership and control of
private property of any citizen.

The police power rests upon the necessity and the right of self-protection.

GUIDELINES: The state may interfere (with property rights), whenever the
public interests demand it. The legislature determines not only what the
interests of the public require, but what measures are necessary for the
protection of such interests. To justify the state in interposing its authority in
behalf of the public, it must appear
1) that the interest of the public generally, as distinguished from those of a
particular class, really requires such interference,
2) that the means are reasonably necessary for the accomplishment of the
purpose, and
3) not unduly oppressive upon individuals.

Note that the determination of what is a proper exercise of police power is not
final or conclusive, but always subject to review by the courts.







13. PEOPLE VS. VENTURA
substantive due process

Ventura was charged and convicted of illegal practice of medicine,
sentencing him to pay a fine of P500.
Natayan (who was an NBI undercover agent of some sort) was at that time
suffering from pains in his back and he asked the accused to see his
sickness. The accused attended to Natayan; wrote something on a piece of
paper; and then he told him that he (Natayan) 'was sick of lumbago'.
Thereupon, the accused asked Natayan to pay P5.00 and then asked him to
Updated by K Pascual. 2014. 4C. Bill of Rights Poli Digests. Atty. Jack Jimenez. 4C. 14


pay the amount to a lady employee in the clinic which Natayan did. At the
request of the accused, Natayan, then went around the other side of the
clinic where he was given an enema of hot water by a male attendant. Then
Natayan was asked to lie down on a table where his back was exposed to a
big bulb for around fifteen minutes and afterwards to a red colored bulb for
another ten minutes. Thereafter Natayan went back to the accused, who
told him. to come back to his clinic for six consecutive days. Natayan came
back. While Natayan was lying on a table about to be given treatment the
National Bureau of Investigation agents raided the place.
He now appeals his conviction.
Appellant, testifying on his behalf admitted that for the past 35 years, he
had been practicing as a naturopathic physician, "treating human ailments
without the use of drugs and medicines" and employing in his practice
"electricity, water and hand", without a license to practice medicine; that
during this time he had treated 500,000 patients, more or less, about 90%
of whom were healed, and that he had studied drugless healing in the
American University, Chicago, Illinois for about four years
The records reveal that the accused begun practicing his method of
drugless healing 35 years ago. This practice was first discovered by the
authorities in 1949. He was prosecuted and convicted therefore the same
year. Sometime after he again set up a clinic. He had a lucrative clientele
and nobody bothered him. the Philippine Federation of Private Medical
Practitioners, complained to the National Bureau of Investigation that
appellant was advertising himself as capable of treating human ailments
without drugs. Upon investigation, apellant was found to be without
certificate of registration to practice such profession either from the Board
of Medical Examiners or from the Committee of Examiners of Masseurs.

ISSUE: Appellant also questions the constitutionality of Section 770 /775 of the
Revised Administrative Code. He contends that to require, of any person whose
business is merely to stimulate by mechanical means the nerves of the body,
many years of study in medical schools, taking up obstetrics, general surgery,
gynecology, bacteriology and many other sciences, is curtailment of the exercise
of one's calling, a violation of the constitutional principle that all men have the
right to life, liberty, and the pursuit of happiness and are entitled to the equal
protection of the law. It is furthermore theorized that inasmuch as drugless
healing is not taught in any of the medical schools prescribed, how could the
members of the Medical Board of Examiners pass on the competence of these
drugless healers?

SC: it is within the police power of the State to require that persons who devote
themselves to the curing of human ills should possess such knowledge
necessary for the proper diagnosis of diseases of the human body.

We must again uphold those immutable concepts of the police power of the
State. Under this power, the State may prescribe such regulations as in its
judgment will secure or tend to secure the general welfare of the people, to
protect them against the consequences of ignorance and incapacity as well as of
deception and fraud. As one means to this end, it has been the practice of
different States, from time immemorial to exact in any pursuit, profession or
trade, a certain degree of skill and learning upon which the community may
confidently rely, their possession being generally ascertained in an examination
of parties by competent persons, or inferred from a certificate to them in the
form of a diploma or license from an institution established for instruction on
the subjects, scientific and otherwise, with which such pursuits have to deal.

ISSUE: He also claims that his act of stimulating the affected nerves of the
patients without use of any drug or medicine is not practice of medicine; that
"practice of medicine" is confined only to the systems taught by the medical
schools, namely, the regular, the homeopathic and the electric schools or
systems.

SC: The statutory definition as to what acts constitute illegal practice of
medicine as provided in said Section 770 includes the acts and practices
performed by appellant. By his own statements, he admitted to have
continuously diagnosed and treated more or less 500,000 instances of different
kinds of human ailments and to have prescribed remedies therefor.
Updated by K Pascual. 2014. 4C. Bill of Rights Poli Digests. Atty. Jack Jimenez. 4C. 15



ISSUE: He tried to show that medical practitioners, members of Congress,
provincial governors, city mayors and municipal board members wrote to him
requesting his help for persons suffering from all kinds of ailments; that
municipal ordinances and resolutions were also passed authorizing him not
only to practice his method of healing but also to put up clinics in some
municipalities, that he was even extended free transportation facilities to work
in the Central Luzon Sanitarium in Tala, Caloocan, Rizal.

SC: Above plea cannot be sustained by this Court. The doctrine of estoppel does
not apply to the government. It is never estopped by mistakes or errors on the
part of its agents, even assuming without conceding that said municipalities had
encouraged appellant's practice. We cannot allow the bargaining away of public
health and safety for the semblance of benefit to a few government officials,
people or even municipalities.
Similarly, there is no such thing as implied license to practice drugless healing
by the mere fact that the Chairman of the Board of Medical Examiners had
permitted appellant to serve free in the Central Luzon Sanitarium in Tala,
Caloocan, Rizal, or that countless people persisted in engaging his services. For
one thing, these people might have contracted his services on the mistaken
notion that he was duly licensed to practice his profession; for another, a
repetition of illegal acts can never make them legal.

Last saved by CHAMP
8/21/2005 9:30:45 PM

NOTES:
1) treating human ailments by means of drugless healing without the required
license constitutes illegal practice of medicine.
2) the power of the state to prescribe qualifications of practitioners is within the
police powers of the state. it may prescribe regulations as in its judgment
will secure the general welfare of the people, to protect them against
ignorance and incapacity, deception or fraud.

14. YNOT VS. IAC

The essence of due process is distilled in the immortal cry of Themistocles to
Alcibiades: "Strike-but hear me first!' "It is this cry that the petitioner in effect
repeats here as he challenges the constitutionality of Executive Order No. 626-A.

Under EO 626-A, the President prohibited interprovincial movement of
carabaos and the slaughtering of carabaos not complying with the age
requirements. Said law further provides that no carabeef shall be
transported from one province to another, and the any carabao or carabeef
transported shall be subject to confiscation and forfeiture of government, to
be distributed to the charitable organizations which the NMIC Chair may
see fit.
Ynot was charged for violation of said law, after having transported 6
carabaos in a pump boat from Masbate to Iloilo. He filed for recovery of said
carabao, but the trial court sustained the confiscation.
The thrust of his petition is that the executive order is unconstitutional
insofar as it authorizes outright confiscation of the carabao or carabeef
being transported across provincial boundaries. His claim is that the
penalty is invalid because it is imposed without according the owner a right
to be heard before a competent and impartial court as guaranteed by due
process. He complains that the measure should not have been presumed,
and so sustained, as constitutional.

Updated by K Pascual. 2014. 4C. Bill of Rights Poli Digests. Atty. Jack Jimenez. 4C. 16


ISSUE: Is the law valid? Was due process observed in its enactment? Was there
proper exercise of police powers?
SC: NO.

The due process clause was kept intentionally vague so it would remain also
conveniently resilient. This was felt necessary because due process is not, like
some provisions of the fundamental law, an "iron rule" laying down an
implacable and immutable command for all seasons and all persons. Flexibility
must be the best virtue of the guaranty. The very elasticity of the due process
clause was meant to make it adapt easily to every situation, enlarging or
constricting its protection as the changing times and circumstances may
require. Instead, they have preferred to leave the import of the protection
openended, as it were, to be "gradually ascertained by the process Of inclusion
and exclusion in the course of the decision of cases as they arise."

The minimum requirements of due process are notice and hearing which,
generally speaking, may not be dispensed with because they are intended as a
safeguard against official arbitrariness. It is a gratifying commentary on our
judicial system that the jurisprudence of this country is rich with applications of
this guaranty as proof of our fealty to the rule of law and the ancient rudiments
of fair play.

The protection of the general welfare is the particular function of the police
power which both restraints and is restrained by due process. The police power
is simply defined as the power inherent in the State to regulate liberty and
property for the promotion of the general welfare.18 By reason of its function, it
extends to all the great public needs and is described as the most pervasive, the
least limitable and the most demanding of the three inherent powers of the
State, far outpacing taxation and eminent domain. The individual, as a member
of society, is hemmed in by the police power, which affects him even before he
is born and follows him still after he is dead-from the womb to beyond the
tomb-in practically everything he does or owns. Its reach is virtually limitless. It
is a ubiquitous and often unwelcome intrusion. Even so, as long as the activity
or the property has some relevance to the public welfare, its regulation under
the police power is not only proper but necessary. And the justification is found
in the venerable Latin maxims, SALUS POPULI EST SUPREMA LEX AND SIC
UTERE TUO UT ALIENUM NON LAEDAS, which call for the subordination of
individual interests to the benefit of the greater number.

It is this power that is now invoked by the government to justify Executive
Order No. 626-A, amending the basic rule in Executive Order No. 626,
prohibiting the slaughter of carabaos except under certain conditions. The
original measure was issued for the reason, as expressed In one of its
Whereases, that "present conditions demand that the carabaos and the
buffaloes be conserved for the benefit of the small farmers who rely on them for
energy needs." We affirm at the outset the need for such a measure.

But while conceding that the amendatory measure has the same lawful subject
as the original executive order, we cannot say with equal certainty that it
complies with the second requirement, viz., that there be a lawful method. We
note that to strengthen the original measure, Executive Order No. 626-A
imposes an absolute ban not on the slaughter of the carabaos but on their
movement, providing that "no carabao, regardless of age, sex, physical condition
or purpose (sic) and no carabeef shall be transported from one province to
another." The object of the prohibition escapes us. The reasonable connection
between the means employed and the purpose sought to be achieved by the
questioned measure is missing.

We do not see how the prohibition of the interprovincial transport of carabaos
can prevent their indiscriminate slaughter, considering that they can be killed
anywhere, with no less difficulty in one province than in another. Obviously,
retaining the carabaos in one province will not prevent their slaughter there,
any more than moving them to another province will make it easier to kill them
them As for the carabeef, the prohibition is made to apply to it as otherwise, so
says executive order, it could be easily circumvented by simply killing the
animal. Perhaps so. However, if the movement of the live animal for the purpose
of preventing their slaughter cannot be prohibited, it should follow that there is
no reason either to prohibit their transfer as, not to be flippant, dead meat.
Updated by K Pascual. 2014. 4C. Bill of Rights Poli Digests. Atty. Jack Jimenez. 4C. 17



Even if a reasonable relation between the means and the end were to be
assumed, we would still have to reckon with the sanction that the measure
applies for violation of the prohibition. The penalty is outright confiscation of
the carabao or carabeef being transported, to be meted out by the executive
authorities, usually the police only. In the Toribio Case, the statute was
sustained because the penalty prescribed was fine and imprisonment, to be
imposed by the court after trial and conviction of the accused. Under the
challenged measure, significantly, no such trial is prescribed, and the property
being transported is immediately impounded by the police and declared, by the
measure itself, as forfeited to the government.

To sum up then, we find that the challenged measure is an invalid exercise of
the police power because the method employed to conserve the carabaos is not
reasonably necessary to the purpose of the law and, worse, is unduly
oppressive. Due process is violated because the owner of the property
confiscated is denied the right to be heard in his defense and is immediately
condemned and punished. The conferment on the administrative authorities of
the power to adjudge the guilt of the supposed offender is a clear encroachment
on judicial functions and militates against the doctrine of separation of powers.

15. LUPANGCO VS. CA
substantive due process

Professional Regulation Commission (PRC) issued Resolution No. 105 as
part of its "Additional Instructions to Examinees to all those applying for
admission to take the licensure examinations in accountancy.
Under said resolution, : No examinee shall attend any review class, briefing,
conference or the like conducted by, or shall receive any hand-out, review
material, or any tip from any school, college or university, or any review
center or the like or any reviewer, lecturer, instructor official or employee
of any of the aforementioned or similar institutions during the three days
immediately preceding every examination day including the examination
day.
Petitioners, all reviewees preparing to take the licensure examinations in
accountancy filed with the RTC Manila, a complaint for injunction for the
issuance of a writ of preliminary injunction against PRC to restrain the
latter from enforcing the above-mentioned resolution and to declare the
same unconstitutional.

ISSUE: Was the regulation valid?
SC: NO.
We realize that the questioned resolution was adopted for a
commendable purpose which is "to preserve the integrity and purity of the
licensure examinations However, its good aim cannot be a cloak to conceal its
constitutional infirmities. On its face, it can be readily seen that it is
unreasonable in that an examinee cannot even attend any review class, briefing,
conference or the like, or receive any hand-out, review material, or any tip from
any school, college or university, or any review center or the like or any
reviewer, lecturer, instructor, official or employee of any of the aforementioned
or similar institutions The unreasonableness is more obvious in that one who is
caught committing the prohibited acts even without any ill motives will be
barred from taking future examinations conducted by the respondent PRC.
Furthermore, it is inconceivable how the Commission can manage to have a
watchful eye on each and every examinee during the three days before the
examination period.
Resolution No. 105 is not only unreasonable and arbitrary, it also
infringes on the examinees' right to liberty guaranteed by the Constitution. PRC
has no authority to dictate on the reviewees as to how they should prepare
themselves for the licensure examinations
They cannot be restrained from taking all the lawful steps needed to assure the
fulfillment of their ambition to become public accountants. They have every
right to make use of their faculties in attaining success in their endeavors. They
should be allowed to enjoy their freedom to acquire useful knowledge that will
promote their personal growth. As defined in a decision of the United States
Updated by K Pascual. 2014. 4C. Bill of Rights Poli Digests. Atty. Jack Jimenez. 4C. 18


Supreme Court: "The term 'liberty' means more than mere freedom from
physical restraint or the bounds of a prison. It means freedom to go where one
may choose and to act in such a manner not inconsistent with the equal rights of
others, as his judgment may dictate for the promotion of his happiness, to
pursue such callings and vocations as may be most suitable to develop his
capacities, and giv to them their highest enjoyment."
Also, it violates the academic freedom of the schools concerned. PRC
cannot interfere with the conduct of review that review schools and centers
believe would best enable their enrolees to meet the standards required before
becoming a fullfledged public accountant.
Making the examinees suffer by depriving them of legitimate means of review
or preparation on those last three precious days-when they should be
refreshing themselves with all that they have learned in the review classes and
preparing their mental and psychological make-up for the examination day
itself-would be like uprooting the tree to get ride of a rotten branch. What is
needed to be done by the respondent is to find out the source of such leakages
and stop it right there. If corrupt officials or personnel should be terminated
from their loss, then so be it. Fixers or swindlers should be flushed out. Strict
guidelines to be observed by examiners should be set up and if violations are
committed, then licenses should be suspended or revoked. These are all within
the powers of the respondent commission as provided for in Presidential
Decree No. 223. But by all means the right and freedom of the examinees to
avail of all legitimate means to prepare for the examinations should not be
curtailed.
17. OPLE VS. TORRES
substantive due process

Ople assailed the validity of Administrative Order No. 308 entitled
"Adoption of a National Computerized Identification Reference System" on
two important constitutional grounds, viz: one, it is a usurpation of the
power of Congress to legislate, and two, it impermissibly intrudes on our
citizenry's protected zone of privacy.
Under A.O. No. 308, a citizen cannot transact business with government
agencies delivering basic services to the people without the contemplated
identification card. No citizen will refuse to get this identification card for
no one can avoid dealing with government. It is thus clear as daylight that
without the ID, a citizen will have difficulty exercising his rights and
enjoying his privileges.

SC: Facially, it violates the right to privacy. The essence of privacy is the "right to
be let alone."

The concept of limited government has always included the idea that
governmental powers stop short of certain intrusions into the personal life of
the citizen. This is indeed one of the basic distinctions between absolute and
limited government. Ultimate and pervasive control of the individual, in all
aspects of his life, is the hallmark of the absolute state. In contrast, a system of
limited government safeguards a private sector, which belongs to the individual,
firmly distinguishing it from the public sector, which the state can control.

The right to privacy is a fundamental right guaranteed by the Constitution,
hence, it is the burden of government to show that A.O. No. 308 is justified by
some compelling state interest and that it is narrowly drawn. A.O. No. 308 is
predicated on two considerations: (1) the need to provide our citizens and
foreigners with the facility to conveniently transact business with basic service
and social security providers and other government instrumentalities and (2)
the need to reduce, if not totally eradicate, fraudulent transactions and
misrepresentations by persons seeking basic services. It is debatable whether
these interests are compelling enough to warrant the issuance of A.O. No. 308.
But what is not arguable is the broadness, the vagueness, the overbreadth of
A.O. No. 308 which if implemented will put our people's right to privacy in clear
and present danger.

The heart of A.O. No. 308 lies in its Section 4 which provides for a Population
Reference Number (PRN) as a "common reference number to establish a linkage
Updated by K Pascual. 2014. 4C. Bill of Rights Poli Digests. Atty. Jack Jimenez. 4C. 19


among concerned agencies" through the use of "Biometrics Technology" and
"computer application designs."
Biometry or biometrics is "the science of the application of statistical methods
to biological facts; a mathematical analysis of biological data." 45 The term
"biometrics" has now evolved into a broad category of technologies which
provide precise confirmation of an individual's identity through the use of the
individual's own physiological and behavioral characteristics. 46 A
physiological characteristic is a relatively stable physical characteristic such as a
fingerprint, retinal scan, hand geometry or facial features

A.O. No. 308 should also raise our antennas for a further look will show that it
does not state whether encoding of data is limited to biological information
alone for identification purposes. In fact, the Solicitor General claims that the
adoption of the Identification Reference System will contribute to the
"generation of population data for development planning." This is an admission
that the PRN will not be used solely for identification but for the generation of
other data with remote relation to the avowed purposes of A.O. No. 308. Clearly,
the indefiniteness of A.O. No. 308 can give the government the roving authority
to store and retrieve information for a purpose other than the identification of
the individual through his PRN . The potential for misuse of the data to be
gathered under A.O. No. 308 cannot be underplayed.

An individual must present his PRN everytime he deals with a government
agency to avail of basic services and security. His transactions with the
government agency will necessarily be recorded - whether it be in the computer
or in the documentary file of the agency. The individual's file may include his
transactions for loan availments, income tax returns, statement of assets and
liabilities, reimbursements for medication, hospitalization, etc. The more
frequent the use of the PRN, the better the chance of building a huge and
formidable information base through the electronic linkage of the files. 55 The
data may be gathered for gainful and useful government purposes; but the
existence of this vast reservoir of personal information constitutes a covert
invitation to misuse, a temptation that may be too great for some of our
authorities to resist.

Even that hospitable assumption will not save A.O. No. 308 from constitutional
infirmity for again said order does not tell us in clear and categorical terms how
these information gathered shall be handled. It does not provide who shall
control and access the data, under what circumstances and for what purpose.
These factors are essential to safeguard the privacy and guaranty the integrity
of the information. 58 Well to note, the computer linkage gives other
government agencies access to the information. Yet, there are no controls to
guard against leakage of information. When the access code of the control
programs of the particular computer system is broken, an intruder, without fear
of sanction or penalty, can make use of the data for whatever purpose, or worse,
manipulate the data stored within the system.

The lack of proper safeguards in this regard of A.O. No. 308 may interfere with
the individual's liberty of abode and travel by enabling authorities to track
down his movement; it may also enable unscrupulous persons to access
confidential information and circumvent the right against self-incrimination; it
may pave the way for "fishing expeditions" by government authorities and
evade the right against unreasonable searches and seizures. 61 The possibilities
of abuse and misuse of the PRN, biometrics and computer technology are
accentuated when we consider that the individual lacks control over what can
be read or placed on his ID, much less verify the correctness of the data
encoded. 62 They threaten the very abuses that the Bill of Rights seeks to
prevent.

The right to privacy is one of the most threatened rights of man living in a mass
society. The threats emanate from various sources - governments, journalists,
employers, social scientists, etc. 88 In the case at bar, the threat comes from the
executive branch of government which by issuing A.O. No. 308 pressures the
people to surrender their privacy by giving information about themselves on
the pretext that it will facilitate delivery of basic services.

DISSENTS:
KAPUNAN: I submit that it is premature for the Court to determine the
constitutionality or unconstitutionality of the National Computerized
Updated by K Pascual. 2014. 4C. Bill of Rights Poli Digests. Atty. Jack Jimenez. 4C. 20


Identification Reference System. A.O. No. 308 does not create any concrete or
substantial controversy. It provides the general framework of the National
Computerized Identification Reference System and lays down the basic
standards (efficiency, convenience and prevention of fraudulent transactions)
for its creation. But as manifestly indicated in the subject order, it is the Inter-
Agency Coordinating Committee (IACC) which is tasked to research, study and
formulate the guidelines and parameters for the use of Biometrics Technology
and in computer application designs that will define and give substance to the
new system. 13 This petition is, thus, premature considering that the IACC is
still in the process of doing the leg work and has yet to codify and formalize the
details of the new system. Before the assailed system can be set up, it is
imperative that the guidelines be issued first.

Without the essential guidelines, the principal contention for invalidating the
new identification reference system - that it is an impermissible encroachment
on the constitutionally recognized right to privacy - is plainly groundless. There
is nothing in A.O. No. 308 to serve as sufficient basis for a conclusion that the
new system to be evolved violates the right to privacy. Said order simply
provides the system's general framework. Without the concomitant guidelines,
which would spell out in detail how this new identification system would work,
the perceived violation of the right to privacy amounts to nothing more than
mere surmise and speculation.

Last saved by CHAMP
8/21/2005 10:49:46 PM
Read orig for other issues.

18. TANADA VS. TUVERA
substantive due process prior publication

Due process was invoked by the petitioners in demanding the disclosure of
a number of presidential decrees which they claimed had not been
published as required by law.
The government argued that while publication was necessary as a rule, it
was not so when it was "otherwise provided," as when the decrees
themselves declared that they were to become effective immediately upon
their approval.
The petitioners suggest that there should be no distinction between laws of
general applicability and those which are not; that publication means
complete publication; and that the publication must be made forthwith in
the Official Gazette.
Solicitor General on the other hand claimed that the clause "unless it is
otherwise provided" in Article 2 of the Civil Code meant that the publication
required therein was not always imperative; that publication, when
necessary, did not have to be made in the Official Gazette;

SC: The clause "unless it is otherwise provided" refers to the date of effectivity
and not to the requirement of publication itself, which cannot in any event be
omitted. This clause does not mean that the legislature may make the law
effective immediately upon approval, or on any other date, without its previous
publication.
Publication is indispensable in every case, but the legislature may in its
discretion provide that the usual fifteen-day period shall be shortened or
extended.

It is not correct to say that under the disputed clause publication may be
dispensed with altogether. The reason is that such omission would offend due
process insofar as it would deny the public knowledge of the laws that are
supposed to govern it. Surely, if the legislature could validly provide that a law
shall become effective immediately upon its approval notwithstanding the lack
of publication (or after an unreasonably short period after publication), it is not
unlikely that persons not aware of it would be prejudiced as a result; and they
would be so not because of a failure to comply with it but simply because they
Updated by K Pascual. 2014. 4C. Bill of Rights Poli Digests. Atty. Jack Jimenez. 4C. 21


did not know of its existence. Significantly, this is not true only of penal laws as
is commonly supposed. One can think of many non-penal measures, like a law
on prescription, which must also be communicated to the persons they may
affect before they can begin to operate.

We note at this point the conclusive presumption that every person knows the
law, which of course presupposes that the law has been published if the
presumption is to have any legal justification at all. It is no less important to
remember that Section 6 of the Bill of Rights recognizes "the right of the people
to information on matters of public concern," and this certainly applies to,
among others, and indeed especially, the legislative enactments of the
government.

We hold therefore that all statutes, including those of local application and
private laws, shall be published as a condition for their effectivity, which shall
begin fifteen days after publication unless a different effectivity date is fixed by
the legislature.
Covered by this rule are presidential decrees and executive orders promulgated
by the President in the exercise of legislative powers whenever the same are
validly delegated by the legislature or, at present, directly conferred by the
Constitution. Administrative rules and regulations must also be published if
their purpose is to enforce or implement existing law pursuant also to a valid
delegation. Interpretative regulations and those merely internal in nature, that
is, regulating only the personnel of the administrative agency and not the public,
need not be published. Neither is publication required of the so-called letters of
instructions issued by administrative superiors concerning the rules or
guidelines to be followed by their subordinates in the performance of their
duties.

We agree that the publication must be in full or it is no publication at all since its
purpose is to inform the public of the contents of the laws.

Last saved by CHAMP 8/21/2005 11:01 PM

16. ESTRADA VS. SANDIGAN BAYAN
substantive due process

Erap assails the constitutionality of the Plunder Law (RA 7080 / RA 7659),
on 3 grounds: (a) it suffers from the vice of vagueness; (b) it dispenses with
the "reasonable doubt" standard in criminal prosecutions; and, (c) it
abolishes the element of mens rea in crimes already punishable under RPC,
all of which are purportedly clear violations of the fundamental rights of the
accused to due process and to be informed of the nature and cause of the
accusation against him.
The Plunder Law provides, Any public officer who, by himself or in
connivance with members of his family, relatives by affinity or consanguinity,
business associates, subordinates or other persons, amasses, accumulates or
acquires ill-gotten wealth through a combination or series of overt or
criminal acts as described in Section 1 (d) hereof, in the aggregate amount
or total value of at least fifty million pesos (P50,000,000.00) shall be guilty of
the crime of plunder and shall be punished by reclusion perpetua to death.
Erap, however, bewails the failure of the law to provide for the statutory
definition of the terms "combination" and "series" in the key phrase "a
combination or series of overt or criminal acts" found in Sec. 1, par. (d), and
Sec. 2, and the word "pattern" in Sec. 4. These omissions, according to
petitioner, render the Plunder Law unconstitutional for being
impermissibly vague and overbroad and deny him the right to be informed
of the nature and cause of the accusation against him, hence, violative of his
fundamental right to due process.

SC: VALID.
As it is written, the Plunder Law contains ascertainable standards and
well-defined parameters which would enable the accused to determine the
Updated by K Pascual. 2014. 4C. Bill of Rights Poli Digests. Atty. Jack Jimenez. 4C. 22


nature of his violation. Section 2 is sufficiently explicit in its description of
the acts, conduct and conditions required or forbidden, and prescribes the
elements of the crime with reasonable certainty and particularity. Thus -
1. That the offender is a public officer who acts by himself or in connivance
with members of his family, relatives by affinity or consanguinity, business
associates, subordinates or other persons;
2. That he amassed, accumulated or acquired ill-gotten wealth through a
combination or series of the following overt or criminal acts: (a) through
misappropriation, conversion, misuse, or malversation of public funds or raids
on the public treasury; (b) by receiving, directly or indirectly, any commission, gift,
share, percentage, kickback or any other form of pecuniary benefits from any
person and/or entity in connection with any government contract or project or by
reason of the office or position of the public officer; (c) by the illegal or fraudulent
conveyance or disposition of assets belonging to the National Government or any
of its subdivisions, agencies or instrumentalities of Government owned or
controlled corporations or their subsidiaries; (d) by obtaining, receiving or
accepting directly or indirectly any shares of stock, equity or any other form of
interest or participation including the promise of future employment in any
business enterprise or undertaking; (e) by establishing agricultural, industrial or
commercial monopolies or other combinations and/or implementation of decrees
and orders intended to benefit particular persons or special interests; or (f) by
taking advantage of official position, authority, relationship, connection or
influence to unjustly enrich himself or themselves at the expense and to the
damage and prejudice of the Filipino people and the Republic of the Philippines;
and,
3. That the aggregate amount or total value of the ill-gotten wealth amassed,
accumulated or acquired is at least P50,000,000.00.
As long as the law affords some comprehensible guide or rule that would
inform those who are subject to it what conduct would render them liable to its
penalties, its validity will be sustained. It must sufficiently guide the judge in its
application; the counsel, in defending one charged with its violation; and more
importantly, the accused, in identifying the realm of the proscribed conduct.
Indeed, it can be understood with little difficulty that what the assailed statute
punishes is the act of a public officer in amassing or accumulating ill-gotten
wealth of at least P50,000,000.00 through a series or combination of acts
enumerated in Sec. 1, par. (d), of the Plunder Law. We discern nothing in the
foregoing that is vague or ambiguous - as there is obviously none - that will
confuse petitioner in his defense. Although subject to proof, these factual
assertions clearly show that the elements of the crime are easily understood
and provide adequate contrast between the innocent and the prohibited acts.
Upon such unequivocal assertions, petitioner is completely informed of the
accusations against him as to enable him to prepare for an intelligent defense.
A statute is not rendered uncertain and void merely because general terms
are used therein, or because of the employment of terms without defining them;
much less do we have to define every word we use. Besides, there is no
positive constitutional or statutory command requiring the legislature to define
each and every word in an enactment. Congress is not restricted in the form of
expression of its will, and its inability to so define the words employed in a
statute will not necessarily result in the vagueness or ambiguity of the law so
long as the legislative will is clear, or at least, can be gathered from the whole
act, which is distinctly expressed in the Plunder Law.
Moreover, it is a well-settled principle of legal hermeneutics that words of
a statute will be interpreted in their natural, plain and ordinary acceptation and
signification, unless it is evident that the legislature intended a technical or
special legal meaning to those wordsThe intention of the lawmakers - who are,
ordinarily, untrained philologists and lexicographers - to use statutory
phraseology in such a manner is always presumed. Thus, Webster's New
Collegiate Dictionary contains the following commonly accepted definition of
the words "combination" and "series:"
Combination - the result or product of combining; the act or process of
combining. To combine is to bring into such close relationship as to obscure
individual characters.
Series - a number of things or events of the same class coming one after
another in spatial and temporal succession.
That Congress intended the words "combination" and "series" to be
understood in their popular meanings is pristinely evident from the legislative
deliberations on the bill which eventually became RA 7080 or the Plunder Law:
Thus when the Plunder Law speaks of "combination," it is referring to at
least two (2) acts falling under different categories of enumeration provided in
Sec. 1, par. (d), e.g., raids on the public treasury in Sec. 1, par. (d), subpar. (1),
and fraudulent conveyance of assets belonging to the National Government
under Sec. 1, par. (d), subpar. (3).
Updated by K Pascual. 2014. 4C. Bill of Rights Poli Digests. Atty. Jack Jimenez. 4C. 23


On the other hand, to constitute a series" there must be two (2) or more
overt or criminal acts falling under the same category of enumeration found in
Sec. 1, par. (d), say, misappropriation, malversation and raids on the public
treasury, all of which fall under Sec. 1, par. (d), subpar. (1). Verily, had the
legislature intended a technical or distinctive meaning for "combination" and
"series," it would have taken greater pains in specifically providing for it in the
law.
Under the circumstances, petitioner's reliance on the "void-for-vagueness"
doctrine is manifestly misplaced. The doctrine has been formulated in various
ways, but is most commonly stated to the effect that a statute establishing a
criminal offense must define the offense with sufficient definiteness that
persons of ordinary intelligence can understand what conduct is prohibited by
the statute. It can only be invoked against that specie of legislation that is
utterly vague on its face, i.e., that which cannot be clarified either by a saving
clause or by construction.
A statute or act may be said to be vague when it lacks comprehensible
standards that men of common intelligence must necessarily guess at its
meaning and differ in its application. In such instance, the statute is repugnant
to the Constitution in two (2) respects - it violates due process for failure to
accord persons, especially the parties targeted by it, fair notice of what conduct
to avoid; and, it leaves law enforcers unbridled discretion in carrying out its
provisions and becomes an arbitrary flexing of the Government muscle. But the
doctrine does not apply as against legislations that are merely couched in
imprecise language but which nonetheless specify a standard though
defectively phrased; or to those that are apparently ambiguous yet fairly
applicable to certain types of activities. The first may be "saved" by proper
construction, while no challenge may be mounted as against the second
whenever directed against such activities. With more reason, the doctrine
cannot be invoked where the assailed statute is clear and free from ambiguity,
as in this case.
The test in determining whether a criminal statute is void for uncertainty
is whether the language conveys a sufficiently definite warning as to the
proscribed conduct when measured by common understanding and practice. It
must be stressed, however, that the "vagueness" doctrine merely requires a
reasonable degree of certainty for the statute to be upheld - not absolute
precision or mathematical exactitude, as petitioner seems to suggest.
Flexibility, rather than meticulous specificity, is permissible as long as the metes
and bounds of the statute are clearly delineated. An act will not be held invalid
merely because it might have been more explicit in its wordings or detailed in
its provisions, especially where, because of the nature of the act, it would be
impossible to provide all the details in advance as in all other statutes.
The void-for-vagueness doctrine states that "a statute which either forbids
or requires the doing of an act in terms so vague that men of common
intelligence must necessarily guess at its meaning and differ as to its
application, violates the first essential of due process of law. The overbreadth
doctrine, on the other hand, decrees that "a governmental purpose may not be
achieved by means which sweep unnecessarily broadly and thereby invade the
area of protected freedoms."

This rationale does not apply to penal statutes.
Criminal statutes have general in terrorem effect resulting from their very
existence, and, if facial challenge is allowed for this reason alone, the State may
well be prevented from enacting laws against socially harmful conduct. In the
area of criminal law, the law cannot take chances as in the area of free speech.
The overbreadth and vagueness doctrines then have special application
only to free speech cases. They are inapt for testing the validity of penal
statutes. As for the vagueness doctrine, it is said that a litigant may challenge a
statute on its face only if it is vague in all its possible applications. "A plaintiff
who engages in some conduct that is clearly proscribed cannot complain of the
vagueness of the law as applied to the conduct of others
.

DISSENTS: KAPUNAN:
The meanings of combination and series as used in R.A. No. 7080 are not
clear.
R.A. No. 7080 does not define pattern, an essential element of the crime of
plunder.
R.A. No. 7080 makes it possible for a person conspiring with the accused in
committing one of the acts constituting the charge of plunder to be convicted
for the same crime.
R.A. No. 7080 does not clearly statethe prescriptive period of the crime of plunder.

READ ORIGINAL..
Updated by K Pascual. 2014. 4C. Bill of Rights Poli Digests. Atty. Jack Jimenez. 4C. 24







19. VILLEGAS VS. HUI CHIONG TSAI PAO HO
equal protection

1. There was an Ordinance in Manila, enacted making it unlawful for any
person not a citizen of the Philippines to be employed in any place of
employment or to be engaged in any kind of trade business or occupation
within manila without first securing an employment permit from the
mayors office.
2. Pao, an employee in Manila, sought an injunction to restrain the
enforcement of Ordinance 6537. He argues that:
a. As a revenue measure, the measure imposed on aliens is
discriminatory and violative of the rule on uniformity of taxation.
b. As a police power measure, it makes no distinction between useful
and non-useful occupations, imposing a fixed P50.00 employment
permit, and that it fails to prescribe any standard to guide or limit
the action of the Mayor
c. It is arbitrary, oppressive and unreasonable, being applied only to
aliens, thereby violative of equal protection clause.

ISSUE: does it violate equal protection clause?
SC: YES.

The P50.00 fee is unreasonable not only because it is excessive but because it
fails to consider valid substantial differences in situation among individual
aliens who are required to pay it. Although the equal protection clause of the
Constitution does not forbid classification, it is imperative that the classification
should be based on real and substantial differences having a reasonable relation
to the subject of the particular legislation. The same amount of P50.00 is being
collected from every employed alien, whether he is casual or permanent, part
time or full time or whether he is a lowly employee or a highly paid executive.

The ordinance in question violates the due process of law and equal protection
rule of the Constitution. Requiring a person before he can be employed to get a
permit from the City Mayor of Manila who may withhold or refuse it at will is
tantamount to denying him the basic right of the people in the Philippines to
engage in a means of livelihood. While it is true that the Philippines as a State is
not obliged to admit aliens within its territory, once an alien is admitted, he
cannot be deprived of life without due process of law. This guarantee includes
the means of livelihood. The shelter of protection under the due process and
equal protection clause is given to all persons, both aliens and citizens.

Ordinance No. 6537 does not lay down any criterion or standard to guide the
Mayor in the exercise of his discretion. It has been held that where an ordinance
of a municipality fails to state any policy or to set up any standard to guide or
limit the mayor's action, expresses no purpose to be attained by requiring a
permit, enumerates no conditions for its grant or refusal, and entirely lacks
standard, thus conferring upon the Mayor arbitrary and unrestricted power to
grant or deny the issuance of building permits, such ordinance is invalid, being
an undefined and unlimited delegation of power to allow or prevent an activity
per se lawful.

20. PEOPLE VS. CAYAT
equal protection
Updated by K Pascual. 2014. 4C. Bill of Rights Poli Digests. Atty. Jack Jimenez. 4C. 25



Cayat, who was a native of Baguio and a non-Christian, was imprisoned for
violation of Act 1639, being a member of a non-Christian tribe, illegally had
in his possession one (1) bottle of A1 gin, an intoxicating liquor.
Under Act 1639, It shall be unlawful for any native of the Philippine Islands
who is a member of a non-Christian tribe to buy, receive, have in his
possession, or drink any ardent spirits, ale, beer, wine, or intoxicating
liquors of any kind, other than the so-called native wines and liquors which
the members of such tribes have been accustomed themselves.
Cayat challenges the constitutionality of the Act on the following grounds:
o (1) That it is discriminatory and denies the equal protection of the
laws;
o (2) That it is violative of the due process clause of the Constitution;
and
o (3) That it is an improper exercise of the police power of the state.

ISSUE: Is the law in accordance with equal protection?
SC: YES. VALID.

The guaranty of the equal protection of the laws is not violated by a legislation
based on reasonable classification. And the classification, to be reasonable,
(1) must rest on substantial distinctions;
(2) must be germane to the purposes of the law;
(3) must not be limited to existing conditions only; and
(4) must apply equally to all members of the same class.

Act No. 1639 satisfies these requirements. The classification rests on real or
substantial, not merely imaginary or whimsical, distinctions. It is not based
upon "accident of birth or parentage," as counsel for the appellant asserts, but
upon the degree of civilization and culture. "The term 'non-Christian tribes'
refers, not to religious belief, but, in a way, to the geographical area, and, more
directly, to natives of the Philippine Islands of a low grade of civilization, usually
living in tribal relationship apart from settled communities." This distinction is
unquestionably reasonable, for the Act was intended to meet the peculiar
conditions existing in the non-Christian tribes.

That it is germane to the purposes of law cannot be doubted. The prohibition "to
buy, receive, have in his possession, or drink any ardent spirits, ale, beer, wine,
or intoxicating liquors of any kind, other than the so-called native wines and
liquors which the members of such tribes have been accustomed themselves to
make prior to the passage of this Act," is unquestionably designed to insure
peace and order in and among the non-Christian tribes. It has been the sad
experience of the past, as the observations of the lower court disclose, that the
free use of highly intoxicating liquors by the non-Christian tribes have often
resulted in lawlessness and crimes, thereby hampering the efforts of the
government to raise their standard of life and civilization.

The law is not limited in its application to conditions existing at the time of its
enactment. It is intended to apply for all times as long as those conditions exist.
The Act was not predicated, as counsel for appellant asserts, upon the
assumption that the non-Christians are "impermeable to any civilizing
influence." On the contrary, the Legislature understood that the civilization of a
people is a slow process and that hand in hand with it must go measures of
protection and security.
Finally, that the Act applies equally to all members of the class is evident from a
perusal thereof. That it may be unfair in its operation against a certain number
of non-Christians by reason of their degree of culture, is not an argument
against the equality of its application.

Last saved by CHAMP
Updated by K Pascual. 2014. 4C. Bill of Rights Poli Digests. Atty. Jack Jimenez. 4C. 26


8/22/2005 11:10 PM

Other issues:
DUE PROCESS:
To constitute due process of law, notice and hearing are not always necessary.
This rule is especially true where much must be left to the discretion of the
administrative officials in applying a law to particular cases. Due process of law
means simply:
(1) that there shall be a law prescribed in harmony with the general powers of
the legislative department of the government;
(2) that it shall be reasonable in its operation;
(3) that it shall be enforced according to the regular methods of procedure
prescribed; and
(4) that it shall be applicable alike to all citizens of the state or to all of a class.

POLICE POWERS:
Neither is the Act an improper exercise of the police power of the state. Any
measure intended to promote the health, peace, morals, education and good
order of the people or to increase the industries of the state, develop its
resources and add to its wealth and prosperity is a legitimate exercise of the
police power, and unless shown to be whimsical or capricious as to unduly
interfere with the rights of an individual, the same must be upheld.
Act No. 1639, as above stated, is designed to promote peace and order in the
non-Christian tribes so as to remove all obstacles to their moral and intellectual
growth and, eventually, to hasten their equalization and unification with the
rest of their Christian brothers. Its ultimate purpose can be no other than to
unify the Filipino people with a view to a greater Philippines.

The law, then, does not seek to mark the non-Christian tribes as "an inferior or
less capable race." On the contrary, all measures thus far adopted in the
promotion of the public policy towards them rest upon a recognition of their
inherent right to equality in the enjoyment of those privileges now enjoyed by
their Christian brothers. But as there can be no true equality before the law, if
there is, in fact, no equality in education, the government has endeavored, by
appropriate measures, to raise their culture and civilization and secure for them
the benefits of their progress, with the ultimate end in view of placing them
with their Christian brothers on the basis of true equality.
21. ORMOC SUGAR VS. TREASURER
equal protection

The municipal Board of Ormoc City, enacted an Ordinance #4, imposing on
any and all productions of centrifugal sugar milled at the Ormoc Sugar
Company, Inc., in Ormoc City a municipal tax equivalent to one per centum
(1%)
Ormoc Sugar Company, Inc. filed a complaint against the City of Ormoc as
well as its Treasurer, Municipal Board and Mayor, alleging that the
ordinance is unconstitutional for being violative of the equal protection
clause.

ISSUE: Is the ordinance valid? Does it comply with equal protection?
SC: NO.

We ruled that the equal protection clause applies only to persons or things
identically situated and does not bar a reasonable classification of the subject of
legislation, and a classification is reasonable where
(1) it is based on substantial distinctions which make real differences;
(2) these are germane to the purpose of the law;
Updated by K Pascual. 2014. 4C. Bill of Rights Poli Digests. Atty. Jack Jimenez. 4C. 27


(3) the classification applies not only to present conditions but also to future
conditions which are substantially identical to those of the present;
(4) the classification applies only to those who belong to the same class.

A perusal of the requisites instantly shows that the questioned ordinance does
not meet them, for it taxes only centrifugal sugar produced and exported by the
Ormoc Sugar Company, Inc. and none other. At the time of the taxing
ordinance's enactment, Ormoc Sugar Company, Inc., it is true, was the only
sugar central in the city of Ormoc. Still, the classification, to be reasonable,
should be in terms applicable to future conditions as well. The taxing ordinance
should not be singular and exclusive as to exclude any subsequently established
sugar central, of the same class as plaintiff, from the coverage of the tax. As it is
now, even if later a similar company is set up, it cannot be subject to the tax
because the ordinance expressly points only to Ormoc Sugar Company, Inc. as
the entity to be levied upon.

22. PEOPLE VS. VERA
equal protection

Mario Cu-Unjieng was convicted in a criminal case.
He applied for PROBATION under the provisions of ACT # 4221. He insists
that he is innocent of the crime for which he has convicted, and that he has
no prior criminal record and the he would observe good conduct in the
future.
Nevertheless, Judge Vera heard the petition.
The City Fiscal obviously opposed the grant of probation. Among the
arguments raised was that Act 4221 (which granted probation) was
violative of the Constitution because its applicability is not uniform
throughout the Islands and because section 11 of said Act No. 4221 endows
the provincial boards with the power to make said law effective or
otherwise in their respective provinces.
Cu-Unjieng maintains that Act No. 4221 is constitutional because, it does
not constitute an undue delegation of legislative power, does not infringe
the equal protection clause of the Constitution, and does not encroach upon
the pardoning power of the Executive.

ISSUE: Is the Act valid? Does it comply with equal protection?
SC: NO. NULL AND VOID.

The equal protection of the laws, is a pledge of the protection of equal laws." No
rule that will cover every case can be formulated. Class legislation
discriminating against some and favoring others is prohibited. But classification
on a reasonable basis, and not made arbitrarily or capriciously, is permitted.
The classification, however, to be reasonable must be based on substantial
distinctions which make real differences; it must be germane to the purposes of
the law; it must not be limited to existing conditions only, and must apply
equally to each member of the class.

Here, one province may appropriate the necessary fund to defray the salary of a
probation officer, while another province may refuse or fail to do so. In such a
case, the Probation Act would be in operation in the former province but not in
the latter. This means that a person otherwise coming within the purview of the
law would be liable to enjoy the benefits of probation in one province while
another person similarly situated in another province would be denied those
same benefits. This is obnoxious discrimination.

Contrariwise, it is also possible for all the provincial boards to appropriate the
necessary funds for the salaries of the probation officers in their respective
provinces, in which case no inequality would result for the obvious reason that
probation would be in operation in each and every province by the affirmative
Updated by K Pascual. 2014. 4C. Bill of Rights Poli Digests. Atty. Jack Jimenez. 4C. 28


action of appropriation by all the provincial boards. On that hypothesis, every
person coming within the purview of the Probation Act would be entitled to
avail of the benefits of the Act. Neither will there be any resulting inequality if
no province, through its provincial board, should appropriate any amount for
the salary of the probation officer-which is the situation now-and, also, if we
accept the contention that, for the purposes of the Probation Act, the City of
Manila should be considered as a province and that the municipal board of said
city has not made any appropriation for the salary of a probation officer.

These different situations suggested show, indeed, that while inequality may
result in the application of the law and in the conferment of the benefits therein
provided, inequality is not in all cases the necessary result. But whatever may
be the case, it is clear that section 11 of the Probatoin Act creates a situation in
which discrimination and inequality are permitted or allowed. There are, to be
sure, abundant authorities requiring actual denial of the equal protection of the
law before courts should assume the task of setting aside a law vulnerable on
that score, but premises and circumstances considered, we are of the opinion
that section 11 of Act No. 4221 permits of the denial of the equal protection of
the law and is on that account bad.

We see no difference between a law which denies equal protection and a law
which permits of such denial. A law may appear to be fair on its face and
impartial in appearance, yet, if it permits of unjust and illegal discrimination, it
is within the constitutional prohibition.

Under section 11 of the Probation Act, not only may said Act be in force in one
or several provinces and not be in force in the other provinces, but one province
may appropriate for the salary of a probation officer of a given year-and have
probation during that year-and thereafter decline to make further
appropriation, and have no probation in subsequent years. While this situation
goes rather to the abuse of discretion which delegation implies, it is here
indicated to show that the Probation Act sanctions a situation which is
intolerable in a government of laws, and to prove how easy it is, under the Act,
to make the guaranty of the equality clause but "a rope of sand".

The clear policy of the law, as may be gleaned from a careful examination of the
whole context, is to make the application of the system dependent entirely upon
the affirmative action of the different provincial boards through appropriation
of the salaries for probation officers at rates not lower than those provided for
provincial fiscals. Without such action on the part of the various boards, no
probation officers would be appointed by the Secretary of Justice to act in the
provinces. The Philippines is divided or subdivided into provinces and it needs
no argument to show that if not one of the provinces-and this is the actual
situation now appropriates the necesasry fund for the salary of a probation
officer, probation under Act No. 4221 would be illusory. There can be no
probation without a probation officer. Neither can there be a probation officer
without a probation system.



23. TAXICAB OPERATORS VS. BOT
equal protection

Board of Transportation, (BOT) issued Memorandum Circular No. 77-42,
phasing out and Replacement of Old and Dilapidated Taxis. It follows the
policy of the government to insure that only safe and comfortable units are
used as public conveyances.
Under said Memo, no car beyond six years shall be operated taxi. (it also
provided for a scheme of phasing out and disallowing registration of older
taxis).
The Regulation also provided that, the rules shall immediately be effective
in Metro-Manila only. Its implementation outside Metro-Manila shall be
carried out only after the project has been implemented in Metro-Manila
and only after the date has been determined by the board.
Updated by K Pascual. 2014. 4C. Bill of Rights Poli Digests. Atty. Jack Jimenez. 4C. 29


Petitioners filed a Petition with the BOT, seeking to nullify the memo, and to
stop its implementation; to allow the registration and operation in 1981
and subsequent years of taxicabs of model 1974, as well as those of earlier
models which were phased-out, provided that, at the time of registration,
they are road worthy and fit for operation. They claim that fixing the ceiling
at six (6) years is arbitrary and oppressive because the roadworthiness and
the use to which they are subjected, and, therefore, their actual physical
condition should be taken into consideration at the time of registration.
Petitioners also allege that the Circular in question violates their right to
equal protection of the law because the same is being enforced in Metro
Manila only and is directed solely towards the taxi industry.
On the other hand, BOT contends, it is impractical to subject every taxicab
to constant and recurring evaluation, not to speak of the fact that it can
open the door to the adoption of multiple standards, possible collusion, and
even graft and corruption. A reasonable standard must be adopted to apply
to all vehicles affected uniformly, fairly, and justly. The span of six years
supplies that reasonable standard. The product of experience shows that by
that time taxis have fully depreciated, their cost recovered. and a fair return
on investment obtained .

ISSUE: Is the Memo valid?
SC: VALID.

Older taxicabs are generally dilapidated and no longer fit for safe and
comfortable service to the public specially considering that they are in
continuous operation practically 24 hours everyday in three shifts of eight
hours per shift. With that standard of reasonableness and absence of
arbitrariness, the requirement of due process has been met.

The Board's reason for enforcing the Circular initially in Metro Manila is that
taxicabs in this city, compared to those of other places, are subjected to heavier
traffic pressure and more constant use. This is of common knowledge.
Considering that traffic conditions are not the same in every city, a substantial
distinction exist so that infringement of the equal protection clause can hardly
be successfully claimed.

In fact, the scheme is already being effect also in Cebu City. The BOT is also in
the process of conducting studies regarding the operation of taxicab in other
cities.
As enunciated in the preambular clauses of the challenged BOT Circular, the
overriding consideration is the safety and comfort of the riding public from the
dangers posed by old and dilapidated taxis. The State, in the exercise of its
police power, can prescribed regulations to promote the health, morals, peace,
good order, safety and general welfare of the people. It can prohibit all things
hurtful to comfort, safety and welfare of society.5 It may also regulate property
rights.6 In the language of Chief Justice Enrique M. Fernando "the necessities
imposed by public welfare may justify the exercise of governmental authority to
regulate even if thereby certain groups may plausibly assert that their interests
are disregarded."

In so far as the non-application of the assailed Circulars to other transportation
services is concerned, it need only be recalled that the equal protection clause
does not imply that the same treatment he accorded all and sundry. It applies to
things or persons identically or similarly situated. It permits of classification of
the object or subject of the law provided classification is reasonable or based on
substantial distinction, which make for real differences, and that it must apply
equally to each member of tire class. What is required under the equal
protection clause is the uniform operation by legal means so that all persons
under identical or similar circumstance would be accorded the same treatment
both in privilege conferred and the liabilities imposed. The challenged Circulars
satisfy the foregoing criteria.

Last saved by CHAMP
8/22/2005 11:42 PM
Updated by K Pascual. 2014. 4C. Bill of Rights Poli Digests. Atty. Jack Jimenez. 4C. 30


24. TABLARIN VS. GUTIERREZ
equal protection

Petitioners sought admission into colleges or schools of medicine for the
school year 1987-1988. However, the petitioners either did not take or did
not successfully take the National Medical Admission Test (NMAT) required
by the Board of Medical Education.
They sought to enjoin the DECS Sec., and the Board of Medical Education
from enforcing Republic Act No. 2382, and MECS Order No. 52, which
required the taking and passing of the NMAT as a condition for securing
certificates of eligibility for admission in MedSchools.
The Medical Act of 1959 created the Board of Medical Education,
authorizing it to promulgate rules or requirements for admission into
medical schools.
The MECS Order No. 52, also established a uniform admission test called the
National Medical Admission Test (NMAT) as an additional requirement for
issuance of a certificate of eligibility for admission into medical schools of
the Philippines. The NMAT, an aptitude test, is considered as an instrument
toward upgrading the selection of applicants for admission into the medical
schools and its calculated to improve the quality of medical education in the
country. The cutoff score for the successful applicants, based on the scores
on the NMAT, shall be determined every year by the Board of Medical
Education after consultation with the Association of Philippine Medical
Colleges. The NMAT rating of each applicant, together with the other
admission requirements as presently called for under existing rules, shall
serve as a basis for the issuance of the prescribed certificate of elegibility
for admission into the medical colleges.
ISSUE ON EQUAL PROTECTION: They claim that the provision which says
that "the cutoff score for the successful applicants, based on the scores on
the NMAT, shall be determined every year by the Board after consultation
with the Association of Philippine Medical Colleges infringes the
requirements of equal protection. They assert, in other words, that students
seeking admission during a given school year, e.g., 1987-1988, when
subjected to a different cutoff score than that established for an, e.g., earlier
school year, are discriminated against and that this renders the MECS Order
"arbitrary and capricious."

ISSUE: Is the law in accordance with equal protection?
SC: YES. LAW VALID.

Different cutoff scores for different school years may be dictated by differing
conditions obtaining during those years. Thus, the appropriate cutoff score for a
given year may be a function of such factors as the number of students who
have reached the cutoff score established the preceding year; the number of
places available in medical schools during the current year; the average score
attained during the current year; the level of difficulty of the test given during
the current year, and so forth. To establish a permanent and immutable cutoff
score regardless of changes in circumstances from year to year, may well result
in an unreasonable rigidity. The above language in MECS Order No. 52, far from
being arbitrary or capricious, leaves the Board of Medical Education with the
measure of flexibility needed to meet circumstances as they change.

We conclude that prescribing the NMAT and requiring certain minimum scores
therein as a condition for admission to medical schools in the Philippines, do
not constitute an unconstitutional imposition.

Last saved by CHAMP
8/22/2005 11:56 PM
Other Issues:

Validity of exercise of police powers/ due process:
Updated by K Pascual. 2014. 4C. Bill of Rights Poli Digests. Atty. Jack Jimenez. 4C. 31


Perhaps the only issue that needs some consideration is whether there is some
reasonable relation between the prescribing of passing the NMAT as a condition
for admission to medical school on the one hand, and the securing of the health
and safety of the general community, on the other hand. This question is
perhaps most usefully approached by recalling that the regulation of the
practice of medicine in all its branches has long been recognized as a reasonable
method of protecting the health and safety of the public.8 That the power to
regulate and control the practice of medicine includes the power to regulate
admission to the ranks of those authorized to practice medicine, is also well
recognized.

The government is entitled to prescribe an admission test like the NMAT as a
means for achieving its stated objective of "upgrading the selection of applicants
into [our] medical schools" and of "improving) the quality of medical education
in the country." We hold that the NMAT is reasonably related to the securing of
the ultimate end of legislation and regulation in this area. That end, it is useful
to recall, is the protection of the public from the potentially deadly effects of
incompetence and ignorance in those who would undertake to treat our bodies
and minds for disease or trauma.

The right of every citizen to choose a profession:
Also, the State is not really enjoined to take appropriate steps to make quality
education "accessible to all who might for any number of reasons wish to enroll
in a professional school but rather merely to make such education accessible to
all who qualify under "fair, reasonable and equitable admission and academic
requirements."








25. BACHE VS. RUIZ
searches, seizures, arrests

The CIR Vera, wrote a letter to Judge Vivencio M. Ruiz requesting the
issuance of a search warrant against Bache & Co, a corporation duly
organized and existing under the laws of the Philippines, for violation of
Section 46(a) of the National Internal Revenue Code.
A BIR examiner, de Leon, was tasked to file the application.
De Leon and his witness, Arturo, went to the CFI. They brought with them
an application for search warrant already filled up but still unsigned by De
Leon; an affidavit of Arturo subscribed before respondent De Leon; a
deposition in printed form of Arturo already accomplished and signed by
him but not yet subscribed; and a search warrant already accomplished but
still unsigned by the Judge.
At that time Judge Ruiz was busy hearing a certain case; so, by means of a
note, he instructed his Deputy Clerk of Court to take the depositions of De
Leon and Arturo. The stenographer, upon request of Judge Ruiz, read to the
Judge her stenographic notes; and thereafter, Judge asked the witness
Arturo to take the oath and warned him that if his deposition was found to
be false and without legal basis, he could be charged for perjury. Judge Ruiz
then signed the search warrant and issued the same.
Later, BIR agents served the search warrant at the office of Bache & Co.
along Ayala Ave. The search yielded six boxes of documents.
Bache & Co protested the search on the ground that no formal complaint or
transcript of testimony was attached to the warrant.

Updated by K Pascual. 2014. 4C. Bill of Rights Poli Digests. Atty. Jack Jimenez. 4C. 32


ISSUE: Can a corporation invoke the constitutional right against unreasonable
searches and seizures?
SC: YES.
A corporation is, after all, but an association of individuals under an assumed
name and with a distinct legal entity. In organizing itself as a collective body it
waives no constitutional immunities appropriate to such body. Here, the
corporation to whom the seized documents belong, and whose rights have
thereby been impaired, is itself the petitioner. The rights of a corporation
against unlawful search and seizure are to be protected even if the same result
might have been achieved in a lawful way.

ISSUE: Was the search warrant (SW) validly issued?
SC: NO.
Judge Ruiz failed to personally examine the complainant and his witness.
The examination of the complainant and the witnesses he may produce, should
be conducted by the judge himself and not by others. The Constitution and the
Rules of Court (ROC) require the judge, before issuing a search warrant, to
"personally examine on oath or affirmation the complainant and any witnesses
he may produce."
Personal examination by the judge of the complainant and his
witnesses is necessary to enable him to determine the existence or non-
existence of a probable cause, because the law prohibits the issuance of
warrants except "upon probable cause." The determination of whether or not a
probable cause exists calls for the exercise of judgment after a judicial appraisal
of facts and should not be allowed to be delegated in the absence of any rule to
the contrary.
Here, no personal examination at all was conducted by Judge Ruiz of
the complainant (De Leon) and his witness (Arturo). While it is true that the
complainant's application for search warrant and the witness' printed-form
deposition were subscribed and sworn to before Judge, the latter did not ask
either of the two any question the answer to which could possibly be the basis
for determining whether or not there was probable cause against herein
petitioners. Indeed, the participants seem to have attached so little significance
to the matter that notes of the proceedings before respondent Judge were not
even taken.
The participation of Judge in the proceedings which led to the issuance
of the SW was thus limited to listening to the stenographer's readings of her
notes, to a few words of warning against the commission of perjury, and to
administering the oath to the complainant and his witness. This cannot be
considered a personal examination. If there was an examination at all of the
complainant and his witness, it was the one conducted by the Deputy Clerk of
Court. But, as stated, the Constitution and the rules require a personal
examination by the judge.
The reading of the stenographic notes to the Judge did not constitute
sufficient compliance with the constitutional mandate and the rule; for by that
manner Judge did not have the opportunity to observe the demeanor of the
complainant and his witness, and to propound initial and follow-up questions
which the judicial mind, on account of its training, was in the best position to
conceive. These were important in arriving at a sound inference on the all-
important question of whether or not there was probable cause.

The search warrant was issued for more than one specific offense.
The SW was issued simply for "violation of Sec. 46(a) of the National Internal
Revenue Code in relation to all other pertinent provisions thereof particularly
Secs. 53, 72, 73, 208 and 209."
The search warrant in question was issued for at least four distinct
offenses under the Tax Code. (the filing of income tax returns), (withholding
of income taxes at source), (unlawful pursuit of business or occupation), and
(failure to make a return of receipts, sales, business or gross value of output).
Under the ROC, no search warrant shall issue for more than one specific offense.

The search warrant does not particularly describe the things to be seized.
The SW merely stated, Unregistered and private books of accounts (ledgers,
journals, columnars, receipts and disbursements books, customers ledgers);
Updated by K Pascual. 2014. 4C. Bill of Rights Poli Digests. Atty. Jack Jimenez. 4C. 33


receipts for payments received; certificates of stocks and securities; contracts,
promissory notes and deeds of sale; telex and coded messages; business
communications, accounting and business records; checks and check stubs;
records of bank deposits and withdrawals; and records of foreign remittances,
covering the years 1966 to 1970."
The SW tends to defeat the major objective of the Bill of Rights, i.e., the
elimination of general warrants, for the language used therein is so all-
embracing as to include all conceivable records of petitioner corporation, which,
if seized, could possibly render its business inoperative.
The evident purpose and intent of this requirement is to limit the things
to be seized to those, and only those, particularly described in the search
warrant - to leave the officers of the law with no discretion regarding what
articles they shall seize, to the end that 'unreasonable searches and seizures'
may not be made, - that abuses may not be committed.
In this event, the description contained in the herein disputed warrant
should have mentioned, at least, the dates, amounts, persons, and other
pertinent data regarding the receipts of payments, certificates of stocks and
securities, contracts, promissory notes, deeds of sale, messages and
communications, checks, bank deposits and withdrawals, records of foreign
remittances, among others, enumerated in the warrant.
26. LIM VS. FELIX
searches and seizures

On March 17, 1989, at about 7:30 o'clock in the morning, at the vicinity of
the airport road of the Masbate Domestic Airport, Congressman Moises
Espinosa and his security escorts were attacked and killed by a lone
assassin.
After preliminary investigation, the investigator filed an amended
complaint accusing Vicente Lim and Mayor Susan Lim of multiple murder.
The MTC of Masbate concluded that a probable cause has been established
for the issuance of a warrant of arrest. The court thus issued a warrant of
arrest (WOA).
In the meantime, the SC ordered the transfer of the venue to Makati. The
Accused Lims asked the RTC of Makati to verify again the existence of
probable cause regarding the issuance of the WOA. The RTC of Makati
denied the request and still issued the WOA.
It will be noted that the preliminary investigation was conducted by the
MTC of Masbate, which found the existence of probable cause that the
offense of multiple murder was committed arid that all the accused are
probably guilty thereof, which was affirmed upon review by the Provincial
Prosecutor who properly filed with the Regional Trial Court four separate
informstions for murder. Considering that both the two competent officers
to whom such duty was entrusted by law have declared the existence of
probable cause, each information is complete in form and substance, and
there is no visible defect on its face, the Makati Court finds it just and
proper to rely on the prosecutor's certification in each information which
reads.

ISSUE: Can the judge issue a warrant of arrest without bail by simply relying on
the prosecutions certification and recommendation that a probable cause
exists?
SC: NO.

The issuance of a warrant is not a mere ministerial function; it calls for the
exercise of judicial discretion on the part of the issuing magistrate. The judge
must satisfy himself of the existence of probable cause before issuing a warrant
or order of arrest. If on the face of the information the judge finds no probable
cause, he may disregard the fiscal's certification and require the submission of
the affidavits of witnesses to aid him in arriving at a conclusion as to the
existence of a probable cause.

Updated by K Pascual. 2014. 4C. Bill of Rights Poli Digests. Atty. Jack Jimenez. 4C. 34


What the Constitution underscores is the exclusive and personal responsibility
of the issuing judge to satisfy himself of the existence of probable cause. In
satisfying himself of the existence of probable cause for the issuance of a
warrant of arrest, the judge is not required to personally examine the
complainant and his witnesses. Following established doctrine and procedures,
he shall:
(1) personally evaluate the report and the supporting documents submitted by
the fiscal regarding the existence of probable cause and, on the basis thereof,
issue a warrant of arrest; or
(2) if on the basis thereof he finds no probable cause, he may disregard the
fiscal's-report and require the submission of supporting affidavits of witnesses
to aid him in arriving at a conclusion as to the existence of probable cause.
Sound policy dictates this procedure, otherwise judges would be unduly laden
with the preliminary examinations and investigation of criminal complaints
instead of concentrating on hearing and deciding cases filed before their
courts."

First, the determination of probable cause is a function of the Judge. It is
not for the Provincial Fiscal or Prosecutor nor for the Election Supervisor to
ascertain. Only the Judge and the Judge alone make this determination.
Second the preliminary inquiry made by a Prosecutor does not bind
the Judge. It merely assists him to make the determination of probable cause.
The Judge does not have to follow what the Prosecutor presents to him. By itself
the Prosecutor's certification of probable cause is ineffectual. It is the report, the
affidavits, the transcripts of stenographic notes (if any), and all other supporting
documents behind the Prosecutor's certiftcation which are material in assisting
the Judge to make his determination.
Third, Judges and Prosecutors alike should distinguish the preliminary
inquiry which determines probable cause for the issuance of a warrant of arrest
from the preliminary investigation proper which ascertains whether the
offender should be held for trial or released. Even if the two inquiries are
conducted in the course of one and the same proceeding, there should be no
confusion about the objectives. The determination of probable cause for the
warrant of arrest is made by the Judge. The preliminary investigation proper-
whether or not there is reasonable ground to believe that the accused is guilty
of the offense charged and, therefore, whether or not he should be subjected to
the expense, rigors and embarrassment of trial-is the function of the Prosecutor.

If a Judge relies solely on the certification of the Prosecutor as in this
case where all the records of the investigation are in Masbate, he or she has not
personally determined probable cause. The determination is made by the
Provincial Prosecutor. The constitutional requirement has not been satisfied.
The Judge commits a grave abuse of discretion.
The records of the preliminary investigation conducted by the
Municipal Court of Masbate and reviewed by the respondent Fiscal were still in
Masbate when the Fiscal issued the warrants of arrest against the petitioners.
There was no basis for the Judge to make his own personal determinate
regarding the existence of a probable cause fee the issuance of a warrant of
arrest as mandated by the Constitution. He could not possibly have known what
transpired in Masbate as he had nothing but a certification. Significantly, the
respondent Judge denied the petitioners' motion for the transmittal of the
records on the ground that the mere certification and recommendation of the
respondent Fiscal that a probable cause exists is sufficient for him to issue a
warrant of arrest.

The Judge has to exercise sound discretion for, after all, the personal
determination is vested in the Judge by the Constitution. It can be as brief or as
detailed as the circumstances of each case require. To be sure, the Judge must
go beyond the Prosecutor's certification and investigation report whenever
necessary. He should call for the complainant and witnesses themselves to
answer the court's probing questions when the circumstances of the case so
require.

CHAMP Page 34 9/16/2014

Updated by K Pascual. 2014. 4C. Bill of Rights Poli Digests. Atty. Jack Jimenez. 4C. 35


27. WEBB VS. DE LEON
searches and seizures

Again, this is the case of Hubert Webb Vizconde Massacre.
During the preliminary investigation, the NBI presented the following: (1)
the sworn statement of their principal witness, Maria Jessica M. Alfaro who
allegedly saw the commission of the crime; (2) the sworn statements of two
(2) of the former housemaids of the Webb family
The DOJ Panel later issued a 26 page resolution finding probable cause to
hold the accused for trial and recommended the filing of information of
Rape with Homicide.
The case was raffled to Branch 25 presided by Judge Escano. It was,
however, another Judge, Judge de Leon, the pairing judge of Judge Escano,
who issued the warrants of arrest against the petitioners.
Judge Escano later voluntarily inhibited himself from the case to avoid any
suspicion about his impartiality considering his employment with the NBI
before his appointment to the bench.
The case was re-raffled to Branch 274, presided by Judge Amelita Tolentino
who issued new warrants of arrest against the petitioners and their co-
accused.
Later, Webb voluntarily surrendered to the police authorities.
Webb contends that Judges de Leon and Tolentino gravely abused their
discretion when they failed to conduct a preliminary examination before
issuing warrants of arrest against them. Webb cites, (1) the issuance of
warrants of arrest in a matter of few hours, (2) the failure of said judges to
issue orders of arrest; (3) the records submitted to the trial court were
incomplete and insufficient from which to base a finding of probable cause;
He also assails the finding of probable cause by the DOJ.

SC:
WHAT IS PROBABLE CAUSE: The need to find probable cause is dictated by the
Bill of Rights which protects "the right of the people to be secure in their
persons x x x against unreasonable searches and seizures of whatever nature x
X X."An arrest without a probable cause is an unreasonable seizure of a person,
and violates the privacy of persons which ought not to be intruded by the State.
Probable cause to warrant arrest is not an opaque concept in our jurisdiction.
Continuing accretions of case law reiterate that they are facts and
circumstances which would lead a reasonably discreet and prudent man to
believe that an offense has been committed by the person sought to be arrested
. Other jurisdictions utilize the term man of reasonable caution or the term
ordinarily prudent and cautious man. The terms are legally synonymous and
their reference is not to a person with training in the law such as a prosecutor
or a judge but to the average man on the street. It ought to be emphasized that
in determining probable cause, the average man weighs facts and circumstances
without resorting to the calibrations of our technical rules of evidence of which
his knowledge is nil. Rather, he relies on the calculus of common sense of which
all reasonable men have in abundance.

The DOJ Panel did not gravely abuse its discretion when it found probable cause
against the petitioners. A finding of probable cause needs only to rest on
evidence showing that more likely than not a crime has been committed and
was committed by the suspects. Probable cause need not be based on clear and
convincing evidence of guilt neither on evidence establishing guilt beyond
reasonable doubt and definitely, not on evidence establishing absolute certainty
of guilt. While probable cause demands more than "bare suspicion," it requires
"less than evidence which would justify x x x conviction," A finding of probable
cause merely binds over the suspect to stand trial. It is not a pronouncement of
guilt. (In short, a low quantum and quality of evidence needed to support a
finding of probable cause.)


ISSUE: Was the WOA properly issued?
SC: YES.
Updated by K Pascual. 2014. 4C. Bill of Rights Poli Digests. Atty. Jack Jimenez. 4C. 36


In arrest cases there must be probable cause that a crime has been committed
and that the person to be arrested committed it, which of course can exist
without any showing that evidence of the crime will be found at premises under
that person's control."

What the Constitution underscores is the exclusive and personal responsibility
of the issuing judge to satisfy himself of the existence of probable cause. In
satisfying himself of the existence of probable cause for the issuance of a
warrant of arrest, the judge is not required to personally examine the
complainant and his witnesses. Following established doctrine and procedure,
he shall: (1) personally evaluate the report and the supporting documents
submitted by the fiscal regarding the existence of probable cause and, on the
basis thereof, issue a warrant of arrest; or (2) if on the basis thereof he finds no
probable cause, he may disregard the fiscal's report and require the submission
of supporting affidavits of witnesses to aid him in arriving at a conclusions as to
the existence of probable cause.
Sound policy dictates this procedure, otherwise judges would be unduly laden
with the preliminary examination and investigation of criminal complaints
instead of concentrating on hearing and deciding cases filed before their
courts,"

In the case at bar, the DOJ Panel submitted to the trial court its 26page report,
the two (2) sworn statements of Alfaro, and the sworn statements of Carlos
Cristobal and Lolita Birrer as well as the counter-affidavits of the petitioners.
Apparently, the painstaking recital and analysis of the parties' evidence made in
the DOJ Panel Report satisfied both judges that there is probable cause to issue
warrants of arrest against petitioners. Again, we stress that before issuing
warrants of arrest, judges merely determine personally the probability, not the
certainty of guilt of an accused. In doing so, judges do not conduct a de novo
hearing to determine the existence of probable cause. They just personally
review the initial determination of the prosecutor finding a probable cause to
see if it is supported by substantial evidence. The sufficiency of the review
process cannot be measured by merely counting minutes and hours. The fact
that it took the respondent judges a few hours to review and affirm the
probable cause determination of the DOJ Panel does not mean they made no
personal evaluation of the evidence attached to the records of the case.36

As priorly discussed, the various types of evidence extant in the records of the
case provide substantial basis for a finding of probable cause against the
petitioner. The corpus delicti of the crime is a given fact. There is an eyewitness
account of the imputed crime given by Alfaro. The alibi defense of petitioner
Webb is also disputed by sworn statements of their former maids. It was
therefore unnecessary for the respondent judges to take the further step of
examining ex parte the complainant and their witnesses with searching
questions.

28. ALVAREZ VS. CFI TAYABAS
searches and seizures

The chief of the secret service of the Anti-Usury Board, of the DOJ,
presented to Judge David an affidavit alleging that according to reliable
information, the petitioner kept in his house books, documents, receipts,
lists, chits and other papers used by him in connection with his activities as
a money-lender, charging usurious rates of interest in violation of the law.
In his oath at the end of the affidavit, the chief of the secret service stated
that his answers to the questions were correct to the best of his knowledge
and belief. He did not swear to the truth of his statements upon his own
knowledge of the facts but upon the information received by him from a
reliable person. Upon the affidavit in question the judge, on said date,
issued the warrant which is the subject matter of the petition, ordering the
search of the petitioner's house.
Among the articles seized were: internal revenue licenses for the years
1933 to 1936, one ledger, two journals. two cashbooks. nine order books,
four notebooks, four check stubs, two memorandums, three bankbooks, two
contracts, four stubs, forty-eight stubs of purchases of copra, two
Updated by K Pascual. 2014. 4C. Bill of Rights Poli Digests. Atty. Jack Jimenez. 4C. 37


inventories, two bundles of bills of lading, one bundle of credit receipts, one
bundle of stubs of purchases of copra, two packages of correspondence, one
receipt book belonging to Luis Fernandez, fourteen bundles of invoices and
other papers, many documents and loan contracts with security and
promissory notes, 504 chits, promissory notes and stubs of used checks of
the HSBC.
Alvarez asks that the search warrant (SW) be declared illegal and set aside,
and prays that all the articles in question be returned to him. He claims that
the search warrant issued by the court is illegal because it has been based
upon the affidavit of agent Mariano G. Almeda in whose oath he declared
that he had no personal knowledge of the facts which were to serve as a
basis for the issuance of the warrant but that he had knowledge thereof
through mere information secured from a person whom he considered
reliable.

SC: SW INVALID.
A search warrant is an order in writing, issued in the name of the People of the
Philippine Islands, signed by a judge or a justice of the peace, and directed to a
peace officer, commanding him to search for personal property and bring it
before the court.

It will be noted that both provisions require that there be not only probable
cause before the issuance of a search warrant but that the search warrant must
be based upon an application supported by oath of the applicant and the
witnesses he may produce. In its broadest sense, an oath includes any form of
attestation by which a party signifies that he is bound in conscience to perform
an act faithfully and truthfully; and it is sometimes defined as an outward
pledge given by the person taking it that his attestation or promise is made
under an immediate sense of his responsibility to God.

The oath required must refer to the truth of the facts within the personal
knowledge of the petitioner or his witnesses, because the purpose thereof is to
convince the committing magistrate, not the individual making the affidavit and
seeking the issuance of the warrant, of the existence of probable cause. The true
test of sufficiency of an affidavit to warrant issuance of a search warrant is
whether it has been drawn in such a manner that perjury could be charged
thereon and affiant be held liable for damages caused.
It appears that the affidavit, which served as the exclusive basis of the search
warrant, is insufficient and fatally defective by reason of the manner in which
the oath was made, and therefore, it is hereby held that the search warrant in
question and the subsequent seizure of the books, documents and other papers
are illegal and do not in any way warrant the deprivation to which the
petitioner was subjected.

It is the practice in this jurisdiction to attach the affidavit of at least the
applicant or complainant to the application. It is admitted that the judge who
issued the search warrant in this case, relied exclusively upon the affidavit
made by agent Mariano G. Almeda and that he did not require nor take the
deposition of any other witness. The purpose of both in requiring the
presentation of depositions is nothing more than to satisfy the committing
magistrate of the existence of probable cause. Therefore, if the affidavit of the
applicant or complainant is sufficient, the judge may dispense with that of other
witnesses. Inasmuch as the affidavit of the agent in this case was insufficient
because his knowledge of the facts was not personal but merely hearsay, it is
the duty of the judge to require the affidavit of one or more witnesses for the
purpose of determining the existence of probable cause to warrant the issuance
of the search warrant. When the affidavit of the applicant or complainant
contains sufficient facts within his personal and direct knowledge, it is sufficient
if the judge is satisfied that there exists probable cause; when the applicant's
knowledge of the facts is mere hearsay, the affidavit of one or more witnesses
having a personal knowledge of the facts is necessary. We conclude, therefore,
that the warrant issued is likewise illegal because it was based only on the
affidavit of the agent who had no personal knowledge of the facts.

PARTICULARITY OF DESCRIPTION: One of the grounds alleged by the petitioner
in support of his contention that the warrant was issued illegally is the lack of
an adequate description of the books and documents to be seized. But where, by
the nature of the goods to be seized, their description must be rather general, it
Updated by K Pascual. 2014. 4C. Bill of Rights Poli Digests. Atty. Jack Jimenez. 4C. 38


is not required that a technical description be given, as this would mean that no
warrant could issue. The only description of the articles given in the affidavit
presented to the judge was as follows "that there are being kept in said
premises books, documents, receipts, lists, chits and other papers used by him
in connection with his activities as money-lender, charging a usurious rate of
interest, in violation of the law." Taking into consideration the nature of the
articles so described, it is clear that no other more adequate and detailed
description could have been given, particularly because it is difficult to give a
particular description of the contents thereof. The description so made
substantially complies with the legal provisions because the officer of the law
who executed the warrant was thereby placed in a position enabling him to
identify the Articles, which he did.

(read orig for other issues)..
CHAMP Page 38 9/16/2014










29. STONEHILL VS. DIOKNO
searches and seizures

A total of 42 SW were issued against Stonehill and the corporations of
which they were officers. The SW ordered the seizure of books of accounts,
financial records, vouchers, receipts, ledgers, journals, portfolios, balance
sheets, etc. etc.
They were allegedly the subject of the offense, either as the proceeds or
fruits, or used in the commission of the crime. (The crime being the
violation of Central Bank Laws, Tariff and Customs Codes, Revenue Code,
Penal Code).
Stonehill assailed the SW on the ground that it does not describe with
particularity the books, documents to be seized. (they also contend that the
SW were used as a fishing expedition in the deportation cases filed against
them)

SC: SW INVALID.
The documents and papers seized may be classified into 2: 1) those found in the
offices of the corporations, and 2) those found in the residences of Stonehill, et.
al

As to the first group: Stonehill has no cause of action to assail the legality of the
SW, since the corporation has a separate juridical personality from them. Hence,
regardless of the amount of shares they had, it is the corporation who can
question the validity of the SW. The legality of the seizure can only be contest
by the party whose rights have been impaired thereby. The objection to an
unlawful search or seizure is STRICTLY PERSONAL. The right to question the
SW belongs exclusively to the corporation, to whom the seized documents
belong, and may not be invoked by the corporate officers like Stonehill.

Updated by K Pascual. 2014. 4C. Bill of Rights Poli Digests. Atty. Jack Jimenez. 4C. 39


As to the second group: Stonehill argues that the SW was in the nature of a
general warrant, and that accordingly the seizures made are null and void.
There are 2 points to the constitutional mandate:
1) that no warrant shall be issued except upon probable cause to be determined
personally by the judge purpose: to grant moral assurance to the court
2) that the warrant shall particularly describe the things to be seized.
purpose: to deny discretion to police officers as to what will be seized, to prevent
arbitrariness

HERE, NONE OF THESE REQUIREMENTS WERE MET.
The SW merely stated for violation of central bank laws, tariff codes, RPC, etc.
There was no specific offense alleged in the application. The averments with
respect to the offense committed were abstract. Thus, it was impossible for the
judge to have found the existence of probable cause for the same presupposes
the introduction of competent proof that the party against whom it is sought has
performed particular acts, or committed specific offenses.

It would be legal heresy of the highest order to convict anybody by simply
allegation of violation of Central Bank Laws, Tariff Codes, Revenue Codes, RPC.

Furthermore, under the Rules of Court, a SW shall not issue except upon
probable cause in connection with one specific offense. In this case, the SW
authorized the seizure of all records pertaining to all business transactions of
Stonehill, regardless of whether the transactions were legal or illegal. The SW
thus contravenes the Constitutional mandate of particularity of description of
the things to be seized. It is in a nature of a general warrant.


30. SEC OF JUSTICE VS. MARCOS
searches and seizures

An application for SW filed by Chief of Police of Baguio City, following an
affidavit of one Romeo Amansec, regarding a illegal possession of firearms
and ammunition by the accused, Rogelio, as well as for violation of Central
Bank Regulations for storing a golden Buddha kept at the premises.
The Secretary of Justice argued that the search warrant was not limited to
one offense covering both illegal possession of firearms and violation of
Central Bank rules and regulations; that it did not particularly describe the
property to be seized; that he did not carefully examine under oath the
applicant and his witnesses; (Hence this is an administrative case against
the Judge).
An investigation was conducted.

SC:
There can be no question that from a reading of the application for search
warrant by Col. Calano, and the affidavit by witness Romeo Amansec, as well as
the search warrant itself, it can be deduced that-the deposition in writing of Sgt.
De Vera, was valid; and that the Judge did examine under oath, both applicant,
Col. Calano, and witnesses, Romeo Amansec and Sgt. De Vera.

What the Secretary of Justice presented as his sole witness, was the Clerk of
Court, Fernando R. Romero, who declared that, 'Q:Would you be able to tell us
more or less what sorts of questions were propounded by Judge Marcos by way
of interrogating these witnesses? A: I cannot repeat the words because it was a
long time ago, but if I may be permitted to make a gist I may be able to relate. Q:
Please do so. A: After administering the oath individually to Col. Calano,
Amansec and de Vera, after giving their names and other personal
circumstances, he dealt on the questions "What were those articles that were
Updated by K Pascual. 2014. 4C. Bill of Rights Poli Digests. Atty. Jack Jimenez. 4C. 40


the subject of the application? And they described the articles, what I heard are
a golden Buddha, a rifle with some ammunitions."'

This being the case, we are bound to accept this testimony and must hold that
outside of the literal defect in that Respondent had not taken the written
deposition of Sgt. de Vera, the proceedings he had adopted were not fatally
wrong, in fact, might as well be added that there is something very probable and
therefore credible in the testimony of this witness that time was of the extreme
essence, the urgency of the situation could no longer permit -further going back
to the Office of the Clerk of Court for renewed typing of application and
affidavits and warrant, at any rate, since it was complainant Secretary of Justice
himself who presented Clerk of Court Romero, he should be bound by latter's
testimony.

SPECIFIC OFFENSE NOT OK!: As to the second ground consisting of the
warrant itself being defective because it was issued for two offenses and the
description of the premises to be searched and the objects to be seized being
too general, "Since the warrant really stated that it was issued for, 'Illegal
possession of firearms and Violation of Central Bank Rules and Regulations,'
and the body recited that, 'Accused Rogelio Ro as illegally possess[es] firearms
and ammunition without license or permit to possess the same and a golden
Buddha which he is keeping and concealing at his premises at No. 47 Ledesma
St., Baguio City in violation of Central Bank Rules and Regulations; this warrant
violated Sec. 3 of Rule 126 which provides that, 'No Search warrant shall issue
for more than one specific offense.' Investigator must even add that the
particular Central Bank circular or regulation is not determined;

PARTICULARITY OF DESCRIPTION --OK! : As to the attack on the description of
the premises to be searched and the objects to be seized, the test of a good
description for purposes of sufficiency of the warrant is that it be one that
would not permit seizure of the wrong property.

Taking into consideration the nature of the articles so described, it is clear that
no other more adequate and detailed description could be given, particularly
because it is difficult to give a particular description of the contents thereof. The
description so made substantially complies with the legal provisions because
the officer of the law who executed the warrant was thereby placed in a position
enabling him to identify the articles in question, which he did,' * * * so that here,
since certainly, no one would be mistaken in identifying the Buddha, whose
image image is well known, and even the firearms and ammunition because
these were those without permit to possess, and all located at No. 47 Ledesma
St., Baguio City, so far as description was concerned, the search warrant perhaps
could not be said to have suffered fatal defects.

PROBABLE CAUSE OK! : As to the third charge that the search warrant was
issued without probable cause, Justice Gatmaitan started with the affidavit of
Amansec showing that " 'on or about 6:30 a.m. of March 31,1971, I went to
Baguio City and while I passed by a house at No. 47 Ledesma Street, Baguio City
I was attracted by several persons inside the house; That I peeped from outside
the house and when the curtain was moved I saw a Buddha that was inside the
house; That I observed what was going on inside the house and I heard
someone say that the golden Buddha was actually for sale and when I observed
them closer I overheard that it was being offered for sale for 100,000 pesos;
That I peeped again and I actually saw for myself again the Buddha and I heard
one of the persons inside whom I later found out to be Mr. Rogelio Roxas that it
was a golden Buddha but that a down payment was needed; That I am executing
this affidavit because I actually saw the Buddha and that I also saw a firearm
and some bullets inside the house.' * * * which can easily sustain, contrary to
complainant's position, that Amansec's knowledge was not hearsay at all. Also,
Sgt. de Vera's knowledge neither was hearsay either. Neither should it be
overlooked that all these if they existed at all were directed to the conscience of
examining Judge who was the one called upon to grade their credibility, to act
with precision; the point is that while at the beginning the knowledge of
witnesses had come was that they were able to confirm the same with their own
eyes, the existence of the suspected articles within the premises, and that was
what they certified under interrogation - from respondent, - as declared no less
by cornplaint's own witness, the Clerk of Court, Fernando R. Romero was
present thereat.

Updated by K Pascual. 2014. 4C. Bill of Rights Poli Digests. Atty. Jack Jimenez. 4C. 41


Please read orig. ang gulo kasi ni Justice Fernando.




31. UY VS. BIR
searches and seizures

A certain Abos reported to the BIR that Unifish Packing Corpo, and Frank Uy
were engaged in activities violating the NIRC. The Corporation is allegedly
selling sardines without issuing receipts, and of various tax frauds such as
(dictating the value of the sardines without any receipt, payments are made
pay to cash and without any receipt, smuggling, illegally availing of tax
exemption, etc.).
The BIR thus applied for SW to search the premises of Unifish. The SW was
issued. The BIR raided the premises of Unifish and confiscated records and
books of accounts of the corporation.
Unifish assails the validity of the SW claiming that it was issued in gadalej
by the Judge.

SC:
1. DESCRIPTION OF THE PLACE TO BE SEARCHED. The constitution requires
that three be a particular description of the place to be searched and the
persons or things to be seized. The description is sufficient if the officer can
with reasonable effort, ascertain, identify the place intended and distinguish it
from other places in the community. As long as the designation or description
points out the place to the exclusion of all others, and that the officers will be led
to it, then the description is sufficient.
Here, it was not shown that a street similarly named Hernan Cortes
could be found in Cebu City. Nor was it established that the enforcing officers
had any difficulty in locating the premises of the corporation. That the first SW
inconsistently identified the city where the premises will be searched is NOT A
DEFECT that would invalidate the SW.

2. DESCRIPTION OF THE PERSONS NAMED IN THE 2 SW. Frank Uy claims that
there was inconsistency in the 2 SW since under SW-1 it was issued solely
against Uy Chin Ho, alias Frank Uy, while SW-2 was issued against Uy Chin Ho
alias Frank Uy AND Unifish Packing Corpo.
The discrepancies are hardly relevant. The Constitution does not
require the warrant to name the person who occupies the premises. Since the
SW is solely for the search of the specifically described premises, the failure to
name the owner or occupant of such property does NOT INVALIDATE the
warrant. Where the name of the owner of the premises is incorrectly inserted in
the SW, it is NOT A FATAL DEFECT if the description is nonetheless correct so
that no discretion is left to the officer in making the search. Inconsistencies in
stating the names NOT FATAL.

3. TWO WARRANTS ISSUED AT ONE TIME FOR ONE CRIME AND ONE PLACE.
Even if two SW were issued for the same crime (violation of sec 253 of NIRC),
SW-1 should be deemed superseded by SW-2. Sw-2 merely corrects the
inconsistencies in the address, and was just to include UNifish Corpo. SW-2 was
issued so that the names of the persons and the description of the place would
be more precise.

4. PROBABLE CAUSE. Probable cause is defined as such facts and circumstances
which would lead a reasonably discreet and prudent man to believe that an
offense has been committed and that the objects sought in connection with the
offense are in the place sought to be searched. In the determination of probable
cause, the Consti requires the examination of witnesses under oath. The
examination must be probing, exhaustive and not merely routinary or pro-
forma. The judge must not simply rehash the contents of the affidavit but must
make his own inquiry. The witnesses must also testify under oath of their own
Updated by K Pascual. 2014. 4C. Bill of Rights Poli Digests. Atty. Jack Jimenez. 4C. 42


personal knowledge. The oath must refer to the truth of the facts within the
personal knowledge of the petitioner or his witnesses. A SW cannot be issued on
loose, vague, or doubtful basis of fact, nor on mere suspicion or belief.
Here, the judge deposed 2 witnesses, the BIR agent and the former
employee of Unifish. (the transcript was included in the decision I think what
was said was that...) the testimony of the BIR agent was found to be NOT from
his own perception but merely hearsay, while the testimony of the former
employee however is based on personal knowledge because he detailed the
schemes employed by Frank Uy for tax evasion. The employee also described
the place where the documents were kept. In short, the court found that the
judge made inquiries that were sufficiently probing.

5. PARTICULARITY OF DESCRIPTION. A SW sufficiently describes the things to
be seized when the description is as specific as the circumstances will ordinarily
allow, or when the description expresses a conclusion of fact, by which the
officer may be guided in making the search and seizure. It is also sufficient when
the things described are limited to those which bear direct relation to the
offense for which the warrant is being issued.
Here, the SW does not conform to these tests. The description should
have mentioned the dates, amounts, persons and other data regarding the
receipts of payments, etc. The applicant must necessarily have some other
evidence, other than the articles to be seized, to prove the said offense. Here, the
description is all embracing, which includes all conceivable records of the
corporation, which if seized, could paralyze its business. (The SW merely
described books of accounts, ledgers, journals, cash books, sales books,
receipts, delivery receipts, sales invoices, financial statements, bank
statements. ) in short they were merely generic descriptions, and hence
invalid.
HOWEVER, with respect to the UNREGISTERED DELIVERY RECEIPTS,
THEY ARE SUFFICIENTLY DESCRIBED AS SUCH. There is no need to include in
the description the serial markings because it is impossible to do so precisely
because they are unregistered. It would also be impractical to specify each and
every receipt and the contents thereof. It is not required that a technical
description be given. Taking into account the nature of the articles, no other
more detailed description could be given. THUS, THE SW IS VALID AS TO THE
UNREGISTERED RECEIPTS. The SW is severable, and those items not
particularly described may be cut off without destroying the whole warrant.

CHAMP Page 42 9/16/2014
32. PEOPLE VS. VELOSO
searches and seizures

Jose M.a Veloso, was found guilty of the crime of resistance of the agents of
the authority.
Jose M.a Veloso was it that time a member of the House of Representatives,
and the manager of the Parliamentary Club, an organization located at No.
124 Calle Arzobispo, City of Manila.
The police of Manila had reliable information that the so-called
Parliamentary Club was nothing more than a gambling house.
The secret service of the City of Manila, applied for, and obtained a search
warrant from Judge Garduo of the municipal court. The police attempted
to raid the Parliamentary Club but when they found the doors to the
premises were closed and barred, one policeman, ascended a telephone
pole, so as to enter a window of the house.
Once inside the Parliamentary Club, Veloso was arrested. Veloso read the
search warrant but told the police that he was Representative Veloso and
not John Doe, and that the police had no right to search the house. The
police replied that Veloso was considered as a John Doe. The policemen
insisted on searching Veloso (because his pockets were bulging with
gambling materials), but Veloso insisting in his refusal to submit to the
search.
Veloso was later convicted of gambling.
Updated by K Pascual. 2014. 4C. Bill of Rights Poli Digests. Atty. Jack Jimenez. 4C. 43


In appealing his conviction, he argues that since the name of Veloso did not
appear in the search warrant, but instead the pseudonym John Doe was
used, the SW cannot be used against him.

SC: VALID SW.

The affidavit for the search warrant and the search warrant itself described the
building to be searched as "the building No. 124 Calle Arzobispo, City of Manila,
Philippine Islands." This, without doubt, was a sufficient designation of the
premises to be searched. It is the prevailing rule that a description of a place to
be searched is sufficient if the officer with the warrant can, with reasonable
effort, ascertain and identify the place intended.

The police officers were accordingly authorized to break down the door and
enter the premises of the building occupied by the so-called Parliamentary Club.
When inside, they then had the right to arrest the persons presumably engaged
in a prohibited game, and to confiscate the evidence of the commission of the
crime. It has been held that an officer making an arrest may take from the
person arrested any money or property found upon his person, which was used
in the commission of the crime or was the fruit of the crime, or did not do an
unlawful act by lawful means, or a lawful act by unlawful means, and so could
not be convicted of the misdemeanor of a riot, with which they are charged in
the indictment."

While it is true that the SW failed to name Jose M.a Veloso as the person to be
seized, it however stated that "John Doe has illegally in his possession in the
building occupied by him, and which is under his control, namely, in the
building numbered 124 Calle Arzobispo, City of Manila, Philippine Islands,
certain devices and effects used in violation of the Gambling Law."

The warrant in this case sufficiently described the place and the gambling
apparatus, and, in addition, contained a description of the person to be seized.
Under the authorities cited by the appellant, it is invariably recognized that the
warrant for the apprehension of an unnamed party is void, "except in those cases
where it contains a descriptio personae such as will enable the, officer to identify
the accused." The description must be sufficient to indicate clearly the proper
person upon, whom the warrant is to be served. As the search warrant stated
that John Doe had gambling apparatus in his possession in the building,
occupied by him at No. 124 Calle Arzobispo, City of Manila, and as this John Doe
was Jose M.a Veloso, the manager of the club, the Police could identify John Doe
as Jose M.a Veloso without difficulty.

CHAMP Page 43 9/16/2014


Other Principles:
"'John Doe' Warrants. It follows, on principle, from what has already been said
regarding the essential requirements of warrants for the apprehension of
persons accused, and about blank warrants, that a warrant for the apprehension
of a person whose true name is unknown, by the name of 'John Doe' or 'Richard
Roe,' 'whose other or true name is unknown,' is void, without other and further
descriptions of the person to be apprehended, and such warrant might as well
have been arrested, as being included in the, description, as the defendant
himself. Such a warrant was contrary to elementary principles, and in direct
violation of the constitutional right of the citizen. It was always necessary to
express the name or give some description of a party to be arrested on a
warrant; and if one was granted with the name in blank, and without other
designation of the person to be arrested, it was void.

Form and Sufficiency of Warrant. Technical accuracy is not required.
Updated by K Pascual. 2014. 4C. Bill of Rights Poli Digests. Atty. Jack Jimenez. 4C. 44


"Name and description of the accused should be inserted in the body of the
warrant; and where the name is unknown there must be such a description of
the person accused as will enable the officer to identify him when found.
"Warrant for apprehension of unnamed party, or containing a wrong name for
the party to be apprehended is void, except in those cases where it contains a
descriptio, personae such as will enable the officer to identify the accused.

33. PEOPLE VS. MUSA
searches and seizures plain view

Musa was charged and convicted of selling marijuana, following a buy-bust
operation conducted by the police. Narcom agents.
The buy-bust operation took place at Musas house, with one of the Narcom
agents acting as the buyer who went inside the house. After the exchange,
the Agent, Sgt Ani, made the arrest.
It appears that Sgt Ani gave a pre-arranged signal to the other narcom
agents to move it and make the arrest. They searched him to retrieve the
marked money but did not find it.
Musa then said that he gave the marked money to his wife. But the other
narcom agents, Sgt Belgra went to the kitchen and noticed something, a
cellophane colored white striped thing at the corner of the kitchen. They
asked Musa about its contents but Musa did not reply. The Narcom agents
thus opened the contents and found dried marijuana leaves.
Musa assails the seizure and admission as evidence of a plastic bag
containing marijuana which the Narcom agents found in the kitchen.

SC: SEARCH and SEIZURE INVALID. NOT ADMISSIBLE.

THERE WAS PROBABLE CAUSE. The testimony of Sgt Ani regarding the buy-
bust operation is credible, direct, lucid and forthright. The day before the buy-
bust, Sgt Ani made a test-buy and successfully bought marijuana from Musa.
Thus, Sgt Ani was able to gain the trust and confidence of Musa to be able to sell
to him more marijuana the following day (during the buy-bust ops).

Also, even if the other narcom agent (Sggt Belarga) did not actually see the sale,
it was not totally impossible for him to have seen Sgt Ani hand to Musa
something, and for Musa to give Sgt Ani something. Hence, the testimony of
the other narcom Agent corroborates the direct evidence given by Sgt. Ani. The
testimony of the poseur-buyer (Sgt Ani) is sufficient to prove the consummation
of the sale.

WARRANTLESS SEARCH. There is no doubt that the warrantless search
incidental to a lawful arrest authorizes the arresting officer to make a search
upon the person of the person arrested. He may take from the person arrested
any money or property found upon his person which was used in the
commission of the crime or was the fruit of the crime or which might furnish the
prisoner with the means of committing violence or of escaping, or which may
be used as evidence in the trial of the cause.

Hence, in a buy-bust operation conducted to entrap a drug-pusher, the law
enforcement agents may seize the marked money found on the person of the
pusher immediately after the arrest even without arrest and search warrants.
Here, the NARCOM agents searched the person of the appellant after arresting
him in his house but found nothing. They then searched he entire house and, in
the kitchen, found and seized a plastic bag hanging in a comer.

The warrantless search and seizure, as an incident to a suspect's lawful arrest,
may extend beyond the person of the one arrested to include the premises or
surroundings under his immediate control. Objects in the "plain view" of an
officer who has the right to be in the position to have that view are subject to
seizure and may be presented as evidence.
Updated by K Pascual. 2014. 4C. Bill of Rights Poli Digests. Atty. Jack Jimenez. 4C. 45



The "plain view" doctrine may not, however, be used to launch unbridled
searches and indiscriminate seizures nor to extend a general exploratory search
made solely to find evidence of defendant's guilt. The "plain view" doctrine is
usually applied where a police officer is not searching for evidence against the
accused, but nonetheless inadvertently comes across an incriminating object.

The justification is that the police officers came inadvertently across a piece of
evidence incriminating the accused. The doctrine serves to supplement the
prior justification-whether it be a warrant for another object, hot pursuit,
search incident to lawful arrest, or some other legitimate reason for being
present unconnected with a search directed against the accused-and permits
the warrantless seizure. It is allowed only where it is immediately apparent to
the police that they have evidence before them; the 'plain view' doctrine may
not be used to extend a general exploratory search from one object to another
until something incriminating at last emerges. Stated differently, it must be
immediately apparent to the police that the items that they observe may be
evidence of a crime, contraband, or otherwise subject to seizure.

In this case, Musa was arrested and searched in the living room.Failing to
retrieve the marked money which they hoped to find, the NARCOM agents
searched the whole house and found the plastic bag in the kitchen. The plastic
bag was, therefore, not within their "plain view' when they arrested the
appellant as to justify its seizure The NARCOM agents had to move item one
portion of the house to another before they sighted the plastic bag. The
NARCOM agents in this case went from room to room with the obvious
intention or fishing for more evidence. Moreover, when the NARCOM agents
saw the plastic bag hanging in one corner of the kitchen, they had no clue as to
its contents. They had to ask the appellant what the bag contained.

The NARCOM agents in this case could not have discovered the inculpatory
nature of the contents of the bag had they not forcibly opened it. Even assuming
then, that the NARCOM agents inadvertently came across the plastic bag
because it was within their "plain view," what may be said to be the object in
their "plain view" was just the plastic bag and not the marijuana. The
incriminating nature of the contents of the plastic bag was not immediately
apparent from the "plain view" of said object. It cannot be claimed that the
plastic bag clearly betrayed its contents, whether by its distinctive
configuration, its transprarency, or otherwise, that its contents are obvious to
an observer.

34. DEL ROSARIO VS. PEOPLE
G.R. No. 142295 May 31, 2001
searches and seizures plain view

Vicente del Rosario was convicted of illegal possession of firearms.
Sometime in May 1996, the police received a report that Vicente del Rosario
was in possession of certain firearms without the necessary licenses. The
PNP Firearms and Explosives Division investigated and found out that the
appellant is not a licensed/registered firearm holder of any kind and
caliber. Thus PNP applied for a search warrant to enable his team to search
the house of del Rosario.
Upon arrival at the house of appellant, the police officers introduced
themselves to the wife of appellant. When the appellant came out, the police
informed him that they had a search warrant and that they were authorized
to search his house. After appellant gave his permission, the police officers
conducted a search of the house.1wpWhen asked about his license to
possess the firearms, the appellant failed to produce any. This prompted the
police officers to seize the subject firearms. H
He appeals his conviction claiming that the search conducted at his
residence was illegal, and the search warrant was issued in violation of the
Constitution and consequently, the evidence seized was inadmissible. He
also submits that he had a license for the .45 caliber firearm and
ammunition seized in his bedroom. The other firearm, a .22 caliber revolver
seized in a drawer at the kitchen of his house, a magazine for 5.56 mm. cal.
Updated by K Pascual. 2014. 4C. Bill of Rights Poli Digests. Atty. Jack Jimenez. 4C. 46


Armalite rifle, and two 2-way radios found in his daughter's bedroom, were
either planted by the police or illegally seized, as they were not mentioned
in the search warrant.

ISSUE: was the seizure of the revolver and magazine valid even if they were not
mentioned in the SW?

SC: ILLEGAL.

With respect to the .22 caliber revolver that the police raiding team found in a
drawer at the kitchen of Vicente Del Rosarios house, suffice it to say that the
firearm was not mentioned in the search warrant applied for and issued for the
search of petitioner's house.

The requisites for a search warrant's validity are: (in the absence of even one
will cause" its downright nullification):
(1) it must be issued upon probable cause;
(2) the probable cause must be determined by the judge himself and not by the
applicant or any other person;
(3) in the determination of probable cause, the judge must examine, under oath
or affirmation, the complainant and such witnesses as the latter may produce;
and
(4) the warrant issued must particularly describe the place to be searched and
persons or things to be seized."

Seizure is limited to those items particularly described in a valid search
warrant. Searching officers are without discretion regarding what articles they
shall seize. Evidence seized on the occasion of such an unreasonable search and
seizure is tainted and excluded for being the proverbial "fruit of a poisonous
tree." In the language of the fundamental law, it shall be inadmissible in
evidence for any purpose in any proceeding.

In this case, the firearm was not found inadvertently and in plain view. It was
found as a result of a meticulous search in the kitchen of petitioner's house. This
firearm, to emphasize, was not mentioned in the search warrant. Hence, the
seizure was illegal.
50
The seizure without the requisite search warrant was in
plain violation of the law and the Constitution. True that as an exception, the
police may seize without warrant illegally possessed firearm or any contraband
for that matter, inadvertently found in plain view. However, "[t]he seizure of
evidence in 'plain view' applies only where the police officer is not searching for
evidence against the accused, but inadvertently comes across an incriminating
object."
52
Specifically, seizure of evidence in "plain view" is justified when there
is:

(a) a prior valid intrusion based on the valid warrantless arrest in which the
police are legally present in the pursuit of their official duties;
(b) the evidence was inadvertently discovered by the police who had the right
to be where they are.
(c) the evidence must be immediately apparent, and
(d) "plain view" justified mere seizure of evidence without further search.
53

The same is true with respect to the 5.56 cal. magazine found in the bedroom of
petitioner's daughter. The seizure was invalid and the seized items were
inadmissible in evidence.

With particular reference to the two 2-way radios that the raiding policemen
also seized in the bedroom of petitioner's daughter, there was absolutely no
reason for the seizure. The 2-way radios were not contraband per se. The
Updated by K Pascual. 2014. 4C. Bill of Rights Poli Digests. Atty. Jack Jimenez. 4C. 47


burden is on the prosecution to show that the two-way radios were not
licensed. The National Telecommunication Commission is the sole agency
authorized to seize unlicensed two-way radios. More importantly, admittedly,
the two-way radios were not mentioned in the search warrant. We condemn the
seizure as illegal and a plain violation of a citizen's right. Worse, the petitioner
was not charged with illegal possession of the two-way radios.1wphi1.nt

CHAMP Page 47 9/16/2014


35. PAPA VS. MAGO
warrantless searches and seizures customs

Martin Alagao, head of the counter-intelligence unit of the Manila Police
Department, obtained a reliable information that certain shipment of
personal effects, allegedly misdeclared and undervalued, would be released
from the customs zone of the port of Manila and loaded on two trucks.
Thus, the Manila Police conducted surveillance at gate No. 1 of the customs
zone.
When the trucks came out, the police intercepted them. The load of the two
trucks, consisting of nine bales of goods, and the two trucks, were seized.
Later, a person claimed ownership of the goods and showed to the
policemen a "Statement and Receipts of Duties Collected on Informal Entry
No. 147-5501", issued by the Bureau of Customs, in the name of a certain
Bienvenido Naguit.
Claiming to have been prejudiced by the seizure and detention of the two
trucks and their cargo, Remedios Mago and Valentin B. Lanopa claimed that
they were the owner of the goods, having purchased them from the Sta.
Monica Grocery in San Fernando, Pampanga, and that she hired the trucks
owned by Valentin B. Lanopa to transport the goods from said place to her
residence at Sampaloc, Manila;
The claim that that the goods and the trucks were seized without search
warrant issued by a competent court. They also claimed that even assuming
that the goods were misdeclared and undervalued, they should not be
subject of seizure, because Mago had bought them from another person
without knowledge that they were imported illegally.
Later, Mago also filed a motion to release the goods, alleging that since the
inventory of the goods seized did not show any article of prohibited
importation, the same should be released as per agreement of the parties
upon her posting of the appropriate bond that may be determined by the
court.
The police however opposed arguing that the court had no jurisdiction to
order the release of the goods in view of the fact that the court had no
jurisdiction over the case, and that most of the goods, as shown in the
inventory, were not declared and were, therefore, subject to forfeiture.
Mago was able to put up a bond of P40,000 and the goods were released.

SC: SEARCH AND SEIZURE VALID.

The Chief of the Manila Police Department, Ricardo G. Papa, having been
deputized in writing by the Commissioner of Customs, could, for the purpose of
the enforcement of the customs and tariff laws, effect searches, seizures, and
arrests, of any cargo, articles or other movable property when the same may be
subject to forfeiture or liable for any fine imposed under customs and tariff
laws. He could lawfully open and examine any box trunk, envelope or other
container wherever found when he had reasonable cause to suspect the
presence therein of dutiable articles introduced into the Philippines contrary to
law; and likewise to stop, search and examine any vehicle, beast or person
reasonably suspected of holding or conveying such article as aforesaid.

Updated by K Pascual. 2014. 4C. Bill of Rights Poli Digests. Atty. Jack Jimenez. 4C. 48


The Tariff and Customs Code does not require said warrant in the instant case.
The Code authorizes persons having police authority under Section 2203 of the
Tariff and Customs Code to enter, pass through or search any land, inclosure,
warehouse, store or building, not being a dwelling house; and also to inspect,
search and examine any vessel or aircraft and any trunk, package, box or
envelope or any person on board, or stop and search and examine any vehicle,
beast or person suspected of holding or conveying any dutiable or prohibited
article introduced into the Philippines contrary to law, without mentioning the
need of a search warrant in said cases.

But in the search of a dwelling house, the Code provides that said "dwelling
house may be entered and searched only upon warrant issued by a judge or
justice of' the peace . . ." Thus, except in the case of the search of a dwelling
house, persons exercising police authority under the customs law may effect
search and seizure without a search warrant in the enforcement of customs
laws.

Note also that the policemen did not have to make any search before they seized
the two trucks and their cargo. But even if there was a search, there is still
authority to the effect that no search warrant would be needed under the
circumstances obtaining in the instant case. This is because it was made on a
moving vehicle. The guaranty of freedom from unreasonable searches and
seizures is construed as recognizing a necessary difference between a search of
a dwelling house or other structure in respect of which a search warrant may
readily be obtained and a search of a ship, motorboat, wagon, or automobile for
contraband goods, where it is not practicable to secure a warrant, because the
vehicle can be quickly moved out of the locality or jurisdiction in which the
warrant must be sought."

Other issues:
Where the goods in question were imported from Hongkong as shown in the
statement and receipts of duties collected on informal entry and where the
importation has not been terminated, the imported goods remain under the
jurisdiction of the Bureau of Customs. Importation is terminated only upon the
full payment of duties, taxes and other charges upon the articles, or secured to
be paid, at the port of entry and the legal permit for withdrawal shall have been
granted. here, the BoC still had jurisdiction because the duties were still not
paid in full.

When the goods were intercepted by the police (who were deputized by the
Boc), then the BoC re-acquired jurisdiction over the goods. Thereafter, the BoC
has exclusive jurisdiction over the imported goods even if no warrant of seizure
has been issued.

36. ANIAG VS. COMELEC
warrantless searches and seizures - checkpoints

Comelec promulgated the Gun Ban starting December 11, 1991, in time for
the May 1992 elections.
Pursuant to this, the Sergeant-at-Arms of the House of Representatives,
wrote Congressman Aniag of Bulacan, requesting the return of the two (2)
firearms issued to him by the House of Representatives.
Thus, Congressman Aniag complied and immediately instructed his driver,
Ernesto Arellano, to pick up the firearms from his house at Valle Verde and
return them to Congress.
There was a checkpoint at the Batasan Complex.
The policemen manning flagged down the car driven by Arellano as it
approached the checkpoint. They searched the car and found the firearms
neatly packed in their gun cases and placed in a bag in the trunk of the car.
Arellano was then apprehended and detained.
The driver explained that he was merely instructed to return the guns to
Congress.
Updated by K Pascual. 2014. 4C. Bill of Rights Poli Digests. Atty. Jack Jimenez. 4C. 49


At the inquest proceedings, Congressman Aniag explained that Arellano did
not violate the firearms ban as he in fact was complying with it when
apprehended by returning the firearms to Congress; and, that he was
petitioner's driver, not a security officer nor a bodyguard.
Comelec however still filed charges against Arellano. Comelec also wanted
to disqualify Aniag from running in the elections because of the incident.
Congressman Aniag strongly protests against the manner by which the PNP
conducted the search. According to him, without a warrant and without
informing the driver of his fundamental rights the policemen searched his
car. The firearms were not tucked in the waist nor within the immediate
reach of Arellano but were neatly packed in their gun cases and wrapped in
a bag kept in the trunk of the car.

SC: SEARCH INVALID.
The checkpoint was set up twenty (20) meters from the entrance of the
Batasan. There was no evidence to show that the policemen were impelled to do
so because of a confidential report leading them to reasonably believe that
certain motorists matching the description furnished by their informant were
engaged in gunrunning, transporting Firearms or in organizing special strike
forces. Nor was there any indication from the package or behavior of Arellano
that could have triggered the suspicion of the policemen. Absent such justifying
circumstances specifically pointing to the culpability of Aniag and Arellano, the
search could not be valid.

The action then of the policemen unreasonably intruded into petitioner's
privacy and the security of his property. Consequently, the firearms obtained in
violation of petitioner's right against warrantless search cannot be admitted for
any purpose in any proceeding.

In short, an extensive search of the car without a warrant can only be resorted
to if the officers conducting the search had PROBABLE CAUSE BEFORE THE
SEARCH that the drvier violated the law, or that they would find evidence
pertaining to the commission of a crime in the vehicle.

ISSUE: Did the seeming acquiescence of Arellano to the search constitute an
implied waiver of petitioner's right to question the reasonableness of the search
of the vehicle and the seizure of the firearms?
SC: NO. NO WAIVER OF RIGHT.
Only his driver was at the at that time it was stopped for inspection. Driver
Arellano did not know the purpose of the checkpoint. In the face of fourteen
(14) armed policemen conducting the operation, driver Arellano being alone
and a mere employee of petitioner could not have marshalled the strength and
the courage to protest against the extensive search conducted in the vehicle. In
such scenario, the "implied acquiescence," if there was any, could not be more
than a mere passive conformity on Arellano's part to the search, and "consent"
given under intimidating or coercive circumstances is no consent within the
purview of the constitutional guaranty.

Guidelines should have been made to ensure that no infringement of civil and
political rights results from the implementation of this authority. PNP installed
the checkpoint at about five o'clock in the afternoon and the search was made
soon thereafter, or thirty minutes later. There was no news of impending
checkpoints, and the reason for the same have been announced in the media to
forewarn the citizens. Nor did the informal checkpoint that afternoon carry
signs informing the public of the purpose of its operation. As a result, motorists
passing that place did not have any inkling whatsoever about the reason behind
the exercise. With the authorities in control to stop and search passing vehicles,
the motorists did not have any choice but to submit to the PNP's scrutiny.

CHAMP Page 49 9/16/2014

37. PEOPLE VS. USANA / ESCANO
Updated by K Pascual. 2014. 4C. Bill of Rights Poli Digests. Atty. Jack Jimenez. 4C. 50


G.R. Nos. 129756-58 January 28, 2000
warrantless searches and seizures - checkpoints

During a COMELEC gun ban, the Makati Police were manning a checkpoint
at the corner of Senator Gil Puyat Ave. and the South Luzon Expressway
(SLEX).They were checking the cars going to Pasay City, stopping those they
found suspicious, and imposing merely a running stop on the others.
At about past midnight, they stopped a Kia Pride car with Plate No. TBH
493. The policemen saw a long firearm on the lap of the person seated at
the passenger seat, who was later identified as Virgilio Usana. They asked
the driver, identified as Escao, to open the door. The police seized the long
firearm, an M-1 US Carbine, from Usana. They were ordered to park their
car and the other passengers were search for more weapons. Their search
yielded a .45 caliber firearm which they seized from Escao.
The three passengers were thereafter brought to the police station. (At this
point, it was the policeman who drove the car to the police station.)
At the police HQ, the investigating policeman became suspicious of the
vehicle. So, he requested Escao to open the trunk. Escao readily agreed
and opened the trunk. The police noticed a blue bag inside it, which they
asked Escao to open. The bag contained a parcel wrapped in tape, and
found HASHHISH.
They were charged with and convicted for violation of the Dangerous
Drugs Act, as well as Illegal Possession of Firearms.
Accused-appellants assail the manner by which the checkpoint in question
was conducted. They contend that the checkpoint manned by elements of
the Makati Police should have been announced. They also complain of its
having been conducted in an arbitrary and discriminatory manner.

SC: VALID SEARCH AND SEIZURE.

The checkpoint herein conducted was in pursuance of the gun ban enforced by
the COMELEC. The COMELEC would be hard put to implement the ban if its
deputized agents were limited to a visual search of pedestrians. It would also
defeat the purpose for which such ban was instituted. Those who intend to
bring a gun during said period would know that they only need a car to be able
to easily perpetrate their malicious designs.

PO3 Suba admitted that they were merely stopping cars they deemed
suspicious, such as those whose windows are heavily tinted just to see if the
passengers thereof were carrying guns. At best they would merely direct their
flashlights inside the cars they would stop, without opening the car's doors or
subjecting its passengers to a body search. There is nothing discriminatory in
this as this is what the situation demands.

We see no need for checkpoints to be announced, as the accused have invoked.
Not only would it be impractical, it would also forewarn those who intend to
violate the ban. Even so, badges of legitimacy of checkpoints may still be
inferred from their fixed location and the regularized manner in which they are
operated.

Not all checkpoints are illegal. Those which are warranted by the exigencies of
public order and are conducted in a way least intrusive to motorists are
allowed. For, admittedly, routine checkpoints do intrude, to a certain extent, on
motorists' right to "free passage without interruption," but it cannot be denied
that, as a rule, it involves only a brief detention of travelers during which the
vehicle's occupants are required to answer a brief question or two. For as long
as the vehicle is neither searched nor its occupants subjected to a body search,
and the inspection of the vehicle is limited to a visual search, said routine checks
cannot be regarded as violative of an individual's right against unreasonable
search. In fact, these routine checks, when conducted in a fixed area, are even
less intrusive.

Updated by K Pascual. 2014. 4C. Bill of Rights Poli Digests. Atty. Jack Jimenez. 4C. 51


(note: there must be an exigency which will justify the establishment of a
checkpoint. Such checkpoint should be limited only to a visual search. If it is going
to be beyond mere visual search, then the police must first establish probable
cause).

ISSUE: Was there waiver of the right?
SC: YES. Even though there was ample opportunity to obtain a search warrant,
we cannot invalidate the search of the vehicle, for there are indications that the
search done on the car of Escao was consented to by him. Both Lopez and
Usana testified that Escao was with the police officers when they searched the
car. There was no apparent objection made by Escao as he seemed to have
freely accompanied the police officers to the car. PO3 Suba, on the other hand,
testified that "Escao readily agreed to open the trunk," upon request of SPO4
de los Santos.

ISSUE: Can they be convicted under the Dangerous Drugs Act?
SC: NO.
Despite the validity of the search, we cannot affirm the conviction of Usana and
Lopez for violation of R.A. No. 6425. The following facts militate against a
finding of conviction: (1) the car belonged to Escao; (2) the trunk of the car
was not opened soon after it was stopped and after the accused were searched
for firearms; (3) the car was driven by a policeman from the place where it was
stopped until the police station; (4) the car's trunk was opened, with the
permission of Escao, without the presence of Usana and Lopez; and (5) after
arrival at the police station and until the opening of the car's trunk, the car was
in the possession and control of the police authorities. No fact was adduced to
link Usana and Lopez to the hashish found in the trunk of the car. Their having
been with Escao in the latter's car before the "finding" of the hashish sometime
after the lapse of an appreciable time and without their presence left much to be
desired to implicate them to the offense of selling, distributing, or transporting
the prohibited drug. In fact, there was no showing that Usana and Lopez knew
of the presence of hashish in the trunk of the car or that they saw the same
before it was seized.
38. PEOPLE VS. SUSAN CANTON
G.R. No. 148825 December 27, 2002
warrantless searches and seizure incidental to arrest
SUSAN CANTON was at the NAIA, being a departing passenger bound for
Saigon, Vietnam. When she passed through the metal detector booth, a
beeping sound was emitted. The airport official and frisker on duty called
her attention, saying "Excuse me maam, can I search you?"
Upon frisking SUSAN, the frisker felt something bulging at her
abdominal area. She then inserted her hand under the skirt of SUSAN,
pinched the package several times and noticed that the package
contained what felt like rice granules.
When she passed her hand, she felt similar packages in front of SUSANs
genital area and thighs.
She asked SUSAN to bring out the packages, but the latter refused and said:
"Money, money only." The frisker then reported the matter to the
supervisor.
Customs officials brought SUSAN to the to a comfort room for a thorough
physical examination. Upon further frisking in the ladies room, the frisker
touched something in front of SUSANs sex organ. She directed SUSAN to
remove her skirt, girdles and panty. SUSAN obliged.
The frisker and the customs officials discovered three packages individually
wrapped and sealed in gray colored packing tape, which SUSAN voluntarily
handed to them.
o The first was taken from SUSANs abdominal area; the second, from
in front of her genital area; and the third, from her right thigh.
They were turned over to the police on duty. Together with SUSAN, they
brought the gray plastic packs to the customs examination table, opened the
same and found that they contained white crystalline substances which,
when submitted for laboratory examination, yielded positive results for
methamphetamine hydrochloride or shabu.
Updated by K Pascual. 2014. 4C. Bill of Rights Poli Digests. Atty. Jack Jimenez. 4C. 52


SUSAN was convicted under the Dangerous Drugs Act.
SUSAN however assailed the propriety of the search and seizure without
warrant on the ground that the seized items were not in plain view.
Here are her arguments:
o SUSAN asserts that the strip search conducted on her in the ladies
room was constitutionally infirm because it was not "incidental to
an arrest." The arrest could not be said to have been made before
the search because at the time of the strip search, the arresting
officers could not have known what was inside the plastic
containers hidden in her body, which were wrapped and sealed
with gray tape. At that point then, they could not have determined
whether SUSAN was actually committing a crime. The strip search
was therefore nothing but a fishing expedition. Verily, it is
erroneous to say that she was caught flagrante delicto and that the
warrantless search was incidental to a lawful arrest.
o SUSAN maintains that, following the doctrine enunciated in Terry v.
Ohio, such stop and frisk search should have been limited to the
patting of her outer garments in order to determine whether she
was armed or dangerous and therefore a threat to the security of
the aircraft.
o SUSAN alleges that from the moment frisker Mylene felt a package
at her abdominal area, started inquiring about the contents thereof,
detained her, and decided to submit her to a strip search in the
ladies room, she was under custodial investigation without
counsel,
o SUSAN questions the application of People v. Johnson
22
because of
its sweeping statement allowing searches and seizures of departing
passengers in airports in view of the gravity of the safety interests
involved. She stresses that the pertinent case should have been
Katz v. United States,
23
which upholds the Fourth Amendment of
the United States of America that "protects people and not places."
The SOLGEN however contends that:
o the search was valid since was caught flagrante delicto and that the
warrantless search was incidental to a lawful arrest.
o Moreover, SUSAN voluntarily submitted herself to the search and
seizure when she allowed herself to be frisked and brought to the
comfort room for further inspection by airport security personnel.
o It likewise maintains that the methamphetamine hydrochloride
seized from SUSAN during the routine frisk at the airport was
acquired legitimately pursuant to airport security procedures.

SC: VALID WARRANTLESS SEARCH. VALID WARRANTLESS ARREST.
CONVICTION AFFIRMED.
The interdiction against warrantless searches and seizures is not absolute. The
recognized exceptions established by jurisprudence are
(1) search of moving vehicles;
(2) seizure in plain view;
(3) customs searches;
(4) waiver or consented searches;
(5) stop and frisk situations (Terry search); and
(6) search incidental to a lawful arrest.
ISSUE: Was it a search incidental to lawful arrest?
SC: NO. We do not agree with the trial court and the OSG that the search and
seizure conducted in this case were incidental to a lawful arrest. SUSANs arrest
did not precede the search. When the metal detector alarmed while SUSAN was
passing through it, the lady frisker on duty forthwith made a pat down search
on the former. In the process, the latter felt a bulge on SUSANs abdomen. The
strip search that followed was for the purpose of ascertaining what were the
packages concealed on SUSANs body. If ever at the time SUSAN was deprived of
her will and liberty, such restraint did not amount to an arrest. Under Section 1
of Rule 113 of the Revised Rules of Criminal Procedure, as amended, arrest is
Updated by K Pascual. 2014. 4C. Bill of Rights Poli Digests. Atty. Jack Jimenez. 4C. 53


the "taking of a person into custody in order that he may be bound to answer for
the commission of an offense."lawphi1.et
As pointed out by the appellant, prior to the strip search in the
ladies room, the airport security personnel had no knowledge yet of what
were hidden on SUSANs body; hence, they did not know yet whether a
crime was being committed. It was only after the strip search upon the
discovery by the police officers of the white crystalline substances inside the
packages, which they believed to be shabu, that SUSAN was arrested. The
search cannot, therefore, be said to have been done incidental to a lawful
arrest. In a search incidental to a lawful arrest, the law requires that there
be first a lawful arrest before a search can be made; the process cannot be
reversed.

ISSUE: Was it a stop and frisk search?
SC: NO. The Terry search or the "stop and frisk" situation refers to a case
where a police officer approaches a person who is acting suspiciously, for
purposes of investigating possibly criminal behavior in line with the
general interest of effective crime prevention and detection. To assure
himself that the person with whom he is dealing is not armed with a weapon
that could unexpectedly and fatally be used against him, he could validly
conduct a carefully limited search of the outer clothing of such person to
discover weapons which might be used to assault him. The Terry Search is
limited to WEAPONS.

ISSUE: Then what was it?
SC: A NEW EXCEPTION! SEARCH PURSUANT TO ROUTINE AIRPORT
SECURITY PROCEDURE (RA 6235)
SEC. 9. Every ticket issued to a passenger by the airline or air carrier concerned
shall contain among others the following condition printed thereon: "Holder
hereof and his hand-carried luggage(s) are subject to search for , and seizure of,
prohibited materials or substances. Holder refusing to be searched shall not be
allowed to board the aircraft," which shall constitute a part of the contract
between the passenger and the air carrier.
This constitutes another exception to the proscription against warrantless
searches and seizures. As admitted by SUSAN and shown in Annex "D" of her
Brief, the afore-quoted provision is stated in the "Notice to All Passengers"
located at the final security checkpoint at the departure lounge. From the said
provision, it is clear that the search, unlike in the Terry search, is not limited to
weapons. Passengers are also subject to search for prohibited materials or
substances.
In this case, after the metal detector alarmed SUSAN consented to be frisked,
which resulted in the discovery of packages on her body. It was too late in the
day for her to refuse to be further searched because the discovery of the
packages whose contents felt like rice granules, coupled by her
apprehensiveness and her obviously false statement that the packages
contained only money, aroused the suspicion of the frisker that SUSAN
was hiding something illegal. It must be repeated that R.A. No. 6235
authorizes search for prohibited materials or substances. To limit the action of
the airport security personnel to simply refusing her entry into the aircraft and
sending her home (as suggested by appellant), and thereby depriving them of
"the ability and facility to act accordingly, including to further search without
warrant, in light of such circumstances, would be to sanction impotence and
ineffectivity in law enforcement, to the detriment of society." Thus, the strip
search in the ladies room was justified under the circumstances.
This is related to the Johnson case.
(Johnson case) Persons may lose the protection of the search and seizure
clause by exposure of their persons or property to the public in a manner
reflecting a lack of subjective expectation of privacy, which expectation
society is prepared to recognize as reasonable. Such recognition is implicit
in airport security procedures. With increased concern over airplane hijacking
and terrorism has come increased security at the nations airports. Passengers
attempting to board an aircraft routinely pass through metal detectors; their
carry-on baggage as well as checked luggage are routinely subjected to x-ray
scans. Should these procedures suggest the presence of suspicious objects,
physical searches are conducted to determine what the objects are. There is
little question that such searches are reasonable, given their minimal
intrusiveness, the gravity of the safety interests involved, and the reduced
privacy expectations associated with airline travel. Indeed, travelers are often
notified through airport public address systems, signs, and notices in their
airline tickets that they are subject to search and, if any prohibited materials or
Updated by K Pascual. 2014. 4C. Bill of Rights Poli Digests. Atty. Jack Jimenez. 4C. 54


substances are found, such would be subject to seizure. These announcements
place passengers on notice that ordinary constitutional protections against
warrantless searches and seizures do not apply to routine airport procedures.

ISSUE: Was the arrest valid?
SC: YES. The appellant, having been caught flagrante delicto, was lawfully
arrested without a warrant. (When, in his presence, the person to be arrested
has committed, is actually committing, or is attempting to commit an offense;).
The search conducted on SUSAN resulted in the discovery and recovery of three
packages containing white crystalline substances, which upon examination
yielded positive results for methamphetamine hydrochloride or shabu. As
discussed earlier, such warrantless search and seizure were legal. Armed with
the knowledge that SUSAN was committing a crime, the airport security
personnel and police authorities were duty-bound to arrest her. As held in
People v. Johnson, her subsequent arrest without a warrant was justified, since
it was effected upon the discovery and recovery of shabu in her person
flagrante delicto.

Other issues: Was her right to counsel violated? (but this is for Sec 12.)
SC: NO. The constitutional right to counsel afforded an accused under
custodial investigation was not violated. In this case, as testified to by the
lone witness for the defense, SPO2 Jerome Cause, no custodial investigation was
conducted after SUSANs arrest. She affixed her signature to the receipt of the
articles seized from her, but before she did so, she was told that she had the
option to sign or not to sign it. In any event, her signature to the packages was
not relied upon by the prosecution to prove its case. Moreover, no statement
was taken from her during her detention and used in evidence against her.
36

Hence, her claim of violation of her right to counsel has no leg to stand on.

39. CAMARA VS. MUNCIPAL COURT SFO.
Administrative searches and seizures.
Camara was charged with violating the San Francisco Housing Code
Section 507.
He refused (3 times) to let the housing inspectors to allow a
warrantless inspection of his ground-floor headquarters which he
leased.
He allegedly violated the apratments occupancy permit by using the
rear of the leasehold as a personal residence. (this was not allowed by the
code).
He questioned the constitutionality of the Code, which authorizes municipal
officials to enter the private dwelling even without a search warrant, and
without probable cause to believe that there is a violation of the Housing
Code.
He claimed that this was in violation of his 4
th
Amendment rights (that is, to
safeguard the privacy and security of individuals against arbitrary invasion
of government officials)
City officials rely on the Frank case, citing that administrative inspections
are allowed even without a warrant because they are merely designed to
make the lease possible demand on the individuals.
City officials also claim that warrantless administrative searches are in the
interest of the public since the health and safety of the public is dependent
on proper enforcement of housing codes. They claim that this is the only
effective means of enforcing such laws by systematic inspection of
physical structures.

SC: SEARCH INVALID.
Except in certain carefully defined classes of cases, a search of private property
without proper consent is unreasonable, unless it has been authorized by a
valid search warrant. The right of officers to thrust themselves into a home is
also a grave concern, not only to the individual but to a society which chooses to
dwell in reasonable security and freedom from surveillance. When the right of
privacy must reasonably yield to the right of search, it is as a rule, to be decided
by a judicial officer, not by a policeman or government enforcement agent.
Updated by K Pascual. 2014. 4C. Bill of Rights Poli Digests. Atty. Jack Jimenez. 4C. 55


FRANK VS. MARYLAND. A routine inspection of the physical condition of
private property is a less hostile intrusion than a typical policemans search for
the fruits and instrumentalities of a crime. That is why the court allowed the
warrantless inspection in the Frank vs. Maryland case. (this doctrine in that case
was that: municipal fire, health, and housing inspection programs touch at most
upon the periphery of the important interests safeguarded by the 4
th

amendments protection against official intrustion, because the inspections are
merely to determine whether physical conditions exists which do not comply
with the standards prescribed by the local ordinances. Also, the Frank case does
not require the property owner to open his doors for a search of evidence of
criminal action which might be used for his conviction. Thus, 4
th
amendment
rights are not involved there, but only the less intense right to be secure from
intrusion into personal privacy.)
NOW IN THIS CASE, the court does not agree that the interests at stake in
these inspection cases are merely peripheral of the 4
th
amendment rights.
The court now says it is surely anomalous to say that the individual and his
private property are still fully protected by the 4
th
Am., only when the
individual is suspected of criminal behavior. This is wrong.
The reason the court gave is that: like most regulatory laws, fire health and
housing codes are enforced by criminal processes!! Discovery of a
violation by the inspector leads to a criminal complaint. And as seen in
Camaras case, refusal to permit an inspection is in itself a crime,
punishable by fine or jail sentence.

Furthermore, under the present Housing Code, when the inspector demands
entry, the occupant has no way of knowing whether enforcement of the
municipal code involved requires the inspection of his premises, no way of
knowing the lawful limits of the inspectors power to search, and no way of
knowing whether the inspector himself is acting under proper authorization.
These are questions which should be addressed to the JUDICIAL BODIES. Under
this setup, it is only after refusing entry and risking criminal conviction can the
occupant be able to challenge the inspection in court. He may never learn any
more about the reason for the inspection than that the law generally allows to
gain entry for inspectors. The practical effect of this system is to leave the
occupant subject to the discretion of the inspector. This is precisely the
discretion to invade private property.
The argument that the inspections are for the public interest misses the point.
The question is NOT whether the inspections may be made at all, but
whether they may be made without a warrant.
The real question is whether the public interest demands the creation of
another exception to the general rule requiring warrants for searches.
The real question is whether the burden of obtaining a warrant would
likely frustrate the governmental purpose behind the search. Here, there
was NO allegation that the housing codes inspection program could NOT
achieve their purpose if they complied with search warrant requirements.
We hold that administrative searches of this kind are SIGNIFICANT
INTRUSIONS UPON THE 4
TH
AM. RIGHTS. Such searches when conducted
without a warrant, lack the traditional safeguards of the 4
th
Am. The arguments
raised in Frank vs. Maryland are insufficient to justify a weakening of the 4
th

Am. Protection.
But the court does not entirely overturn the Frank case. That case gave
recognition to the unique character of such kinds of inspection programs
because it did not require search warrants. To reject that proposition does not
justify ignoring the question whether some other accommodation between
public need and individual rights is essential.
Nothing is intended to foreclose prompt inspections without a warrant. This is
because warrantless searches are traditionally upheld in emergency situations.
It seems likely that the warrants should be normally sought only after entry is
refused unless there are satisfactory reasons for securing immediate entry. (I
think, what the court is simply saying is that authorizing entry to inspect is
allowed, provided it is not done by force.)
Here, Camara was charged for refusing to allow inspectors to enter his
property without warrant. There was NO EMERGENCY DEMANDING
IMMEDIATE ACCESS. In fact, the officials made 3 trips to the building. Yet,
no warrant was obtained and thus Camara was unable to verify the need for or
the appropriate limits of the inspection. Thus, Camara had the constitutional
right to insist that the inspectors obtain a search warrant. He CANNOT BE
CONVICTED FOR REFUSING TO PERMIT INSPECTION.

Updated by K Pascual. 2014. 4C. Bill of Rights Poli Digests. Atty. Jack Jimenez. 4C. 56


Note: Is there a legitimate expectation of privacy?
The more intrusive these searches are, the greater the need to obtain a SW.
40. PADILLA VS. CA
warrantless arrest
This is the case of Robin Padilla charged with illegal possession of
firearms.
Manarang and Perez were in a Manukan Restaurant in Angeles City. While
inside the restaurant, they notice a Mitsubishi Pajero (PMA-777), running
fast in the highway. Manarang remarked that the vehicle might get into an
accident considering the heavy rain. True enough, immediately after the
Pajero passed the restaurant, they heard a screeeeching sound and then a
hard brake of a vehicle.
Manarang then reported the incident to the PNP. The Pajero however
started to move and leave the accident site. Manarang approached and
found out that the vehicle had hit somebody. He was able to get the plate
number (PMA-777)
Manarang rode his motorcycle and chased the Pajero. The policemen also
joined him. After a long police chase, they were able to stop the Pajero.
When police approached the vehicle, the driver rolled down the window
and put his head out while raising both his hands. There was no one else
inside the vehicle. They recognized the driver as Robin Padilla (idol!!)
wearing a short leather jacket. When he alighted with both his hands raised,
the police noticed a gun in his left waist.
He was disarmed.
He was informed that he was being arrest for hit-and-run incident. The
police pointed out the fact that his plate number was dangling and the hood
was dented.
Padilla however denied the allegations and played with the crown by
holding their hands with one hand and pointing to one of the policemen
with his right hand saying, iyan kinuha ang baril ko!.
Because of this, another magazine of a armalite rifle was discovered in
Padillas right pocket. This was also confiscated.
Then when Padilla was going back to his car, the police saw a baby armalite
at the drivers seat.
He was again informed that he will be arrested, this time for illegal
possession of firearms.
He was brought to the police headquarters.
Robin claims that his arrest was illegal, and consequently the searches and
seizures taken were inadmissible. (he also claims he was a confidential
agent on a secret mission order). He claims that the policemen who actually
arrested him were not at the scene of the hit and run.

SC: WARRANTLESS ARREST VALID. SUBSSEQUENT SEARCH VALID.
ARREST:A peace officer or a private person may, without a warrant, arrest a
person:
(a) When, in his presence, the person to be arrested has committed, is actually
committing, or is attempting to commit an offense.

Paragraph (a) requires that the person be arrested (1) after he has
committed or while he is actually committing or is at least attempting to
commit an offense, (ii) in the presence of the arresting officer or private
person. Both elements concurred here, as it has been established that
petitioner's vehicle figured in a hit and run - an offense committed in the
"presence" of Manarang, a private person, who then sought to arrest petitioner.
It must be stressed at this point that "presence" does not only require that the
arresting person sees the offense, but also when he "hears the disturbance
created thereby AND proceeds at once to the scene. " As testified to by
Updated by K Pascual. 2014. 4C. Bill of Rights Poli Digests. Atty. Jack Jimenez. 4C. 57


Manarang, he heard the screeching of tires followed by a thud, saw the
sideswiped victim (balut vendor), reported the incident to the police and
thereafter gave chase to the erring Pajero vehicle using his motorcycle in order
to apprehend its driver. After having sent a radio report to the PNP for
assistance, Manarang proceeded to the Abacan bridge where he found
responding policemen SP02 Boda and SP02 Miranda already positioned near
the bridge who effected the actual arrest of petitioner.
That Manarang decided to seek the aid of the policemen (who admittedly were
nowhere in the vicinity of the hit and run) in effecting petitioner's arrest, did
not in any way affect the propriety of the apprehension. It was in fact the most
prudent action Manarang could have taken rather than collaring
petitioner by himself, inasmuch as policemen are unquestionably better
trained and well-equipped in effecting an arrest of a suspect (like herein
petitioner) who, in all probability, could have put up a degree of resistance
which an untrained civilian may not be able to contain without endangering his
own life. Moreover. it is a reality that curbing lawlessness gains more
success when law enforcers function in collaboration with private citizens.
It is precisely through this cooperation that the offense herein involved
fortunately did not become an additional entry to the long list of unreported
and unsolved crimes.
The exigent circumstances of - hot pursuit, a fleeing suspect, a moving vehicle,
the public place and the raining nighttime - all created a situation in which
speed is essential and delay improvident.
The Court acknowledges police authority, to make the forcible stop since they
had more than mere "reasonable and articulable" suspicion that the occupant of
the vehicle has been engaged in criminal activity.36 Moreover, when caught in
flagrante delicto With possession of an unlicensed firearm (Smith & Wesson)
and ammunition (M-1 6 magazine), petitioner's warrantless arrest was proper
as he was again actually committing another offense (illegal possession of
firearm and ammunitions) and this time in the presence of a peace officer.
Also, the policemen's warrantless arrest of petitioner could likewise be
justified under paragraph (b) as he had in fact just committed an offense.
There was no supervening event or a considerable lapse of time between
the hit and run and the actual apprehension. The policemen saw for
themselves the fast approaching Pajero of petitioner,38 its dangling plate
number, and the dented hood and railings thereof . These formed part of the
arresting police officer's personal knowledge of the facts indicating that
petitioner's Pajero was indeed the vehicle involved in the hit and run incident.
Verily then, the arresting police officers acted upon verified personal knowledge
and not on unreliable hearsay information .
SEARCHES AND SEIZURE: the authorities stumbled upon petitioners
firearms and ammunitions without even undertaking any active search
which, as it is commonly understood, is a prying into hidden places for that
which is concealed. The seizure of the Smith & Wesson revolver and an M-16
rifle magazine was justified for they came within "plain view" of the
policemen who inadvertently discovered the revolver and magazine tucked in
petitioner's waist and back pocket respectively, when he raised his hands after
alighting from his Pajero. The same justification applies to the confiscation of
the M-16 armalite rifle which was immediately apparent to the poicemen as
they took a casual glance at the Pajero and saw said rifle lying horizontally near
the driver's seat. Thus it has been held that: performing their duties as police
officers for the apprehension of the guilty person and the taking of the corpus
delicit."
"Objects whose possesion are prohibited by law inadvertently found in plain
view are subject to seizure even without a warrant."

With respect to the Berreta pistol and a black bag containing assorted
magazines, petitioner voluntarily surrendered them to the police. This latter
gesture of petitioner indicated a waiver of his right against the alleged search
and seizure, and that his failure to quash the information estopped him from
assailing any purported defect.

Even assuming that the firearms and ammunitions were products of an active
search done by the authorities on the person and vehicle of petitioner, their
seizure without a search warrant nonetheless can still be Justified under a
search incidental to a lawful arrest (first instance). Once the lawful arrest was
effected, the police may undertake a protective search58 of the passenger
compartment and containers in the vehicle" which are within petitioner's
grabbing distance regardless of the nature of the offense. This satisfied the two-
tiered test of an incidental search:
Updated by K Pascual. 2014. 4C. Bill of Rights Poli Digests. Atty. Jack Jimenez. 4C. 58


(1) the item to be searched (vehicle) was within the arrestee's custody or area
of immediate control" and
(2) the search was contemporaneous with the arrest.

The products of that search are admissible evidence not excluded by the
exclusionary rule.

Another justification is a search of a moving vehicle (third instance). In
connection therewith, a warrantless search is constitutionally permissible
when, as in this case, the officers conducting the search have reasonable or
probable cause to believe, before the search, that either the motorist is a law-
offender (like herein petitioner with respect to the hit and run) or the contents
or cargo of the vehicle are or have been instruments or the subject matter or the
proceeds of some criminal offense.
41. GO VS. CA
warrantless arrests.

This is the case of Eldon Maguan, who was shot by Rolito Go. (traffic
altercation).
Eldon Maguan was driving his car along Wilson St., San Juan, heading
towards P. Guevarra St.. Rolito Go entered Wilson St., where it is a one-way
street and started travelling in the opposite or "wrong" direction. The 2 cars
nearly bumped each other. Rolito Go alighted from his car, walked over and
shot Maguan inside his car. Petitioner then boarded his car and left the
scene.
6 days later, Rolito Go presented himself before the San Juan Police Station
to verify news reports that be was being hunted by the police. The police
forthwith detained him. An eyewitness to the shooting, who was at the
police station at that time, positively identified petitioner as the
gunman.
He was later charged with murder.
He assails the warrantless arrest claiming that it was unlawful.
The prosecution argues that:
o The warrantless arrest was valid because the offense for which he
was arrested and charged had been 'freshly committed." His
identity had been established through investigation. At the time he
showed up at the police station, there had been an existing
manhunt for him. During the confrontation at the San Juan Police
Station, one witness positively identified petitioner as the culprit.
o Since Gos identity as the gunman who had shot Eldon Maguan had
been sufficiently established by police work, he was validly
arrested six (6) days later at the San Juan Police Station.
Rolito Go however claims:
o that he was not lawfully arrested without warrant because he went
to the police station six (6) days after the shooting which he had
allegedly perpetrated. Thus, the crime had not been "just
committed" at the time that he was arrested. Moreover, none of
the police officers who arrested him had been an eyewitness
to the shooting of Maguan and accordingly none had the "personal
knowledge" required for the lawfulness of a warrantless arrest.

ISSUE: Was the warrantless arrest valid?
SC: NOT VALID.
First, there was no continuing crime. The offense for which petitioner was
arrested was murder, an offense which was obviously commenced and
completed at one definite location in time and space. The fatal shooting of
Maguan was not "continuing crime."

Updated by K Pascual. 2014. 4C. Bill of Rights Poli Digests. Atty. Jack Jimenez. 4C. 59


Second, Gos "arrest' took place six (6) days after the shooting of Maguan.
The "arresting" officers obviously were not present, within the meaning of
Section 5(a), at the time petitioner had allegedly shot Maguan. Neither
could the "arrest" effected six (6) days after the shooting be reasonably
regarded as effected when [the shooting had] in fact just been committed"
within the meaning of Section 5(b). Moreover, none of the "arresting"
officers had any "personal knowledge" of facts indicating that petitioner was
the gunman who had shot Maguan.
The information upon which the police acted had been derived from statemints
made by alleged eyewitnesses to the shooting-one stated that Petitioner was the
gunman; another was able to take down the alleged gunman's car's plate
number which turned out to be registered in petitioner's wife's name. That
information did not, however, constitute "personal knowledge." It is thus
clear to the Court that there was no lawful warrantless arrest of petitioner
within the meaning of Section 5 of Rule 113.

Go was not arrested at all. When he walked into the San Juan Police Station,
accompanied by two (2) lawyers, he in fact placed himself at the disposal
of the police authorities. He did not state that he was "surrendering'
himself, in all probability to avoid the implication he was admitting that
he had slain Eldon Maguan or that he was otherwise guilty of a crime.
When the police filed a complaint for frustrated homicide with the Prosecutor,
the latter should have immediately scheduled a preliminary investigation to
determine whether there was probable cause for charging petitioner in court
for the killing of Eldon Maguan. Instead, as noted earlier, the Prosecutor
proceeded under the erroneous supposition that Section 7 of Rule 112 was
applicable and required petitioner to waive the provisions of Article 125 of the
Revised Penal Code as a condition for carrying out a preliminary investigation.
This was substantive error, for petitioner was entitled to a preliminary
investigation and that right should have been accorded him without any
conditions. Moreover, Since petitioner had not be been arreste,with or without a
warrant, he was also entitled to be released forthwith subject only to his
appearing at the preliminary investigation.

Note under Rule 113, warrantless arrests are allowed only for:
a) in flagrante delicto - When, in his presence, the person to be arrested has
committed, is actually committing, or is attempting to commit an offense;

b) When an offense has in fact just been committed, and he has personal
knowledge of facts indicating that the person to be arrested has committed it;
and

c) When the person to be arrested is a prisoner who has escaped from a penal
establishment or place where he is serving final judgment or temporarily
confined while his case is pending, or has escaped while being transferred from
one confinement to another.
42. PEOPLE VS. BURGOS
warrantless arrests
Burgos was charged with illegal possession of firearms in furtherance of
subversion.
The police obtained information from former members (Masamlok) of the
NPA that it was Burgos who recruited them to join the NPA.
Thus the police arrested Burgos at his house, while he was plowing his
field.
Then inside the house the police asked the accused about his firearm, as
reported by Cesar Masamlok.
At first accused denied possession of said firearm but later, the wife
pointed to a place below their house where a gun was buried in the
ground.
Subversive documents were also recovered (some notebooks, pamphlets
of the NPA).
He was thus arrested without a warrant.
Updated by K Pascual. 2014. 4C. Bill of Rights Poli Digests. Atty. Jack Jimenez. 4C. 60



ISSUE1: Was the arrest of Ruben Burgos lawful? Were the search of his house
and the subsequent confiscation of a firearm and documents allegedly found
therein conducted in a lawful and valid manner?
SC: ARREST NOT VALID.

Under Section 6(a) of Rule 113, the officer arresting a person who has just
committed, is committing, or is about to commit an offense must have
personal knowledge of that fact.
o The offense must also be committed in his presence or within his
view.
o There is no such personal knowledge in this case.
o Whatever knowledge was possessed by the arresting officers, it
came in its entirety from the information furnished by Cesar
Masamlok. The location of the firearm was given by the appellant's
wife.
At the time of the appellant's arrest, he was not in actual possession of
any firearm or subversive document.
o Neither was he committing any act which could be described as
subversive.
o He was, in fact, plowing his field at the time of the arrest.

DOCTRINE: What is sought to be guarded is a man's prerogative to choose who
is allowed entry to his residence. In that haven of refuge, his individuality can
assert itself not only in the choice of who shall be welcome but likewise in the
kind of objects he wants around him. There the state, however powerful,
does not as such have access except under the circumstances above noted,
for in the traditional formulation, his house, however humble, is his castle.
Thus is outlawed any unwarranted intrusion by government, which is
called upon to refrain from any invasion of his dwelling and to respect the
privacies of his life. the belief that to value the privacy of home and person and
to afford its constitutional protection against the long reach of government is no
less than to value human dignity, and that his privacy must not be disturbed
except in case of overriding social need, and then only under stringent
procedural safeguards.'

ISSUE2: Sol-gen claims that warrantless arrest was proper under Section 6(b)
using the test of reasonableness. He claims that the information given by Cesar
Masamlok was sufficient to induce a reasonable ground that a crime has been
committed and that the accused is probably guilty thereof.
SC: ARREST NOT VALID.
DOCTRINE: In arrests without a warrant under Section 6(b), however, it is not
enough that there is reasonable ground to believe that the person to be arrested
has committed a crime. A crime must in fact or actually have been
committed first. That a crime has actually been committed is an essential
precondition. It is not enough to suspect that a crime may have been
committed. The fact of the commission of the offense must be undisputed.
(there must be no question that a crime has actually been committed.) The
test of reasonable ground applies only to the identity of the perpetrator. (the
only question remaining is the identity)
In this case, the accused was arrested on the sole basis of Masamlok's
verbal report. Masamlok led the authorities to suspect that the accused
had committed a crime.
o They were still fishing for evidence of a crime not yet ascertained.
o The subsequent recovery of the subject firearm on the basis of
information from the lips of a frightened wife cannot make the
arrest lawful.
o If an arrest without warrant is unlawful at the moment it is
made, generally nothing that happened or is discovered
afterwards can make it lawful.
The fruit of a poisoned tree is necessarily also tainted.
More important, we find no compelling reason for the haste with which the
arresting officers sought to arrest the accused. We fail to see why they failed
to first go through the process of obtaining a warrant of arrest, if indeed
they had reasonable ground to believe that the accused had truly
committed a crime. There is no showing that there was a real apprehension
that the accused was on the verge of flight or escape. Likewise, there is no
showing that the whereabouts of the accused were unknown.
The basis for the action taken by the arresting officer was the verbal
report made by Masamlok who was not required to subscribe his
allegations under oath. There was no compulsion for him to state
Updated by K Pascual. 2014. 4C. Bill of Rights Poli Digests. Atty. Jack Jimenez. 4C. 61


truthfully his charges under pain of criminal prosecution. Consequently, the
need to go through the process of securing a search warrant and a warrant
of arrest becomes even more clear. The arrest of the accused while he was
plowing his field is illegal. The arrest being unlawful, the search and seizure
which transpired afterwards could not likewise be deemed legal as being
mere incidents to a valid arrest.

ISSUE: WAS THERE A WAIVER?
SC: NO WAIVER.
Neither can it be presumed that there was a waiver, or that consent was
given by the accused to be searched simply because he failed to object.
DOCTRINE: To constitute a waiver, it must appear first that the right exists;
secondly, that the person involved had knowledge, actual or constructive, of the
existence of such a right; and lastly, that said person had an actual intention to
relinquish the right
Since arrest was illegal, then the search is also also illegal.
43. PEOPLE VS. MARTI
warrantless searches and seizures
Marti and his common-law wife Reyes, went to the booth of the "Manila
Packing and Export Forwarders" carrying with them four (4) gift-wrapped
packages.
o They were sending the packages to a friend in Zurich,
Switzerland.
The attendant at the booth asked if she could examine and inspect the
packages.
o Marti, however refused, assuring her that the packages simply
contained books, cigar&, and gloves and were gifts to his friend in
Zurich. Thus, the packages were no longer inspected. It was
marked and sealed, ready for shipment
However, before delivery of the box to the Bureau of Customs and/ or
Bureau of Posts, the owner of the forwarding company, following
standard operating procedure, opened the boxes for final inspection.
o When he opened it, a peculiar odor emitted therefrom.
o His curiousity amused, he squeezed one of the bundles allegedly
containing gloves and felt dried leaves inside.
o Opening one of the bundles, he pulled out a cellophane wrapper
protruding from the opening of one of the gloves. He made an
opening on one of the cellophane wrappers and took several grams
of the contents thereof.
He reported the shipment to the NBI. He later went to the NBI with the
box.
o In the presence of the NBI agents, he opened the top flaps, removed
the styro-foam and took out the cellophane wrappers from inside
the gloves. Dried marijuana leaves were found to have been
contained inside the cellophane wrappers.
NBI searched for Marti but to no avail. An information for violation of DDA
was filed against Marti.
Marti contends that the evidence had been obtained in violation of his
constitutional rights against unreasonable search and seizure and
privacy of communication and therefore argues that the same should be
held inadmissible in evidence.
The case at bar assumes a peculiar character since the evidence sought to
be excluded was primarily discovered and obtained by a private
person, acting in a private capacity and without the intervention and
participation of State authorities.

ISSUE: Under the circumstances, can accused/appellant validly claim that his
constitutional right against unreasonable searches and seizure has been
violated? Stated otherwise, may an act of a private individual, allegedly in
violation of appellant's constitutional rights, be invoked against the State?
Updated by K Pascual. 2014. 4C. Bill of Rights Poli Digests. Atty. Jack Jimenez. 4C. 62


SC: NO. In the absence of governmental interference, the liberties guaranteed by
the Constitution cannot be invoked against the State. EVIDENCE ADMISSIBLE.
The Fourth Amendment gives protection against unlawful searches and
seizures, and as shown in previous cases, its protection applies to
governmental action. Its origin and history clearly show that it was
intended as a restraint upon the activities of sovereign authority, and was
not intended to be a limitation upon other than governmental agencies; as
against such authority it was the purpose of the Fourth Amendment to
secure the citizen in the right of unmolested occupation of his dwelling and
the possession of his property, subject to the right of seizure by process
duly served."
DOCTRINE: The fourth amendment does not require exclusion of evidence
obtained through a search by a private citizen. Rather, the amendment
only proscribes governmental action.
The contraband in the case at bar having come into possession of the
Government without the latter transgressing appellant's rights against
unreasonable search and seizure, the Court sees no cogent reason why the same
should not be admitted against him in the prosecution of the offense charged.
Records of the case clearly indicate that it was Mr. Job Reyes, the proprietor of
the forwarding agency, who made search/inspection of the packages. Said
inspection was reasonable and a standard operating procedure on the part of
Mr. Reyes as a precautionary measure before delivery of packages to the Bureau
of Customs or the Bureau of Posts.
It will be recalled that after Reyes opened the box containing the illicit cargo, he
took samples of the same to the NBI and later summoned the agents to his place
of business. Thereafter he opened the parcels containing the rest of the
shipment and entrusted the care and custody thereof to the NBI agents. Clearly,
the NBI agents made no search and seizure, much less an illegal one, contrary to
the postulate of accused/appellant.
Second, the mere presence of the NBI agents did not convert the
reasonable search effected by Reyes into a warrantless search and seizure
proscribed by the Constitution. Merely to observe and look at that which is
in plain sight is not a search. Having observed that which is open, where no
tresspass has been committed in aid thereof, is not search. Where the
contraband articles are identified without a trespass on the part of the arresting
officer, there is not the search that is prohibited by the constitution
If the search is made upon the request of law enforcers, a warrant must
generally be first secured if it is to pass the test of constitutionality. However, if
the search is made at the behest or initiative of the proprietor of a private
establishment for its own and private purposes, as in the case at bar, and
without the intervention of police authorities, the right against unreasonable
search and seizure cannot be invoked for only the act of private individual, not
the law enforcers, is involved.
In sum, the protection against unreasonable searches and seizures cannot
be extended to acts committed by private individuals so as to bring it
within the ambit of alleged unlawful intrusion by the government.
43.5 LUZ V. PEOPLE
667 SCRA 421
G.R. No. 197788. February 29, 2012.*

DIGEST
PETITION for review on certiorari of the decision and resolution of the Court of
Appeals.

Facts (Prosecution Version):
PO2 Emmanuel L. Alteza, who was then assigned at the Sub-Station 1 of the
Naga City Police Station as a traffic enforcer, substantially testified that on
March 10, 2003 at around 3:00 oclock in the morning, he saw the accused,
who was coming from the direction of Panganiban Drive and going to
Diversion Road, Naga City, driving a motorcycle without a helmet;
o that this prompted him to flag down the accused for violating a
municipal ordinance which requires all motorcycle drivers to wear
helmet (sic) while driving;
o that he invited the accused to come inside their sub-station since
the place where he flagged down the accused is almost in front of
the said sub-station; that while he and SPO1 Rayford Brillante were
issuing a citation ticket for violation of municipal ordinance, he
noticed that the accused was uneasy and kept on getting something
from his jacket;
Updated by K Pascual. 2014. 4C. Bill of Rights Poli Digests. Atty. Jack Jimenez. 4C. 63


o that he was alerted and so, he told the accused to take out the
contents of the pocket of his jacket as the latter may have a weapon
inside it; that the accused obliged and slowly put out the contents
of the pocket of his jacket which was a nickel-like tin or metal
container about two (2) to three (3) inches in size, including two
(2) cellphones, one (1) pair of scissors and one (1) Swiss knife;
o that upon seeing the said container, he asked the accused to open
it; that after the accused opened the container, he noticed a cartoon
cover and something beneath it;
and that upon his instruction, the accused spilled out the
contents of the container on the table which turned out to
be four (4) plastic sachets, the two (2) of which were
empty while the other two (2) contained suspected shabu.
Arraigned on 2 July 2003, petitioner, assisted by counsel, entered a plea of
Not guilty to the charge of illegal possession of dangerous drugs.
RTC convicted petitioner of illegal possession of dangerous drugs
committed.
o It found the prosecution evidence sufficient to show that he had
been lawfully arrested for a traffic violation and then subjected to a
valid search, which led to the discovery on his person of two plastic
sachets later found to contain shabu. The RTC also found his
defense of frame-up and extortion to be weak, self-serving and
unsubstantiated.
Upon review, the CA affirmed the RTCs Decision. Hence, this Petition for
review on Certiorari to SC.
Petitioner claims that there was no lawful search and seizure, because there
was no lawful arrest. He claims that the finding that there was a lawful
arrest was erroneous, since he was not even issued a citation ticket or
charged with violation of the city ordinance. Even assuming there was a
valid arrest, he claims that he had never consented to the search conducted
upon him.

ISSUE: WON the Search and Seizure of the Shabu is invalid? INVALID!
HELD: WHEREFORE, the Petition is GRANTED. REVERSED and SET ASIDE.
Petitioner Rodel Luz y Ong is hereby ACQUITTED.

RATIO: We find the Petition to be impressed with merit, but not for the
particular reasons alleged.
First, there was no valid arrest of petitioner. When he was flagged down
for committing a traffic violation, he was not, ipso facto and solely for this
reason, arrested.
Arrest is the taking of a person into custody in order that he or she may be
bound to answer for the commission of an offense.
o It is effected by an actual restraint of the person to be arrested or
by that persons voluntary submission to the custody of the one
making the arrest.
o Neither the application of actual force, manual touching of the
body, or physical restraint, nor a formal declaration of arrest, is
required.
o It is enough that there be an intention on the part of one of the
parties to arrest the other, and that there be an intent on the
part of the other to submit, under the belief and impression
that submission is necessary.
Under R.A. 4136, or the Land Transportation and Traffic Code, the
general procedure for dealing with a traffic violation is not the arrest of
the offender, but the confiscation of the drivers license of the latter.
Similarly, the Philippine National Police (PNP) Operations Manual provides
the following procedure for flagging down vehicles during the conduct of
checkpoints:
o m. If it concerns traffic violations, immediately issue a Traffic
Citation Ticket (TCT) or Traffic Violation Report (TVR). Never
indulge in prolonged, unnecessary conversation or argument with
the driver or any of the vehicles occupants;
At the time that he was waiting for PO3 Alteza to write his citation
ticket, petitioner could not be said to have been under arrest. There
was no intention on the part of PO3 Alteza to arrest him, deprive him
of his liberty, or take him into custody.
Prior to the issuance of the ticket, the period during which petitioner was at
the police station may be characterized merely as waiting time.
In fact, as found by the trial court, PO3 Alteza himself testified that the only
reason they went to the police sub-station was that petitioner had been
flagged down almost in front of that place.
Hence, it was only for the sake of convenience that they were waiting there.
There was no intention to take petitioner into custody.
In Berkemer v. McCarty: US Supreme Court discussed at length whether the
roadside questioning of a motorist detained pursuant to a routine traffic
stop should be considered custodial interrogation. The Court held that, such
Updated by K Pascual. 2014. 4C. Bill of Rights Poli Digests. Atty. Jack Jimenez. 4C. 64


questioning does not fall under custodial interrogation, nor can it be
considered a formal arrest, by virtue of the nature of the questioning, the
expectations of the motorist and the officer, and the length of time the
procedure is conducted.
The U.S. Court in Berkemer thus ruled that, since the motorist therein was
only subjected to modest questions while still at the scene of the traffic
stop, he was not at that moment placed under custody (such that he should
have been apprised of his Miranda rights), and neither can treatment of this
sort be fairly characterized as the functional equivalent of a formal arrest.
Similarly, neither can petitioner here be considered under arrest at the
time that his traffic citation was being made.
It also appears that, according to City Ordinance No. 98-012, which
was violated by petitioner, the failure to wear a crash helmet while
riding a motorcycle is penalized by a fine only. Under the Rules of Court,
a warrant of arrest need not be issued if the information or charge was filed
for an offense penalized by a fine only. It may be stated as a corollary that
neither can a warrantless arrest be made for such an offense.
This ruling does not imply that there can be no arrest for a traffic
violation. Certainly, when there is an intent on the part of the police
officer to deprive the motorist of liberty, or to take the latter into
custody, the former may be deemed to have arrested the motorist. In
this case, however, the officers issuance (or intent to issue) a traffic
citation ticket negates the possibility of an arrest for the same
violation.
Even if one were to work under the assumption that petitioner was deemed
arrested upon being flagged down for a traffic violation and while
awaiting the issuance of his ticket, then the requirements for a valid arrest
were not complied with. (i.e. Miranda rights)
In Berkemer, the U.S. Court also noted that the Miranda warnings must also
be given to a person apprehended due to a traffic violation.
If it were true that petitioner was already deemed arrested when he
was flagged down for a traffic violation and while he was waiting for
his ticket, then there would have been no need for him to be arrested
for a second timeafter the police officers allegedly discovered the
drugsas he was already in their custody.

Second, there being no valid arrest, the warrantless search that
resulted from it was likewise illegal.
The following are the instances when a warrantless search is allowed:
o (i) a warrantless search incidental to a lawful arrest;
o (ii) search of evidence in plain view;
o (iii) search of a moving vehicle;
o (iv) consented warrantless search;
o (v) customs search;
o (vi) a stop and frisk search; and
o (vii) exigent and emergency circumstances.
None of the above-mentioned instances, especially a search incident to a
lawful arrest, are applicable to this case.
It must be noted that the evidence seized, although alleged to be
inadvertently discovered, was not in plain view. It was actually concealed
inside a metal container inside petitioners pocket. Clearly, the evidence
was not immediately apparent.
Neither was there a consented warrantless search. Consent to a search is
not to be lightly inferred, but shown by clear and convincing evidence. It
must be voluntary in order to validate an otherwise illegal search; that is,
the consent must be unequivocal, specific, intelligently given and
uncontaminated by any duress or coercion.
o While the prosecution claims that petitioner acceded to the
instruction of PO3 Alteza, this alleged accession does not suffice to
prove valid and intelligent consent. In fact, the RTC found that
petitioner was merely told to take out the contents of his pocket.
Whether consent to the search was in fact voluntary is a question of fact to
be determined from the totality of all the circumstances. Relevant to this
determination are the following characteristics of the person giving consent
and the environment in which consent is given:
o (1) the age of the defendant;
o (2) whether the defendant was in a public or a secluded location;
o (3) whether the defendant objected to the search or passively
looked on;
o (4) the education and intelligence of the defendant;
o (5) the presence of coercive police procedures;
o (6) the defendants belief that no incriminating evidence would be
found;
o (7) the nature of the police questioning;
o (8) the environment in which the questioning took place; and
o (9) the possibly vulnerable subjective state of the person
consenting.
Updated by K Pascual. 2014. 4C. Bill of Rights Poli Digests. Atty. Jack Jimenez. 4C. 65


Neither does the search qualify under the stop and frisk rule. While the
rule normally applies when a police officer observes suspicious or unusual
conduct, which may lead him to believe that a criminal act may be afoot, the
stop and frisk is merely a limited protective search of outer clothing for
weapons.
In Knowles v. Iowa: US Supreme Court held that when a police officer stops a
person for speeding and correspondingly issues a citation instead of
arresting the latter, this procedure does not authorize the officer to conduct
a full search of the car. Instead, police officers may only conduct minimal
intrusions, such as ordering the motorist to alight from the car or doing a
patdown.
o In Robinson, we stated that a custodial arrest involves danger to
an officer because of the extended exposure which follows the
taking of a suspect into custody and transporting him to the police
station. We recognized that [t]he danger to the police officer
flows from the fact of the arrest, and its attendant proximity, stress,
and uncertainty, and not from the grounds for arrest. A routine
traffic stop, on the other hand, is a relatively brief encounter and is
more analogous to a so-called Terry stop . . . than to a formal
arrest.
o Where there is no formal arrest . . . a person might well be less
hostile to the police and less likely to take conspicuous, immediate
steps to destroy incriminating evidence.
The foregoing considered, petitioner must be acquitted. While he may have
failed to object to the illegality of his arrest at the earliest opportunity, a
waiver of an illegal warrantless arrest does not, however, mean a waiver of
the inadmissibility of evidence seized during the illegal warrantless arrest.
The Constitution guarantees the right of the people to be secure in their
persons, houses, papers and effects against unreasonable searches and
seizures. Any evidence obtained in violation of said right shall be
inadmissible for any purpose in any proceeding.

44. MANALILI VS. CA
stop and frisk
Police were conducting a surveillance near the Kalookan City Cemetery
because of information that drug addicts were roaming the area.
They chanced Upon a male person in front of the cemetery who appeared
high on drugs. The male person was observed to have reddish eyes and to
be walking in a swaying manner. When this male person tried to avoid the
policemen, the latter approached him and introduced themselves as police
officers. The policemen then asked the male person what he was holding in
his hands. The male person tried to resist. The police asked the male person
if he could see what said male person had in his hands. The latter showed
the wallet and allowed the policeman to examine the same. The policeman
took the wallet and examined it. He found Suspected Crushed marijuana
residue inside. He kept the wallet and its marijuana contents.
The male person (Manalili) was brought to the police station.
The trial court convicted petitioner of illegal possession of marijuana
residue largely on the strength of the arresting officers' testimony. The
policemen were "neutral and disinterested" witnesses, testifying only on
what transpired during the performance of their duties. Substantially, they
asserted that the appellant was found to be in possession of a substance
which was later identified as crushed marijuana residue.
Manalili assails the admissibility of the evidence seized, contending that
they were products of an illegal search.

SC: VALID SEARCH. EVIDENCE ADMISSIBLE.
The search was valid, being akin to a stop-and-frisk. Stop-and-frisk was defined
as the right of a police officer to stop a citizen on the street, interrogate him, and
pat him for weapon(s).

According to the case of Terry vs. Ohio, where a police officer observes an
unusual conduct which leads him reasonably to conclude in light of his
experience that criminal activity may be afoot and that the persons with whom
he is dealing may be armed and presently dangerous, where in the course of
investigating this behavior lie identified himself as a policeman and makes
reasonable inquiries, and where nothing in the initial stages of the encounter
serves to dispel his reasonable fear for his own or others' safety, he is entitled
for the protection of himself and others in the area to conduct a carefully limited
Updated by K Pascual. 2014. 4C. Bill of Rights Poli Digests. Atty. Jack Jimenez. 4C. 66


search of the outer clothing of such persons in an attempt to discover weapons
which might be used to assault him. Such a search is a reasonable search under
the Fourth Amendment, and any weapon seized may properly be introduced in
evidence against the person from whom they were taken.

Stop-and-frisk has already been adopted as another exception to the general
rule against a search without a warrant. The interest of effective crime
prevention and detection allows a police officer to approach a person, in
appropriate circumstances and manner, for purposes of investigating possible
criminal behavior even though there is insufficient probable cause to make an
actual arrest.

In the case at hand, the policemen observed during their surveillance that
appellant had red eyes and was wobbling like a drunk along the Caloocan City
Cemetery, which according to police information was a popular hangout of drug
addicts. From his experience as a member of the AntiNarcotics Unit of the
Caloocan City Police, such suspicious behavior was characteristic of drug
addicts who were "high." The policemen therefore had sufficient reason to stop
petitioner to investigate if he was actually high on drugs. During such
investigation, they found marijuana in petitioner's possession.
45. MALACAT VS. CA
stop and frisk
Malacat was charged with illegal possession of firearms.
The police at Quiapo Manila, in response to bomb threats reported seven
days earlier, were on foot patrol near the Mercury Drug store at Plaza
Miranda.
o They chanced upon two groups of Muslim-looking men, with each
group, comprised of three to four men, posted at opposite sides of
the corner
o These men were acting suspiciously with "their eyes ... moving very
fast."
After minutes of surveillance, the police approached one group of
men, who then fled in different directions.
o As the policemen gave chase, they were able to apprehend Malacat.
o Upon searching him, police found a fragmentation grenade
tucked inside petitioner's "front waist line."
Police admitted that Malacat et. al were merely standing on the corner.
o Although they were not creating a commotion, since they were
supposedly acting suspiciously, the police approached them.
RTC: VALID ruled that the warrantless search and seizure of petitioner
was akin to a "stop and frisk," where a "warrant and seizure can be effected
without necessarily being preceded by an arrest" and "whose object is
either to maintain the status quo momentarily while the police officer seeks
to obtain more information." Probable cause was not required as it was not
certain that a crime had been committed, however, the situation called for
an investigation, hence to require probable cause would have been
"premature."
o The RTC emphasized that the police were "confronted with an
emergency, in which the delay necessary to obtain a warrant,
threatens the destruction of evidence. The officers "had to act in
haste," as petitioner and his companions were acting suspiciously,
considering the time, place and "reported cases of bombing."
o Further, petitioner's group suddenly ran away in different
directions as they saw the arresting officers approach, thus "it is
reasonable for an officer to conduct a limited search, the purpose of
which is not necessarily to discover evidence of a crime, but to
allow the officer to pursue his investigation without fear of
violence."
The RTC also held the grenade admissible since it was a search incidental to
lawful arrest.
Updated by K Pascual. 2014. 4C. Bill of Rights Poli Digests. Atty. Jack Jimenez. 4C. 67



SC: ARREST NOT VALID. SEARCH NOT VALID.
Here, there could have been no valid inflagrante delicto or hot pursuit arrest
preceding the search in light of the lack of personal knowledge on the
part of Yu, the arresting officer, or an overt physical act, on the part of
petitioner, indicating that a crime had just been committed, was being
committed or was going to be committed.
Having thus shown the invalidity of the warrantless arrest in this case,
plainly, the search conducted on petitioner could not have been one
incidental to a lawful arrest.
Three (3) reasons why the "stop-and-frisk" was 'invalid:
1) We harbor grave doubts as to Yu's claim that petitioner was a member of
the group which attempted to bomb Plaza Miranda two days earlier. This
claim is neither supported by any police report or record nor corroborated by
any other police officer who allegedly chased that group. Aside from impairing
Yu's credibility as a witness, this likewise diminishes the probability that a
genuine reason existed so as to arrest and search petitioner.
2) There was nothing in petitioner's behavior or conduct which could have
reasonably elicited even mere suspicion other than that his eyes were
"moving very fast" - an observation which leaves us incredulous since Yu and
his teammates were nowhere near petitioner and it was already 6:30 p.m., thus
presumably dusk. Petitioner and his companions were merely standing at the
comer and were not creating any commotion or trouble.
3) There was at all no ground, probable or otherwise, to believe that
petitioner was armed with a deadly weapon. None was visible to Yu, for as
he admitted, the alleged grenade was "discovered" "inside the front waistline"
of petitioner, and from all indications as to the distance between Yu and
petitioner, any telltale bulge, assuming that petitioner was indeed hiding a
grenade, could not have been visible to Yu. When the policemen approached
the accused and his companions, they were not yet aware that a
handgrenade was tucked inside his waistline. They did not see any bulging
object in [sic] his person.
DOCTRINE: In a search incidental to a lawful arrest, the law requires that
there first be a lawful arrest before a search can be made - the process
cannot be reversed. Assuming a valid arrest, the arresting officer may
search the person of the arrestee and the area within which the latter may
reach for a weapon or for evidence to destroy, and seize any money or property
found which was used in the commission of the crime, or the fruit of the crime,
or that which may be used as evidence, or which might furnish the arrestee with
the means of escaping or committing violence.
ACQUITTED.

Notes on Stop and Frisk:
The allowable scope of a "stop and-frisk" is a "limited protective search of
outer clothing for weapons,".

We merely hold today that where a police officer observes unusual conduct
which leads him reasonably to conclude in light of his experience that criminal
activity may be afoot and that the persons with whom he is dealing may be
armed and presently dangerous, where in the course of investigating this
behavior he identifies himself as a policeman and makes reasonable inquiries,
and where nothing in the initial stages of the encounter serves to dispel his
reasonable fear for his own or others' safety, he is entitled for the protection
of himself and others in the area to conduct a carefully limited search of
the outer clothing of such persons in an attempt to discover weapons which
might be used to assault him. Such a search is a reasonable search under the
Fourth Amendment* * *

DOCTRINE: While probable cause is not required to conduct a "stop and
frisk," it nevertheless holds that mere suspicion or a hunch will not validate
a "stop and frisk." A genuine reason must exist, in light of the police
officer's experience and surrounding conditions, to warrant the belief that
the person detained has weapons concealed about him.
Finally, a "stop and-frisk" serves a two-fold interest:
Updated by K Pascual. 2014. 4C. Bill of Rights Poli Digests. Atty. Jack Jimenez. 4C. 68


(1) the general interest of effective crime prevention and detection, which
underlies the recognition that a police officer may, under appropriate
circumstances and in an appropriate manner, approach a person for purposes
of investigating possible criminal behavior even without probable cause; and
(2) the more pressing interest of safety and self-preservation which permit
the police officer to take steps to assure himself that the person with whom he
deals is not armed with a deadly weapon that could unexpectedly and fatally be
used against the police officer.
45.5 ALAJANO V. CABUAY
468 SCRA 188
G.R. No. 160792. August 25, 2005.*

Privacy of Communications and Correspondence; While letters containing
confidential communication between detainees and their lawyers enjoy a
limited protection in that prison officials can open and inspect the mail for
contraband but could not read the contents thereof without violating the
inmates right to correspondence, letters that are not of that nature could
be read by prison officials.

DIGEST

PETITION for review on certiorari of the decision and resolution of the Court of
Appeals.

This petition for review seeks to nullify the Decision of the Court of Appeals.
The Court of Appeals dismissed the petition for habeas corpus filed of Capt.
Gary Alejano (PN-Marines), Capt. Nicanor Faeldon (PN-Marines), Capt.
Gerardo Gambala (PA), Lt. SG James Layug (PN), Capt. Milo Maestrecampo
(PA), and Lt. SG Antonio Trillanes IV (PN) (detainees).
Early morning of 27 July 2003, some 321 armed soldiers, led by the
now detained junior officers, entered and took control of the Oakwood
Premier Luxury Apartments (Oakwood), an upscale apartment complex,
located in the business district of Makati City.
The soldiers disarmed the security officers of Oakwood and planted
explosive devices in its immediate surroundings. The junior officers
publicly renounced their support for the administration and called for the
resignation of President Gloria Macapagal-Arroyo and several cabinet
members.
Around 7:00 p.m. of the same date, the soldiers voluntarily
surrendered to the authorities after several negotiations with
government emissaries. The soldiers later defused the explosive devices
they had earlier planted. The soldiers then returned to their barracks.
On 31 July 2003, Gen. Abaya, as the Chief of Staff of the AFP, issued a
directive to all the Major Service Commanders to turn over custody of ten
junior officers to the ISAFP Detention Center. The transfer took place while
military and civilian authorities were investigating the soldiers
involvement in the Oakwood incident.
On 1 August 2003, government prosecutors filed an Information for
coup detat with the RTC of Makati City, against the soldiers involved in
the 27 July 2003 Oakwood incident.
o The government prosecutors accused the soldiers of coup detat as
defined and penalized under Article 134-A of the Revised Penal
Code of the Philippines, as amended.
o The trial court later issued the Commitment Orders giving custody
of junior officers Trillanes and Gambala to the Commanding
Officers of ISAFP.
On 2 August 2003, Gen. Abaya issued a directive to all Major Service
Commanders to take into custody the military personnel under their
command who took part in the Oakwood incident except the detained
junior officers who were to remain under the custody of ISAFP.
On 11 August 2003, petitioners filed a petition for habeas corpus with
the Supreme Court.
SC issued a Writ of Habeas Corpus directing respondents to make a
return of the writ and to appear and produce the persons of the detainees
before the Court of Appeals on the scheduled date for hearing and further
proceedings.
On 18 August 2003, pursuant to the directives of the Court, respondents
submitted their Return of the Writ and Answer to the petition and produced
the detainees before the Court of Appeals during the scheduled hearing.
On 17 September 2003, the Court of Appeals rendered its decision
dismissing the petition: the detainees are already charged of coup detat
before the RTC of Makati.

ISSUE: WON the detainees right to counsel was violated? NO

Updated by K Pascual. 2014. 4C. Bill of Rights Poli Digests. Atty. Jack Jimenez. 4C. 69


Petitioners admit that they do not question the legality of the detention of
the detainees. Neither do they dispute the lawful indictment of the
detainees for criminal and military offenses. What petitioners bewail is
the regulation adopted by Gen. Cabuay in the ISAFP Detention Center
preventing petitioners as lawyers from seeing the detaineestheir
clientsany time of the day or night. The regulation allegedly curtails the
detainees right to counsel and violates Republic Act No. 7438 (RA 7438).
Petitioners claim that the regulated visits made it difficult for them to prepare
for the important hearings before the Senate and the Feliciano Commission.

Pre-trial detainees do not forfeit their constitutional rights upon
confinement.16 However, the fact that the detainees are confined makes their
rights more limited than those of the public.17

The schedule of visiting hours does not render void the detainees indictment
for criminal and military offenses to warrant the detainees release from
detention. The ISAFP officials did not deny, but merely regulated, the detainees
right to counsel. The purpose of the regulation is not to render ineffective the
right to counsel, but to secure the safety and security of all detainees. American
cases are instructive on the standards to determine whether regulations on pre-
trial confinement are permissible.

In our jurisdiction, the last paragraph of Section 4(b) of RA 7438 provides the
standard to make regulations in detention centers allowable: such reasonable
measures as may be necessary to secure the detainees safety and prevent his
escape. In the present case, the visiting hours accorded to the lawyers of the
detainees are reasonably connected to the legitimate purpose of securing the
safety and preventing the escape of all detainees.

While petitioners may not visit the detainees any time they want, the fact that
the detainees still have face-to-face meetings with their lawyers on a daily basis
clearly shows that there is no impairment of detainees right to counsel.
Petitioners as counsels could visit their clients between 8:00 a.m. and 5:00 p.m.
with a lunch break at 12:00 p.m. The visiting hours are regular business hours,
the same hours when lawyers normally entertain clients in their law offices.
Clearly, the visiting hours pass the standard of reasonableness. Moreover, in
urgent cases, petitioners could always seek permission from the ISAFP officials
to confer with their clients beyond the visiting hours.


ISSUE: (IMPORTANT) WON ISAFP Detention Center violated the detainees
right to privacy of communication when the ISAFP officials opened and
read the personal letters of Trillanes and Maestrecampo. NO.

We now pass upon petitioners argument that the officials of the ISAFP
Detention Center violated the detainees right to privacy when the ISAFP
officials opened and read the letters handed by detainees Trillanes and
Maestrecampo to one of the petitioners for mailing. Petitioners point out that
the letters were not in a sealed envelope but simply folded because there were
no envelopes in the ISAFP Detention Center. Petitioners contend that the
Constitution prohibits the infringement of a citizens privacy rights unless
authorized by law. The Solicitor General does not deny that the ISAFP officials
opened the letters.

Courts in the U.S. have generally permitted prison officials to open and
read all incoming and outgoing mail of convicted prisoners to prevent the
smuggling of contraband into the prison facility and to avert coordinated
escapes.

Inmates were deemed to have no right to correspond confidentially with
anyone. The only restriction placed upon prison authorities was that the
right of inspection should not be used to delay unreasonably the
communications between the inmate and his lawyer.

Eventually, the inmates outgoing mail to licensed attorneys, courts, and court
officials received respect.44 The confidential correspondences could not be
censored.45 The infringement of such privileged communication was held to be
a violation of the inmates First Amendment rights. A prisoner has a right to
consult with his attorney in absolute privacy, which right is not abrogated
by the legitimate interests of prison authorities in the administration of
the institution. Moreover, the risk is small that attorneys will conspire in plots
that threaten prison security.

American jurisprudence initially made a distinction between the privacy rights
enjoyed by convicted inmates and pre-trial detainees.
The case of Palmigiano v. Travisono49 recognized that pre-trial
detainees, unlike convicted prisoners, enjoy a limited right of
privacy in communication.
Censorship of pre-trial detainees mail addressed to public officials,
courts and counsel was held impermissible.
Updated by K Pascual. 2014. 4C. Bill of Rights Poli Digests. Atty. Jack Jimenez. 4C. 70


While incoming mail may be inspected for contraband and read in
certain instances, outgoing mail of pre-trial detainees could not be
inspected or read at all.

In the subsequent case of Wolff v. McDonnell,50 involving convicted prisoners,
the U.S. Supreme Court held that prison officials could open in the presence of
the inmates incoming mail from attorneys to inmates. However, prison officials
could not read such mail from attorneys.

However, while persons imprisoned for crime enjoy many protections of
the Constitution, it is also clear that imprisonment carries with it the
circumscription or loss of many significant rights.

State v. Dunn: noted the considerable jurisprudence in the United States holding
that inmate mail may be censored for the furtherance of a substantial
government interest such as security or discipline. State v. Dunn declared that
if complete censorship is permissible, then the lesser act of opening the
mail and reading it is also permissible.

A pre-trial detainee has no reasonable expectation of privacy for his
incoming mail. However, incoming mail from lawyers of inmates enjoys
limited protection such that prison officials can open and inspect the mail
for contraband but could not read the contents without violating the
inmates right to correspond with his lawyer. The inspection of privileged
mail is limited to physical contraband and not to verbal contraband.

The letters alleged to have been read by the ISAFP authorities were not
confidential letters between the detainees and their lawyers. The
petitioner who received the letters from detainees Trillanes and Maestrecampo
was merely acting as the detainees personal courier and not as their counsel
when he received the letters for mailing. In the present case, since the letters
were not confidential communication between the detainees and their
lawyers, the officials of the ISAFP Detention Center could read the letters.
If the letters are marked confidential communication between the detainees and
their lawyers, the detention officials should not read the letters but only open
the envelopes for inspection in the presence of the detainees.

Moreover, the junior officers are detained with other high-risk persons from the
Abu Sayyaf and the NPA. Thus, we must give the military custodian a wider
range of deference in implementing the regulations in the ISAFP Detention
Center. The military custodian is in a better position to know the security risks
involved in detaining the junior officers, together with the suspected Abu Sayyaf
and NPA members.

HELD: WHEREFORE, we DISMISS the petition. We AFFIRM the Decision of the
Court of Appeals in CA-G.R. SP No. 78545.
46. ADIONG VS. COMELEC
freedom of speech and the electoral process - censorship

Comelec promulgated Resolution 2347 which prohibited the posting of
dcals and stickers on mobile places, and limiting their location or
publication to authorized posting areas.
It is unlawful: (f) To draw, paint, inscribe, post, display or publicly exhibit
any election propaganda in anyplace, whether public or private, mobile or
stationary, except in the COMELEC common posted areas and/or billboards,
at the campaign headquarters of the candidate or political party,
organization or coalition, or at the candidate's own residential house or one
of his residential houses, if he has more than one: Provided, that such
posters or election propaganda shall not exceed two (2) feet by three (3)
feet in size."
Blo Umpar Adiong, a senatorial candidate 1992 elections now assails the
COMELEC's Resolution insofar as it prohibits the posting of decals and
stickers in ,'mobile" places like cars and other moving vehicles. According to
him such prohibition is violative of the Omnibus Election Code and Republic
Act No. 6646. In addition, the petitioner believes that there is already a ban
on radio, television and print political advertisements, he, being a neophyte
in the field of politics stands to suffer grave and irreparable injury with this
prohibition. The posting of decals and stickers on cars and other moving
vehicles would be his last medium campaigning.

ISSUE: Does it violate freedom of speech and expression?
Updated by K Pascual. 2014. 4C. Bill of Rights Poli Digests. Atty. Jack Jimenez. 4C. 71


SC: YES.

The prohibition unduly infringes on the citizen's fundamental right of free
speech enshrined in the Constitution (Sec. 4, Article III). There is no public
interest substantial enough to warrant the kind of restriction involved in this
case.

All of the protections expressed in the Bill of Rights are important but we have
accorded to free speech the status of a preferred freedom. We have adopted the
principle that debate on public issues should be uninhibited, robust, and wide
open and that it may well include vehement, caustic and sometimes
unpleasantly sharp attacks on government and public officials. Too many
restrictions will deny to people the robust, uninhibited, and wide open debate,
the generating of interest essential if our elections will truly be free, clean and
honest.

We have also ruled that the preferred freedom of expression calls all the more
for the utmost respect when what may be curtailed is the dissemination of
information to make more meaningful the equally vital right of suffrage.

When faced with border line situations where freedom to speak by a candidate
or party and freedom to know on the part of the electorate are invoked against
actions intended for maintaining clean and free elections, the police, local
officials and COMELEC should lean in favor of freedom. For in the ultimate
analysis, the freedom of the citizen and the State's power to regulate are not
antagonistic. There can be no free and honest elections if in the efforts to
maintain them, the freedom to speak and the right, to know are undully
curtailed.

The posting of decals and stickers in mobile places like cars and other moving
vehicles does not endanger any substantial government interest. There is no
clear public interest threatened by such activity so as to justify the curtailment
of the cherished citizen's right of free speech and expression. Under the clear
and present danger rule not only must the danger be patently clear and
pressingly present but the evil sought to be avoided must be so substantive as
to justify a clamp over one's mouth or a writing instrument to be stilled.

The freedom of expression curtailed by the questioned prohibition is not so
much that of the candidate or the political party. The regulation strikes at the
freedom of an individual to express his preference and, by displaying it on his
car, to convince others to agree with him. A sticker may be furnished by a
candidate but once the car owner agrees to have it placed on his private vehicle,
the expression becomes a statement by the owner, primarily his own and not of
anybody else.

Also, the restriction as to where the decals and stickers should be posted is so
broad that it encompasses even the citizen's private property, which in this case
is a privately-owned vehicle. In consequence of this prohibition, another
cardinal rule prescribed by the Constitution would be violated, Section 1, Article
III of the Bill of Rights provides that no person shall be deprived of his property
without due process of law.

We have to consider the fact that in the posting of decals and stickers on cars
and other moving vehicles, the candidate needs the consent of the owner of the
vehicle. In such a case, the prohibition would not only deprive the owner who
consents to such posting of the decals and stickers the use of his property but
more important, in the process, it would deprive the citizen of his right to free
speech and information.

Section 11 of Rep. Act 6646 is so encompassing and invasive that it prohibits the
posting or display of election propaganda in any place, whether public or
private, except in the common poster areas sanctioned by COMELEC. This
means that a private person cannot post his own crudely prepared personal
poster on his own front door or on a post in his yard. The provisions allowing
Updated by K Pascual. 2014. 4C. Bill of Rights Poli Digests. Atty. Jack Jimenez. 4C. 72


regulations are so loosely worded that they include the posting of decals or
stickers in the privacy of one's living room or bedroom. This is delegation
running riot.

Finally, the constitutional objective to give a rich candidate and a poor
candidate equal opportunity to inform the electorate as regards their
candidacies, is not impaired by posting decals and stickers on cars and other
private vehicles. Compared to the paramount interest of the State in
guaranteeing freedom of expression, any financial considerations behind the
regulation are of marginal significance.

It is to be reiterated that the posting of decals and stickers on cars, calesas,
tricycles, pedicabs and other moving vehicles needs the consent of the owner of
the vehicle. Hence, the preference of the citizen becomes crucial in this kind of
election propaganda not the financial resources of the candidate. Whether the
candidate is rich and, therefore, can afford to doleout more decals and stickers
or poor and without the means to spread out the number of decals and stickers
is not as important as the right of the owner to freely express his choice and
exercise his right of free speech. The owner can even prepare his own decals or
stickers for posting on his personal property. To strike down this right and
enjoin it is impermissible encroachment of his liberties.

The prohibition becomes CENSORSHIP.
47. NEAR VS. MINNESOTA
freedom of expression prior restraint

The Session Laws of Minnesota provides for the abatement, as a public
nuisance, of a malicious, scandalous and defamatory newspaper, magazine
or periodical.
The law provided that any person who is engaged in the business of
producing publishing or circulating, selling or possessing obscene lewd
magazines, etc, or malicious, scandalous or defamatory newspaper etc., will
be deemed nuisance.
The Attorney General sought to enjoin one publication, known as the
SATURDAY PRESS, which allegedly comes under the Session Laws
The malicious articles mentioned that a Jewish gangster was in control of
gambling, bootlegging and racketeering, and that law enforcers were not
energetically performing their duties. Most of the charges were directed
against the Chief of Police for being grossly negligent of his duties, having
illicit relations with the gangsters, and graft. It also accused the Mayor of
dereliction. In short, the articles made serious accusations against the
public officers named in connection with the prevalence of crimes and the
failure to expose and punish them.
Near, the defendant and owner of the publication, denied that the articles
were malicious, scandalous and defamatory. He also assails the legality of
the statute as violative of freedom of expression.

SC: STATUTE INVALID. CENSORSHIP. Infringes the liberty of the press (14
th

Am.)

The liberty of the Press and of Speech is within the liberty protected by the due
process clause. The statute in question must be tested based on its OPERATION
AND ITS EFFECTS, as follows:

1) The statute is not aimed at the redress of individual or private wrongs. The
remedies for libel remain available and unaffected. The statute is directed at the
DISTRIBUTION of scandalous matter, detrimental to public morals and general
welfare. In order to suppress publication of a newspaper, it is not necessary to
prove the falsity of the charges. In this case, there was no allegation that the
matter published was false. What the statute requires is that the publication be
malicious.
Updated by K Pascual. 2014. 4C. Bill of Rights Poli Digests. Atty. Jack Jimenez. 4C. 73



Here, the statute still permits the defense of TRUTH, but also that the TRUTH
WAS PUBLISHED WITH GOOD MOTIVES AND FOR JUSTIFIABLE ENDS.

2) The Statute directed not simply at the circulation of scandalous or
defamatory statements, but at the CONTINUED PUBLICATION of such articles.

3) The object of the statute is not punishment but suppression of the offending
newspaper. It is the continued publication of the article that constitutes the
business and the declared nuisance. Under the statute, a publisher faces not
simply a possibility of verdict against him in a suit for libel, but goes further to
declare the business as a nuisance, unless the publisher is able to put up a
defense to prove the truth of the charges, and that it was published with good
motives and justifiable ends. Otherwise, if he fails to do these, his business is
abated.

Thus, suppression is accomplished by enjoining the publication and that
restraint is the object and effect of the statute.

4) The statute not only operates to suppress the offending newspaper, but to
put the publisher under effective censorship. This is because when a malicious
or scandalous newspaper which has been declared as a nuisance resumes
business, it will be punished again as a contempt of court, by fine or
imprisonment.

Note also that the law does not give any definition of the words scandalous,
malicious and defamatory.

gathering from these 4 points, the OPERATION AND EFFECT of the statute is
that public authorities may bring the owner or publisher of a newspaper before
a judge upon a charge of publishing scandalous and defamatory material (such
as the charges against the public officers) and unless he is able to bring evidence
to prove that the charges are true and that they are publish with good motives
and for justifiable ends, his newspaper will be suppressed, and further publication
is made punishable as contempt. THIS IS THE ESSENCE OF CENSORSHIP.

The security of freedom of the press (and expression) requires that it should be
exempt not only from prior restraint from the executive, but also from
legislative restraint.

The purpose of the constitutional provision is to prevent all such prior
restraints upon publication. For whatever wrong the appellant has committed
or may commit by his publications, there is still redress with libel laws. As has
been noted, the statute in question does not deal with punishments, as it provides
for no punishments (except in case of contempt) but the statute provides for the
suppression and injunction, and ultimately, restraint, upon publication.

The statute in question cannot be justified by the reason that the publisher is
permitted to show anyway, that the material is true and published with good
motives. If it were so, then the legislature could provide that any time the
publisher can bring his proof of the truth and his good motives. Then the
legislature can also provide for a machinery of determining in the complete
exercise of its discretion what would be justifiable ends and restrain publication
accordingly. THIS IS A STEP TO A COMPLETE SYSTEM OF CENSORSHIP.

CHAMP Page 73 9/16/2014
48. TOLENTINO VS. SEC OF FINANCE
freedom of press
Updated by K Pascual. 2014. 4C. Bill of Rights Poli Digests. Atty. Jack Jimenez. 4C. 74



This is the E-VAT case. The Philippine Press Institute assails the
constitutionality of the EVAT law insofar as it repeals the tax exemption of
publishers of newspapers and magazines. Under the EVAT, they are also
required to register.
Now it is contended by the PPI that by removing the exemption of the press
from the VAT while maintaining those granted to others, the law
discriminates against the press. At any rate, it is averred, "even
nondiscriminatory taxation of constitutionally guaranteed freedom is
unconstitutional."


SC: VALID LAW.
The press is not exempt from the taxing power of the State and that what the
constitutional guarantee of free press prohibits are laws which single out the
press or target a group belonging to the press for special treatment or which in
any way discriminate against the press on the basis of the content of the
publication, and R.A. No. 7716 is none of these.

With respect to the first contention, it would suffice to say that since the law
granted the press a privilege, the law could take back the privilege anytime
without offense to the Constitution. The reason is simple: by granting
exemptions, the State does not forever waive the exercise of its, sovereign
prerogative.
Indeed, in withdrawing the exemption, the law merely subjects the press to the
same tax burden to which other businesses have long ago been subject.

The VAT is, however, different. It is not a license tax. It is not a tax on the
exercise of a privilege, much less a constitutional right. It is imposed on the sale,
barter, lease or exchange of goods or properties or the sale or exchange of
services and the lease of properties purely for revenue purposes. To subject the
press to its payment is not to burden the exercise of its night any more than to
make the press pay income tax or subject it to general regulation is not to
violate its freedom under the Constitution. (in short the VAT is not regulatory
but for revenue purposes!)

Other issue: Freedom of Religion
The Philippine Bible Society, Inc. claims that although it sells bibles, the
proceeds derived from the sales are used to subsidize the cost of pointing copies
which are given free to those who cannot afford to pay so that to tax the sales
would be to increase the price, while reducing the volume of sale. Granting that
to be the case, the resulting burden on the exercise of religious freedom is so
incidental as to make it difficult to differentiate it from any other economic
imposition that might make the right to disseminate religious doctrines costly.
Otherwise, to follow the petitioner's argument, to increase the tax on the sale of
vestments would be to lay an impermissible burden on the right of the preacher
to make a sermon.

The registration fee of P1,000.00 imposed by 107 of the NIRC, as amended by
7 of R.A. No. 7716, although fixed in amount is really just to pay for the expenses
of registration and enforcement of provisions such as those relating to
accounting in 108 of the NIRC. That the PBS distributes free bibles and
therefore is not liable to pay the VAT does not excuse it from the payment of
this fee because it also sells same copies. At any rate whether the PBS is liable
for the VAT must be decided in concrete cases, in the event it is assessed this tax
by the Commissioner of Internal Revenue.
49. SWS VS. COMELEC
freedom of speech

Updated by K Pascual. 2014. 4C. Bill of Rights Poli Digests. Atty. Jack Jimenez. 4C. 75


Sec 5.4 of the Fair Election Act prohibited the publication of surveys
affecting national elections 15 days prior to the elections, and 7 days for
surveys affecting local elections.
The SWS argued that said restriction is an abridgment of freedom of speech.
They argue that the restriction on the publication of election survey results
constitutes a prior restraint on the exercise of freedom of speech without
any clear and present danger to justify such restraint. They claim that SWS
and other pollsters conducted and published the results of surveys prior to
the 1992, 1995, and 1998 elections up to as close as two days before the
election day without causing confusion among the voters and that there is
neither empirical nor historical evidence to support the conclusion that
there is an immediate and inevitable danger to tile voting process posed by
election surveys. They point out that no similar restriction is imposed on
politicians from explaining their opinion or on newspapers or broadcast
media from writing and publishing articles concerning political issues up to
the day of the election. Consequently, they contend that there is no reason
for ordinary voters to be denied access to the results of election surveys,
which are relatively objective.
On the other hand, COMELEC contends: that the provision is necessary to
prevent the manipulation and corruption of the electoral process by
unscrupulous and erroneous surveys just before the election. It contends
that (1) the prohibition on the publication of election survey results during
the period proscribed by law bears a rational connection to the objective of
the law, i.e., the prevention of the debasement of the electoral process
resulting from manipulated surveys, bandwagon effect, and absence of
reply; and (2) the impairment of freedom of expression is minimal, the
restriction being limited both in duration, i.e., the last 15 days before the
national election and the last 7 days before a local election, and in scope as
it does not prohibit election survey results but only require timeliness.

SC: PROVISION INVALID.

There is no showing that the following considerations outweigh freedom of
expression:
to prevent last-minute pressure on voters,
the creation of bandwagon effect to favor candidates,
misinformation,
the junking" of weak and "losing" candidates by their parties, and
the form of election cheating called "dagdag-bawas" and
invoking the State's power to supervise media of information during the
election period
What test should then be employed to determine the constitutional validity of
5.4? The OBrien Test. (United States v. O 'Brien)
Content-Based Government regulation is sufficiently justified:
[1] if it is within the constitutional power of the Government;
[2] if it furthers an important or substantial compelling governmental interest;
[3] if the governmental interest is unrelated to the suppression of free
expression; and
[4] if the incidental restriction on alleged First Amendment freedoms [of speech,
expression and press] is no greater than is essential to the furtherance of that
interest.
8


Here, the 3
RD
CRITERION WAS NOT MET.
Sec. 5.4 fails to meet criterion [3] of the O 'Brien test because the causal
connection of expression to the asserted governmental interest makes such
interest "not related to the suppression of free expression." By prohibiting the
publication of election survey results because of the possibility that such
publication might undermine the integrity of the election, 5.4 actually
suppresses a whole class of expression, while allowing the expression of
opinion concerning the same subject matter by newspaper columnists, radio
and TV commentators, armchair theorists, and other opinion takers. In effect,
5.4 shows a bias for a particular subject matter, if not viewpoint, by referring
Updated by K Pascual. 2014. 4C. Bill of Rights Poli Digests. Atty. Jack Jimenez. 4C. 76


personal opinion to statistical results. The constitutional guarantee of freedom
of expression means that "the government has no power to restrict expression
because of its message, its ideas, its subject matter, or its content.
Also, the 4
th
CRITERION WAS NOT MET.
It fails to meet criterion [4] of the O 'Brien test, namely, that the restriction be
not greater than is necessary to further the governmental interest. As already
stated, 5.4 aims at the prevention of last-minute pressure on voters, the
creation of bandwagon effect, "junking" of weak or "losing" candidates, and
resort to the form of election cheating called "dagdag-bawas." Praiseworthy as
these aims of the regulation might be, they cannot be attained at the sacrifice of
the fundamental right of expression, when such aim can be more narrowly
pursued by punishing unlawful acts, rather than speech because of
apprehension that such speech creates the danger of such evils.
Under the Administrative Code of 1987, the COMELEC is given the power: To
stop any illegal activity, or confiscate, tear down, and stop any unlawful,
libelous, misleading or false election propaganda, after due notice and hearing.
This is surely a less restrictive means than the prohibition contained in 5.4.
Pursuant to this power of the COMELEC, it can confiscate bogus survey results
calculated to mislead voters. Candidates can have their own surveys conducted.
No right of reply can be invoked by others. No principle of equality is involved.
It is a free market to which each candidate brings his ideas. As for the purpose
of the law to prevent bandwagon effects, it is doubtful whether the Government
can deal with this natural-enough tendency of some voters. Some voters want to
be identified with the "winners." Some are susceptible to the herd mentality.
Also, , contrary to the claim of the Solicitor General, the prohibition imposed by
5.4 cannot be justified on the ground that it is only for a limited period and is
only incidental. The prohibition may be for a limited time, but the curtailment of
the right of expression is direct, absolute, and substantial. It constitutes a total
suppression of a category of speech and is not made less so because it is only for
a period of fifteen (15) days immediately before a national election and seven
(7) days immediately before a local election. ..
We hold that 5.4 is invalid because (1) it imposes a prior restraint on the
freedom of expression, (2) it is a direct and total suppression of a category of
expression even though such suppression is only for a limited period, and (3)
the governmental interest sought to be promoted can be achieved by means
other than suppression of freedom of expression.

50. PRIMICIAS VS. FUGOSO
assembly and petition

Cipriano Primicias, a campaign manager of the Coalesced Minority Parties,
filed a Mandamus against Valeriano Fugoso, as Mayor Manila, to compel the
latter to issue a permit for the holding of a public meeting at Plaza Miranda
for the purpose of petitioning the government for redress to grievances.
Earlier, the Municipal Board of Manila, several ordinances which prohibit as
an offense against public peace, and penalizes as a misdemeanor, "any act,
in any public place, meeting, or procession, tending to disturb the peace or
excite a riot; or collect with other persons in a body or crowd for any
unlawful purpose; or disturb or disquiet any congregation engaged in any
lawful assembly.
It also provided that, The streets and public places of the city shall be kept
free and clear for the use of the public, and the sidewalks and crossings for
the pedestrians, and the same shall only be used or occupied for other
purposes as provided by ordinance or regulation, and that the holding of
any parade or procession in any streets or public places is prohibited unless
a permit therefor is first secured from the Mayor.
The Mayor refused to issue the permit on the ground that that there is a
reasonable ground to believe, basing upon previous utterances and upon
the fact that passions, specially on the part of the losing groups, remains
bitter and high, that similar speeches will be delivered tending to
undermine the faith and confidence of the people in their government, and
in the duly constituted authorities, which might threaten breaches of the
peace and a disruption of public order."

ISSUE: Said provision is susceptible of two constructions:
Updated by K Pascual. 2014. 4C. Bill of Rights Poli Digests. Atty. Jack Jimenez. 4C. 77


1) one is that the Mayor of the City of Manila is vested with unregulated
discretion to grant or refuse to grant permit for the holding of a lawful assembly
or meeting, parade, or procession in the streets and other public places of the
City of Manila; and
2) the other is that the applicant has the right to a permit which shall be granted
by the Mayor, subject only to the latter's reasonable discretion to determine, or
specify the streets or public places to be used for the purpose, with a view to
prevent confusion by overlapping, to secure convenient use of the streets and
public places by others, and to provide adequate and proper policing to
minimize the risk of disorder.

SC: 2
ND
VIEW IS ADOPTED.
It does not confer upon the Mayor the power to refuse to grant the permit, but
only the discretion, in issuing the permit, to determine or specify the streets or
public places where the parade or procession may pass or the meeting may be
held.

A statute requiring persons using the public streets for a parade or procession
to procure a special license therefor from the local authorities is not an
unconstitutional abridgment of the rights of assembly or of freedom of speech
and press, where, the licensing authorities are strictly limited, in the issuance of
licenses, to a consideration of the time, place, and manner of the parade or
procession, with a view to conserving the public convenience and of affording
an opportunity to provide proper policing, and are not invested with arbitrary
discretion to issue or refuse license.

As the Municipal Board is empowered only to regulate the use of streets, parks,
and other public places, and the word "regulate," as used in section 2444 of the
Revised Administrative Code, means and includes the power to control, to
govern, and to restrain, but can not be construed as synonymous with
"suppress" or "prohibit".

The privilege of a citizen of the to use the streets may be regulated in the
interest of all; it is not absolute, but relative, and must be exercised in
subordination to the general comfort and convenience, and in consonance with
peace and good order; but it must not, in the guise of regulation, be abridged or
denied.'

The authority of a municipality to impose regulations in order to assure the
safety and convenience of the people in the use of public highways has never
been regarded as inconsistent with civil liberties but rather as one of the means
of safeguarding the good order upon which they ultimately depend.

Section 2434 of the Administrative Code, a part of the Charter of the City of
Manila, which provides that the Mayor shall have the power to grant and refuse
municipal licenses or permits of all classes, cannot be cited as an authority for
the Mayor to deny the application of the petitioner, for the simple reason that
said general power is predicated upon the ordinances enacted by the Municipal
Board requiring licenses or permits to be issued by the Mayor. It is not a specific
or substantive power independent from the corresponding municipal
ordinances which the Mayor, as Chief Executive of the City, is required to
enforce under the same section 2434.

Finally, the request for the permit was "to hold a peaceful public meeting," and
there is no denial of that fact or any doubt that it was to be a lawful assemblage,
the reason given for the refusal of the permit can not be given any
consideration. The Ordinance enables the Mayor to refuse the permit on his
mere opinion that such refusal will prevent riots, disturbances or disorderly
assemblage. It can thus, be made the instrument of arbitrary suppression of free
expression of views on national affairs, for the prohibition of all speaking will
undoubtedly 'prevent' such eventualities.

Fear of serious injury cannot alone justify suppression of free speech and
assembly. Men feared witches and burned women. It is the function of speech to
free men from the bondage of irrational fears. To justify suppression of free
Updated by K Pascual. 2014. 4C. Bill of Rights Poli Digests. Atty. Jack Jimenez. 4C. 78


speech there must be reasonable ground to fear that serious evil will result if
free speech is practiced. There must be reasonable ground to believe that the
danger apprehended is imminent. There must be reasonable ground to believe
that the evil to be prevented is a serious one * * *.

"Those who won our independence by revolution were not cowards. They did
not fear political change. They did not exalt order at the cost of liberty. * * *

Moreover, even imminent danger cannot justify resort to prohibition of these
functions essential effective democracy, unless the evil apprehended is
relatively serious. Prohibition of free speech and assembly is a measure so
stringent that it would be inappropriate as the means for averting a relatively
trivial harm to a society. * * * The fact that speech is likely to result in some
violence or in destruction of property is not enough to justify its suppression.
There must be the probability of serious injury to the state. Among freemen, the
deterrents ordinarily to be applied to prevent crimes are education and
punishment for violations of the law, not abridgment of the rights of free speech
and assembly.
MANDAMUS GRANTED.

51. NAVARRO VS. VILLEGAS
assembly and petition

Navarro, who was part of the movement for democratic Philippines, an
association of students and workers and peasants, wrote a letter to the
Mayor of Manila, Villegas, applying for a permit to hold a rally at the Plaza
Miranda.
The Mayor denied the request, because a month ago, during the opening of
the session of Congress, there was a series of demonstrations and rallies,
which ended in destruction of private and public property, loss of lives and
injuries to others. Schools, offices and stores were forced to close then. The
Mayor thought that a rally at Plaza Miranda poses a clearer and more
imminent danger of public disorder.
The Mayor instead suggested that that the group of Navarro use the Sunken
Garden near Intramuros instead. He also said he can grant the rally at Plaza
Miranda during Saturdays, Sundays and Holidays so as not to cause great
disruption of the normal activities of the community.
Navarro filed a suit contesting the Mayors denial, on the ground that it is
violative of right to peaceably assemble and petition the govt for redress of
grievances.
He claimed that the right to peaceful assembly cannot be fully enjoyed if
they cannot be allowed to use public places. He contends that for the
complete enjoyment of the right, a particular public place may be used for
greater publicity and effectiveness. He claims that Plaza Miranda is the most
convenient place for rallies and it has acquired a certain significance in the
national life as the Congress of the People, Court of Last Resort, Forum of
the Masses. Navarro claims that the denial by the mayor was designed to
minimize the effectiveness of the rally.

SC: The petition is denied because Navarro failed to show a clear specific legal
duty on the part of the Mayor to grant their application for permit
unconditionally.

Justice Fernando dissented and voted to granted the petition. The right of
freedom of assembly is to be accorded the utmost deference and respect. The
reasons given by the mayor do not satisfy the standards in the Primicias case.
The effect is one of prior restraint! This is not allowed.

ANNOTATIONS: read orig kasi puro doctrines:

Updated by K Pascual. 2014. 4C. Bill of Rights Poli Digests. Atty. Jack Jimenez. 4C. 79


Free Speech and Right to Assembly complement each other. They are
inseparable and cognate rights. A republican form of government implies the
right of citizens to meet peaceably for consultation and to express protest.

Right to Assembly is NOT ABOSLUTE. It is subject to regulation under the police
power of the state. It may be regulated in order that it may not be injurious to
the equal enjoyment of others having an equal right not injurious to the rights of
community. It must be exercised in subordination to the general comfort and
convenience and in consonance with peach and good order. However, the
regulation must not be in the guise of abridgment or denial of said right.

The statute authorizing municipalities to impose regulations to insure the safety
and convenience of the people in the use of public places is consistent with civil
liberties and is one of the means of safeguarding peace and good order upon
which they ultimately depend.

The holding of meetings for peaceful political action cannot be proscribed.
Those who assist such meetings cannot be branded as criminals. The rights of
free speech and peaceful assembly are preserved not as to the auspices under
which the meeting is held but as to its purpose; not as to the relation of the
speakers, but whether their utterances transcend the bounds of freedom of speech.

Freedom of expression cannot be denied except on a showing of a clear and
present danger of a substantive evil that Congress has the right to prevent.
There should be no prior restraint on the communication of views nor
subsequent punishment unless there is clear and present danger.

other matters:

Right to peaceful assembly is an attribute of citizenship. It is found wherever
civilization exists. It was not a right granted to the people by the Constitution. It
was there long before the adoption of the constitution (of the US).

Guide to Interpretation the trend is to recognize the broadest scope and the
widest latitude in public parades and demonstrations, whether religious or
political. Only the those which endangers paramount public interest will give
occasion for permissible limitation of the right. (widest room for discussion,
narrowest range for restriction both for freedom of speech and peaceful
assembly)

Limitations: Criticism should be specific and constructive, specifying particular
objectionable actuations of the government. It must be reasoned, tempered, not
contemptuous condemnation of the entire government set-up (in which case
the intention would be seditious).

Criterion for possible limitation: Standards:
1. Dangerous tendency
2. Clear and Present Danger
3. Balancing of Interests
52. IN RE VALMONTE:
assembly and petition

Valmonte applied for a Mayors permit to hold a rally in front of the Justice
Hall of Las Pinas to protest the delay in the cases of his clients pending in
the RTC.
Updated by K Pascual. 2014. 4C. Bill of Rights Poli Digests. Atty. Jack Jimenez. 4C. 80


The Office of the Mayor refused to issue the permit on the ground that a
rally is prohibited under an SC En Banc Resolution.
He assails the validity of the SC Resolution, claiming that the SC acted in
gadalej in promulgating the guidelines. The SC resolution prohibits rallies
within 200 meter radius from every courthouse.
He argues that the resolution violates the Public Assembly Act, and violates
the freedom of speech, expression and peoples right to peaceful assembly
and petition the government for redress of grievances.

SC:
Freedom of speech and expression has its limitations. It has never been
understood as the absolute right to speech whenever, however, and wherever
one pleases, for the manner, time and place of public discussion can be
constitutionally controlled. Justice Cruz says, the better policy is not liberty
untamed, but liberty regulated by law, where every freedom is exercised in
accordance with law and with due regard for the rights of others.

The Court reiterates that judicial independence and the fair and orderly
administration of justice constitute paramount governmental interest that can
justify the regulation of the publics right to free speech and peacefully assembly
in the vicinity of court houses. Freedom of expression needs to be adjusted to
and accommodated with the requirements of equally important public interests.
One of these fundamental public interests is the maintenance of the integrity
and orderly functioning of the administration of justice. There is no antimony
between free expression and the integrity of the justice system.

Judicial independence is undermined by the irascible demonstrations outside
the courthouses. They wittingly or unwittingly spoil the ideal of sober, non-
partisan proceedings before an cold and neutral judge. A state may thus adopt
safeguards necessary to assure that the administration of justice is at all stages,
free from outside control and influence.

It is of course true that most judges will be influence only b what they sea and
hear in court. However, JUDGES ARE HUMAN, and the will be consciously or
unconsciously influenced by demonstrations in or near their courtrooms. A
state must thus protect the judicial process from being misjudged in the minds
of the public.

The administration of justice must not only be fair but must also appear to be
fair and it is the duty of this court to eliminate everything that will diminish or
destroy this judicial desideratum.

WAS IT JUDICIAL LEGISLATION? NO.
The Public Assembly Act does not preclude the court from promulgating rules
regulating the conduct of demonstrations within the vicinity of courts. While the
BP 880 imposes general restrictions as to the time place and manner of
conducting concerted actions, the SC resolution on the other hand merely adds
specific restrictions as they involve judicial independence and orderly
administration of justice. There is no discrepancy between the 2 sets of
regulatory measures.
Courts have full plenary power to control their proceedings to effectuate fair
and impartial administration of justice. Such authority is essential to the courts
existence, dignity and functions. This Court is vested by the Constitution to
adopt measures to ensure orderly administration of justce.
53. GONZALES VS. KALAWA KATIGBAK
obscenity

Justice Fernando ulit.
The motion picture in question, Kapit sa Patalim was classified "For Adults
Only."
Updated by K Pascual. 2014. 4C. Bill of Rights Poli Digests. Atty. Jack Jimenez. 4C. 81


Jose Antonio U. Gonzalez, President of the Malaya Films, assailed the
classification by the Board of Review for Motion Pictures and Television. He
claims the classification was without basis. He claims that such
classification "is without legal and factual basis and is exercised as
impermissible restraint of artistic expression. The film is an integral whole
and all its portions. including those to which the Board now offers belated
objection, are essential for the integrity of the film. Viewed as a whole, there
is no basis even for the vague speculations advanced by the Board as basis
for its classifiestion.
The objectionable parts showed some contents of Kapit are not fit for the
young. Some of the scenes in the picture were taken in a theater-club and a
good portion of the film shots concentrated on some women erotically
dancing naked, or at least nearly naked, on the theater stage. Another scene
on that stage depicted the women kissing and caressing as lesbians. And
toward the end of the picture, there exists scenes of excessive violence
attending the battle between a group of robbers and the police.

SC: THE BOARD ACTED WITH ABUSE OF DISCRETION. BUT IT WAS NOT
GRAVE.
Motion pictures are important both as a medium for the communication of ideas
and the expression of the artistic impulse. Their effects on the perception by our
people of issues and public officials or public figures as well as the prevailing
cultural traits is considerable.

Emphasis should rightly be on freedom from censorship. It is the opinion of this
Court, therefore, that to avoid an unconstitutional taint on its creation, the
power of respondent Board is limited to the classification of films. It can, to
safeguard other constitutional objections, determine what motion pictures are
for general patronage and what may require either parental guidance or be
limited to adults only. That is to abide by the principle that freedom of
expression is the rule and restrictions the exemption. The power to exercise
prior restraint is not to be presumed, rather the presumption is against its
validity.

The test, to repeat, to determine whether freedom of expression may be limited
is the clear and present danger of an evil of a substantive character that the
State has a right to prevent. Such danger must not only be clear but also present.
There should be no doubt that what is feared may be traced to the expression
complained of. The causal connection must be evident. Also, there must be
reasonable apprehension about its imminence. The time element cannot be
ignored. Nor does it suffice if such danger be only probable. There is the
requirement of its being well-nigh inevitable. The basic postulate, therefore, as
noted earlier, is that where the movies, theatrical productions, radio scripts,
television programs, and other such media of expression are concerned-
included as they are in freedom of expression-censorship, especially so if an
entire production is banned, is allowable only under the clearest proof of a clear
and present danger of a substantive evil to public safety, public morals, public
health or any other legitimate public interest.

The law, however, frowns on obscenity-and rightly so. All ideas having even the
slightest redeeming social importance-unorthodox ideas, controversial ideas,
even ideas hateful to the prevailing climate of opinion-have the full protection of
the guaranties, unless excludable bemuse they encroach upon the limited area
of more important interests. But implicit in the history of the First Amendment
is the rejection of obscenity as utterly without redeeming social importance.

The early leading standard of obscenity was the HICKLIN TEST: whether to the
average person, applying contemporary community standards, the dominant
theme of the material taken as a whole appeals to prurient interest.

Sex and obscenity are not synonymous." Further: "Obscene material is material
which deals with sex in a manner appealing to prurient interest. The portrayal
of sex e.g., in art, literature and scientific works, is not itself sufficient reason to
deny material the constitutional protection of freedom of speech and press. Sex,
a great and mysterious motive force in human life, has indisputably been a
subject of absorbing interest to mankind through the ages; it is one of the vital
problems of human interest and public concern."

Updated by K Pascual. 2014. 4C. Bill of Rights Poli Digests. Atty. Jack Jimenez. 4C. 82


EO 876 creating the board was tasked to "apply contemporary Filipino cultural
values as standard,. As far as the question of sex and obscenity are concerned, it
cannot be stressed strongly that the arts and letters "shall be under the
patronage of the State."27 That is a constitutional mandate. It will be less than
true to its function it any government office or agency would invade the sphere
of autonomy that an artist enjoys. There is no orthodoxy in what passes for
beauty or for reality. It is for the artist to determine what for him is a true
representation. It is not to be forgotten that art and belles lettres deal primarily
with imagination, not so much with ideas in a strict sense. What is seen or
perceived by an artist is entitled to respect, unless there is a showing that the
product of his talent rightfully may be considered obscene. As so well put by
Justice Frankfurter in a concurring opinion, "the widest scope of freedom is to
be given to the adventurous and imaginative exercise of the human spirit" a in
this sensitive area of a man's personality.

The question before the Court is whether or not there was a grave abuse of
discretion. That there was an abuse of discretion by respondent Board is
evident in the light of the difficulty and travail undergone by petitioners before
Kapit sa Patalim was classified as "For Adults Only," without any deletion or cut.
Moreover its perception of what constitutes obscenity appears to be unduly
restrictive. This Court concludes then that there was an abuse of discretion.
Nonetheless, there are not enough votes to maintain that such an abuse can be
considered grave. Accordingly, certiorari does not lie.

The ruling is to be limited to the concept of obscenity applicable to motion
pictures. It is the consensus of this Court that where television is concerned: a
less liberal approach calls for observance. This is so because unlike motion
pictures where the patrons have to pay their way, television reaches every
home where there is a set. Children then will likely will be among the avid
viewers of the programs therein shown.

CHAMP Page 82 9/16/2014

55. BORJAL VS. CA
libel

PERPETUALLY HAGRIDDEN as the public is about losing one of the most basic
yet oft hotly contested freedoms of man, the issue of the right of free expression
bestirs and presents itself time and again, in cyclic occurrence, to inveigle, nay,
challenge the courts to re-survey its ever shifting terrain, explore and furrow its
heretofore uncharted moors and valleys and finally redefine the metes and
bounds of its controversial domain.

Borjal and Soliven were charged with libel following a series of articles in
the Philippine Star regarding alleged anomalous activities of a self-
proclaimed hero of the EDSA. Without expressly referring to Wensceslao,
the articles mentioned that the transportation conference was merely a
money-making gimmick of the organizer. The conference allegedly solicits
funds from the public.
Wency was the director of the First National Conference on Land Tranpo
(which was taksed to draft bills embodying long-term transportation
policies of the government).
Wency felt that he was the one being alluded to and thus sued for damages.
The RTC and CA ruled in favor of Wency citing that although Wency was not
named in the articles, he was sufficiently identifiable. The court also ruled
that the articles defamed Wency because Borjal described him as self-
proclaimed hero, one with shady deals, thick face, and person with dubious
ways.

SC: NOT LIBELOUS.

Updated by K Pascual. 2014. 4C. Bill of Rights Poli Digests. Atty. Jack Jimenez. 4C. 83


In order to maintain a libel suit, it is essential that the victim be identifiable
although it is not necessary that he be named. It is also not sufficient, that the
offended party recognized himself as the person attacked or defamed, but it
must be shown that at least a third person could identify him as the object of the
libelous publication.10 Regrettably, these requisites have not been complied
with in the case at bar.

The questioned articles written by Borjal do not identify private respondent
Wenceslao as the organizer of the conference. Surely, there were millions of
"heroes" of the EDSA Revolution and anyone of them could be "self-proclaimed"
or an "organizer of seminars and conferences." Also, the organizers of the
conference were not identified, the article only enumerated Wency as one of the
Executive Director and Spokesman, and not as, a conference organizer.

Even Wency himself entertained doubt that he was the person spoken of in
Borjal's columns. The former even called up columnist Borjal to inquire if he
(Wenceslao) was the one referred to in the subject articles. Identification is
grossly inadequate when even the alleged offended party is himself unsure that
he was the object of the verbal attack. It is well to note that the revelation of the
identity of the person alluded to came not from Borjal but from Wency himself
when he supplied the information through his 4 June 1989 letter to the editor.
Had private respondent not revealed that he was the "organizer" of the FNCLT
referred to in the Borjal articles, the public would have remained in blissful
ignorance of his identity. It is therefore clear that on the element of
identifiability alone the case falls.

Art. 354 of The Revised Penal Code which state-
Art. 354. Requirement for publicity. - Every defamatory imputation is presumed
to be malicious, even if it be true, if no good intention and justifiable motive for
making it is shown, except in the following cases:
1) A private communication, made by any person to another in the performance
of any legal, moral or social duty; and,
2) A fair and true report, made in good faith, without any comments or remarks,
of any judicial, legislative or other official proceedings which are not of
confidential nature, or of any statement, report or speech delivered in said
proceedings, or of any other act performed by public officers in the exercise of
their functions.

A privileged communication may be either absolutely privileged, or qualifiedly
privileged. Absolutely privileged communications are those which are not,
actionable even if the author has acted in bad faith. An example is found in Sec.
11, Art. VI, of the 1987 Constitution which exempts a member of Congress from
liability for any speech or debate in the Congress or in any Committee thereof.
Upon the other hand, qualifiedly privileged communications containing
defamatory amputations are not actionable unless found to have been made
without good intention or justifiable motive. To this genre belong "private
communications" and "fair and true report without any comments or remarks."
Indisputably, petitioner Borjal's questioned writings are not within the
exceptions of Art. 354 of The Revised Penal Code for they are neither private
communications nor fair and true report without any comments or remarks.
However this does not necessarily mean that they are not privileged. To be sure,
the enumeration under Art. 354 is not an exclusive list of qualifiedly
privileged communications since fair commentaries on matters of public
interest are likewise privileged. The concept of privileged communications is
implicit in the freedom of the press.

The doctrine of fair comment means that while in general every, discreditable
imputation publicly made is deemed false, because every man is presumed
innocent until his guilt is judicially proved, and every false imputation is
deemed malicious, nevertheless, when the discreditable imputation is directed
against a public person in his public capacity, it is not necessarily actionable. In
order that such discreditable imputation to a public official may be actionable, it
must, either be a false allegation of fact or a comment based on a false
supposition. If the comment is an expression of opinion, based on established
facts, then it is immaterial that the opinion happens to be mistaken, as long as it
might reasonably be inferred from the facts. To reiterate, fair commentaries
Updated by K Pascual. 2014. 4C. Bill of Rights Poli Digests. Atty. Jack Jimenez. 4C. 84


on matters of public interest are privileged and constitute a valid defense
in an action for libel or slander.

There is no denying that the questioned articles dealt with matters of public
interest.. The nature and functions of his position which included solicitation of
funds, dissemination of information about the FNCLT in order to generate
interest in the conference, and the management and coordination of the various
activities of the conference demanded from him utmost honesty, integrity and
competence. These are matters about which the public has the right to be
informed, taking into account the very public character of the conference itself.

While, generally, malice can be presumed from defamatory words, the
privileged character of a communication destroys the presumption of malice.
The onus of proving actual malice then lies on plaintiff, Wenceslao. He must
bring home to the defendant, Borjal, the existence of malice as the true motive
of his conduct. Malice connotes ill will or spite and speaks not in response to
duty but merely to injure the reputation of the person defamed, and implies an
intention to do ulterior and unjustifiable harm.34 Malice is bad faith or bad
motive.35 It is the essence of the crime of libel.
Wency failed to substantiate that Borjal was animated by a desire to inflict
unjustifiable harm on his reputation, or that the articles were written and
published without good motives or justifiable ends. On the other hand, we find
Borjal to have acted in good faith. Moved by a sense of civic duty and prodded
by his responsibility as a newspaperman, he proceeded to expose and
denounce what he perceived to be a public deception.

JACKS COMMENTS: fair comment is not derogatory as it expresses not a fact
but an opinion. Fair comments do not attack the person, but only his policies.

56. WISCONSIN VS. YODER
freedom of religion

Wisconsin enacted a Compulsory School Attendance Law, which required
the families to send their children to public or private school until age 16.
The Yoders are members of the Old Amish Religion. They refuse to send
their children to school after age 14 (8
th
grade).
The School District Administrator complained, and charged the Amish
families. They were later convicted.
The defense of the Amish families was that the attendance of their children
to high school was contrary to Amish religion and Amish way of life. They
believed that sending their children to high school would expose them to
the danger of the censure of their church, and would endanger their own
salvation.
The main contention was that the Amish religion has the fundamental belief
that salvation requires life in a church community separate and apart from
the world or worldly influence. They are devoted to a life in harmony with
nature and the soil, making their living by farming or closely related
activities.
They contend that high school education has a worldly influence because it
tends to emphasize intellection and scientific accomplishments,
competitiveness, success, social life with other students. Amish people
emphasize informal learning through doing, a life of goodness rather than
life of intellect. Wisdom rather than technical knowledge, community
welfare rather than competition, separation rather than integration with
worldly society.
Their belief is that once a child has learned basic reading, writing and math,
these are better learned through doing rather than in a classroom.
The contention of the state was that the law was a reasonable exercise of
governmental power, where the State has a high responsibility for
education of its citizens.

Updated by K Pascual. 2014. 4C. Bill of Rights Poli Digests. Atty. Jack Jimenez. 4C. 85


ISSUE: A states interest in universal education however is not totally free from
a BALANCING PROCESS., when it infringes on other fundamental rights and
interests such as those protected by the FREE EXERCISE CLAUSE. There is also
the TRADITIONAL INTERESTS OF PARENTS with respect t the religious
upbringing of the children. Thus the test must be:
Whether there is a denial of the free exercise of religious belief, or
Whether there is a state interest of sufficient magnitude to override the
interest under the free exercise clause.

SC: FREE EXERCISE CLAUSE UPHELD. STATE CANNOT COMPEL THE AMISH
FAMILIES TO SEND THEIR CHILDREN TO FORMAL HIGH SCHOOL AT AGE 16.
A way of life may not be interposed as a barrier to reasonable regulation of
education if it is based on purely secular considerations. To have the protection
of the Religion Clause, the claims must be rooted in religious belief. The traditional
way of life of the Amish people is not merely a matter of personal preference,
but one of deep religious conviction shared by an organized group and
intimately related to daily living.

The impact of the compulsory attendance law on Amish religion is not only
severe but inescapable because the law COMPELS THEM UNDER THREAT OF
CRIMINAL SANCTION, to perform act undeniably at odds with their
fundamental religious beliefs.

It carries with it an objective danger to the free exercise of religion. It creates a
very real threat of undermining the Amish community and religious practice.
They must either abandon the belief and be assimilated into society, or be
forced to migrate to some other place.

The state argues tat while religious beliefs are absolutely free from state control,
actions are not protected such that activities of individuals, even religiously
based, can be regulated.
SC: There are still certain acts which are under the protection of free exercise
clause. The protection does not cover only beliefs. A regulation may in its
application, still infringe the free exercise clause, if it unduly burdens the free
exercise of religion.

The state argues that this formal education is to enable the Amish to participate in
a democratic process.
SC: The Amish alternative has already enabled them to function effectively in
their day to day lives. They have survived for more than 200 years in this
country. (They are self-sufficient and able to survive).

The state invokes Parens Patriae role.
SC: The duty to prepare the child for additional obligations includes the
inculcation of moral standards, religious beliefs and elements of good
citizenship. Thus, this case involves a more fundamental interest of the parents,
as contrasted from that of the state, to guide the religious future and education
of their children.
The power of the parent may be limited only if it appears that their decisions
jeopardize the health and safety of the child or home. But in the case of the
Amish people, they have not. The parents decision not to send the child to
school does NOT impair the physical nor mental health of the child, nor result in
the inability to be self-supporting. (in short, the claim of paren patriae is too
broad and sweeping.)

57. AMERICAN BIBLE SOCIETY (ABS) VS. CITY OF MANILA.
Freedom of religion
Updated by K Pascual. 2014. 4C. Bill of Rights Poli Digests. Atty. Jack Jimenez. 4C. 86



The ABS is a non-stock non-profit religious and missionary corporation in
the Phils. It is engaged in selling bibles and gospel portions as well as
translating them into local dialects.
The City treasurer of Manila informed them that they were conduction the
business of general merchandise without the necessary mayors permit
(making them subject to prosecution), in violation of local ordinances. The
treasurer also required them to get a permit and pay the fees pursuant to
another ordinance.
The ABS contends that the ordinances are unconstitutional because they
provide for religious censorship and restraint on free exercise and
enjoyment of religious profession, (that is, the distribution and sale of
bibles).

ISSUE: whether the ordinances are constitutional?
SC: NO. The constitutional guaranty of free exercise and enjoyment of religious
profession and worship carries with it the RIGHT TO DISSEMINATE RELIGIOUS
INFORMATION. Any restraint of such right can only be justified on the ground
that there is a clear and present danger of any substantive evil which the state
has the right to prevent. RIGHT TO PROPAGE RELIGIOUS BELIEF
In the Murdock vs. Pennsylvannia case, the tax imposed there is a flat license
tax, payment of which is a condition of the exercise of the constitutional
privileges. The power to tax the exercise of a privilege is the power to control or
suppress its enjoyment. This power to impose a license tax on the exericise of
freedoms iis as potent as the power of censorship, which the court has struck
down repeatedly. The power to tax the exercise of religious practice can make its
exercise so costly that it would effectively deprive it of the resources necessary for
its maintenances.

ISSUE: The City claims that ABS is already engaged in business because the price
charged is a bit higher than the actual costs, hence no longer exempt from the
permit.
SC: ABS is not engaged in the business of selling merchandise for profit. (Its
purpose was to distribute bibles, not to engage in profit-making business). Thus,
the ordinance cannot be applied to ABS for in doing so it would impair its free
exercise and enjoyment of religious profession and worship, as well as the right
to disseminate religious belief.

HOWEVER, the ordinance requiring that they obtain a mayors permit, IS VALID.
We do not find that it imposes any charge upon the enjoyment of a right granted
by the Constitution, nor tax the exercise of religious practices.

Since ABS is not engaged in a business (not covered by ordinance 2529), then
ordinance 3000 is also not applicable to ABS, since a permit is required only for
those engaged in business.

CHAMP Page 86 9/16/2014

(Here the tax is invalid but the permit is valid, although the requirement of
permit would not be applicable.)
58. TOLENTINO VS. SEC OF FINANCE
freedom of religion

This is the E-VAT case. The Philippine Press Institute assails the
constitutionality of the EVAT law insofar as it repeals the tax exemption of
publishers of newspapers and magazines. Under the EVAT, they are also
required to register.
Now it is contended by the PPI that by removing the exemption of the press
from the VAT while maintaining those granted to others, the law
Updated by K Pascual. 2014. 4C. Bill of Rights Poli Digests. Atty. Jack Jimenez. 4C. 87


discriminates against the press. At any rate, it is averred, "even
nondiscriminatory taxation of constitutionally guaranteed freedom is
unconstitutional."
The Philippine Bible Society, Inc. claims that although it sells bibles, the
proceeds derived from the sales are used to subsidize the cost of pointing
copies which are given free to those who cannot afford to pay so that to tax
the sales would be to increase the price, while reducing the volume of sale.
They claim that the EVAT law violates freedom of religion.

SC: LAW VALID.
Freedom of religion does not prohibit the imposition of a tax on the sale of
religious materials by a religious organization which is GENERALLY
APPLICABLE.
Also, the resulting burden on the exercise of religious freedom is so
INCIDENTAL as to make it difficult to differentiate it from any other
economic imposition that might make the right to disseminate religious
doctrines costly.
In short, the EVAT is a non-discriminatory tax.
The registration fee of P1,000.00 imposed by 107 of the NIRC, as amended
by 7 of R.A. No. 7716, although fixed in amount is really just to pay for the
expenses of registration and enforcement of provisions such as those
relating to accounting in 108 of the NIRC. That the PBS distributes free
bibles and therefore is not liable to pay the VAT does not excuse it from the
payment of this fee because it also sells same copies. At any rate whether
the PBS is liable for the VAT must be decided in concrete cases, in the event
it is assessed this tax by the Commissioner of Internal Revenue.

Note: compare this with the American Bible Society case with respect to the
registration free. In the ABS case, it was a form of regulation, in this Tolentino
case, it is a form or revenue.
59. EBRALINAG VS. SUPERINTENDENT
freedom of religion

The petitioners are members of Jehovahs Witnesses, who were expelled
from their classes by the public school authorities for refusing to salute the
flag, sing the national anthem, and recite the pledge, as required by RA
1265.
The state argues that under a previous case of Gerona vs. Secretary of
Education, the expulsion was upheld because the flag is not an image but a
symbol of the Republic, an emblem of national sovereignty and under a
system of complete separation of Church and State, the flag is utterly devoid
of any religious significance. Saluting the flag does not involve any religious
ceremony, and is simply similar to taking an oath. The state does not
impose a religion or religious belief or a religious test on students. The state
merely carries out its duty to supervise education and develop civic
conscience on the students.

SC: It is time to re-examine the 30year old Gerona ruling.
Religious freedom is a fundamental right which is entitled to the highest priorty
and the amplest protection for it involves the relationship of man to his Creator.
The right to religious profession and worship has a 2-fold aspect: THE
FREEDOM TO BELIEVE AND THE FREEDOM TO ACT ON ONES BELIEF. The first
is absolute, as long as the belief is confined within the realm of thought. The
second is subject to regulation where the belief is translated into external acts
that affect public welfare.

Petitioners argue that they are not engaged in external acts for they simply quietly
stand at attention during the flag ceremony to show their respect to other
students who chose to participate. Thus, they do not engage in any disruptive
behavior. There is no warrant for their expulsion. Absent a threat to public
safety (clear and present danger of a substantive evil which the state has the
Updated by K Pascual. 2014. 4C. Bill of Rights Poli Digests. Atty. Jack Jimenez. 4C. 88


right to prevent), expulsion is unjustified. Any expulsion would violate religious
freedom.

While the state has responsibility to inculcate patriotism in the youth, this
interest is not free from the balancing process when it intrudes into
fundamental rights such as freedom of religion.

Refusal to take part in the flag ceremony is not so offensive as to prompt
legitimate state intervention. Exempting the Jehovahs witnesses from the flag
ceremony, which comprise a small portion of the school population, will NOT
shake up this part of the globe. It does not automatically produce a nation bereft
of patriotism. These virtues of patriotism may be taught in school. In fact,
forcing a small religious group to participate in a ceremony that will violate
their belief will hardly be conducive to love of country.

The essence of the guarantee of free exercise of religion is freedom from
conformity with religious dogma.

There is also a violation of right to receive education if they will be expelled.

Justice Cruz concurring:
The real issue is the interpretation of the bible. It is not for the courts to
interpret. It is the religion which provides its own interpretation of the bible.

CHAMP Page 88 9/16/2014

60. ESTRADA VS. ESCRITOR
freedom of religion

these are the bare facts only.. pls read orig nalang. - champ
Soledad Escritor, a court interpreter in the Las Pias regional trial court,
was accused of cohabiting with a certain Luciano Quilapio, while they were
still married to other persons. Their relationship of 23 years had borne a
son.
The Complainant, Alejandro Estrada, alleged that this constituted
disgraceful and immoral conduct under existing laws and civil service rules.
He filed the charge against Escritor as he believes that she is committing an
immoral act that tarnishes the image of the court, thus she should not be
allowed to remain employed therein as it might appear that the court
condones her act.
In her defense, Escritor testified that when she entered the judiciary in
1999,she was already a widow, her husband having died in 1998. She
admitted that she has been living with Luciano Quilapio, Jr. without the
benefit of marriage for twenty years and that they have a son.
Escritor claimed that as members of the Jehovah's Witnesses, their living
together had the approval of their sect as proven by a DECLARATION OF
PLEDGING FAITHFULNESS which they executed in 1991. By virtue of this
act, their union was legitimized by the Jehovah's Witnesses and they were,
therefore, to be regarded as husband and wife. Ergo, no violation of any law
or rule as this would be an unconstitutional violation of their religious
freedom. Escritor and Quilapios declarations are recorded in the Watch
Tower Central office. They were executed in the usual and approved form
prescribed by the Watch Tower Bible and Tract Society.
Estrada reiterates: that the Declaration of Pledging Faithfulness recognizes
the supremacy of the proper public authorities such that she bound
herself to seek means to . . . legalize their union. It is binding only to her
co-members in the congregation and serves only the internal purpose of
displaying to the rest of the congregation that she and her mate are a
Updated by K Pascual. 2014. 4C. Bill of Rights Poli Digests. Atty. Jack Jimenez. 4C. 89


respectable and morally upright couple. Their religious belief and practice,
however, cannot override the norms of conduct required by law for
government employees. To rule otherwise would create a dangerous
precedent as those who cannot legalize their live-in relationship can simply
join the Jehovahs Witnesses congregation and use their religion as a
defense against legal liability.

ISSUES:
1) whether or not the relationship between respondent Escritor and Quilapio is
valid and binding in their own religious congregation, the Jehovahs Witnesses.
2) Whether or not Escritor should be found guilty of the administrative charge
of gross and immoral conduct.
3) whether or not respondents right to religious freedom should carve out an
exception from the prevailing jurisprudence on illicit relations for which
government employees are held administratively liable.

SC:
(there was a very loooong narration of the history of religious freedom. See orig.)

Escritor does not claim that there is error in the settled jurisprudence that an
illicit relation constitutes disgraceful and immoral conduct for which a
government employee is held liable. Nor is there an allegation that the norms of
morality with respect to illicit relations have shifted towards leniency from the
time these precedent cases were decided. The Court finds that there is no such
error or shift, thus we find no reason to deviate from these rulings that such
illicit relationship constitutes disgraceful and immoral conduct
punishable under the Civil Service Law. Escritor having admitted the
alleged immoral conduct, she, could be held administratively liable.
HOWEVER, there is a distinguishing factor that sets the case at bar apart from
the cited precedents, i.e., as a defense, respondent invokes religious freedom
since her religion, the Jehovahs Witnesses, has, after thorough investigation,
allowed her conjugal arrangement with Quilapio based on the churchs religious
beliefs and practices. This distinguishing factor compels the Court to apply
the religious clauses to the case at bar.

(and then theres another loooong narration of what morality is. See orig.)

SC: When the law speaks of immorality in the Civil Service Law or immoral
in the Code of Professional Responsibility for lawyers, or public morals in the
Revised Penal Code, or morals in the New Civil Code, or moral character in
the Constitution, the distinction between public and secular morality on the one
hand, and religious morality, on the other, should be kept in mind., The morality
referred to in the law is public and necessarily secular, not religious.

Otherwise, if government relies upon religious beliefs in formulating public
policies and morals, the resulting policies and morals would require conformity
to what some might regard as religious programs or agenda. The non-believers
would therefore be compelled to conform to a standard of conduct buttressed
by a religious belief, i.e., to a compelled religion, anathema to religious
freedom. Likewise, if government based its actions upon religious beliefs, it
would tacitly approve or endorse that belief and thereby also tacitly disapprove
contrary religious or non-religious views that would not support the policy. As
a result, government will not provide full religious freedom for all its citizens, or
even make it appear that those whose beliefs are disapproved are second-class
citizens. Expansive religious freedom therefore requires that government be
neutral in matters of religion; governmental reliance upon religious justification
is inconsistent with this policy of neutrality.

In other words, government action, including its proscription of immorality as
expressed in criminal law like concubinage, must have a secular purpose. That
is, the government proscribes this conduct because it is detrimental (or
dangerous) to those conditions upon which depend the existence and progress
of human society and not because the conduct is proscribed by the beliefs of
one religion or the other.
Updated by K Pascual. 2014. 4C. Bill of Rights Poli Digests. Atty. Jack Jimenez. 4C. 90



Recognizing the religious nature of the Filipinos and the elevating influence of
religion in society, however, the Philippine constitutions religion clauses
prescribe not a strict but a benevolent neutrality. Benevolent neutrality
recognizes that government must pursue its secular goals and interests but at
the same time strives to uphold religious liberty to the greatest extent possible
within flexible constitutional limits. Thus, although the morality contemplated
by laws is secular, benevolent neutrality could allow for accommodation of
morality based on religion, provided it does not offend compelling state
interests.

The distinction between public and secular morality as expressed - albeit not
exclusively - in the law, on the one hand, and religious morality, on the other, is
important because the jurisdiction of the Court extends only to public and
secular morality.

Having distinguished between public and secular morality and religious
morality, the more difficult task is determining which immoral acts under this
public and secular morality fall under the phrase disgraceful and immoral
conduct for which a government employee may be held administratively liable.

Application of Benevolent Neutrality and the
Compelling State Interest Test to the Case at Bar

In applying the test, the first inquiry is whether respondents right to
religious freedom has been burdened. There is no doubt that choosing
between keeping her employment and abandoning her religious belief and
practice and family on the one hand, and giving up her employment and keeping
her religious practice and family on the other hand, puts a burden on her free
exercise of religion.

The second step is to ascertain respondents sincerity in her religious
belief. Respondent appears to be sincere in her religious belief and practice
and is not merely using the Declaration of Pledging Faithfulness to avoid
punishment for immorality. She did not secure the Declaration only after
entering the judiciary where the moral standards are strict and defined, much
less only after an administrative case for immorality was filed against her. The
Declaration was issued to her by her congregation after ten years of living
together with her partner, Quilapio, and ten years before she entered the
judiciary. Ministers from her congregation testified on the authenticity of the
Jehovahs Witnesses practice of securing a Declaration and their doctrinal or
scriptural basis for such a practice. As the ministers testified, the Declaration is
not whimsically issued to avoid legal punishment for illicit conduct but to make
the union of their members under respondents circumstances honorable
before God and men.

In any event, even if the Court deems sufficient respondents evidence on
the sincerity of her religious belief and its centrality in her faith, the case
at bar cannot still be decided using the compelling state interest test.
The case at bar is one of first impression.

It is apparent from the OCAs reliance upon this ruling that the state
interest it upholds is the preservation of the integrity of the judiciary by
maintaining among its ranks a high standard of morality and decency.
However, there is nothing in the OCAs memorandum to the Court that
demonstrates how this interest is so compelling that it should override
respondents plea of religious freedom nor is it shown that the means employed
by the government in pursuing its interest is the least restrictive to
respondents religious exercise.
Indeed, it is inappropriate for the complainant, a private person, to present
evidence on the compelling interest of the state. The burden of evidence should
be discharged by the proper agency of the government which is the Office of the
Solicitor General. To properly settle the issue in the case at bar, the government
should be given the opportunity to demonstrate the compelling state interest it
seeks to uphold in opposing the respondents stance that her conjugal
Updated by K Pascual. 2014. 4C. Bill of Rights Poli Digests. Atty. Jack Jimenez. 4C. 91


arrangement is not immoral and punishable as it comes within the scope of free
exercise protection. Should the Court prohibit and punish her conduct
where it is protected by the Free Exercise Clause, the Courts action would
be an unconstitutional encroachment of her right to religious freedom
We cannot therefore simply take a passing look at respondents claim of
religious freedom, but must instead apply the compelling state interest test.
The government must be heard on the issue as it has not been given an
opportunity to discharge its burden of demonstrating the states compelling
interest which can override respondents religious belief and practice. To
repeat, this is a case of first impression where we are applying the compelling
state interest test in a case involving purely religious conduct. The careful
application of the test is indispensable as how we will decide the case will make
a decisive difference in the life of the respondent who stands not only before the
Court but before her Jehovah God.
IN VIEW WHEREOF, the case is REMANDED to the Office of the Court
Administrator. The Solicitor General is ordered to intervene in the case where it
will be given the opportunity (a) to examine the sincerity and centrality of
respondents claimed religious belief and practice; (b) to present evidence on
the states compelling interest to override respondents religious belief and
practice; and (c) to show that the means the state adopts in pursuing its interest
is the least restrictive to respondents religious freedom.
(Note from SC Spokesperson Ismael Khan from the Inquirer: In other words, the
decision shows that the Supreme Court is prepared to adopt a policy of
benevolent neutrality in its interpretation of the establishment and free
exercise of religion clauses of the Constitution, unless it can be demonstrably
proven that there is compelling state interest that would negate such an
accommodation, such as a clear and present danger to established institutions
of society and the law.)

CHAMP Page 91 9/16/2014
please read orig..

61. IGLESIA NI KRISTO VS. CA
freedom of religion.

Iglesia Ni Kristo (INK) has a tv program aired on Channel 2, every Saturday.
It presents and propagates religious beliefs, doctrines etc, and makes
comparative studies with other religions.
The MTRCB classified one program as X, not for public viewing because
they allegedly offend and attack another religion. (In that program, the INK
criticizes the Catholic Church for its veneration of the Virgin Mary. The INK
suggests a very literal translation of the Bible and nowhere in the Bible is
there a veneration for the Virgin Mary.)
The INK appealed to the Office of the President, which reversed the MTRCB
decision (allowed the showing of the program). A civil case was filed by the
INK against the MTRCB for gadalej in giving them an X-rating and requiring
them to submit VTR tapes.
MTRCB invoked their powers under PD1986.
The RTC ruled that INK should stop the program and refrain from attacking
other religions.
The CA reversed.

ISSUES:
Whether the program is constitutionally protected exercise of religious
expression. Whether it is subject of the police power of the state.
Whether there was clear and present danger to stop the program.
Whether MTRC has the power to censor religious programs.
Whether the religious program is indecent and contrary to law and good
customs.

Updated by K Pascual. 2014. 4C. Bill of Rights Poli Digests. Atty. Jack Jimenez. 4C. 92


SC: MTRCB WAS WRONG. INK IS CORRECT.

The law gives the Board the power to screen, review and examine all
television programs. By the clear terms of the law, the Board has the power
to approve, delete x x x and/or prohibit the x x x exhibition and/or television
broadcast of x x x television programs x x x. The law also directs the Board to
apply contemporary Filipino cultural values as standard to determine those
which are objectionable for being immoral, indecent, contrary to law and/or
good customs, injurious to the prestige of the Republic of the Philippines and its
people, or with a dangerous tendency to encourage the commission of violence
or of a wrong or crime.

Freedom of religion has been accorded a preferred status by the framers of
our fundamental laws, past and present. We have affirmed this preferred status
well aware that it is designed to protect the broadest possible liberty of
conscience, to allow each man to believe as his conscience directs, to profess his
beliefs, and to live as he believes he ought to live, consistent with the liberty of
others and with the common good.

The right to religious profession and worship has a two-fold aspect, viz.,
freedom to believe and freedom to act on ones beliefs. The first is absolute as
long as the belief is confined within the realm of thought. The second is
subject to regulation where the belief is translated into external acts that
affect the public welfare.

Its essence is freedom from conformity to religious dogma, not freedom
from conformity to law because of religious dogma.

We thus reject petitioners postulate that its religious program is per se beyond
review by the respondent Board. Its public broadcast on TV of its religious
program brings it out of the bosom of internal belief. Television is a medium
that reaches even the eyes and ears of children. For sure, we shall continue to
subject any act pinching the space for the free exercise of religion to a
heightened scrutiny but we shall not leave its rational exercise to the
irrationality of man. For when religion divides and its exercise destroys,
the State should not stand still.

First. Deeply ensconced in our fundamental law is its hostility against all prior
restraints on speech, including religious speech. Hence, any act that restrains
speech is hobbled by the presumption of invalidity and should be greeted with
furrowed brows. It is the burden of the respondent Board to overthrow this
presumption. If it fails to discharge this burden, its act of censorship will be
struck down. It failed in the case at bar.
(here the MTRCB violated the prohibition against prior restraint). Prior restraint
includes religious speech.

Second. The evidence shows that the Board x-rated the TV series for
attacking other religions, especially the Catholic church. An examination of
the evidence, especially Exhibits A, A-1, B, C, and D will show that the so-
called attacks are mere criticisms of some of the deeply held dogmas and
tenets of other religions. The CA also did not review the VTR Tapes. The
respondent Board may disagree with the criticisms of other religions by petitioner
but that gives it no excuse to interdict such criticisms, however, unclean they may
be. Under our constitutional scheme, it is not the task of the State to favor any
religion by protecting it against an attack by another religion. Religious dogmas
and beliefs are often at war and to preserve peace among their followers,
especially the fanatics, the establishment clause of freedom of religion prohibits
the State from leaning towards any religion. Vis-a-vis religious differences, the
State enjoys no banquet of options. Neutrality alone is its fixed and immovable
stance. In fine, respondent board cannot squelch the speech of petitioner Iglesia ni
Cristo simply because it attacks other religions, even if said religion happens to be
the most numerous church in our country. In a State where there ought to be no
difference between the appearance and the reality of freedom of religion, the
remedy against bad theology is better theology. The bedrock of freedom of
religion is freedom of thought and it is best served by encouraging the
marketplace of dueling ideas. When the luxury of time permits, the marketplace
Updated by K Pascual. 2014. 4C. Bill of Rights Poli Digests. Atty. Jack Jimenez. 4C. 93


of ideas demands that speech should be met by more speech for it is the spark of
opposite speech, the heat of colliding ideas that can fan the embers of truth.
(ESSENCE OF FREEDOM TO DIFFER).

Third. The respondents cannot also rely on the ground attacks against another
religion in x-rating the religious program of petitioner. Even a sideglance at
Section 3 of PD 1986 will reveal that it is not among the grounds to justify an
order prohibiting the broadcast of petitioners television program. The ground
attack against another religion was merely added by the respondent Board in
its Rules.

Fourth. In x-rating the TV program of the petitioner, the respondents failed to
apply the clear and present danger rule. The records show that the decision
of the respondent Board, affirmed by the respondent appellate court, is
completely bereft of findings of facts to justify the conclusion that the subject
video tapes constitute impermissible attacks against another religion. There is
no showing whatsoever of the type of harm the tapes will bring about
especially the gravity and imminence of the threatened harm. Prior restraint
on speech, including religious speech, cannot be justified by hypothetical
fears but only by the showing of a substantive and imminent evil which
has taken the life of a reality already on ground.

62. PAMIL VS. TELERON
freedom of religion

Fr. Gonzaga was elected and proclaimed as municipal mayor of
Albuquerque, Bohol.
Pamil, who was the losing candidate, filed a quo warranto against Fr.
Gonzaga on the ground that under the Administrative Code, ecclesiastics,
soldiers in active duty, persons receiving salaries from govt, cannot be
elected into municipal office.
Judge Teleron sustained Fr. Gonzaga. He ruled that the statutory prohibition
was impliedly repealed by the Election Code of 1971.

SC: The Court is divided. But since the majority of 8 votes was not reached,
there is a presumption of validity. The votes of the 7 would be inadequate to
render the challenged provision invalid. This means that the prohibition still
applies and that Fr. Gonzaga must vacate his office.

(5 justices) DISQUALIFIED. (Makasiar, Barredo, etc)
Prohibition still applies because it is constitutional.
The provision in the Administrative Code is NOT a religious test but merely
defines a disqualification. It prohibits priests from running for public office. It
does NOT deprive such individuals of their political right of suffrage, to elect
and to vote.

Furthermore, it does NOT inquire into the religion of any citizen. If it does, all
citizens would be disqualified for election and there would be no need to single
them out in the law. All these persons enumerated in the Adminstrative Code
profess some religion or religious belief. But one is disqualified under the
Administrative Code not by reason of his religion or lack of it, but because of his
religious profession or vocation. Fr. Gonzaga is disqualified not because of
religion but because he is a priest.

The prohibition means that a person may exercise civil or political rights
without being required to belong to a certain church or to hold a particular
religious belief. This is different from disqualifying ALL priests from holding
municipal offices. It applies to ALL regardless of religious belief.
Updated by K Pascual. 2014. 4C. Bill of Rights Poli Digests. Atty. Jack Jimenez. 4C. 94



To allow him to run would also permit an erosion of the separation between
church and state. The Administrative Code was designed to preserve that wall of
separation. The no-religious test clause however does not guarantee the right to
run for public office and thereafter to use such public office to compel the
citizenry to conform to his religious belief, thereby gaining for his Church
dominance over the State.

Also, since a priest, once elected, enjoys salary pertaining to his office, this
would be direct violation of the prohibition that no money should be
appropriated in favor of priests or ministers.

Finally, the 1971 Election Code is NOT inconsistent with the prohibition in the
Administrative Code. It does NOT enumerate the disqualified but merely defines
the effect of filing a certificate of candidacy that is, the cessation from public
office).



(7 justices) - NOT DISQUALIFIED. (Teehankee, Fernando, etc.)
The judgment is affirmed because the provision is no longer operative either
because it was superseded by the 1935 constitution or repealed. The overriding
principle of the supremacy of the Constitution, or at the very least the repeal of
such provision bars a reversal.
They argue that under the present constitution, no religious test shall be
required fro the exercise of civil and political rights. The Administrative Code is
on its face, inconsistent with the religious freedom guaranteed by the
Constitution. To so exclude them would be to impose a religious test. There
being an incompatibility between the Administrative Code and an express
constitutional mandate, the priest must be allowed to hold office.

The no-religious test clause bars the state from disqualifying a non-believer
from voting or being voted for a public office because it is tantamount to a
religious test and compelling them to profess a belief in God and in religion. By
the same token, the same clause is equally applicable to those who are full
believers. To disqualify them from being voted for and elected into office under
the Administrative Code is to exact a religious test for the exercise of their
political rights for its amounts to compelling them to shed off their religious
ministry or robe for the exercise of their political right to run for public office.

The right of the citizen cannot be discriminated against simply because he is a
priest.
There is also no violation of the separation of the church and state because it
simply seeks to achieve government neutrality in religious matters.
Finally, the prohibition under the Administrative Code has been repealed by the
1971 Election Code. In that Code, there is no more mention of ecclesiastics as
among those disqualified.
63. US. VS. BALLARD
freedom of religion

Ballard was indicted and convicted for using mails to defraud under the US
Criminal Code. He alleged employed a scheme to defraud by organizing the
I AM movement through the use of mails, distributing literature etc.
Ballard a.k.a. Saint Germain was represented as the divine messenger and
that his other cohorts were the ascertained masters. They claim that the
words of the alleged divine entity would be transmitted to mankind
through Ballard.
They represented in their mails, that they had supernatural powers to heal
persons of all ailments. They also tried to solicit and obtain money or
Updated by K Pascual. 2014. 4C. Bill of Rights Poli Digests. Atty. Jack Jimenez. 4C. 95


property from their patients for their use and benefit by means of false and
fraudulent pretenses and promises.

ISSUE: It is immaterial what Ballard et al preached or wrote in their mails. Their
religious beliefs cannot be an issue in this court. The real issue is, did they
honestly and in good faith believe in those things? If they did they should be
acquitted.
The jury would thus be called to decide only on the question of whether Ballard
believed the representations they made and that the benefits they promised
would flow from their belief to those who embrace and followed their teachings.

The Circuit Court of Appeals ruled however that the question of truth of the
representations concerning their beliefs should be submitted to the jury. Thus it
remanded the case for new trial.

SC: The lower court erred and we do not agree that the truth or verity of the
respondents beliefs should have been submitted to the jury.

The law knows no heresy and is committed to the support of no dogma, the
establishment of no sect. The 1
st
Am. Has dual aspect. It not only forestalls
compulsion by law of the acceptance of any creed or the practice of any form of
worship, but also safeguards the free exercise of the chose form of religion. The
1
st
Am., embraces 2 concepts, the freedom to believe and the freedom to act.
The first is absolute but the second is not.

Freedom of though includes freedom of religious belief. It embraces the right to
maintain theories of life and death. Heresy trials are foreign to our Constitution.
Men may believe what they cannot prove. They may not be put to the poof
of their religious beliefs. If one could be sent to jail because a jury found those
teachings false, then what would be left of religious freedom?

Mans relation to his God is not a concern of the State. He is granted the right to
worship as he pleases and to answer to no man for the verity of his religious
views. The religious beliefs espoused by Ballard etc, might seem incredible and
preposterous, but if those doctrines are subject to trial, then the same can be
done with religious sects or beliefs. This a forbidden domain. The 1
st
Am does
not select any one group or any one type of religion for preferred treatment. It
puts them all in that position.

CHAMP Page 95 9/16/2014
64. ABINGTON SCHOOL DISTRICT VS. SCHEMPP
freedom of religion

There was a Pennsylvania law requiring that at least 10 verses of the Bible
shall be read at the opening of each public school on every school day. This
was followed by the recitation of the Lords Prayer over the schools PA
system. There are no prefatory statements, no questions asked or solicited,
no comments or explanations made and no interpretations given at or
during the exercises. To be excused from such Bible reading, the student
must preset a written request by his parents.
The Schempp family sought to enjoin the enforcement of the statute, as
violative of the Non-establishment clause.
The Schempps are members of the Unitarian faith in Germantown.

In another case, the City of Baltimore adopted the same practice.
The Murrays however were atheists. They also sought the nullification of
the statute as it threatens their religious liberty by placing a premium of
belief as against non-belief.
Updated by K Pascual. 2014. 4C. Bill of Rights Poli Digests. Atty. Jack Jimenez. 4C. 96



SC:
Religious freedom is one of "absolute equality before the law, of all religious
opinions and sects. "Neither a state nor the Federal Government can set up a
church. Neither can pass laws which aid one religion, aid all religions, or prefer
one religion over another. It was to create a complete and permanent
separation of the spheres of religious activity and civil authority by
comprehensively forbidding every form of public aid or support for religion.

Freedom conscience and freedom to adhere to such religious organization or
form of worship as the individual may choose cannot be restricted by law. On
the other hand, it safeguards the free exercise of the chosen form of religion.
Thus the Amendment embraces two concepts, - freedom to believe and freedom
to act. The first is absolute but, in the nature of things, the second cannot be.

It requires the state to be a neutral in its relations with groups of religious
believers and non-believers; it does not require the state to be their adversary.
State power is no more to be used so as to handicap religions than it is to favor
them.

Separation is a requirement to abstain from fusing functions of Government and
of religious sects, not merely to treat them all equally. So far as interference
with the `free exercise' of religion and an `establishment' of religion are
concerned, the separation must be complete and unequivocal. The First
Amendment within the scope of its coverage permits no exception; the
prohibition is absolute. The First Amendment, however, does not say that in
everyand all respects there shall be a separation of Church and State. Rather, it
studiously defines the manner, the specific ways, in which there shall be no
concert or union or dependency one on the other.

Neither a State nor the Federal Government can constitutionally force a person
`to profess a belief or disbelief in any religion.' Neither can constitutionally pass
laws or impose requirements which aid all religions as against non-believers,
and neither can aid those religions based on a belief in the existence of God as
against those religions founded on different beliefs.

The Establishment Clause, unlike the Free Exercise Clause, does not depend
upon any showing of direct governmental compulsion and is violated by the
enactment of laws which establish an official religion whether those laws
operate directly to coerce non-observing individuals or not. When the power,
prestige and financial support of government is placed behind a particular
religious belief, the indirect coercive pressure upon religious minorities to
conform to the prevailing officially approved religion is plain.

The test may be stated as follows: what are the purpose and the primary effect
of the enactment? If either is the advancement or inhibition of religion then the
enactment exceeds the scope of legislative power as circumscribed by the
Constitution. To be valid
1) there must be a secular legislative purpose
2) the statute should not advance nor inhibit any religion
3) three must be no excessive entanglement between government and religion.

Applying the Establishment Clause principles to the cases at bar we find that the
States are requiring the selection and reading at the opening of the school day of
verses from the Holy Bible and the recitation of the Lord's Prayer by the
students in unison. These exercises are prescribed as part of the curricular
activities of students who are required by law to attend school. They are held in
the school buildings under the supervision and with the participation of
teachers employed in those schools.

Updated by K Pascual. 2014. 4C. Bill of Rights Poli Digests. Atty. Jack Jimenez. 4C. 97


The conclusion follows that in both cases the laws require religious exercises
and such exercises are being conducted in direct violation of the rights of the
appellees and petitioners. It is no defense to urge that the religious practices
here may be relatively minor encroachments on the First Amendment.

It might well be said that one's education is not complete without a study of
comparative religion or the history of religion and its relationship to the
advancement of civilization. It certainly may be said that the Bible is worthy of
study for its literary and historic qualities. Nothing we have said here indicates
that such study of the Bible or of religion, when presented objectively as part of
a secular program of education, may not be effected consistently with the First
Amendment. But the exercises here do not fall into those categories. They
are religious exercises, required by the States in violation of the command
of the First Amendment that the Government maintain strict neutrality,
neither aiding nor opposing religion.

Finally, we cannot accept that the concept of neutrality, which does not permit a
State to require a religious exercise even with the consent of the majority of
those affected, collides with the majority's right to free exercise of
religion.While the Free Exercise Clause clearly prohibits the use of state action
to deny the rights of free exercise to anyone, it has never meant that a majority
could use the machinery of the State to practice its beliefs.

It is not within the power of government to invade that citadel, whether its
purpose or effect be to aid or oppose, to advance or retard. In the relationship
between man and religion, the State is firmly committed to a position of
neutrality.

CHAMP Page 97 9/16/2014
65. LEMON VS. KURTZMAN
freedom of religion

Pennsylvania and Rhode Island adopted a statute which provides financial
support to schools by way of reimbursement to the cost of teaching salaries,
textbooks, instructional materials in specified secular subjects. There is also
a 15% annual subsidy to salaries of teachers.
The State relied upon the Everson case where the court upheld a statute
which reimbursed parents of parochial school children for their bus transpo
expenses.

SC: UNCONSTITUTIONAL.
The 3 evils which the non-establishment clause seeks to prevent are
sponsorship, financial support, and active involvement of the sovereign in
religious activities.

The criteria are as follows:
1) the statute must have a secular religious purpose
2) its principal or primary effect must be that it neither advances nor inhibits
religion
3) it must not foster an excessive government entanglement with religion.

1
st
: The secular and religious education are identifiable and separable in these
statutes. The have created statutory restrictions designed to guarantee the
separation between the secular and religious education functions and to ensure
that the State will financially support only the former.

2
nd
: Here, the legislative intent of the statutes was NOT to advance religion. The
statute themselves clearly state that they are intended to enhance the quality of
Updated by K Pascual. 2014. 4C. Bill of Rights Poli Digests. Atty. Jack Jimenez. 4C. 98


secular education. A state has legitimate concern for maintaining minimum
standards.

3
rd
: However, the cumulative impact of the entire relationship arising under the
statutes in each State is an excessive entanglement between government and
religion. In order to determine how excessive the entanglement is, we must
examine the character and purpose of the institutions which are benefited and
the nature of the aid that the state provides, and the resulting relationship
between government and religious authority.

RHODE ISLAND: Under the Rhode Island Statute, nuns have a teaching role,
thus enhancing the religious atmosphere in the parochial schools. There was
also an attempt to maintain a 1-to-1 ratio between nuns and lay teachers. Also,
it was found that the parochial schools are an integral part of the mission of the
Catholic Church. This makes it a power vehicle for transmitting Catholic faith to
the next generation. In short, parochial schools involve substantial religious
activity and purpose.

We cannot ignore the dangers that a teacher under religious control poses to
the separation of the religious from the purely secular aspects of education.
Furthermore, the Rhode Island Schools are under the general supervision of the
Bishop of Providence and the Diocese Superintendent. The school principals are
also nuns appointed by either the Mother Provincial of the order whose
members staff the school. With respect to lay teachers, before they are accepted
to teach, they are first interviewed by the Superintendent and their contracts
are co-signed by the parish priest, who also has the discretion in negotiating
salary levels. HENCE, RELIGIOUS AUTHORITY PERVADES IN THE SCHOOL
SYSTEM.

Unlike a book, a teacher cannot be inspected once so as to determine the extent
and intent of his personal beliefs. Thus a dedicated religious person teaching in
a school affiliated with his faith and operated to inculcate its tenets, cannot
remain religious neutral. Thus, there will be excessive and enduring
entanglement between church and state.

PENNSYLVANIA: Same. The very restrictions and surveillance necessary to
ensure that teachers play a strictly non-ideological role give rise to
entanglements between church and state. Here, reimbursements of expenses is
not only limited to courses offered in public schools, but also any subject matter
expressing religious teaching or the morals or forms of worship of any sect.

It also provides state financial aid DIRECTLY to church-related school. A DIRECT
MONEY SUBSIDY would be a relationship pregnant with entanglement and
could encompass sustained and detailed administrative relationship for
enforcement of the standards. A government post-audit power to inspect and
evaluate a church-related schools financial records and to determine which
expenditures are religious and which are secular creates an intimate and
continuing relationship between church and state.

The constitution decrees that religion must be a private matter for the
individual, the family and the institutions of private choice.
66. AGLIPAY VS. RUIZ
freedom of religion

Gregorio Aglipay is the head of the Phil. Indep. Church. He filed a
prohibition against the Director of Posts to stop the issuance and sale of
commemorative postage stamps of the 33
rd
International Eucharistic
Congress.
The stamps were issued during the celebration in Manila of said event,
organized by the Roman Catholic Church. The stamp had the design of a
chalice with grape vines and stalks of wheat.
Updated by K Pascual. 2014. 4C. Bill of Rights Poli Digests. Atty. Jack Jimenez. 4C. 99


Aglipay alleges that Act 4052, which authorizes the Director of Posts to
dispose of 60,000 pesos in a manner it deems advantageous to the
government, violates the prohibition that no public money shall be
appropriated for or in support of any church, as well as the separation
between church and state.

SC: VALID LAW.
Act 4052 does not contemplate any religious purpose in view. What it gives the
Director of Posts is merely the discretionary power to determine when the
issuance of the special postage stamps would be advantageous to the
government. The issuance of the postage stamp was NOT inspired by any
sectarian feeling to favor a particular church or religious domination. The
stamps were not issued and sold for the benefit of the Roman Catholic Church.
Nor were money derived from that sale of the stamps given to that church.

The only purpose of the selling of the stamps was to advertise the Philippines
and to attract more tourists to the country. The officials merely took advantage
of an event that has considerable international importance to give publicity to
the Philippines.

Note also that instead of showing a Catholic Church chalice, as originally
planned, the stamp contains a map of the Philippines and City of Manila. What is
therefore emphasized in not the Eucharistic Congress itself but Manila, as the
seat of that congress.

Even if the event had religious character, the resulting propaganda was not the
aim and purpose of the Government. The government should not be
embarrassed in its activities simply because of incidental results, more or less
religious in character, if the purpose in view could be undertaken by
appropriate legislation. The main purpose should not be frustrated by its
subordination to mere incidental results not contemplated.
67. VICTORIANO VS. ELIZALDE ROPE WORKERS
freedom of religion

Victoriano was a member of the INK and an employee of the Elizalde Rope
Factory. He was also a member of the Union which had a CBA containing a
closed shop provision.
Under their CBA, membership in the Union is required as a condition for
employment.
Being a member of the INK which prohibits the affiliation of its members
with any labor organization, Victoriano resigned from the Union.
The Union then asked the Company to terminate him for violating the CBA
condition, invoking the union security clause.
Under RA 875, the Company had liberty impose conditions as a
requirement for continued employment.
Later, RA 3350 was enacted amending RA 875 which exempted members of
religious sects from the affiliation with labor organizations.
Victoriano now questions his dismissal.
The Company and the Union however invoke the unconstitutionality of RA
3350 insofar as it impairs the obligation of contracts. This is because while
the Union is obliged to comply with its closed shop provision, the
amendment relieves the employer company from its reciprocal obligation
of cooperation in the maintenance of the union membership as a condition
for employment. The Union also averred that said Act discriminates in favor
of members of said religious sects. They also contend that Republic Act No.
3350 violates the constitutional prohibition against requiring a religious
test for the exercise of a civil right or a political right.


Updated by K Pascual. 2014. 4C. Bill of Rights Poli Digests. Atty. Jack Jimenez. 4C. 100


SC: RA 3350 VALID. VICTORIANO CANNOT BE DISMISSED.
Republic Act No. 3350 merely excludes ipso jure from the application and
coverage of the closed shop agreement the employees belonging to any religious
sects which prohibit affiliation of their members with any labor organization.
What the exception provides, therefore, is that members of said religious sects
cannot be compelled or coerced to join labor unions even when said unions
have closed shop agreements with the employers; that in spite of any closed
shop agreement, members of said religious sects cannot be refused employment
or dismissed from their jobs on the sole ground that they are not members of
the collective bargaining union. It is clear, therefore, that the assailed Act, far
from infringing the constitutional provision on freedom of association, upholds
and reinforces it. It does not prohibit the members of said religious sects from
affiliating with labor unions. It still leaves to said members the liberty and the
power to affiliate, or not to affiliate, with labor unions. If, notwithstanding their
religious beliefs, the members of said religious wets prefer to sign up with the
labor union, they can do so. If in deference and fealty to their religious faith,
they refuse to sign up, they can do so; the law does not coerce them to join;
neither does the law prohibit them from joining, and neither may the employer
or labor union compel them to join. Republic Act No. 3350, therefore, does not
violate the constitutional provision on freedom of association.

What then was the purpose sought to be achieved by Republic Act No. 3350? Its
purpose was to insure freedom of belief and religion, and to promote the
general welfare by preventing discrimination against those members of
religious sects which prohibit their members from joining labor unions,
confirming thereby their natural, statutory and constitutional right to work. It
cannot be gainsaid that said purpose is legitimate.

It would be unthinkable indeed to refuse employing a person who, on account of
his religious beliefs and convictions, cannot accept membership in a labor
organization although he possesses all the qualifications for the job. This is
tantamount to punishing such person for believing in a doctrine he has a right
under the law to believe in. The law would not allow discrimination to flourish
to the detriment of those whose religion discards membership in any labor
organization. Likewise, the law would not commned the deprivation of their
right to work and pursue a modest means of livelihood, without in any manner
violating their religious faith and/or belief."

It cannot be denied, furthermore, that the means adopted by the Act to achieve
that purpose - exempting the members of said religious sects from coverage of
union security agreements - is reasonable.

It may not be amiss to point out here that the free exercise of religious
profession or belief is superior to contract rights. In case of conflict, the latter
must, therefore, yield to the former.

The purpose of Republic Act No. 3350 is secular, worldly, and temporal, not
spiritual or religious or holy and eternal. It was intended to serve the secular
purpose of advancing the constitutional right to the free exercise of religion, by
averting that certain persons be refused work, or be dismissed from work. It
acted merely to relieve the exercise of religion, by certain persons, of a burden
that is imposed by union security agreements.

It is our view that the exemption from the effects of closed shop agreement does
not directly advance, or diminish, the interests of any particular religion.
Although the exemption may benefit those who are members of religious sects
that prohibit their members from joining labor unions, the benefit upon the
religious sects is merely incidental and indirect. The "establishment clause" (of
religion) does not ban regulation on conduct whose reason or effect merely
happens to coincide or harmonize with the tenets of some or all religions. The
free exercise clause of the Constitution has been interpreted to require that
religious exercise be preferentially aided.

The Act does not require as a qualification, or condition, for joining any lawful
association membership in any particular religion or in any religious sect;
neither does the Act require affiliation with a religious sect that prohibits its
Updated by K Pascual. 2014. 4C. Bill of Rights Poli Digests. Atty. Jack Jimenez. 4C. 101


members from joining a labor union as a condition or qualification for
withdrawing from a labor union. Joining or withdrawing from a labor union
requires a positive act Republic Act No. 3350 only exempts members with such
religious affiliation from the coverage of closed shop agreements. So, under this
Act, a religious objector is not required to do a positive act-to exercise the right
to join or to resign from the union. He is exempted ipso jure without need of any
positive act on his part. A conscientious religious objector need not perform a
positive act or exercise the right of resigning from the labor union-he is
exempted from the coverage of any closed shop agreement that a labor union
may have entered into. How then can there be a religious test required for the
exercise of a right when no right need be exercised?

(read orig for other arguments and issues)
68. VILLAVECENCIO VS. LUKBAN
liberty of abode and travel

One hundred and seventy women, who had lived in the segregated district
for women of ill repute in the city of Manila, were by orders of the Mayor of
the city of Manila and the chief of police of that city isolated from society
and then at night without their consent and without any opportunity to
consult with friends or to defend their rights, were forcibly hustled on
board steamers for transportation to regions unknown.
No law order, or regulation authorized the Mayor of the city of Manila or the
chief of the police of that city to force citizens of the Philippine Islands to
change their domicile from Manila to another locality.
The city authorities quietly perfected arrangements with the Bureau of
Labor for sending the women to Davao, Mindanao, as laborers.
The women were given no opportunity to collect their belongings, and
apparently were under the impression that they were being taken to a
police station for an investigation. They had no knowledge that they were
destined for a life in Mindanao. 'They had not been asked if they wished to
depart from that region and had neither directly nor indirectly given their
consent to the deportation.
The vessels reached Davao, and they were receipted for as laborers by the
Governor.
The friends and relatives of the women filed for habeas corpus. But
somehow the Mayor refused to comply with the order of the court.
They argue that the habeas corpus is not proper because the women are not
restrained of their liberty. That they are under no restraint; the women
were free in Davao.

ISSUE: By authority of what law did the Mayor and the Chief of Police presume
to act in deporting by duress these persons from Manila to another distant
locality within the Philippine Islands?

SC:
The writ of habeas corpus was properly granted, and that the Mayor of the city
of Manila who was primarily responsible for the deportation, is in contempt of
court for his failure to comply with the order of the court.

There is no law, order, or regulation, which even hints at the right of the Mayor
of the city of Manila or the chief of police of that city to force citizens of the
Philippine Islands-and these women despite their being in a sense lepers of
society are nevertheless not chattels but Philippine citizens protected by the
same constitutional guaranties as are other citizens-to change their domicile
from Manila to another locality.

The privilege of domicile is deemed so important as to be found in the Bill of
Rights of the Constitution. Liberty of abode is a principle so deeply imbedded in
jurisprudence and considered so elementary in nature as not even to require a
Updated by K Pascual. 2014. 4C. Bill of Rights Poli Digests. Atty. Jack Jimenez. 4C. 102


constitutional sanction. Even the Governor-General of the Philippine Islands,
even the President of the United States, who has often been said to exercise
more power than any king or potentate has no such arbitrary prerogative,
either inherent or express. Much less, therefore, has the executive of a
municipality who acts within a sphere of delegated powers. If the mayor and the
chief of police could at their mere behest or even for the most praiseworthy of
motives, render the liberty of the citizen so insecure, then the presidents and
chiefs of police of one thousand other municipalities of the Philippines have the
same privilege. If these officials can take to themselves such power, then any
other official can do the same. And if any official can exercise the power, then all
persons would have just as much right to do so. And if a prostitute could be sent
against her wishes and under no law from one locality to another within the
country, then officialdom can hold the same club over the head of any citizen.

A prime specification of an application for a writ of habeas corpus is restraint of
liberty. The essential object and purpose of the writ of habeas corpus is to
inquire into all manner of involuntary restraint as distinguished from voluntary,
and to relieve a person therefrom if such restraint is illegal. Any restraint which
will preclude freedom of action is sufficient. The forcible taking of these women
from Manila by officials of that city who handed them over to other parties, who
deposited them in a distant region, deprived these women of freedom of
locomotion just as effectively as if they had been imprisoned. Placed in Davao
without either money or personal belongings, they were prevented from
exercising the liberty of going when and where they pleased. The restraint of
liberty which began in Manila continued until the aggrieved parties were
returned to Manila and released or until they freely and truly waived this right.

The court (Davao) ordered the Mayor to produce the bodies of the persons they
did not produce the bodies of the persons in whose behalf the writ was granted;
they did not show impossibility of performance; and they did not present
writings that waived the right to be present by those interested. Instead a few
stereotyped affidavits purporting to show that the women were contented with
their life in Davao, some of which have since been repudiated by the signers,
were appended to the return.

Hence, the mayor is guilty of contempt of court.
69. MANOTOC VS. CA
liberty of abode and travel

Ricardo L. Manotoc, Jr., is one of the two principal stockholders of Trans-
Insular Management, Inc. and the Manotoc Securities, Inc., a stock
brokerage house.
Following the "run" on stock brokerages, Manotoc, who was then in the
United States, came home, and together with his co-stockholders, filed a
petition with the SEC for the appointment of a management committee.
Six of the companys clients filed criminal complaints against Manotoc for
estafa (because the TCT given by the company to its clients was fake). Bail
was fixed at P105,000.
Manotoc filed a motion for permission to leave the country, stating as
ground his desire to go to the United States, relative to his business
transactions and opportunities. He claims he is going to Louisiana, U.S.A. to
obtain foreign investment in Manotoc Securities, Inc.
The motion was denied in all cases.
He thus filed a petition for certiorari and mandamus seeking to annul the
orders denying his leave to travel abroad.
He contends that having been admitted to bail as a matter of right, neither
the courts which granted him bail nor the SEC which has no jurisdiction
over his liberty, could prevent him from exercising his constitutional right
to travel.
He claims that liberty is indivisible and that it operates without boundaries.

SC: MANOTOC CANNOT LEAVE THE COUNTRY.
Updated by K Pascual. 2014. 4C. Bill of Rights Poli Digests. Atty. Jack Jimenez. 4C. 103


A court has the power to prohibit a person admitted to bail from leaving the
Philippines. This is a necessary consequence of the nature and function of a bail
bond.

The condition imposed upon petitioner to make himself available at all times
whenever the court requires his presence operates as a valid restriction on his
right to travel.

The obligation assumed by the bond surety was to hold the accused amenable
at all times to the orders and processes of the lower court. This was to prohibit
said accused from leaving the jurisdiction of the Philippines, because, otherwise,
said orders and processes will be nugatory, and inasmuch as the jurisdiction of
the courts from which they issued does not extend beyond that of the
Philippines they would have no binding force outside of said jurisdiction.

Indeed, if the accused were allowed to leave the Philippines without sufficient
reason, he may be placed beyond the reach of the courts. If the sureties have the
right to prevent the principal from leaving the state, more so then has the court
from which the sureties merely derive such right, and whose jurisdiction over
the person of the principal remains un affected despite the grant of bail to the
latter. In fact, this inherent right of the court is recognized by petitioner himself,
notwithstanding his allegation that he is at all total liberty to leave the country,
for he would not have filed the motion for permission to leave the country in
the first place, if it were otherwise.

A perusal of petitioner's 'Motion for Permission to Leave the Country' will
show that it is solely predicated on petitioner's wish to travel to the United
States where he will, allegedly attend to some business transaction and search
for business opportunities. From the tenor and import of petitioner's motion, no
urgent or compelling reason can be discerned to justify the grant of judicial
imprimatur thereto. Petitioner has not sufficiently shown that there is absolute
necessity for him to travel abroad. Petitioner's motion bears no indication that
the alleged business transactions could not be undertaken by any other person
in behalf. Neither is there any hint that petitioner's absence from the United
States would absolutely preclude him from taking advantage of the business
opportunities therein, nor is there any showing that petitioner's non-presence
in the United States would cause him irreparable damage or prejudice."
Petitioner has not specified the duration of the proposed travel or shown that
his surety has agreed to it. Petitioner merely alleges that his surety has agreed
to his plans as he had posted cash indemnities.

The constitutional right to travel being invoked by petitioner is not an absolute
right. The liberty of abode and of travel shall not be impaired except upon
lawful order of the court, or when necessary in the interest of national
security, public safety or public health."

To our mind, the order of the trial court releasing petitioner on bail constitutes
such lawful order as contemplated by the above-quoted constitutional
provision.
70. SILVERIO VS. CA
liberty of abode and travel

Silverio was charged with violation of Section 20 (4) of the Revised
Securities Act in a Criminal Case. He posted bail for his provisional liberty.
The records will show that the information was filed on October 14,1985.
Until this date (28 July 1988), the case had yet to be arraigned. Several
scheduled arraignments were cancelled and react, mostly due to the failure
of accused Silverio to appear. The reason for accused Silverio's failure to
appear had invariably been because he is abroad in the United States of
America;
Since the information was filed, until this date, accused Silverio had never
appeared in person before the Court.
Updated by K Pascual. 2014. 4C. Bill of Rights Poli Digests. Atty. Jack Jimenez. 4C. 104


The bond posted by Silverio had been cancelled twice and warrants of
arrest had been issued against him all for the same reason-failure to appear
at scheduled arraignments.
Later, more than two (2) years after the filing of the Information,
respondent People of the Philippines filed a motion to cancel the passport
of and to issue a hold departure Order against accused-petitioner on the
ground that he had gone abroad several times without the necessary Court
approval resulting in postponements of the arraignment and scheduled
hearings.
He also assails the finding that the right to travel can be impaired. He claims
that while the Constitution recognizes the power of the Courts to curtail the
liberty of abode within the limits prescribed by law, it restricts the
allowable impairment of the right to travel only on grounds of interest of
national security, public safety or public health.

SC:
The bail bond he had posted had been cancelled and Warrants of Arrest had
been issued against him by reason, in both instances, of his failure to appear at
scheduled arraignments. Warrants of Arrest having been issued against him for
violation of the conditions of his bail bond, he should be taken into custody.

The foregoing condition imposed upon an accused to make himself available at
all times whenever the Court requires his presence operates as a valid
restriction of his right to travel. A person facing criminal charges may be
restrained by the Court from leaving the country or, if abroad, compelled to
return. So it is also that "An accused released on bail may be rearrested without
the necessity of a warrant if he attempts to depart from the Philippines without
prior permission of the Court where the case is pending.

Sec. 6. The liberty of abode and of changing the. same within the limits
prescribed by law shall not be prepaired except upon lawful order of the court.
Neither shall the right to travel be impaired except in the interest of national
security, public safety, or public health, as may be provided by law."

ISSUE: Petitioner thus theorizes that under the 1987 Constitution, Courts can
impair the right to travel only on the grounds of "national security, public safety,
or public health."
SC: This is untenable. The last portion applies only to executive or
administrative officers. The Constitution however should be interpreted to
mean that while the liberty of travel may be impaired even without Court Order,
the appropriate executive officers or administrative authorities are not armed
with arbitrary discretion to impose limitations. They can impose limits only on
the basis of "national security, public safety, or public health" and "as may be
provided by law.
The Constitution should by no means be construed as delimiting the inherent
power of the Courts to use all means necessary to carry their orders into effect
in criminal cases pending before them.

Petitioner is facing a criminal charge. He has posted bail but has violated the
conditions thereof by failing to appear before the Court when required.
Warrants for his arrest have been issued. Those orders and processes would be
rendered nugatory if an accused were to be allowed to leave or to remain, at his
pleasure, outside the territorial confines of the country. Holding an accused in a
criminal case within the reach of the Courts by preventing his departure from
the Philippines must be considered as a valid restriction on his right to travel so
that he may be dealt with in accordance with law. The offended party in any
criminal proceeding is the People of the Philippines. It is to their best interest
that criminal prosecutions should run their course and proceed to finality
without undue delay, with an accused holding himself amenable at all times to
Court Orders and processes.

In short, the condition imposed upon an accused admitted to bail to make
himself available at all times whenever the Court requires his presence operates
as a valid restriction on the right to travel.
Updated by K Pascual. 2014. 4C. Bill of Rights Poli Digests. Atty. Jack Jimenez. 4C. 105


71. LEGASPI VS. CSC
right to information

Legaspi earlier requested for information on the civil service eligibilities of
certain persons employed as sanitarians at the Cebu City Health
Department.
This was denied by the CSC. He filed a mandamus against the CSC, claiming
his right to be informed of the eligibilities of these sanitarians as
guaranteed by the Constitution.

ISSUES:
o Whether the right to information is a self-executing provision.
o Whether Legaspi has standing.
o Whether the Government agency CSC has the duty to disclose
information.
o Whether the information requested is covered by the right to
information.

SC: YES. YES. YES. YES.

The right of the people to information on matters of public concern shall be
recognized. Access to official. records, and to documents, and papers pertaining to
official acts, transactions, or decisions, as well as to government research data
used as basis for policy development, shall be afforded the citizen, subject to such
limitations as may be provided by law.

1) These constitutional provisions are self-executing. The fundamental right
recognized may be asserted by the people upon the ratification of the
constitution without need for any ancillary act of the Legislature. What may be
provided for by the Legislature are reasonable conditions and limitations upon
the access to be afforded which must, of necessity, be consistent with the
declared State policy of full public disclosure of all transactions involving public
interest. However, it cannot be overemphasized that whatever limitation may
be prescribed by the Legislature, the right and the duty have become operative
and enforceable by virtue of the adoption of the New Charter. Therefore, the
right may be properly invoked in a Mandamus proceeding such as this one.

2) Petitioner has firmly anchored his case upon the right of the people to
information an matters of public concern, which, by its very nature, is a public
right.The people are regarded as the real party in interest and the relator at
whose instigation the proceedings are instituted need not show that he has any
legal or special interest in the result, it being sufficient to show that he is a
citizen and as such interested in the execution of the laws. The requirement on
personal interest is satisfied by the mere fact that the petitioner is a citizen, and
therefore, part of the general "public" which possesses the right.

3) Subject to reasonable conditions prescribed by law, the State adopts and
implements a policy of hill public disclosure of all its transactions involving
public interest. The law itself makes a list of what should be published in the
Official Gazette. Such listing, to our mind, leaves respondents with no discretion
whatsoever as to what must be in eluded or excluded from such publication.
There is the absence of discretion on the part of government agencies in
allowing the examination of public records. Except, perhaps when it is clear that
the purpose of the examination is unlawful, or sheer, idle curiosity, we do not
believe it is the duty under the law of registration officers to concern
themselves with the motives, reasons, and objects of the person seeking access
to the records. It is not their prerogative to see that the information which the
records contain is not flaunted before public gaze, or that scandal is not made of
it. (Subject to the regulation of the manner and hours of examination to the end
that damage to or loss of, the records may be avoided, that undue interference
with the duties of the custodian of the books and documents and other
Updated by K Pascual. 2014. 4C. Bill of Rights Poli Digests. Atty. Jack Jimenez. 4C. 106


employees may be prevented, that the right of other persons entitled to make
inspection may be insured.)

Thus, while the manner of examining public records may be subject to
reasonable regulation by the government agency in custody thereof, the duty to
disclose the information of public concern, and to afford access to public
records cannot be discretionary on the part of said agencies. Certainly, its
performance cannot be made contingent upon the discretion of such agencies.
Otherwise, the enjoyment of the constitutional right may be rendered nugatory
by any whimsical exercise of agency discretion. The constitutional duty, not
being discretionary, its performance may be compelled by a writ of Mandamus
in a proper case.

4) But the constitutional guarantee to information on matters of public
concern is not absolute. It does not open every door to any and all information.
Access to official records, papers, etc., are "subject to limitations as may be
provided by law". The law may therefore exempt certain types of information
from public scrutiny, such as those affecting national security. The access to a
particular public record must be circumscribed by the nature of the information
sought, i.e., (a) being of public concern or one that involves public interest, and,
(b) not being exempted by law from the operation of the constitutional
guarantee. The threshold question is, therefore, whether or not the information
sought is of public interest or public concern.

Public concern" like "public interest" is a term that eludes exact definition. Both
terms embrace a broad spectrum of subjects which the public may want to
know, either because these directly affect their lives, or simply because such
matters naturally arouse the interest of an ordinary citizen. In the final analysis,
it is for the courts to determine in a case by case basis whether the matter at
issue is of interest or importance, as it relates to or affects the public.

In case of denial of access, the government agency has the burden of showing
that the information requested is not of public concern, or, if it is of public
concern, that the same has been exempted by law from the operation of the
guarantee.

Here, the information sought by the petitioner in this case is the truth of the
claim of certain government employees that they are civil service eligibles for
the positions to which they were appointed. Public office being a public trust. It
is the legitimate concern of citizens to ensure that government positions
requiring civil service eligibility are occupied only by persons who are eligibles.
Public officers are at all times accountable to the people even as to their
eligibilities for their respective positions.

The CSC has failed to cite any provision in the Civil Service Law which would
limit the petitioner's right to know who are, and who are not, civil service
eligibles. We take judicial notice of the fact that the names of those who pass the
civil service examinations, as in bar examinations and licensure examinations
for various professions, are released to the public. Hence, there is nothing secret
about one's civil service eligibility, if actually possessed. Petitioner's request
is,therefore ,neither unusual nor unreasonable- And when. as in this case, the
government employees concerned claim to be civil service eligibles, the public,
through any citizen, has a right to verify their professed eligibilities from the
Civil Service Commission. The civil service eligibility of a sanitarian being of
public concern, and in the absence of express limitations under the law upon
access to the register of civil service eligibles for said position, the duty of the
respondent Commission to confirm or deny the civil service eligibility of any
person occupying the position becomes imperative. Mandamus, therefore lies.

CHAMP Page 106 9/16/2014
72. VALMONTE VS. BELMONTE
right to information

Updated by K Pascual. 2014. 4C. Bill of Rights Poli Digests. Atty. Jack Jimenez. 4C. 107


Members of the print media invoke the right to information in a mandamus
case against the GSIS.
They are seeking the list of the names of the members of the Batasan who
belong to the PDPLaban who were able to secure clean loans of P2M each,
through the intercession and marginal notes and with the guarantee, of
Former First Lady Imelda Marcos.
The GSIS general manager Sonny Belmonte denied the request citing the
confidential relationship between GSIS and its borrowers, that GSIS had the
duty to its customers to preserve this confidentiality and that there is no
court order requiring them to divulge the identities of the borrowers. GSIS
also invokes the right to privacy, which is also fundamental right.

ISSUE: Whether or not petitioners are entitled to access to the documents
evidencing loans granted by the GSIS.
SC:

An informed citizenry with access to the diverse currents in political, moral and
artistic thought and data relative to them, and the free exchange of ideas and
discussion of issues thereon, is vital to the democratic government envisioned
under our Constitution. The cornerstone of this republican system of
government is delegation of power by the people to the State. In this system,
governmental agencies and institutions operate within the limits of the
authority conferred by the people. Denied access to information on the inner
workings of government, the citizenry can become prey to the whims and
caprices of those to whom the power had been delegated.

Petitioners are practitioners in media. As such, they have both the right to
gather and the obligation to check the accuracy of information they disseminate.
For them, the freedom of the press and of speech is not only critical, but vital to
the exercise of their professions. The right of access to information ensures that
these freedoms are not rendered nugatory by the government's monopolizing
pertinent information. For an essential element of these freedoms is to keep
open a continuing dialogue or process of communication between the
government and the people.

The right to information is an essential premise of a meaningful right to speech
and expression. The right to information goes hand-in-hand with the
constitutional policies of full public disclosure ** and honesty in the public
service. *** It is meant to enhance the widening role of the citizenry in
governmental decision-making as well as in checking abuse in government.

In determining whether or not a particular information is of public concern
there is no rigid test which can be applied. "Public concern" like "public
interest" is a terra that eludes exact definition.

The GSIS is a trustee of contributions from the government and its employees
and the administrator of various insurance programs for the benefit of the
latter. Undeniably, its funds assume a public character. Considering the nature
of its funds, the GSIS is expected to manage its resources with utmost prudence
and in strict compliance with the pertinent laws or rules and regulations. It
must preserve at all times the actuarial solvency of the funds administered by
the System. It is therefore the legitimate concern of the public to ensure that
these funds are managed properly with the end in view of maximizing the
benefits that accrue to the insured government employees. Moreover, the
supposed borrowers were Members of the defunct Batasang Pambansa who
themselves appropriated funds for the GSIS and were therefore expected to be
the first to see to it that the GSIS performed its tasks with the greatest degree of
fidelity and that all its transactions were above board.
In sum, the public nature of the loanable funds of the GSIS and the public office
held by the alleged borrowers make the information sought clearly a matter of
public interest and concern.

Updated by K Pascual. 2014. 4C. Bill of Rights Poli Digests. Atty. Jack Jimenez. 4C. 108


A second requisite must be met before the right to information may be enforced
through mandamus proceedings, that the information sought must not be
among those excluded by law.

GSIS has failed to cite any law granting them the privilege of confidentiality as
regards the documents subject of this petition. His position is apparently based
merely on considerations of policy.

When the information requested from the government intrudes into the privacy
of a citizen, a potential conflict between the rights to information and to privacy
may arise. However, the competing interests of these rights need not be
resolved in this case. The right to privacy belongs to the individual in his private
capacity, and not to public and governmental agencies like the GSIS. Moreover,
the right cannot be invoked by juridical entities like the GSIS. A corporation has
no right of privacy in its name since the entire basis of the right to privacy is an
injury to the feelings and sensibilities of the party and a corporation would have
no such ground for relief. Neither can the GSIS through its General Manager,
Belmonte, invoke the right to privacy of its borrowers. The right is purely
personal in nature.

HOWEVER, Although citizens are afforded the right to information and,
pursuant thereto, are entitled to "access to official records," the Constitution
does not accord them a right to compel custodians of official records to prepare
lists, abstracts, summaries and the like in their desire to acquire information on
matters of public concern. Hence, they must be allowed access to documents
and records, as petitioners may specify, subject to reasonable regulations as to
the time and manner of inspection.
73. UNITED PEPSI VS. LAGUESMA
right to assembly

Petitioner is a union of supervisory employees. The union filed a petition
for certification election on behalf of the route managers at Pepsi-Cola
Products Philippines, Inc. However, its petition was denied on the ground
that the route managers are managerial employees and, therefore, ineligible
for union membership under the first sentence of Art. 245 of the Labor
Code. (Managerial employees are not eligible to join, assist or form any labor
organization. Supervisory employees shall not be eligible for membership in a
labor organization of the rank-and-file employees but may join, assist or form
separate labor organizations of their own.)
The union assailed this decision contending that it impairs their right to
self-organization.

ISSUE: Is Art 245 constitutional?
SC:YES.

(This is a labor case but the relevant part is on the right to join form and assist
labor unions. The distinctions between managerial and supervisory employees
omitted. See original.)

Managerial employee" is one who is vested with powers or prerogatives to lay
down and execute management policies and/or to hire transfer, suspend, lay
off, recall, discharge, assign or discipline employees. Supervisory employees are
those who, in the interest of the employer, effectively recommend such
managerial actions if the exercise of such authority is not merely routinary or
clerical in nature but requires the use of independent judgment.

The guarantee of organizational right in Art. III, 8 is NOT infringed by a ban
against managerial employees forming a union. The right guaranteed in Art. III,
8 is subject to the condition that its exercise should be for purposes "not
contrary to law." In the case of Art. 245, there is a rational basis for prohibiting
managerial employees from forming or joining labor organizations. The
Updated by K Pascual. 2014. 4C. Bill of Rights Poli Digests. Atty. Jack Jimenez. 4C. 109


rationale for this inhibition is, because if these managerial employees would
belong to or be affiliated with a Union, the latter might not be assured of their
loyalty to the Union in view of evident conflict of interests. The Union can also
become company-dominated with the presence of managerial employees in
Union membership. After all, those who qualify as top or middle managers are
executives who receive from their employers information that not only is
confidential but also is not generally available to the public, or to their
competitors, or to other employees.

Relate to government employees During the 1986 Constitutional Convention,
Commissioner Lerum proposed that the prohibition on security guards,
government employees and supervisory employees from forming unions be
lifted. This is because the Labor Code denied this right and there was a need to
reinstate this right. The Commission accepted the proposal. Lerums proposal to
amend the Sec 8 of Art III of the Constitution by including labor union in the
guarantee of the right to organization should be taken in the context of
statements that his aim was the removal of the statutory ban against joining
labor organization. The approval of his proposal can only mean that the
Commission intended the absolute right to organize of government workers,
supervisory employees and security guards to be constitutionally guaranteed.
By implication, NO similar absolute constitutional right to organize for labor
purposes should be deemed to have been granted to top-level and middle-
managers. As to them, the right to self-organization may be regulated and even
abridged conformably to Art III, Sec 8. (Thus the Lerum proposal was never
intended to apply to managerial employees. He simply meant to restore the
right of supervisory employees to organize. )
74. VISAYAN REFINING VS. CAMUS AND PAREDES
eminent domain

The Gov-Gen directed the Atty-Gen to cause the condemnation proceedings
for expropriation of the land known as Camp Thomas Claudio in Pque for
military and aviation purposes.
Among the defendants impleaded were Visayan Refining and Worcester
Leas, who were owners of the different portions of the property.
The govt filed a case to ask the court to give the government the possession
of the land after a DEPOSIT is made (so that there can be immediate
possession). The Judge fixed a provisional value and granted the govt
petition.
The affected parties quested the validity of the proceedings on the ground
that there is no law authorizing the exercise of eminent domain just to
acquire land ffor military or aviation purposes. They also contend that the
P600,000 deposit was made without authority of law.
The govt argues that the money comes from an appropriation made by law,
Act #2784 for the use of the Militia Commission.
The govt further argues that the power of eminent domain is expressly
conferred on the Government by the Philippine Bill (which was the
constitution then I think.), as well as the Jones Act and the Administrative
Code.

ISSUE: Can expropriation proceedings be maintained in the absence of a statute
authorizing the exercise of power of eminent domain? Must there be a special
legislative act for every parcel of land expropriated?

SC. NO NEED FOR SPECIAL LEGISLATION.
If land can be taken by the Government for a public use at all, the use intended
to be made of the land now in question, that is, for military and aviation
purposes, is a public use. It is undeniable that a military establishment is
essential to the maintenance of organized society, and the courts will take
judicial notice of the recent progress of the military and naval arts resulting
from the development of aeronautics.

Updated by K Pascual. 2014. 4C. Bill of Rights Poli Digests. Atty. Jack Jimenez. 4C. 110


Expropriation proceedings may be maintained upon the exclusive initiative of
the Governor-General, without the aid of any special legislative authority other
than that already on the statute books. Furthermore, if the Government
complies with the requirements of law relative to the making of a deposit in
court, provisional possession of the property may be at once given to it, just as
is permitted in the case of any other person or entity authorized by law to
exercise the power of eminent domain. Special legislative authority for the
buying of a piece of land by the Government is no more necessary than for
buying a paper of pins; and in the case of a forced taking of property against the
will of the owner, all that can be required of the government is that it should be
able to comply with the conditions laid down by law as and when those
conditions arise.

It is recognized by all writers that the power of eminent domain is inseparable
from sovereignty being essential to the existence of the State and inherent in
government even in its most primitive forms. No law, therefore, is ever
necessary to confer this right upon sovereignty or upon any government
exercising sovereign or quasi-sovereign powers.

The power of eminent domain does not depend for its existence on a specific
grant in the constitution. It is inherent in sovereignty and exists in a sovereign
state without any recognition of it in the constitution. The provisions found in
most of the state constitutions relating to the taking of property for the public
use do not by implication grant the power to the government of the state, but
limit a power which would otherwise be without limit.

Once authority is given to exercise the power of eminent domain, the matter
ceases to be wholly legislative. The executive authorities may then decide
whether the power will be invoked and to what extent. The power of eminent
domain, with respect to the conditions under which the property is taken, must
of course be exercised in subjection to all the restraints imposed by
constitutional or organic law. The two provisions by which the exercise of this
power is chiefly limited in this jurisdiction are found in the third section of the
Jones Act, which among other things declares (1) that no law shall be enacted
which shall deprive any person of property without due process of law and (2)
that private property shall not be taken for public use without just
compensation. The whole problem of expropriation is resolvable in its ultimate
analysis into a constitutional question of due process of law.

The method of expropriation adopted in this jurisdiction is such as to afford
absolute assurance that no piece of land can be finally and irrevocably taken
from an unwilling owner until compensation is paid.

It will be noted that the title does not actually pass to the expropriator until a
certified copy of the record of the judgment is recorded in the office of the
register of deeds. Before this stage of the proceedings is reached the
compensation is supposed to have been paid; and the court is plainly directed to
make such final order and judgment as shall secure to the defendant just
compensation for the land taken.

The idea is rather to protect the owner by requiring payment as a condition
precedent to the acquisition of the property by the other party. The owner is
completely protected from the possibility of losing his property without
compensation.

In the eventuality that the expropriation shall not be consummated, the owners
will be protected by the deposit from any danger of loss resulting from the
temporary occupation of the land by the Government; for it is. obvious that this
preliminary deposit serves the double purpose of a prepayment upon the value
of the property, if finally expropriated, and as an indemnity against damage in
the eventuality that the proceedings should fall of consummation.

(the court also reiterated the fact that there was really money appropriated for
the purposes of the Militia Commission nder Act no 2826 and 2715.
75. CITY OF MANILA VS. CHINESE COMMUNITY
Updated by K Pascual. 2014. 4C. Bill of Rights Poli Digests. Atty. Jack Jimenez. 4C. 111


eminent domain

The City of Manila filed a petition for the expropriation of certain lands in
the district of Binondo for the purpose of constructing the extension of Rizal
Avenue in Manila.
The Comunidad de Chinos de Manila claimed that it was the owner of
certain lands and that there was NO necessity or expediency of
expropriation for street purposes, and that there were existing roads which
sufficient, that there are other routes available for expropriation which
would not disturb the resting places of the dead (because the land is being
used as a cemetery).
They also claim that the land had become quasi-public property because it
has been used for the burial of the dead for many years and should not be
converted to a street.
The lower court ruled in favor of the Chinese Community.
The City appealed and claim that under the law, it had authority to
expropriate and it may expropriate any land it may desire, and that the only
function of the lower court should have been to ascertain the value of the
land, and neither can the court nor the owners question the expropriation
itself, and that the courts are mere appraisers of the land and fix the value.

ISSUE: What is the function of the courts in expropriation? Is it limited to those
above?
SC: The power of the court is not limited to that question. The right of
expropriation is not an inherent power in a municipal corporation, and before it
can exercise the right some law must exist conferring the power upon it. When
the courts come to determine the question, they must not only find (a) that a
law or authority exists for the exercise of the right of eminent domain, but (b)
also that the right or authority is being exercised in accordance with the law. In
the present case there are two conditions imposed upon the authority conceded
to the City of Manila: First, the land must be private; and, second, the purpose
must be public. If the court, upon trial, finds that neither of these conditions
exists or that either one of them fails, certainly it cannot be contended that the
right is being exercised in accordance with law.
Whether the purpose for the exercise of the right of eminent domain is public, is
a question of fact. Whether the land is public or private is also a question of fact;
and, in our opinion, when the legislature conferred upon the courts of the
Philippine Islands the right to ascertain upon trial whether the right exists for
the exercise of eminent domain, it intended that the courts should inquire into,
and hear proof upon, those questions.

The city of Manila is given authority to expropriate private lands for public
purposes. The legislature, in providing for the exercise of the power of eminent
domain, may directly determine the necessity for appropriating private
property for a particular improvement for public use, and it may select the exact
location of the improvement. In such a case, it is well settled that the utility of
the proposed improvement, the extent of the public necessity for its
construction, the expediency of constructing it, the suitableness of the location
selected and the consequent necessity of taking the land selected for its site, are
all questions exclusively for the legislature to determine, and the courts have no
power to interfere, or to substitute their own views for those of the
representatives of the people.

When the statute does not designate the property to be taken nor how much
may be taken, then the necessity of taking particular property is a question for
the courts. Where the application to condemn or appropriate is made directly to
the court, the question (of necessity) should be raised and decided in limine.

In this case, the law merely conferred -general authority to take land for public
use when a necessity exists therefor. Under such statute, the allegation of the
necessity for the appropriation is an issuable allegation which it is competent
for the courts to decide.

Updated by K Pascual. 2014. 4C. Bill of Rights Poli Digests. Atty. Jack Jimenez. 4C. 112


Whether it was wise, advisable, or necessary to confer upon a municipality the
power to exercise the right of eminent domain, is a question with which the
courts are not concerned. But when that right or authority is exercised for the
purpose of depriving citizens of their property, the courts are authorized, in this
jurisdiction, to make inquiry and to hear proof upon the necessity in the
particular, case, and not the general authority.
It is erroneous to suppose that the legislature is beyond the control of the courts
in exercising the power of eminent domain, either as to the nature of the use or
the necessity to the use of any particular property. For if the use be not public or
no necessity for the taking exists, the legislature cannot authorize the taking of
private property against the will of the owner, notwithstanding compensation
may be required.

The taking of private property for any use which is not required by the
necessities or convenience of the inhabitants of the state, is an unreasonable
exercise of the right of eminent domain, and beyond the power of the legislature
to delegate. The question whether any particular use is a public one or not is
ultimately, at least, a judicial question.

The Charter of the city of Manila authorizes the taking of private property for
public use. Suppose the owner of the property denies and successfully proves
that the taking The very foundation of the right to exercise eminent domain
is a genuine necessity, and that necessity must be of a public character The
ascertainment of the necessity must precede or accompany, NOT FOLLOW,
the taking of the land. If the courts can inquire into whether a public use exists
or not, then it follows that it can examine into the question of necessity.

The general power to exercise the right of eminent domain must not be
confused with the right to exercise it in a particular case. The power of the
legislature to confer, upon municipal corporations and other entities within the
State, general authority to exercise the right of eminent domain cannot be
questioned by the courts, but that general authority of municipalities or entities
must not be confused with the right to exercise it in particular instances. The
moment the municipal corporation or entity attempts to exercise the authority
conferred, it must comply with the conditions accompanying the authority. The
necessity for conferring the authority upon a municipal corporation to exercise
the right of eminent domain is admittedly within the power of the legislature.
But whether or not the municipal corporation or entity is exercising the right in
a particular case under the conditions imposed by the general authority, is a
question which the courts have the right to inquire into.

It is a well known fact that cemeteries may be public or private. The former is a
cemetery used by the general community, or neighborhood, or church, while the
latter is used only by a family, or a small portion of the community or
neighborhood. Where a cemetery is open to the public, it is a public use and no
part of the ground can be taken for other public uses under a general authority.
And this immunity extends to the unimproved and unoccupied parts which are
held in good faith for future use. The cemetery in question seems to have been
established under governmental authority.

APPROPRIATION SHOULD NOT BE MADE UNTIL IT IS FULLY ESTABLISHED
THAT THERE IS A GREAT NECESSITY THEREFOR. EXPROPRIATION DENIED.

In the general acceptation of the definition of a public cemetery, it would make
the cemetery in question public property. If that is true, then, of course, the
petition of the plaintiff must be denied, for the reason that the city of Manila has
no authority or right under the law to expropriate public property.

Even granting that a necessity exists for the opening of the street in question,
there was no proof of the necessity of opening the same through the cemetery.
The record shows that adjoining and adjacent lands have been offered to the
city free of charge, which will answer every purpose of the plaintiff.


Updated by K Pascual. 2014. 4C. Bill of Rights Poli Digests. Atty. Jack Jimenez. 4C. 113


Note: if there is a law granting expropriation of specific land, for a specific
public purpose, then the court has no jurisdiction to inquire into purpose. But if
it was a general authority, then the court may inquire.

Note: Consummated sale before taking. Use land suitable at the time of taking.
76. DE KNECHT VS. BAUTISTA

Cristina De Knecht claims that more than 10 years ago, the DPWH prepared
a plan to extent EDSA to Roxas Blvd.
Being an owner of a house (along Del Pan Street) that would be affected,
she wrote President Marcos to instead adopt the original plan of using
Cuneta Avenue instead of the Del Pan Street as the extension of Edsa. The
matter was referred to Human Settlements.
The government still filed for expropriation along Del Pan (because the
DPWH seemed to insist on this).
The government made the required deposit with the PNB.
Cristina assailed the choice of the Del Pan Street claiming: The condemnor
may not choose any property it wants. Where the legislature has delegated
a power of eminent domain, the question of the necessity for taking a
particular line for the intended improvement rests in the discretion of the
grantee power subject however to review by the courts in case of fraud, bad
faith or gross abuse of discretion. The choice of property must be examined
for bad faith, arbitrariness or capriciousness and due process requires
determination as to whether or not the proposed location was proper in
terms of the public interests. As envisioned by the government, the EDSA
extension would be linked to the Cavite Expressway. Logically then, the
proposed extension must point to the south and not detour to the north.
The government on the other hand claimed: the residents of Del Pan and
Fernando Rein Streets who were to be adversely affected were duly notified
of such proposed project., that both lines, Cuneta Avenue and Del Pan -
Fernando Rein Streets lines, meet satisfactorily planning and design criteria
and therefore are both acceptable. In selecting the Del Pan-Fernando Rein
Streets lines the Government did not do so because it wanted to save the
motels located along Cuneta Avenue but because it wanted to minimize the
social impact factor or problem involved."

SC: DEL PAN ROUTE NOT ALLOWED.
it is a fact that the Department of Public Highways originally establish the
extension of EDSA along Cuneta Avenue. It is to be presumed that the
Department of Public Highways made studies before deciding on Cuneta
Avenue. It is indeed odd why suddenly the proposed extension of EDSA to Roxas
Boulevard was changed to go through Fernando Rein-Del Pan Streets which the
Solicitor General concedes the Del Pan-Fernando Rein Streets line follows
northward and inward direction. While admitting "that both lines, Cuneta
Avenue and Del Pan-Fernando Rein Streets lines, meet satisfactorily planning
and design criteria and therefore are both acceptable x x x", the Solicitor
General justifies the change to Del Pan-Fernando Rein Streets on the ground
that the government "wanted to minimize the social impact factor or problem
involved."
It is doubtful whether the extension of EDSA along Cuneta Avenue can be
objected to on the ground of social impact. The improvements and buildings
along Cuneta Avenue to be affected by the extension are mostly motels.

As found by the Human Settlements: From both engineering and traffic
management viewpoints, it is incontestable that the straighter and shorter
alignment is preferable to one which is not. Alignment 1 is definitely the
contour conforming alignment to EDSA whereas Alignment 2 affords a greater
radius of unnatural curvature as it hooks slightly northward before finally
joining with Roxas Boulevard. It is clear that the choice of Fernando ReinDel Pan
Streets as the line through which the Epifanio de los Santos Avenue should be
extended to Roxas Boulevard is arbitrary and should not receive judicial
approval. The respondent judge committed a grave abuse of discretion in
allowing the Republic of the Philippines to take immediate possession of the
properties sought to be expropriated.
Updated by K Pascual. 2014. 4C. Bill of Rights Poli Digests. Atty. Jack Jimenez. 4C. 114



(read original for the report of the Human Settlements)

77. CITY OF BAGUIO VS. NAWASA
eminent domain

Baguio City operates the Baguio Waterworks System BWS under its charter.
Baguio City filed a complaint against Nawasa contending that the latter does
not include the Baguio Waterworks System. It assailed the constitutionality
of RA 1383 (giving Nawasa the power over the BWS), claiming that the law
deprives Baguio City of ownership of the waterworks system without
compensation.
The Nawasa on the other hand claims that RA 1383 is but an exercise of
police powers, and that the BWS is not a private property but a public
works for a public service.
Under RA 1383, Nawasa owns and or has jurisdiction, supervision and
control over all existing government-owned waterworks in the boundaries
of cities, municipalities and municipal districts in the Philippines including
those served by the waterworks and wells and drills sections.
Furthermore, under said law, all existing government owned waterworks
and sewerage systems in cities, municipalities, and municipal districts, are
transferred to the Nawasa.

ISSUE: Is RA 1383 an exercise of police power (pp) or power of eminent
domain (ed)?
SC: ED.
The Act does not confiscate, nor destroy, nor appropriate property belonging to
appellee. It merely directs that all waterworks belonging to cities, municipalities
and municipal districts in the Philippines be transferred to the NAWASA for the
purpose of placing them under the control and supervision of one agency with a
view to promoting their efficient management, but in so doing it does not
confiscate them because it directs that they be paid with an equal value of the
assets of the NAWASA. Here, , its purpose is to effect a real transfer of the
ownership of the waterworks to the new agency and does not merely
encompass a transfer of administration.

ISSUE: Is BWS public property or patrimonial property of the city?
SC: PATRIMONIAL.
The BWS is not like any public road, park, street or other public property held in
trust by a municipal corporation for the benefit of the public but it is rather a
property owned by the city in its proprietary character. While the cases may
differ as to the public or private character of waterworks, the weight of
authority as far as the legislature is concerned classes them as private affairs.

A waterworks system is patrimonial property of the city that has established it.
Being owned by a municipal corporation in a proprietary character, waterworks
cannot be taken away without observing the safeguards set by our Constitution
for the protection of private property.

ISSUE: Does RA 1383 comply with expropriation requirements?
SC: NO. Because there was no payment of just compensation. Although the law
said that the assets of Nawasa are to be used as payment, these assets have not
been specified. Also, no action was done by Nawasa to contract indebtedness or
issue bonds as payment. Nawasa has not complied with its commitment to pay.

CHAMP Page 114 9/16/2014
Updated by K Pascual. 2014. 4C. Bill of Rights Poli Digests. Atty. Jack Jimenez. 4C. 115



78. CITY OF MANILA VS. ESTRADA
ED just compensation

The city of Manila sought to expropriate an entire parcel of land with its
improvements for use in connection with a new market at that time being
erected in the district of Paco. The land was bounded by Calle Herran, the
Paco Estero, the market site, and Calle Looban.
The commissioners tasked to determine the compensation amount
reported P20 per square meter.
The lower court however, reduced this to P15 per square meter.
It appears that two disinterested witnesses for the city testified that the
land was worth P10 per square meter, their statements being based upon
the prices obtained for land in the open market in the vicinity. The
defendant Estrada testified that it was worth P25 per square meter, basing
his statement on the price obtained three years previously by the owner of
the parcel on the opposite side of Calle Herran of P19.85 per square meter.

ISSUE: How much is just compensation?
SC: For the determination of the market value of land, which is that sum of
money which a person, desirous but not compelled to buy and an owner willing
but not compelled to sell, would agree on as a price to be given and received
therefore. The market value of a piece of land is attained by a consideration of
all those facts which make it commercially valuable.

In determining the value of land appropriated for public purposes, the same
considerations are to be regarded as in a sale of property between private
parties. The inquiry in such cases must be what is the property worth in the
market, viewed not merely with reference to the uses to which it is at the time
applied, but with reference to the uses to which it is plainly adapted; that is to
say, what is it worth from its availability for valuable uses? It is to be estimated
by reference to the uses for which the property is suitable, having regard to the
existing business or wants of the community, or such as may be reasonably
expected in the immediate future."

The market value of the property means its actual value, independent of the
location of plaintiff's road thereon, that is, the fair value of the property as
between one who wants to purchase and one who wants to sell it; not what
could be obtained for it in peculiar circumstances when greater than its fair
price could be obtained; not its speculative value; not the value obtained
through the necessities of another. Nor, on the other hand, is it to be limited to
that price which the property would bring when forced off at aucti.on under the
hammer. The question is, if the defendant wanted to sell its property, what
could be obtained for it upon the market from parties who wanted to buy and
Would give its full value."

The market value of property is the price which it will bring when it is offered
for sale by one who desires, but is not obliged to sell it, and is bought by one
who is under no necessity of having it. In estimating its value all the capabilities
of the property, and all the uses to which it may be applied or for which it is
adapted, are to be considered, and not merely the condition it is in at the time
and the use to which it is then applied by the owner. It is not a question of the
value of the property to the owner. Nor can the damages be enhanced by his
unwillingness to sell. On the other hand, the damages cannot be measured by
the value of the property to the party condemning it, nor by its need of the
particular property. All the facts as to the condition of the property and its
surroundings, its improvements and capabilities, may be shown and considered
in estimating its value."

(read original for the specific objections on the presented evidence of just
compensation)

Updated by K Pascual. 2014. 4C. Bill of Rights Poli Digests. Atty. Jack Jimenez. 4C. 116


SC: THERE WAS JUST COMPENSATION AS IT WAS BASED OF THE FAIR
MARKET VALUE:
1) price paid for the adjoing lots
2) there were 2 real estate agents
3) tax assessments of the value of the land considered.
79. EPZA VS. DULAY
just compensation

PD1811 reserved certain parcels of land in LapuLapu Mactan Cebu, for the
establishment of EPZA and the Mactan EPZ. It however encroached upon
the property of Dulay.
The parties have agreed to the expropriation except for the just
compensation.
EPZA contends that under PD1533, the basis of JC should be the fair and
current market value as declared by the owner, or the market value
determined by the assessor, whichever is lower. Thus, it claims that there is
no more need to appoint commissioners under the Rules of Court, to
determine just compensation. It also claims that local assessors and the
owners themselves may fix the JC.
On the other hand, Dulay relies on the commissioners report fixing the JC at
P15 per sqm.

ISSUE: What is the proper way to determine the JC?
SC:
JC means the equivalent for the value of the property at the time of its taking.
Anything beyond that is more and anything short of that is less, than just
compensation. It means a fair and full equivalent for the loss sustained, which is
the measure of the indemnity, not whatever gain would accrue to the
expropriating entity.

Hence, in estimating the market value, all the capabilities of the property and all
the uses to which it may be applied or for which it is adapted are to be
considered and not merely the condition it is in the time and the use to which it
is then applied by the owner. All the facts as to the condition of the property and
its surroundings, its improvements and capabilities may be shown and
considered in estimating its value.

PD1533 IS NOT VALID. The method of ascertaining just compensation under the
aforecited decrees constitutes impermissible encroachment on judicial
prerogatives. It tends to render this Court inutile in a matter which under the
Constitution is reserved to it for final determination.

Although in an expropriation proceeding the court technically would still have
the power to determine the just compensation for the property, if we follow the
PD, then the court would be relegated to simply stating the lower value of the
property as declared either by the owner or the assessor. As a necessary
consequence, it would be useless for the court to appoint commissioners under
Rule 67 of the Rules of Court. Moreover, the need to satisfy the due process
clause in the taking of private property is seemingly fulfilled since it cannot be
said that a judicial proceeding was not had before the actual taking. However,
the strict application of the decrees during the proceedings would be nothing
short of a mere formality or charade as the court has only to choose between
the valuation of the owner and that of the assessor, and its choice is always
limited to the lower of the two. The court cannot exercise its discretion or
independence in determining what is just or fair. Even a grade school pupil
could substitute for the judge insofar as the determination of constitutional just
compensation is concerned.
COURTS STILL HAVE THE POWER TO DETERMINE THE JC.

Updated by K Pascual. 2014. 4C. Bill of Rights Poli Digests. Atty. Jack Jimenez. 4C. 117


The owner of property expropriated is entitled to recover from expropriating
authority the fair and full value of the lot, as of the time when possession
thereof was actually taken by the province, plus consequential damages-
including attorney's fees-from which the consequential benefits, if any should
be deducted, with interest at the legal rate, on the aggregate sum due to the
owner from and after the date of actual taking.' In fine, the decree only
establishes a uniform basis for determining just compensation which the Court
may consider as one of the factors in arriving at 'just compensation,' as envisage
in the Constitution.

Indeed, where this Court simply follows PD 1533, thereby limiting the
determination of just compensation on the value declared by the owner or
administrator or as determined by the Assessor, whichever is lower, it may
result in the deprivation of the landowner's right of due process to enable it to
prove its claim to just compensation, as mandated by the Constitution. The
valuation in the decree may only serve as a guiding principle or one of the
factors in determining just Compensation but it may not substitute the court's
own judgment as to what amount should be awarded and how to arrive at such
amount.

THE COURT NOW ABANDONS THE NHA VS. REYES DOCTRINE.

The basic unfairness of the decrees is readily apparent.
Just compensation means the value of the property at the time of the taking. It
means a fair and full equivalent for the lose sustained. All the facts as to the
condition of the property and its surroundings, its improvements and
capabilities, should be considered.
In this particular case, the tax declarations presented by the petitioner as basis
for just compensation were made by the LapuLapu municipal, later city assessor
long before martial law, when land was not only much cheaper but when
assessed values of properties were stated in figures constituting only a fraction
of their true market value. The private respondent was not even the owner of
the properties at the time. It purchased the lots for development purposes. To
peg the value of the lots on the basis of documents which are out of date and at
prices below the acquisition cost of present owners would be arbitrary and
confiscatory.

Tthe values given by provincial assessors are usually uniform for very wide
areas covering several barrios or even an entire town with the exception of the
poblacion. Individual differences are never taken into account. The value of land
is based on such generalities as its possible cultivation for rice, corn, coconuts,
or other crops.

To say that the owners are estopped to question the valuations made by
assessors since they had the opportunity to protest before the local board of
appeals is illusory. The overwhelming mass of land owners accept
unquestioningly what is found in the tax declarations prepared by local
assessors or municipal clerks for them. They do not even look at, much less
analyze, the statements.

The determination of "just compensation" in eminent domain cases is a judicial
function. The executive department or the legislature may make the initial
determinations but when a party claims a violation of the guarantee in the Bill
of Rights that private property may not be taken for public use without just
compensation, no statute, decree, or executive order can mandate that its own
determination shall prevail over the court's findings. Much less can the courts
be precluded from looking into the "just-ness" of the decreed compensation.

We, therefore, hold that P.D. No. 1533, which eliminates the court's discretion to
appoint commissioner a pursuant to Rule 67 of the Rules of Court, is
unconstitutional and void.

CHAMP Page 117 9/16/2014
Updated by K Pascual. 2014. 4C. Bill of Rights Poli Digests. Atty. Jack Jimenez. 4C. 118


80. VICENTE NOBLE VS. CITY OF MANILA
eminent domain

Syquia entered into a contract with the City of Manila for the construction
of a school building on the land of the City in Tayuman, Tondo.
Under the contract, Syquia will lease the building to the City and after 3
years, the City will buy the building from Syquia for P46000. But if after 3
years the City is still unable to buy it, then the conrtact of lease will be
extended, and so on successively.
Syquia however conveyed his rights to the building to Sandoval, as well as
the lease contract.
Sandoval then conveyed his rights to Noble.
The City failed to pay the rent and Noble filed a case to compel the City to
just buy the building as per the lease contract.
The City however wanted the rescission of the lease contract and instead
asked for expropriation. (I think this was their strategy to avoid paying the
fixed price in the contract.) The city wanted the appointment of
commissioners to determine the JC for the building.

ISSUE: Was expropriation proper?
SC: NO. The city of Manila has no right to expropriate the building. It should
comply with the terms of the lease contract of paying P46,000 for the price of
the building, plus rent.
Under the contract, the city had to buy the building within 3 years. The
purchase constituted the principal consideration with respect to Syquia, the
lease being merely secondary because it subsists only while there was no
purchase yet. If the purchase is not made after the 3 years, then Syquia can
demand the city to comply with its obligation to buy the building.

When the rights were conveyed to other persons, there was only an amendment
of the period of lease (extension only). But the obligation itself of the city to buy
the building remains. Despite the amendment of the original contract, the
obligation of the city to purchase the building was kept alive, although not
necessarily within the first three years of its occupancy. The defendant itself has
acknowledged this obligation wherein it was stated that the lease was
renewable from year to year until the leased building is purchased pursuant to
the original contract.

The city being bound to buy the building at an agreed price, under a valid and
subsisting contract, and the plaintiff being agreeable to its sale, the
expropriation thereof, as sought by the defendant, is baseless. Expropriation lies
only when it is made necessary by the opposition of the owner to the sale or by
the lack of any agreement as to the price. There being in the present case a valid
and subsisting contract, between the owner of the building and the city, for the
purchase thereof at an agreed price, there is no reason for the expropriation.
Expropriation, as a manifestation of the right of eminent domain of the state and
as a limitation upon private ownership, is based upon the consideration that it
should not be an obstacle to human progress and to the development of the
general welfare of the community. In the circumstances of the present case,
however, the expropriation would depart from its own purposes and turn out to
be an instrument to repudiate compliance with obligations legally and validly
contracted.


CHAMP Page 118 9/16/2014
81. UNITED STATES VS. CAUSBY
eminent domain

Updated by K Pascual. 2014. 4C. Bill of Rights Poli Digests. Atty. Jack Jimenez. 4C. 119


Respondents own 2.8 acres of land near the airport of North Carolina. They
had a house and other buildings for raising chickens.
Near them was the path of the runway, used by the US government under a
lease contract. (this was during WW2, so US fighter planes and bombers use
the airport).
Sometimes, the fighter planes would come close enough to miss the top of
the trees. The noise is startling. At night, the glare from the planes would
brighten the place. As a result the respondents had to give up their chicken
business.
150 chickens were lost.
Thus the property depreciated in value and.
The US relies on the Air Commerce Act of 1926, claiming that the US has
complete and exclusive national sovereignty over the air space in the
country. It claims that navigable air space is the airspace above the
minimum space altitudes of flight prescribed by the Civil Aeronautics
Authority.
It is argued that such navigable airspace is subject to right of interstate and
foreign air navigation. Now the US claims that these flights were an exercise
of this right to travel in airspace. The US claims that the flight are made
within the navigable airspace without any physical invasion of the property
of landowners. There has been no taking of property but merely incidental
damage resulting from authorized air navigation. It is also argued that
landowner does not own the airspace above.

SC: The flights over the property rendered it inhabitable, thus there is
compensable taking of private property under the 5
th
Am.
It is the owners loss, not the takers gain, which is the measure of the value of
the property taken.

Because of the frequency and altitude of the flights, the respondents could not
use their land for any purpose, therefore, their loss would be complete. It is as
complete as if the US had entered upon the surface of the land and taken
exclusive possession thereof. The land is appropriated as if it were used for the
runways themselves.

THUS there would be taking. Though it would only be an easement of flight
which was taken, that easement, if permanent and not merely temporary
normally would be equivalent to a fee. It would be a definite exercise of
complete dominion and control over the surface of the land. The fact that the
planes never touched the land would be irrelevant because the owners right to
possess and exploit the land (his beneficial ownership) would be destroyed.

If the landowner is to have full enjoyment of the land, he must have exclusive
control of the immediate reaches of the enveloping atmosphere. Otherwise,
buildings could not be erected, trees could not be planted, and fences could not
be run. The fact that he does not occupy it ina a physical sense (by the erection
of buildings) is not material. The flight of the airplanes, which skims the surface
although not touch it, is as much as an appropriation of the use of the land as a
more conventional entry upon it.

There is an intrusion so immediate and so direct as to subtract from the owners
full enjoyment of the property. Thus, the adjacent airspace at this low altitude is
so close to the land that invasions of it affects the use of the land itself. This
invasion is the same as the invasion of the surface. It is the character of the
invasion, and not the amount of damage resulting, so long as the damage is
substantial, that determines whether there is a taking.

In short, flights over private lands are generally not a taking, unless they are so
low and so frequent as to be a direct and immediate interference to the
enjoyment and use of land.

Updated by K Pascual. 2014. 4C. Bill of Rights Poli Digests. Atty. Jack Jimenez. 4C. 120


(But since it is not clear yet whether the taking is permanent or temporary, then
the case is remanded to the lower court before the amount of JC is determined.)
CHAMP Page 120 9/16/2014

82. REPUBLIC VS. CASTELLVI
eminent domain elements of taking.

Republic filed for eminent domain against Castellvi, over a parcel of land
Floridablanca, Pampanga.
Republic alleged, that the fair market value according to the Committee on
Appraisal for the Province of Pampanga, was not more than P2,000 per
hectare, or a total market value of P259,669.10.
Castellvi alleged, that being a residential land, it had a fair market value of
P15.00 per square meter, so it had a total market value of P11,389,485.00;
that the Republic, through the Armed Forces of the Philippines, particularly
the Philippine Air Force, had been, despite repeated demands, illegally
occupying her property, preventing her from using and disposing of it, thus
causing her damages by way of unrealized profits.
Republic argues that the "taking" should be reckoned from the year 1947
when by virtue of a special lease agreement between the Republic and
Castellvi, the former was granted the "right and privilege" to buy the
property should the lessor wish to terminate the lease, and that in the event
of such sale, it was stipulated that the fair market value should be as of the
time of occupancy; and that the permanent improvements amounting to
more than half a million pesos constructed during a period of twelve years
on the land, subject of expropriation, were indicative of an agreed pattern of
permanency and stability of occupancy by the Philippine Air Force in the
interest of national security.
Castellvi, on the other hand, maintains that the 'taking" of property under
the power of eminent domain requires two essential elements, to wit: (1)
entrance and occupation by condemnor upon the private property for more
than a momentary or limited period, and (2) devoting it to a public use in
such a way as to oust the owner and deprive him of all beneficial enjoyment
of the property. This appellee argues that the first element is wanting, for
the contract of lease relied upon provides for a lease from year to year; that
the second element is also wanting, because the Republic was paying the
lessor Castellvi a monthly rental of P445.58; and that the contract of lease
does not grant the Republic the "right and privilege" to buy the premises "at
the value at the time of occupancy.

SC:
'Taking' under the power of eminent domain may be defined generally as
1) entering upon private property for
2) more than a momentary period, and,
3) under the warrant or color of legal authority,
4) devoting it to a public use, or otherwise informally appropriating or
injuriously affecting it 5) substantially to oust the owner and deprive him of all
beneficial enjoyment thereof.

1) YES. This element is present in this case when by virtue of the lease, the
Republic (AFP), took possession of the Castellvi property.

2) NO. Momentary should be construed to mean a limited period, not indefinite
or permanent. Here, there was a lease contract for 1 year, renewable. The entry
on the property under lease is temporary and transitory. The fact the Republic
constructed some installations does not alter the fact that the entry into the
land was only momentary or transitory, intended to last only a year, even if
renewable. Even if there was a seeming intention to be permanent such
cannot prevail over the clear and express terms of the lease contract. If it was
Updated by K Pascual. 2014. 4C. Bill of Rights Poli Digests. Atty. Jack Jimenez. 4C. 121


really intended to be more than momentary, then why would the contract of
lease be from year to year basis?

3) YES. There was entry under the warrant or color of legal authority since it
was the republic who entered the property as a lessee.

4) YES. Property was devoted to public use because it was used by the AFP-Air
Force

5) NO. The entry of the republic into the property does NOT oust Castellvi or
deprive him of the use of the property. Castellvi remained the owner and was
continuously recognized as the owner, as shown by the yearly lease contract.
Republic has also been paying its monthly rentals until the time when it filed for
eminent domain.

It is thus clear that the taking of the Castellvi property for purposes of eminent
domain cannot be considered to have taken place in 1947, when the Republic
merely commenced to occupy the property as lessee. Thus the JC should be
determined as of the date of filing of petition for ED, not as of 1947 when the
Republic first occupied it as lessee.


ISSUE: The Republic claims that there was a right to buy the property at the
value it had at the time of first occupation as lessee in 1947.
SC: NO. What was expressly agreed in the lease agreement is that should the
lessor require the lessee to return the premises, then the lessee would have the
right and privilege of paying the lessor what it would fairly cost to put the
premises in the same condition as it was at the commencement of the lease. The
"fair value" at the time of occupancy, mentioned in the lease agreement, does
not refer to the value of the property if bought by the lessee, but refers to the
cost of restoring the property in the same condition as of the time when the
lessee took possession of the property.

Just compensation" is to be determined as of the date of the filing of the
complaint. This Court has ruled that when the taking of the property sought to
be expropriated coincides with the commencement of the expropriation
proceedings, or takes place subsequent to the filing of the complaint for eminent
domain, the just compensation should be determined as of the date of the filing
of the complaint. In the instant case, it is undisputed that the Republic was
placed in possession of the Castellvi property, by authority of the court, on
August 10, 1959. The "taking" of the Castellvi property for the purposes of
determining the just compensation to be paid must, therefore, be reckoned as of
June 26, 1959 when the complaint for eminent domain was filed.
83. HEIRS OF ARDONA VS. REYES
eminent domain

The Phil Tourism Authority filed 4 complaints for the expropriation of land
in Cebu City for the development of a resort complex for tourism
The PTA has already deposit 10% of the amount and so it wanted
immediate possession of the land.
The main contention was that the expropriation is constitutionally infirm
because nowhere in the constitution can a provision be found which allows
the taking of private property for the promotion of tourism. They argue that
it would disregard the land reform nature of the property being
expropriated.
They invoke a strict construction that "public use" means literally use by
the public and that "public use" is not synonymous with "public interest",
"public benefit", or "public welfare" and much less "public convenience."
Updated by K Pascual. 2014. 4C. Bill of Rights Poli Digests. Atty. Jack Jimenez. 4C. 122


They contend that the promotion of tourism is not "public use" because
private concessioners would be allowed to maintain various facilities such
as restaurants, hotels, stores, etc. inside the tourist complex.
Finally, they also rely on the Land Reform Program that the properties
subject of expropriation may not be taken since they are within the
coverage of "operation land transfer" under the land reform program and
that the agrarian reform program occupies a higher level in the order of
priorities than other State policies like those relating to the health and
physical well-being of the people

ISSUE: Is expropriation for tourism public use?

SC: YES. Expropriation valid.
The petitioners look for the word "tourism" in the Constitution. Understandably
the search would be in vain. The policy objectives of the framers can be
expressed only in general terms such as social justice, local autonomy,
conservation and development of the national patrimony, public interest, and
general welfare, among others. The particular mention in the Constitution of
agrarian reform and the transfer of utilities and other private enterprises to
public ownership merely underscores the magnitude of the problems sought to
be remedied by these programs. They do not preclude nor limit the exercise of
the power of eminent domain for such purposes like tourism and other
development programs.

The constitutional restraints are public use and just compensation. First, their
contention which is rather sweeping in its call for a retreat from the public
welfare orientation is unduly restrictive and outmoded. Second, no less than the
lawmaker has made a policy determination that the power of eminent domain
may be exercised in the promotion and development of Philippine tourism.

There can be no doubt that expropriation for such traditional purposes as the
construction of roads, bridges, ports, waterworks, schools, electric and
telecommunications systems, hydroelectric power plants, markets and
slaughterhouses, parks, hospitals, government office buildings, and flood
control or irrigation systems is valid. However, the concept of public use is not
limited to traditional purposes. Here as elsewhere the idea that "public use" is
strictly limited to clear cases of "use by the public" has been discarded.

Private bus firms, taxicab fleets, roadside restaurants, and other private
businesses using public streets and highways do not diminish in the least bit the
public character of expropriations for roads and streets. The lease of store
spaces in underpasses of streets built on expropriated land does not make the
taking for a private purpose. Airports and piers catering exclusively to private
airlines and shipping companies are still for public use. The expropriation of
private land for slum clearance and urban development is for a public purpose
even if the developed area is later sold to private homeowners, commercial
firms, entertainment and service companies, and other private concerns.

The petitioners also have failed to show that the area being developed is indeed
a land reform area and that the affected persons have emancipation patents and
certificates of land transfer.
84. MANOTOK VS. NHA
eminent domain

The president instituted a nationwide slum improvement and resettlement
program. One such program under the Zonal Improvement Program was
the Tambunting Estate in Manila.
However, a fire razed almost the entire estate. Following this calamity, the
President made an announcement that the national government would
acquire the property for the benefit of the fire victims.
Updated by K Pascual. 2014. 4C. Bill of Rights Poli Digests. Atty. Jack Jimenez. 4C. 123


The President issued PD 1669 declaring the Tambunting Estate
expropriated, directing the NHA to administer the land
Manotok, one of the owners of the Tambunting Estate questioned the PD.
Although there was already a deposit of P5M, she did not yet withdraw the
amount nor surrender her titles over the property.
She contends that the Presidential Decrees providing for the direct
expropriation of the properties in question violate their constitutional right
to due process and equal protection of the law because by the mere passage
of the said decrees their properties were automatically expropriated and
they were immediately deprived of the ownership and possession thereof
without being given the chance to oppose such expropriation or to contest
the just compensation to which they are entitled.
They argue that the government must first have filed a complaint with the
proper court under Rule 67 of the Revised Rules of Court in order to fulfill
the requirements of due process. They contend that the determination of
just compensation should not have been vested solely with the City
Assessor and that a maximum or fixed amount of compensation should not
have been imposed by the said decrees. Petitioners likewise state that by
providing for the maximum amount of just compensation and by directing
the City Assessor to take into consideration the alleged existing conditions
of the properties in question, namely: that no "improvement has been
undertaken on the land and that the land is squatted upon by resident
families which should considerably depress the expropriation costs," the
City Assessor is forced to accept, as actual and existing conditions of the
property, the foregoing statements in the decrees when in fact the Sunog-
Apog area has been subdivided into subdivision lots and leased to the
occupants thereof under contracts of lease, making them lessees and not
squatters as assumed by Presidential Decree No. 1670.
The government on the other hand argued, that the power of eminent
domain is inherent in the State and when the legislature itself or the
President through his lawmaking prerogatives exercises this power, the
public use and public necessity of the expropriation, and the fixing of the
just compensation become political in nature, and the courts must respect
the decision of the law-making body, unless the legislative decision is
clearly and evidently arbitrary, unreasonable, and devoid of logic and
reason; and that all that is required is that just compensation be determined
with due process of law which does not necessarily entail judicial process.
(the government argues that it was a political question)
The government also argued that the PD supersedes the Rules of Court
insofar as the procedure for expropriation is concerned.

SC: PD UNCONSTITUTIONAL.
The Tambunting subdivision is summarily proclaimed a blighted area and
directly expropriated by decree without the slightest semblance of a hearing or
any proceeding whatsoever. The expropriation is instant and automatic to take
effect immediately upon the signing of the decree. No deposit before taking is
required under the decree. The P3,400,000.00 appropriated from the general
fund is not a deposit but constitutes an installment payment for the property,
the maximum price of which is fixed so as not to exceed P17,000,000.00. There
is no provision for any interests to be paid on the unpaid installments spread
out over a period of five years. Not only are the owners given absolutely no
opportunity to contest the expropriation, plead their side, or question the
amount of payments fixed by decree, but the decisions, rulings, orders, or
resolutions of the NHA are expressly declared as beyond the reach of judicial
review. An appeal may be made to the Office of the President but the courts are
completely enjoined from any inquiry or participation whatsoever in the
expropriation of the subdivision or its incidents.

Constitutionally suspect methods or authoritarian procedures cannot be the
basis for social justice. A program to alleviate problems of the urban poor which
is well studied, adequately funded, genuinely sincere, and more solidly
grounded on basic rights and democratic procedures is needed.

The due process clause cannot be rendered nugatory everytime a specific
decree or law orders the expropriation of somebody's property and provides its
own peculiar manner of taking the same. Neither should the courts adopt a
hands-off policy just because the public use has been ordained as existing by the
decree or the just compensation has been fixed and determined beforehand by a
statute.
Updated by K Pascual. 2014. 4C. Bill of Rights Poli Digests. Atty. Jack Jimenez. 4C. 124



Although due process does not always necessarily demand that a proceeding be
had before a court of law, it still mandates some form of proceeding wherein
notice and reasonable opportunity to be heard are given to the owner to protect
his property rights. We agree with the public respondents that there are
exceptional situations when, in the exercise of the power of eminent domain,
the requirement of due process may not necessarily entail judicial process. But
where it is alleged that in the taking of a person's property, his right to due
process of law has been violated, the courts will have to step in and probe into
such an alleged violation.

There is no showing whatsoever as to why the properties involved were singled
out for expropriation through decrees or what necessity impelled the particular
choices or selections. The Tambunting estate or at least the western half of the
subdivision fronting Rizal Avenue Extension is valuable commercial property. It
is located at the junction where three main city streets Converge-Rizal Avenue
from downtown Manila, Jose Abad Santos Street from Binondo, and Aurora
Boulevard leading to Retiro Street and other points in Quezon City. The Libiran
Furniture Company, alone, which fronts the entrance to Jose Abad Santos Street
is clearly a multi-million peso enterprise. It is a foregone conclusion that the
favored squatters allowed to buy these choice lots would lose no time, once it is
possible to do so, to either lease out or sell their lots to wealthy merchants even
as they seek other places where they can set up new squatter colonies. The
public use and social justice ends stated in the whereas clauses of P.D. 1669 and
P.D. 1670 would not be served thereby.

The principle of non-appropriation of private property for private purposes,
however, remains. There is no showing how the President arrived at the
conclusion that the Sunog-Apog area is a blighted community. The many
pictures submitted as exhibits by the petitioners show a well-developed area
subdivided into residential lots with either middle-income or upper class
homes. There are no squatters. The provisions of the decree on the relocation of
qualified squatter families and on the re-blocking and re-alignment of existing
structures to allow the introduction of basic facilities and services have no basis
in fact. The area is well-developed.

The decrees, do not by themselves, provide for any form of hearing or
procedure by which the petitioners can question the propriety of the
expropriation of their properties or the reasonableness of the just
compensation. Having failed to provide for a hearing, the Government should
have filed an expropriation case under Rule 67 of the Revised Rules of Court but
it did not do so.

Another infirmity from which the questioned decrees suffer is the
determination of just compensation. The maximum amount of compensation
was imposed by the decrees and these amounts were only a little more than the
assessed value of the properties in 1978 when, according to the government, it
decided to acquire said properties. The 1978 values would deprive the
petitioner of the opportunity to prove a higher value because, the actual or
symbolic taking of such properties occurred only in 1980 when the questioned
decrees were promulgated. The rule is the determination of just compensation
is reckoned either at the time of the actual taking of the government or at the
time of the judgment by the court, whichever came first.

In estimating the market value, all the capabilities of the property and all the
uses to which it may be applied or for which it is adapted are to be considered
and not merely the condition it is in at the time and the use to which it is then
applied by the owner. All the facts as to the condition of the property and its
surroundings, its improvements and capabilities may be shown and considered
in estimating its value.

The market value stated by the city assessor alone cannot substitute for the
court's judgment in expropriation proceedings. It is violative of the due process
and the eminent domain provisions of the Constitution to deny to a property
owner the opportunity to prove that the valuation made by a local assessor is
wrong or prejudiced. The statements made in tax documents by the assessor
may serve as one of the factors to be considered but they cannot exclude or
prevail over a court determination made after expert commissioners have
examined the property and all partinent circumstances are taken into account
Updated by K Pascual. 2014. 4C. Bill of Rights Poli Digests. Atty. Jack Jimenez. 4C. 125


and after the parties have had the opportunity to fully plead their cases before a
competent and unbiased tribunal. To enjoin this Court by decree from looking
into alleged violations of the due process, equal protection, and eminent domain
clauses of the Constitution is impermissible encroachment on its independence
and prerogatives.
85. FAMILARA VS. JM TUASON
eminent domain

The Barrio captain of Barrio Tatalon, in behalf on 1,500 other occupants of
the Tatalon Estate, sought to enjoin the JM Tuason Corporation from
bulldozing and fencing any portion of the Estate.
They claim that the Land Tenure Administration was already directed to
commence the expropriation proceedings of the Estate.
They rely on RA 2616, which provided that after expropriation proceedings
have been initiated, no ejectment proceedings can be made.
The corporation however argued that there is no expropriation proceeding
actually instituted before the courts yet.

SC: CORPORATION IS CORRECT.
In the absence of any proceeding for expropriation instituted before the courts,
petitioner has clearly no cause of action. it is true that Republic Act No. 2616,
insofar as it expropriated singularly a particular private property, had survived
the challenge of being discriminatory, and was declared free from constitutional
infirmity. Nevertheless, this Court has also ruled that the provision which places
a restraint upon the exercise and enjoyment by the owner of certain rights over
its property, is justifiable only if the government takes possession of the land
and is in a position to make a coetaneous payment of just compensation to its
owner.

To hold that the mere declaration of an intention to expropriate, without
instituting the corresponding proceeding therefor before the courts, with
assurance of just compensation, would already preclude the exercise by the
owner of his rights of ownership over the land, or bar the enforcement of any
final ejectment order that the owner may have obtained against any intruder
into the land, is to sanction an act which is indeed confiscatory arid therefore
offensive to the Constitution. For it must be realized that in a condemnation
case, it is from the condemnor's taking possession of the property that the
owner is deprived of the benefits of ownership such as possession, management
and disposition thereof. Before that time, the proprietary right of the owner
over his property must be recognized.

The mere filing of the condemnation proceedings for the benefit of tenants can
not by itself alone, lawfully suspend the condemnee's dominical rights, whether
of possession, enjoyment or disposition. And this is especially the case where
final and executory judgments of ejectment have been obtained against the
occupants of the property."

Here, there was no proof that expropriation proceedings have actually be
instituted and being pursued by the government. There is also no proof that the
supposed amount of P10M has been available for the alleged expropriation.
Thus, the corporation retains its rights over their property.
SECTION 10. NON-IMPAIRMENT OF CONTRACTS
85.5. PRYCE CORPORATION V. CHINA BANKING CORPORATION
G.R. 172302, 14 Feb 2014.
This case resolves conflicting decisions between two divisions. Only one
may serve as res judicata or a bar for the other to proceed. This case also
settles the doctrine as to whether a hearing is needed prior to the issuance
of a stay order in corporate rehabilitation proceedings.
Updated by K Pascual. 2014. 4C. Bill of Rights Poli Digests. Atty. Jack Jimenez. 4C. 126


The present case originated from a petition for corporate rehabilitation
filed by petitioner Pryce Corporation on July 9, 2004 with the RTC of
Makati, Branch 138.
1

The rehabilitation court found the petition sufficient in form and substance
and issued a stay order on July 13, 2004 appointing Mendoza as
rehabilitation receiver.
2
Rehabilitation plan and amended rehab plan was
submitted to Court.
RTC-Makati found petitioner Pryce Corporation "eligible to be placed
in a state of corporate rehabilitation."
5

o The disposition likewise identified the assets to be held and
disposed of by petitioner Pryce Corporation and the manner by
which its liabilities shall be paid and liquidated.
6

On February 23, 2005, respondent China Banking Corporation elevated
the case to the Court of Appeals questioning the order of RTC-Makati as
rehab court. Specifically the following terms:
o The indebtedness to China Banking Corporation and BPI as well
as the long term commercial papers will be paid through a dacion
en pago of developed real estate assets of the petitioner.
All accrued penalties are waived.
Interests shall accrue only up to July 13, 2004, the date of
issuance of the stay order[.]
No interest will accrue during the pendency of petitioners
corporate rehabilitation[.]
Dollar-denominated loans will be converted to Philippine
Pesos on the date of the issuance of this Order using the
reference rate of the Philippine Dealing System as of this
date.
7

Respondent China Banking Corporation contended that the
rehabilitation plans approval impaired the obligations of contracts.
o It argued that neither the provisions of Presidential Decree No.
902-A nor the Interim Rules of Procedure on Corporate
Rehabilitation (Interim Rules) empowered commercial courts "to
render without force and effect valid contractual stipulations."
8

[ff facts pertinent to res judicata issue, not for impairment of contract
issue]
The Bank of the Philippine Islands (BPI), another creditor of petitioner
Pryce Corporation, filed a separate petition with the Court of Appeals
assailing the same order by the rehabilitation court.
CA 7
TH
DIVISION granted respondent China Banking Corporation's petition,
and reversed and set aside the rehabilitation courts:
o (1) July 13, 2004 stay order that also appointed Gener T. Mendoza
as rehabilitation receiver;
o (2) September 13, 2004 order giving due course to the petition and
directing the rehabilitation receiver to evaluate and give
recommendations on petitioner Pryce Corporations proposed
rehabilitation plan; and
o (3) January 17, 2005 order finding petitioner Pryce Corporation
eligible to be placed in a state of corporate rehabilitation,
identifying assets to be disposed of, and determining the manner of
liquidation to pay the liabilities.
12

With respect to BPIs separate appeal, the CA 1
st
DIVISION
o granted its petition initially
o and set aside the January 17, 2005 order of the rehabilitation court
in its decision dated May 3, 2006.
o On reconsideration, the court issued a resolution dated May
23, 2007 setting aside its original decision and dismissing the
petition.
o BPI elevated the case to SC, docketed as G.R. No. 180316.
By resolution dated January 30, 2008, the SC 1st Division
DENIED the petition. MR denied with finality.
Meanwhile, petitioner Pryce Corporation also appealed to SC assailing the
decision of the CA 7th DIVISION granting respondent China Banking
Corporations petition as well as the resolution denying MR.
In the decision dated February 4, 2008, the SC 1st Division of this court
denied its petition with the dispositive portion as follows:
o WHEREFORE, we DENY the petition. The assailed Decision of the
Court of Appeals in CA-G.R. SP No. 88479 is AFFIRMED with the
modification discussed above. Let the records of this case be
REMANDED to the RTC, Branch 138, Makati City, sitting as
Commercial Court, for further proceedings with dispatch to
determine the merits of the petition for rehabilitation.
Updated by K Pascual. 2014. 4C. Bill of Rights Poli Digests. Atty. Jack Jimenez. 4C. 127


Petitioner Pryce Corporation filed an omnibus motion for (1)
reconsideration or (2) partial reconsideration and (3) referral to the SC En
Banc dated February 29, 2008.
o Respondent China Banking Corporation also filed a motion for
reconsideration on same date, only insofar as it ordered the
remand of the case for further proceedings "to determine whether
petitioner's financial condition is serious and whether there is clear
and imminent danger that it will lose its corporate assets."
By resolution dated June 16, 2008, this SC denied with finality the separate
motions for reconsideration filed by the parties.
o On September 10, 2008, petitioner Pryce Corporation filed a 2ND
motion for reconsideration praying that the CAs decision dated
February 4, 2008 be set aside.
The 1ST Division of this court referred this case to the En Banc en consulta
by resolution dated June 22, 2009.
HELD: WHEREFORE, petitioner Pryce Corporation's motion is GRANTED. This
court's February 4, 2008 decision is RECONSIDERED and SET ASIDE.
MAY SKIP RES JUDICATA - ISSUE 1: WHETHER THE ISSUE ON THE
VALIDITY OF THE REHABILITATION ORDER DATED JANUARY 17, 2005 IS
NOW RES JUDICATA IN LIGHT OF BPI V. PRYCE CORPORATION DOCKETED
AS G.R. NO. 180316;
BPI v. Pryce Corporation docketed as G.R. No. 180316 rendered the issue on
the validity of the rehabilitation courts January 17, 2005 order approving
the amended rehabilitation plan as res judicata.
In BPI v. Pryce Corporation, the Court of Appeals ultimately dismissed the
petition. On appeal, this court denied the petition filed by BPI with finality.
An entry of judgment was made for BPI v. Pryce Corporation on June 2,
2008.
32

o In effect, this court upheld the January 17, 2005 order of the
rehabilitation court.
According to the doctrine of res judicata, "a final judgment or decree on the
merits by a court of competent jurisdiction is conclusive of the rights of the
parties or their privies in all later suits on all points and matters
determined in the former suit."
33

The elements for res judicata to apply are as follows:
o (a) the former judgment was final;
o (b) the court that rendered it had jurisdiction over the subject
matter and the parties;
o (c) the judgment was based on the merits; and
o (d) between the first and the second actions, there was an identity
of parties, subject matters, and causes of action.
34

Res judicata embraces two concepts: (1) bar by prior judgment
35
and (2)
conclusiveness of judgment.
36

o Bar by prior judgment exists "when, as between the first case
where the judgment was rendered and the second case that is
sought to be barred, there is identity of parties, subject matter, and
causes of action."
37

o On the other hand, the concept of conclusiveness of judgment
finds application "when a fact or question has been squarely put in
issue, judicially passed upon, and adjudged in a former suit by a
court of competent jurisdiction."
38
This principle only needs
identity of parties and issues to apply.
39

The elements of res judicata through bar by prior judgment are
present in this case.
o On the element of identity of parties, res judicata does not
require absolute identity of parties as substantial identity is
enough.
o Substantial identity of parties exists "when there is a
community of interest between a party in the first case and a
party in the second case, even if the latter was not impleaded
in the first case."
o In the present case, respondent China Banking Corporation and BPI
are creditors of petitioner Pryce Corporation and are both
questioning the rehabilitation courts approval of the amended
rehabilitation plan. Thus, there is substantial identity of parties
since they are litigating for the same matter and in the same
capacity as creditors of petitioner Pryce Corporation.
There is no question that both cases deal with the subject matter of
petitioner Pryce Corporations rehabilitation. The element of identity of
causes of action also exists.
Updated by K Pascual. 2014. 4C. Bill of Rights Poli Digests. Atty. Jack Jimenez. 4C. 128


In separate appeals, respondent China Banking Corporation and BPI
questioned the same January 17, 2005 order of the rehabilitation court
before the Court of Appeals.
Since the January 17, 2005 order approving the amended
rehabilitation plan was affirmed and made final in G.R. No. 180316,
this plan binds all creditors, including respondent China Banking
Corporation.
MAY SKIP ISSUE 2: WON the rehabilitation court is required to hold a
hearing to comply with the "serious situations" test laid down in Rizal
Commercial Banking Corp. v. IAC before issuing a stay order.
The rehabilitation court complied with the Interim Rules in its order dated July
13, 2004 on the issuance of a stay order and appointment of Gener T. Mendoza
as rehabilitation receiver.
53

The 1999 Rizal Commercial Banking Corp. v. IAC
54
case provides for the
"serious situations" test in that the suspension of claims is counted only
upon the appointment of a rehabilitation receiver,
55
and certain situations
serious in nature must be shown to exist before one is appointed, viz:
Furthermore, as relevantly pointed out in the dissenting opinion, a petition for
rehabilitation does not always result in the appointment of a receiver or the
creation of a management committee. The SEC has to initially determine
whether such appointment is appropriate and necessary under the
circumstances. Under Paragraph (d), Section 6 of Presidential Decree No. 902-A,
certain situations must be shown to exist before a management committee
may be created or appointed, such as:
1. when there is imminent danger of dissipation, loss, wastage or
destruction of assets or other properties; or
2. when there is paralization of business operations of such
corporations or entities which may be prejudicial to the interest of
minority stockholders, parties-litigants or to the general public.
On the other hand, receivers may be appointed whenever:
1. necessary in order to preserve the rights of the parties-litigants;
and/or
2. protect the interest of the investing public and creditors.
(Section 6 [c], P.D. 902-A.)
These situations are rather serious in nature, requiring the appointment
of a management committee or a receiver to preserve the existing assets
and property of the corporation in order to protect the interests of its
investors and creditors. Thus, in such situations, suspension of actions for
claims against a corporation as provided in Paragraph (c) of Section 6, of
Presidential Decree No. 902-A is necessary, and here we borrow the words of
the late Justice Medialdea, "so as not to render the SEC management
Committee irrelevant and inutile and to give it unhampered rescue
efforts over the distressed firm" (Rollo, p. 265)."
Otherwise, when such circumstances are not obtaining or when the SEC
finds no such imminent danger of losing the corporate assets, a
management committee or rehabilitation receiver need not be appointed
and suspension of actions for claims may not be ordered by the SEC. When
the SEC does not deem it necessary to appoint a receiver or to create a
management committee, it may be assumed, that there are sufficient assets to
sustain the rehabilitation plan, and that the creditors and investors are amply
protected.
56

IMPORTANT ISSUE 2.2: WON theres a violation of the Constitutional
proscription against IMPAIRMENT of Contractual Obligations?
NOTE: In this case, no law or executive issuance was passed to justify
invocation of the non-impairment clause. What was passed was merely a
COURT ORDER.
Respondent China Banking Corporation mainly argues the violation of the
constitutional proscription against impairment of contractual
obligations
68
in that neither the provisions of Pres. Dec. No. 902-A as
amended nor the Interim Rules empower commercial courts "to render
without force and effect valid contractual stipulations."
69

Updated by K Pascual. 2014. 4C. Bill of Rights Poli Digests. Atty. Jack Jimenez. 4C. 129


HISTORY EKLAVU: The non-impairment clause first appeared in the
United States Constitution as a safeguard against the issuance of
worthless paper money that disturbed economic stability after the
American Revolution.
70

This constitutional provision was designed to promote commercial
stability.
71
At its core is "a prohibition of state interference with
debtor-creditor relationships."
72

HISTORY EKLAVU: This clause first became operative in the Philippines
through the Philippine Bill of 1902, the fifth paragraph of Section 5 which
states "[t]hat no law impairing the obligation of contracts shall be enacted."
It was consistently adopted in subsequent Philippine fundamental laws,
namely, the Jones Law of 1916,73 the 1935 Constitution,
74
the 1973
Constitution,
75
and the present Constitution.
76

Nevertheless, this court has brushed aside invocations of the non-
impairment clause to give way to a valid exercise of police
power
77
and afford protection to labor.
78

Pacific Wide Realty and Development Corporation v. Puerto Azul Land: This
case (involving corporate rehabilitation) does not involve a law or an
executive issuance declaring the modification of the contract among debtor
PALI, its creditors and its accommodation mortgagors. Thus, the non-
impairment clause may not be invoked.
Oposa v. Factoran, Jr.: Even assuming that the same may be invoked, the
non-impairment clause must yield to the police power of the State.
Property rights and contractual rights are not absolute. The
constitutional guaranty of non-impairment of obligations is limited by the
exercise of the police power of the State for the common good of the general
public.
Successful rehabilitation of a distressed corporation will benefit its
debtors, creditors, employees, and the economy in general. The court
may approve a rehabilitation plan even over the opposition of creditors
holding a majority of the total liabilities of the debtor if, in its judgment, the
rehabilitation of the debtor is feasible and the opposition of the creditors is
manifestly unreasonable. The rehabilitation plan, once approved, is binding
upon the debtor and all persons who may be affected by it, including the
creditors, whether or not such persons have participated in the proceedings
or have opposed the plan or whether or not their claims have been
scheduled.
80

Corporate rehabilitation is one of many statutorily provided remedies for
businesses that experience a downturn. Rather than leave the various
creditors unprotected, legislation now provides for an orderly
procedure of equitably and fairly addressing their concerns.
Corporate rehabilitation allows a court-supervised process to
rejuvenate a corporation. Its twin, insolvency, provides for a system of
liquidation and a procedure of equitably settling various debts owed by an
individual or a business. It provides a corporations owners a sound chance
to re-engage the market, hopefully with more vigor and enlightened
services, having learned from a painful experience.
Necessarily, a business in the red and about to incur tremendous losses may
not be able to pay all its creditors. Rather than leave it to the strongest or
most resourceful amongst all of them, the state steps in to equitably
distribute the corporations limited resources.
The cram-down principle adopted by the Interim Rules does, in effect,
dilute contracts. When it permits the approval of a rehabilitation plan
even over the opposition of creditors,
81
or when it imposes a binding effect
of the approved plan on all parties including those who did not participate
in the proceedings,
82
the burden of loss is shifted to the creditors to allow
the corporation to rehabilitate itself from insolvency.
Rather than let struggling corporations slip and vanish, the better
option is to allow commercial courts to come in and apply the process
for corporate rehabilitation.
This option is preferred so as to avoid what Garrett Hardin called the
Tragedy of Commons.
o Here, Hardin submits that "coercive government regulation is
necessary to prevent the degradation of common-pool resources
[since] individual resource appropriators receive the full benefit of
their use and bear only a share of their cost."
83
By analogy to the
game theory, this is the prisoners dilemma: "Since no individual
has the right to control or exclude others, each appropriator has a
very high discount rate [with] little incentive to efficiently manage
the resource in order to guarantee future use."
84
Thus, the cure is
an exogenous policy to equitably distribute scarce resources. This
will incentivize future creditors to continue lending, resulting in
something productive rather than resulting in nothing.
o In fact, these corporations exist within a market. The General
Theory of Second Best holds that "correction for one market
Updated by K Pascual. 2014. 4C. Bill of Rights Poli Digests. Atty. Jack Jimenez. 4C. 130


imperfection will not necessarily be efficiency-enhancing unless
[there is also] simultaneous [correction] for all other market
imperfections."
85
The correction of one market imperfection may
adversely affect market efficiency elsewhere, for instance, "a
contract rule that corrects for an imperfection in the market for
consensual agreements may [at the same time] induce welfare
losses elsewhere."
86
This theory is one justification for the passing
of corporate rehabilitation laws allowing the suspension of
payments so that corporations can get back on their feet.
As in all markets, the environment is never guaranteed. There are
always risks. Contracts are indeed sacred as the law between the parties.
However, these contracts exist within a society where nothing is risk-free,
and the government is constantly being called to attend to the realities of
the times.
Corporate rehabilitation is preferred for addressing social costs. Allowing
the corporation room to get back on its feet will retain if not increase
employment opportunities for the market as a whole. Indirectly, the
services offered by the corporation will also benefit the market as "[t]he
fundamental impulse that sets and keeps the capitalist engine in motion
comes from [the constant entry of] new consumers goods, the new
methods of production or transportation, the new markets, [and] the new
forms of industrial organization that capitalist enterprise creates."
87

86. MERALCO VS. PROVINCE OF LAGUNA
non-impairment of contracts
Laguna provinces, (Bian, Sta Rosa, San Pedro, Luisiana, Calauan and
Cabuyao) by virtue of existing laws then in effect, issued resolutions
through their respective municipal councils granting franchise in favor of
Meralco
Subsequently, the Local Government Code was enacted giving the LGUs
the power to create their own sources of revenue.
o The 1991 Code explicitly authorizes provincial governments,
notwithstanding any exemption granted by any law or other
special law, x x x (to) impose a tax on businesses enjoying a
franchise.
Thus, the Laguna Province enacted an ordinance 01-92 taxing businesses
that are under a franchise, of of 1%.
Then the provincial treasurer sent demand letters to Meralco for tax
payment.
Meralco paid under protest and claimed for a refund.
Meralco argues that the franchise tax already paid to the National
Government already includes that imposed by the Provincial Tax
Ordinance. Meralco also contended that the tax under the Provincial
Ordinance violates PD 551 (which already puts the tax at 2%).
The claim for refund was denied, citing that this PD 551 was repealed by
the new Local Government Code.
ISSUE: Whether the imposition of the tax under the Provincial Ordinance
violates the non-impairment clause of the Constitution and of PD551? Does the
LGC repeal PD551?
Are franchises contracts?
SC:
The LGC explicitly authorizes the provincial governments, notwithstanding any
exemption granted by an other law, to impose a tax on businesses under a
franchise. Thus, the legislative intent was to carry out the constitutional
mandate of vesting broad tax powers to LGUs. As such the LGC has effectively
withdrawn tax exemptions or incentives previously enjoyed by certain entities.

The phrase in lieu of all taxes in the original franchise, has to give way to the
peremptory language of the LGC specifically withdrawing such exemptions and
privileges. Thus, upon the effectivity of the LGC, all exemptions can no longer be
invoked by Meralco.
Updated by K Pascual. 2014. 4C. Bill of Rights Poli Digests. Atty. Jack Jimenez. 4C. 131


While the Court may have frequently referred to tax exemptions
contained in special franchises as being in the nature of contracts,
(because they are part of the inducements for carrying on the franchise),
THESE EXEMPTIONS ARE FAR FROM BEING CONTRACTUAL IN NATURE.
Contractual tax exemptions, (to be considered within the coverage of the non-
impairment clause) are those agreed to by the taxing authority in contracts, such
as those in government bonds and debentures, lawfully entered into by them
under enabling laws which the government acting in its private capacity, sheds its
cloak of authority and waives governmental immunity. It is only these kinds of
tax exemptions that the exemptions cannot be revoked because it would violate
the non-impairment clause. THIS IS DIFFERENT FROM TAX EXEMPTIONS
UNDER A FRANCHISE.

A franchise partakes the nature of a grant which is beyond the purview of
the non-impairment clause. No franchise for the operation of a public utility
shall be granted except under the condition that such privilege shall be subject
to amendment, alteration, repeal by Congress and when the common good
requires.
87. TOLENTINO VS. SEC OF FINANCE
non-impairment clause
This is again the EVAT case.
The Creba or Chamber of Real Estate and Builders Association assails the
validity of the EVAT. CREBA asserts that R.A. No. 7716 (1) impairs the
obligations of contracts.
They claim that the application of the tax to existing contracts (those
entered into before the effectivity of the EVAT) of the sale of real
property by installment or on deferred payment basis would result in
substantial increases in the monthly amortizations to be paid because
of the 10% VAT. The additional amount, it is pointed out, is something that
the buyer did not anticipate at the time he entered into the contract.

SC: LAW VALID. NO IMPAIRMENT OF CONTRACTS.
A lawful tax on a new subject, or an increased tax on an old one, interferes with
a contract or impairs its obligation, within the meaning of the Constitution.
Even though such taxation may affect particular contracts, as it may
increase the debt of one person and lessen the security of another, or may
impose additional burdens upon one class and release the burdens of another,
still the tax must be paid unless prohibited by the Constitution, nor can it
be said that it impairs the obligation of any existing contract in its true
legal sense.

Indeed not only existing laws but also "the reservation of the essential
attributes of sovereignty, is read into contracts as a postulate of the legal
order." Contracts must be understood as having been made in reference to
the possible exercise of the rightful authority of the government and no
obligation of contract can extend to the defeat of that authority.
88. HOME BUILDING VS. BLAISDELL
non-impairment clause
There was a Minnesota Mortgage Moratorium Law providing that
o during times of emergencies, relief may be had through judicial
proceedings with respect to foreclosure of mortgages and
execution sales,
such that these may be postponed and the periods of
redemption may be extended.
o The Law remains in effect only during the times of emergency and
in no case beyond May 1, 1935.
There was a severe financial and economic depression that has existed
for several years, resulting in extremely low prices for the products of
farms and factories, in much unemployment, in almost complete lack of
credit for farmers, business men and property owners, and in extreme
stagnation of business, agriculture and industry;
Updated by K Pascual. 2014. 4C. Bill of Rights Poli Digests. Atty. Jack Jimenez. 4C. 132


o that many owners of real property, by reason of these conditions,
are unable and, it is believed, for some time will be unable, to meet
all payments as they come due, of taxes, interest and principal of
mortgages, and are, therefore, threatened with the loss of their
property through foreclosure sale;
o that much property has been bid in on foreclosure for prices much
below what it is believed was its real value, and often for much less
than the mortgage indebtedness, resulting in deficiency judgments;
o that, under the existing conditions, foreclosure of many real estate
mortgages by advertisement would prevent fair, open and
competitive bidding in the manner contemplated by law.
(there was also a man and a woman whose house was foreclosed by the
Home and Building Loan Association as mortgagor)
The appellant assails the validity of said law as being repugnant to the non-
impairment clause.
The lower court of Minnesota upheld the validity of the law, conceding that
although the obligations of the mortgage contract were impaired, the
impairment was still within the police powers of the state, because it was
called into exercise in times of public economic emergency.
They appealed.

SC: LAW VALID.
In determining whether the relief provided exceeds the non-impairment clause
of the Constitution, we must consider the relation of the emergency to the
constitutional power. Emergency does not create power. Emergency does not
increase granted power or remove or diminish the restrictions imposed upon
the power granted or reserved.
HISTORY SHIT: The purpose of the non-impairment clause is that the
power of changing the relative situation of the debtor and creditor, of
interfering with contracts, touches the interest of all, and controls the conduct
of every individual in those things, had been so much abused by state
legislatures, as to break into the ordinary intercourse of society and destroy
the confidence between man and man. This mischief has become so great and
alarming as to impair commercial intercourse, threaten the existence of credit,
as well as sap the moral out of people and destroy the sanctity of private faith.
Thus, the non-impairment clause was constituted to guard against this
evil.
The obligation of a contract is the law which binds the parties to perform their
agreement. This Court has said that 'the laws which subsist at the time and
place of the making of a contract, and where it is to be performed, enter into
and form a part of it, as if they were expressly referred to or incorporated in its
terms. This principle embraces alike those which affect its validity, construction,
discharge, and enforcement. Nothing can be more material to the obligation
than the means of enforcement.
The distinction between the obligation of a contract, and the remedy given by
the legislature to enforce that obligation, has been taken at the bar, and exists in
the nature of things. Without impairing the obligation of the contract, the
remedy may certainly be modified as the wisdom of the nation shall direct. It is
competent for the States to change the form of the remedy, or to modify it
otherwise, as they may see fit, provided no substantial right secured by the
contract is thereby impaired.
The obligations of a contract are impaired by a law which renders them invalid,
or releases or extinguishes and impairment, has been predicated of laws which
without destroying contracts derogate from substantial contractual rights.
In view of the conditions with which the Minnesota statute seeks to
safeguard the interests of the mortgagee-purchaser during the
extended period,
o not only is the constitutional provision qualified by the measure
of control which the state retains over remedial processes,
o but the state also continues to possess authority to safeguard
the vital interests of its people.
Not only are existing laws read into contracts in order to fix
obligations as between the parties, but the reservation of essential
attributes of sovereign power is also read into contracts as a postulate
of the legal order.
o The government retains authority to secure the peace and good
order of society. But into all contracts, there are conditions which
Updated by K Pascual. 2014. 4C. Bill of Rights Poli Digests. Atty. Jack Jimenez. 4C. 133


arise, not out of the literal terms of the contract itself. They are
super induced by the pre- existing and higher authority of the laws
of nature, of nations, or of the community to which the parties
belong.
o They are always presumed, and must be presumed, to be known
and recognized by all, are binding upon all, and need never,
therefore, be carried into express stipulation, for this could add
nothing to their force. Every contract is made in subordination to
them, and must yield to their control, as conditions inherent and
paramount, wherever a necessity for their execution shall occur.'
The Legislature cannot 'bargain away the public health or the public morals.
Whatever doubt there may have been that the protective power of the state,
its police power, may be exercised-without violating the true intent of
the provision of the Federal Constitution-in directly preventing the
immediate and literal enforcement of contractual obligations by a
temporary and conditional restraint, where vital public interests
would otherwise suffer.

AFTER ESTABLISHING THESE CRITERIA, THE COURT RULED THAT:
1. An emergency existed in Minnesota which furnished a proper occasion
for the exercise of the reserved power of the state to protect the vital
interests of the community. The declarations of the existence of this
emergency by the Legislature and by the Supreme Court of Minnesota cannot be
regarded as a subterfuge or as lacking in adequate basis. the economic
emergency which threatened 'the loss of homes and lands which furnish those
in possession the necessary shelter and means of subsistence' was a 'potent
cause' for the enactment of the statute.
2. The legislation was addressed to a legitimate end; that is, the legislation was
not for the mere advantage of particular individuals but for the protection of a
basic interest of society.
3. In view of the nature of the contracts in question-mortgages of
unquestionable validity-the relief afforded and justified by the emergency, in
order not to contravene the constitutional provision, could only be of a
character appropriate to that emergency, and could be granted only upon
reasonable conditions.
4. The conditions upon which the period of redemption is extended do not
appear to be unreasonable. The initial extension of the time of redemption for
thirty days from the approval of the act was obviously to give a reasonable
opportunity for the authorized application to the court.
As already noted, the integrity of the mortgage indebtedness is
not impaired; interest continues to run; the validity of the sale and the
right of a mortgagee-purchaser to title or to obtain a deficiency
judgment, if the mortgagor fails to redeem within the extended period,
are maintained; and the conditions of redemption, if redemption there
be, stand as they were under the prior law. The mortgagor during the
extended period is not ousted from possession, but he must pay the
rental value of the premises as ascertained in judicial proceedings and
this amount is applied to the carrying of the property and to interest
upon the indebtedness. The mortgagee-purchaser during the time that
he cannot obtain possession thus is not left without compensation for
the withholding of possession.

5. The legislation is temporary in operation.
89. RUTTER VS. ESTEBAN
non-impairment clause
Rutter was the seller. Esteban was the buyer of the 2 parcels of land.
o There was still an unpaid balance of P4800.
To secure payment of the balance, a mortgage was constituted over the
land.
Estaben defaulted. Rutter filed an action to recover the balance.
Esteban invokes RA342, the Moratorium Law. He claims that this
obligation was contracted before the War, and that he is a war
sufferer, such that his obligation cannot be enforced until after the
lapse of 8 years from the settlement of his claim by the Philippine War
Damage Commission.
Updated by K Pascual. 2014. 4C. Bill of Rights Poli Digests. Atty. Jack Jimenez. 4C. 134


Rutter assails the validity of the Moratorium Law, for being violative of the
non-impairment clause.
SC:
The moratorium is a postponement of fulfilment of obligations decreed by
the state through the medium of the courts or the legislature. Its essence is
the application of the sovereign power. Such moratorium laws "were passed by
many state legislatures at the time of the civil war suspending the rights of
creditors for a definite and reasonable time, whether they suspend the right of
action or make dilatory the remedy.
The true test, therefore, of the constitutionality of a moratorium statute lies in
the determination of the period of suspension of the remedy. It is required that
such suspension be definite and reasonable, otherwise it would be violative of
the constitution.
One of the arguments advanced against the validity of the moratorium law
is the fact that it impairs the obligation of contracts which is prohibited by
the Constitution. This argument, however, does not now hold water.
o While this may be conceded, it is however justified as a valid
exercise by the State of its police power.
(the previous case of Home Builders vs. Blaisdell was quotedsee
doctrines)
This decision elicited several comments. Laws altering existing contracts
constitute an impairment within the meaning of the contract clause
only if they are unreasonable in the light of the circumstances
occasioning their enactment.
o Application of this 'rule of reason' was justified on the theory that
all contracts are made subject to an implied reservation of the
protective power of the state, and that therefore statutes
which validly exercise this reserved power, rather than
impairing the obligations of an existing contract, are
comprehended within them.
There are however limitations. It must be noted that the application of the
reserved power of the State to protect the integrity of the government and
the security of the people should be limited to its proper bounds and
must be addressed to a legitimate purpose.
One of them is that the impairment should only refer to the remedy
and not to a substantive right.
o The State may postpone the enforcement of the obligation but
cannot destroy it by making the remedy futile.
o Another limitation refers to the propriety of the remedy. The rule
requires that the alteration or change that the new legislation
desires to write into an existing contract must not be burdened
with restrictions and conditions that would make the remedy
hardly pursuing.
The Blaisdell case postulates that the protective power of the State, the
police power, may only be invoked and justified by an emergency,
temporary in nature, and can only be exercised upon reasonable conditions
in order that it may not infringe the constitutional provision against
impairment of contracts. It is predicated on the ground that the laws
altering existing contracts will constitute an impairment of the contract
clause of the Constitution only if they are unreasonable in the light of the
circumstances occasioning their enactment.

ISSUE: Is the period of eight (8) years under RA 342 reasonable under the
present circumstances?
SC: UNREASONABLE.
The purpose of the law is to afford to prewar debtors an opportunity to
rehabilitate themselves by giving them a reasonable time within which to
pay their prewar debts so as to prevent them from being victimized by their
creditors.
BUT, we should not lose sight of the fact that these obligations had
been pending since 1945 as a result of the issuance of Executive
Orders Nos. 25 and 32 and at present their enforcement is still
inhibited because of the enactment of Republic Act No. 342 and would
continue to be unenforceable during the eight-year period granted to
prewar debtors to afford them an opportunity to rehabilitate
themselves,
o which in plain language means that the creditors would have to
observe a vigil of at least twelve (12) years before they could effect
a liquidation of their investment dating as far back as 1941.
o This period seems to us unreasonable, if not oppressive.
While the purpose of Congress is plausible, and should be commended, the
relief accorded works injustice to creditors who are practically left at the
Updated by K Pascual. 2014. 4C. Bill of Rights Poli Digests. Atty. Jack Jimenez. 4C. 135


mercy of the debtors. Their hope to effect collection becomes extremely
remote, more so if the credits are unsecured.
o And the injustice is more patent when, under the law, the
debtor is not even required to pay interest during the
operation of the relief.
Such an exemption, applied in the case of debts owing before the exemption
was created by the legislature, constitutes an unwarranted interference
with the obligation of contracts in violation of the constitutional provision",
and cannot be sustained even as emergency legislation, because it contains
no limitation as to time, amount, circumstances or need.

RA 342 NULL AND VOID. (Note this case was decided in 1953).

The court also noted that the economic and financial conditions of the country
has improved and returned to normal.

90. DEL ROSARIO VS. DE LOS SANTOS
non-impairment clause
Del ROSARIO was land OWNER. De Los SANTOS was a TENANT.
Santos filed a petition before the Court of Agrarian Relations, manifesting
their desire
o to take advantage of the Agricultural Tenancy Act,
o which gives them the power to change the tenancy contract
from one of share tenancy to leasehold tenancy.
Del Rosario assailed the statute but the CAR ruled in favor of De Los Santos.
o The CAR declared the relationship to be of leasehold tenancy.
Del Rosario appealed, claiming that it impaired the obligation of contracts.

SC: LAW VALID.
Tenancy legislation is a manifestation of a deep and earnest concern
to solve an age-old problem of Philippine society. The framers of the
Constitution mindful of the growing feeling of dissatisfaction with the
government's ability to cope with poverty and misery of the vast majority of
the people inserted the provisions of the protection to labor and social
justice in the Constitution, thus, leaving no doubt about the validity of
remedial legislation intended to minimize if not abolish the oppressive
condition usually associated with agricultural labor.
The attribute of police power, reinforced by the constitutional provisions
giving protection to labor and on social justice justifies the enactment of
statutory provisions.
o The provisions of the Agricultural Tenancy Act, particularly of
Sec. 14 thereof is not an unconstitutional impairment of the
obligation of an existing contract.
o Obligations of contracts must yield to a proper exercise of
the police power when such power is exercised to
preserve the security of the state and the means adopted
are reasonably adapted to the accomplishment of that end
and are not arbitrary or oppressive.
o Neither is the law a transgression on the freedom of contract
embraced in the liberty safeguarded by the due process clause.
That public interest would be served by governmental measures intended
to aid the economically underprivileged is apparent to all. Nor is the means
relied upon to attain such a valid objective unreasonable or oppressive,
Considering that in the adjustment or reconciliation of the conflicting
claims to property and state authority, it suffices that there be a rational
basis for the legislative act, it is easily understandable why, from the
enactment of the Constitution with its avowed concern for those who have
less in life, the constitutionality of such legislation has been repeatedly
upheld.
With its unqualified approval of the power of Congress to abolish share
tenancy, as reflected in the latest legislation on the subject, as against the
contention that with the limitation on the freedom of contract there is a
deprivation of property without due process of law, evinces unmistakably
the firmness with which it adheres to the view that the police power is of
sufficient amplitude and scope to- free from the taint of constitutional
Updated by K Pascual. 2014. 4C. Bill of Rights Poli Digests. Atty. Jack Jimenez. 4C. 136


infirmity legislation intended to ameliorate the sad plight of Filipino tenants
and agricultural workers.
91. NDC VS. PVB
non-impairment clause
Summary:
1) invalid exercise of police power fails the lawful subject, lawful means test
2) taking without due process extinguishes the mortgage and liens arbitrarily.
3) violates equal protection puts secured and unsecured creditors to the same
level
4) violates non-impairment interferes with purely private agreement, no
connection to public interests

FACTS:
[Agrix owed PVB.] Agrix Marketing had executed a REM in favor of PVB,
over 3 parcels of land.
o Agrix however became bankrupt and unable to pay.
President Marcos issued PD 1717 ordering the rehabilitation of Agrix
Group of Companies, to be administered by the NDC.
It provided for the procedure for filing claims against the Agrix companies,
and created a Claims Committee.
o The law provides further that all mortgages and other liens
presently attaching to the assets of the dissolved corporations
are extinguished.
PVB thus filed a claim with Agrix.
o The New Agrix (under the NDC) however invoked the PD 1717.
They argue that this is an exercise of police power for the
promotion of the common welfare.
PVB however claims that it was an impairment of the obligations of
contracts and a denial of due process.

SC: LAW NULL AND VOID. IMPAIRS OBLIGATION OF CONTRACTS
The court is especially disturbed by Sec 4 of PD1717 which
extinguishes all mortgages upon the assets of Agrix.
There was also a provision that unsecured obligations shall bear no
interest.
It fails the lawful subject and lawful means test of the exercise of
police powers.
o The interests of the public are not sufficiently involved to warrant
the interference of the government with the private contracts of
Agrix.
o The decree speaks vaguely of the public, particularly small
investors. However there is no record of how many such investors
are, and who they are, and why they are being preferred over PVB
and other creditors of Agrix.
It is also unduly oppressive.
o The right to property in all mortgages, liens and interests owing to
the creditors of Agrix like PVB, has been arbitrarily destroyed.
o No consideration is paid for the extinction of the mortgage rights.
The accrued interests and other charges are simply rejected.
o The right to property is dissolved by legislative fiat without regard
to the private interests violated, and worse, in favor of another
private interest.
A mortgage lien is a property right derived from contract.
o Thus, it comes under the protection of the Bill of Rights. So are
interests, penalties and charges which become vested rights once
they accrue.
There is also AN IMPARIMENT OF OBLIGATION OF CONTRACTS
between Agrix and PVB.
o While it is true that the police power is superior to the non-
impairment clause, the principle will apply only where the
Updated by K Pascual. 2014. 4C. Bill of Rights Poli Digests. Atty. Jack Jimenez. 4C. 137


contract is so related to the public welfare that it will be
considered congenitally susceptible to change by the
legislature in the interest of the greater number.
o But the contracts of loan and mortgage executed between Agrix
and PVB are PURELY PRIVATE TRANSACTIONS, and are not
affected with public interest. There is no warrant to amend their
provisions and deprive PVB of its vested property rights.

92. PHILCONSA VS. ENRIQUEZ
non-impairment clause
House Bill No. 10900, the General Appropriation Bill of 1994 (GAB of
1994), was passed and approved by both houses of Congress on December
17, 1993. As passed, it imposed conditions and limitations on certain items
of appropriations in the proposed budget previously submitted by the
President. It also authorized members of Congress to propose and identify
projects in the "pork barrels" allotted to them and to realign their
respective operating budgets.
In the appropriation for the Armed Forces of the Philippines (AFP),
the President vetoed the special provision on Use of Fund," which
requires the prior approval of the Congress for the release of the
corresponding modernization funds.
As reason for the veto, the President stated that the said condition and
prohibition violate the Constitutional mandate of non-impairment of
contractual obligations,
o and if allowed, "shall effectively alter the original intent of the AFP
Modernization Fund to cover all military equipment deemed
necessary to modernize the Armed Forces of the Philippines"
The president vetoed it since they were already contracted for.
ISSUE: Was the Presidential Veto valid?
SC: YES. VALID VETO.
Special Provision No. 3, prohibiting the use of the Modernization fund for
payment of the trainer planes and armored personnel carriers, which have
been contracted for by the AFP, is violative of the Constitutional prohibition
on the passage of laws that impair the obligation of contracts (Art. III, Sec.
10), more so, contracts entered into by the Government itself.
The veto of said special provision is therefore valid.
A provision cannot prohibit payment, because to do so would impair the
perfect contracts.
93. FIRST PHILIPPINE INTERNATIONAL BANK VS. CA
non-impairment clause
1) The Bank had 6 parcels of land.
2) Janolo wanted to purchase the property and thus made a formal purchase
offer at P3.5M.
3) The Bank made a counter-offer at P5.5M
4) Janolo made another offer of P4.25M.
5) There was no more counter-offer but a meeting took place among the Bank
officials.
a. Janolo allegedly accepted the counter-offer of P5.5M
6) However the conservator of the bank informed Janolo that the
proposal to buy the property is under study yet as of this time because
there has been a newly created committed for the Conservator of the
bank.
7) Janolo however demanded that there was already a perfected contract of
sale.
8) The bank still refused. The bank did not accept the payment. Worse, the
bank advertised the lot to other persons.
9) Janolo made a second tender of payment this time through the Acting
Conservator.
Updated by K Pascual. 2014. 4C. Bill of Rights Poli Digests. Atty. Jack Jimenez. 4C. 138


10) Since the bank still refused, Janolo filed a suit for specific performance, on
the ground that there has already been a perfected contract of sale.
11) The Bank claimed that Rivera, the Property Department Manager, had no
authority to sell. They claim that it should be the Conservator.
12) It will be noted that the Bank was placed under Conservatorship under the
Central Bank during the time that there was negotiation and perfection of
the contract of sale.
13) The Bank contends that the conservator has the power to revoke or
overrule the actions of the management of the bank under RA 265 or
Central Bank Act.

ISSUE: Was the contract perfected? YES.
SC: Rivera had apparent or implied authority to act for the Bank in the
matter of selling its assets. Rivera was the one in charge of the assets, and it
was Rivera who met with the buyers and made counter-offers. He was also the
one who signed the letters to the buyers.
The Banks repudiation of Riveras authority, through the Conservator,
came only 7 months after there was acceptance by Janolo. Such delay,
clearly characterizes the repudiation as nothing more than a last-minute
attempt by the Bank to opt out of its binding contractual obligation.

ISSUE: May the Conservator Revoke the Perfected and Enforceable Contract?
NO, powers of the Conservator cannot extend to POST FACTO REPUDIATION of
perfected transactions
Under the Central Bank Act, when a bank is in a state of inability to
maintain a state of liquidity, the Monetary Board may appoint a conservator
to take charge of its assets. The Conservator has the power to overrule
and revoke the actions of the management.
SC: There is absolutely no evidence that the Conservator, at the time the
contract was perfected, actually repudiated or overruled the contract of
sale.
The Banks acting Conservator at that time, Mr. Romey, never objected to
the sale of the property to Janolo. It was the subsequent Conservator, Mr.
Encarnacion, who took over AFTER the sale was perfected, who
unilaterally repudiated, NOT THE CONTRACT, but the authority of
Rivera to make a binding offer.
Also, it must be pointed that the powers of the conservator must be
related to the preservation of the banks assets and restoration of its
viability.
o Such powers however cannot extend to POST FACTO
REPUDIATION of perfected transactions, otherwise they would
infringe on the non-impairment clause.
o If the legislature itself cannot revoke an existing valid contract, how
can it delegate such non-existent powers to the conservator under
the Central Bank Act?
Thus the Central Bank Act merely gives the conservator power to
revoke contracts that are, deemed to defective void, voidable,
unenforceable, rescissible.
o Hence the conservator merely takes place of a banks board of
directors.
Whatever the board cannot do such as repudiating a
contract validly entered into under the doctrine of
implied authority neither can the conservator do.
His power is not unilateral and he cannot simply repudiate
valid obligations of the Bank.
o To rule otherwise would be to enable a failing bank to become
solvent, at the expense of third parties, simply by getting the
conservator to unilaterally revoke all previous dealings which
had become unfavorable to the Bank, yielding nothing to perfected
contractual rights, nor vested interests of third parties who have
dealt with the Bank.
93.5. LAKAS NG MANGGAGAWANG MAKABAYAN V. ABIERA
36 SCRA 437 (1970)
Updated by K Pascual. 2014. 4C. Bill of Rights Poli Digests. Atty. Jack Jimenez. 4C. 139


Law is part of contract even without any stipulation.The principle is thus
well-settled that an existing law enters into and forms part of a valid contract
without the need for the parties expressly making reference to it. Only thus
could its validity insofar as some of its provisions are concerned be assured.
Thus, it does not admit of doubt that the collective bargaining contract between
petitioner and the Marinduque Mining and Industrial Corporation of 1967 must
be deemed to have incorporated within its terms the 1961 amendment of the
Industrial Peace Act exempting from the operation of a closed shop or a union
security shop agreement "members of any religious sects which prohibit
affiliation of their members in any such labor organization." If it were not so,
then the collective bargaining agreement itself could be properly assailed as the
freedom of contract recognized by the Civil Code while it empowers the parties
to establish such stipulations, clauses, terms and conditions as they may deem
convenient is limited by the requirement that they should not be "contrary to
law." It is a fundamental postulate that however broad the freedom of
contracting parties may be, it does not go so far as to countenance
disrespect for or failure to observe a legal prescription. The statute takes
precedence. A stipulation in a collective bargaining must yield to do it.
That is to adhere to the rule of law.
Jurisdiction over unfair labor practice.Under the Industrial Peace Act, the
jurisdiction over an unfair labor practice case, whether on the part of
management or of a labor union is vested with the Court of Industrial
Relations.
FACTS
Congress in 1961 amended the pertinent provision of the Industrial Peace
Act by exempting from the operation of a closed shop or a union security
shop agreement "members of any religious sects which prohibit affiliation
of their members in any such labor organization."
1


1
Republic Act No. 3350 amending paragraph 4, subsection (a) of Section 4 of
Republic Act No. 875. The Industrial Peace Act reads as follows: "(4) Provided,
That nothing in this Act or statute of the Republic of the Philippines shall
preclude an employer from making an agreement with a labor organization to
require as a condition of employment membership therein, if such labor
organization is the representative of the employees as provided in Section
The above saving clause notwithstanding, petitioner union, Lakas Ng
Manggagawang Makabayan (LMM), was able to prevail on the employer, the
Marinduque Mining and Industrial Corporation, to terminate the services of
private respondents, adherents of the Iglesia ni Cristo sect,
o one of the tenets of which prohibits membership in any labor
organization.
In the petition filed on September 9, 1968, the main reliance is placed by
petitioner Union on a CBA with the employer, the Marinduque Mining and
Industrial Corporation, entered into on November 1, 1967 and effective for
three years, one of the provisions of which required members of such Union
and those who become such thereafter to continue and remain with that
status "in good standing on a condition of continued employment."
Reference is then made to private respondents having been members
of petitioner Union even PRIOR to such collective bargaining
agreement.
Subsequently, however, on May 7, 1968 (during the effectivity of such CBA)
private respondents tendered their irrevocable resignations as such
members and disauthorized the Union to withdraw the check-off of union
dues private respondents relied on their being adherents of the Iglesia ni
Cristo sect and therefore falling within the terms of the Industrial Peace Act
( RA 3350).
After failing in its attempts to have them resume their membership
petitioner Union recommended to the employer their dismissal and
such recommendation was carried out pursuant to the existing CBA.
To regain their status as such employees, private respondents filed a
petition for mandamus with the Court of First Instance of Negros Occidental
presided by respondent Judge, the Honorable Carlos Abiera.
o CFI favored EE. Union lost.
o Thus unsuccessful, petitioner Union elevated the matter to this
Court and questioned the lower courts jurisdiction because
Union alleges that it should be the Court of Industrial
Relations that should exercise jurisdiction.
ISSUE: WON the CFI of Negros Occ. had jurisdiction? NO, BUT EXEMPTION
FROM CBA IS UPHELD

twelve, but such agreement shall not cover members of any religious sects
which prohibit affiliation of their members in any such labor organization."
Updated by K Pascual. 2014. 4C. Bill of Rights Poli Digests. Atty. Jack Jimenez. 4C. 140


SC: We find for petitioner UNION and proceed to explain why. WHEREFORE,
the writ of certiorari is granted annulling the order of respondent Judge of
August 5, 1968 granting the prayer of private respondents for a preliminary
mandatory injunction as well as the writ of preliminary injunction thereafter
issued on the 8th day of August, 1968. Respondent Judge is prohibited from
proceeding further with the mandamus petition filed by private respondents
which is ordered dismissed. The writ of preliminary injunction issued by this
Court under its resolution of September 12, 1968 is made permanent.
As stated at the outset, petitioner in the light of the controlling statutory
provision could validly impugn the jurisdiction of respondent Judge.
1. It does not admit of doubt that the collective bargaining contract
between petitioner and the Marinduque Mining and Industrial
Corporation of 1967 must be deemed to have incorporated within its
terms the 1961 amendment of the Industrial Peace Act exempting from
the operation of a closed shop or a union security shop agreement
"members of any religious sects which prohibit affiliation of their
members in any such labor organization." If it were not so, then the
collective bargaining agreement itself could be properly assailed as the
freedom of contract recognized by the Civil Code while it empowers the
parties to establish such stipulations, clauses, terms and conditions as
they may deem convenient is limited by the requirement that they
should not be "contrary to law".
2. The principle is thus well-settled that an existing law enters into
and forms part of a valid contract without the need for the parties
expressly making reference to it. Only thus could its validity insofar
as some of its provisions are concerned be assured.
a. On the assumption then that private respondents could lay
claim to the protection of the above exemption provision, the
fundamental question, the only one before this Court, is
whether such a statutory right could be vindicated in an
ordinary court as was done here or in the Court of Industrial
Relations?
3. There can be no dispute as to the answer. Under the Industrial Peace
Act, it is made an unfair labor practice for a labor organization, such as
petitioner here, "[t]o cause or attempt to cause an employer to
discriminate against an employee in violation of subsection (a) (4) or to
discriminate against an employee with respect to whom membership in
such organization has been denied or terminated on some ground other
than the usual terms and conditions under which membership or
continuation of membership is made available to other members."
a. Reference is thus made to an earlier subsection of said act
making it an unfair labor practice for an employer to
discriminate in regard to hire or tenure of employment or any
term or condition of employment to encourage or discourage
membership in any labor organization.
b. There is this proviso which as originally worded when the
measure was enacted in 1953 reads thus: "That nothing in this
Act or in any other Act or statute of the Republic of the
Philippines shall preclude an employer from making an
agreement with a labor organization to require as a condition
of employment membership therein, if such labor organization
is the representative of the employees as provided in section
twelve."15 Then, as was set forth at the outset of this opinion,
came the 1961 amendment with members of religious sects as
beneficiaries and thus entitled to exemption from a closed
shop or a union security shop. It is an integral part of the
section on what constitutes an unfair labor practice.
c. Under the next section of the Industrial Peace Act, the
jurisdiction over an unfair labor practice case, whether on
the part of management or of a labor union, is vested with
the Court of Industrial Relations.
i. Thus: "The Court shall have jurisdiction over the
prevention of unfair labor practices and is empowered
to prevent any person from engaging in any unfair
labor practice. This power shall be exclusive and shall
not be affected by any other means of adjustment or
prevention that has been or may be established by an
agreement, code, law or otherwise."
ii. This Court then ever since the effectivity of such
Act has no choice but to adhere to the view that the
Court of Industrial Relations and not a court of
first instance, is vested with jurisdiction over
every kind of an unfair labor practice case.
Petitioner must thus be sustained.
Updated by K Pascual. 2014. 4C. Bill of Rights Poli Digests. Atty. Jack Jimenez. 4C. 141


4. It only remains to be added that unless the legislative act granting
the above exemption to certain religious sects remains
unmodified and no challenge is hurled against its validity
resulting in its nullification, the parties to any contract must live
up faithfully to its terms. It is a fundamental postulate that
however broad the freedom of contracting parties may be, it does
not go so far as to countenance disrespect for or failure to observe
a legal prescription. The statute takes precedence; a stipulation in a
collective bargaining must yield to it. That is to adhere to the rule of
law.
94. GANZON VS. INSERTO
non-impairment clause
Ganzon wanted to foreclose the REM executed by Tajanglangit for its
obligation under a P/N of P40,000.
The REM covered a residential lot.
A day before the scheduled auction, Tajanglangit filed for injunction, and
sought to declare the extrajudicial foreclosure proceedings as null and void.
They claim that Ganzon had executed a Deed of Absolute Sale of the land in
favor of Tajanglangit, and that there was a proviso to the effect that Ganzon
guaranteed to have the occupants of the land to vacate the premises. It was
claimed that Ganzon failed to discharge this obligation, since there as still
occupants in the land. Hence, the extra-judicial foreclosure should be
nullified since Ganzon committed breach of his warranty.
They also claim that the REM did not contain any stipulation authorizing
extrajudicial foreclosure.
ON the other hand, Ganzons defense was that the REM was an entirely
different transaction from the sale of the lot.
Later, the Tajanglangit filed a motion for the release of the REM, in
exchange for putting up a surety bond worth P80,000.
The court allowed this.

ISSUE: may the court order the cancellation of the REM annotated in the TCT to
secure the payment of a PN and allow a substitution of the REM with a surety
bond to secure the same?
SC: NO.

The REM cannot be substituted by a surety bond. The REM in favor of Ganzon is
inseparable from the property. It is a right in rem, a lien on the property. To
substitute the REM with a surety bond would convert such lien from a right in
rem to a right in personam. This conversion cannot be ordered for it would
abridge the rights of the Mgee under the REM.

The court order violated the non-impairment of contracts clause. Substitution of
the REM with a surety bond to secure the payment of P40,000 would in effect
change the terms and conditions of the REM contract. Even before the trial on
the very issues affecting the contact, the lower court has directed a deviation
from its terms, diminished its efficiency, and dispensed with a primary
condition.

It should also be noted that GANZON DENIED THAT the P40,000 under the PN
which resulted in the execution of the REM to secure its payment was
CONNECTED with the sale of lot. Ganzon insisted that the REM is entirely
different from the sale, and it arose from another transaction.
95. PHILAMLIFE VS. AUDITOR GENERAL
non-impairment clause

Philamlife entered into a reinsurance treaty with other insurance
companies abroad AIRCO.
Updated by K Pascual. 2014. 4C. Bill of Rights Poli Digests. Atty. Jack Jimenez. 4C. 142


The Central Bank collected Forex margin on Philamlifes remittances to
Airco amounting to P268,000. This was pursuant to the Margin Law.
Philamlife however claimed refund on the ground that reinsurance
premiums remitted wre already paid when the reinsurance treaty was
perfected, and that pre-existing obligations were expressly exempt from the
margin fee.
Philamlife claimed that even the Monetary Board ruled that reinsurance
contracts entered into and approved before 1959 were exempt from the
forex margin fees, even if the remittances were made only after 1959
because such remittances are made only in the implementation of the
mother contract, a continuing contract which is the reinsurance treaty.
Still the Auditor General denied the refund. It claimed that the even if the
reinsurance treaty came about way back in 1950, the reinsurance
premiums remitted after the approval of the Margin Law in 1959, would
still be covered by the Margin Fees.
Philamlife argues that the Margin Law impairs the obligation of contracts. It
claims that under the said law, it would have paid much more to have the
continuing benefit of reinsurance of its risk than it has been required to do
so by the reinsurance treaty in question. And that the theoretical equality
between the conrtacting parties would be disturbed and one of them would
be placed at a distinct disadvantage in relation to the other.

ISSUE: Does the margin law violate the non-impairment clause?
SC: NO. MARGIN LAW VALID.

This pose at once loses potency on the face of the rule long recognized that
existing laws form part of the contract "as the measure of the obligation to
perform them by the one party and the right acquired by the other." Stated
otherwise, "the obligation does not inhere, and. subsist in the contract itself,
propio vigore, but in the law applicable to the contract." Indeed, Article 1315 of
the Civil Code gives out the precept that parties to a perfected contract "are
bound ... to all the consequences which, according to their nature, may be in
keeping with law.

Accordingly, when petitioner entered into the reinsurance treaty of January 1,
1950 with Airco, it did so with the understanding that the municipal laws of the
Philippines at the time said treaty was executed, became an unwritten condition
thereof. Such municipal laws constitute part of the obligation of contract. It is in
this context that we say that Republic Act 265, the Central Bank Act, enacted on
June 15, 1948 - previous to the date of the reinsurance treaty - became a part of
the obligation of contract created by the latter. And under Republic Act 265,
reasonable restrictions may be imposed by the State through the Central Bank
on all foreign exchange transactions "in order to protect the international
reserve of the Central Bank during an exchange crisis." The Margin Law is
nothing more than a supplement to the Central Bank Act; it is a reasonable
restriction on transactions in foreign exchange.

The Margin Law, is a remedial currency measure. It was thus passed to reduce
as far as is practicable the excessive demand for foreign exchange. There cannot
be an impairment of the obligation of contracts. For, the State may, through its
police power, adopt whatever economic policy may reasonably be deemed to
promote public welfare, and to enforce that policy by legislation adapted to its
purpose. The freedom of contract, under our system of government, is not
meant to be absolute. The same is understood to be subject to reasonable
legislative regulation aimed at the promotion of public health, morals, safety
and welfare. In other words, the constitutional guaranty of non-impairment of
obligations of contract is limited by the exercise of the police power of the State,
in the interest of public health, safety, morals and general welfare."

The economic interests of the State may justify the exercise of its continuing and
dominant protective power notwithstanding interference with contracts." It
bears repetition to state at this point that the Margin Law is part of the
economic "Stabilization Program" of the country.

Updated by K Pascual. 2014. 4C. Bill of Rights Poli Digests. Atty. Jack Jimenez. 4C. 143


Neither property rights nor contract rights are absolute; for government cannot
exist if the citizen may at will use his property to the detriment of his fellows, or
exercise his freedom of contract to work them harm. Equally fundamental with
the private right is that of the public to regulate it in the common interest.
"Contracts, however express, cannot fetter the constitutional authority of the
Congress. Contracts may create rights of property, but when contracts deal with
a' subject matter which lies within the control of the Congress, they have a
congenital infirmity. Parties cannot remove their transactions from the reach of
dominant constitutional power by making contracts about them."

Not only are existing laws read into contracts in order to fix obligations as
between the parties, but the reservation of essential attributes of
sovereign power is also read into contracts as a postulate of the legal
order.

Refund denied.
SECTION 12.
96. PEOPLE VS. TAMPUS
rights of the accused custodial investigation
Tampus, who was a member of the OXO Gang, was convicted of murder
for avenging the stabbing of his friend Rosales (also an OXO member),
by a member of the rival gang, the Batang Mindanao Gang.
Tampus is a TB patient and a prisoner at the national penitentiary.
o He stabbed the victim as he was emerging from the toilet.
He later surrendered to the prison guards with the knife used.
TWO days later, he made an extrajudicial confession before this prison
guard.
o At arraignment, he pleaded guilty to the charge of murder.
At the arraignment, the trial court inform them the gravity of the charge
and that the death penalty might be imposed. He reiterated his plea of guilt.
The judge required the prosecution to present evidence.
o At the witness stand, he still affirmed his confession.
Upon appeal, Tampus lawyer contended that he was denied the right
to a public trial because the arraignment and hearing were held at the
state penitentiary.
The New Bilbilid Prison was the venue of the arraignment and the
hearing (not at the Makati Court) because one of the co-accused refused, for
security reasons, to be brought to Makati. That is why the Court allowed
arraignment and trial of Tampus to be held at Muntinlupa.
Also upon appeal, Tampus lawyer also argues that the extrajudicial
confession was taken in violation of their constitutional rights.
ISSUE: Right to public trial
SC: The record does not show that the public was actually excluded from the
place where the trial was held or that the accused was prejudiced by the holding
of the trial at the national penitentiary. The fact that for the convenience of
the witnesses a case is tried in Bilibid without any objection on the part of
the accused is not a ground for the reversal of a conviction.
The accused may waive his right to have a public trial as shown in the rule
that the trial court may motu-proprio exclude the public from the courtroom
when the evidence to be offered is offensive to decency or public morals.
ISSUE: Extrajudicial confession.
The lawyer argued that before the confession was taken, Tampus had been
interrogated 2 days before, that is on the day of the killing, and that during such
custodial investigation, Tampus was not informed of his right to counsel and to
remain silent.
SC: VOLUNTARILY AND SPONTANEOUSLY MADE EXTRAJUDICIAL
CONFESSION. As to the extrajudicial confession, there is no doubt that the
Updated by K Pascual. 2014. 4C. Bill of Rights Poli Digests. Atty. Jack Jimenez. 4C. 144


confession was voluntarily made. The truth is that even before the custodial
investigation on the day of the killing, Tampus had already admitted it, when
after coming out of the toilet, they surrendered to the first jail guard whom they
encountered. That was a spontaneous statement, elicited without any
interrogation, and part of the res gestae, at the same time a voluntary
confession of guilt.

The accused gave the confession or statement at the spur of the moment,
without any urging. Thus, they waived their right to remain silent and to have
counsel. That admission was confirmed by their extrajudicial confession,
plead of guilty and testimony in open court.
Furthermore, the crime was proven beyond reasonable doubt by the evidence
of the prosecution. The court during the trial is not duty-bound to apprise the
accused that he has the right to remain silent. It is his counsel who should claim
that right for him. If he does not claim it and he calls the accused to the witness
stand, then he waives that right.
Teehankee dissents: it should be the trial courts duty to inform the accused of
his right to remain silent.

97. GAMBOA VS. CRUZ
rights of the accused custodial investigation right to COUNSEL
Gamboa was arrested for vagrancy WITHOUT a warrant. He was brought
to the HQ.
During the police lineup of 5 detainees, the complainant Bernal, pointed
and identified Gamboa as the one who was the companion of the real
culprit.
While Bernal was being interviewed by the police, Gamboa was also there
seated.
Gamboa was later charged with robbery.
He filed a demurrer to evidence, claiming that the police lineup, was
done without prior notice, and in the absence of his counsel, a clear
violation of the constitutional rights.
ISSUE: Was the right to counsel violated?
SC: NO.
The right to counsel attaches upon the start of an investigation, i.e. when the
investigating officer starts to ask questions to elicit information and for
confessions or admissions from the respondent/accused. At such point or stage,
the person being interrogated must be assisted by counsel to avoid the
pernicious practice of extorting false or coerced admissions or confessions from
the lips of the person undergoing interrogation, for the commission of an
offense.
Any person under investigation must, among other things, be assisted by
counsel. The above-cited provisions of the Constitution are clear. They leave no
room for equivocation. Accordingly, in several cases, this Court has consistently
held that no custodial investigation shall be conducted unless it be in the
presence of counsel, engaged by the person arrested, or by any person in his
behalf, or appointed by the court upon petition either of the detainee himself, or
by anyone in his behalf, and that, while the right may be waived, the waiver
shall not be valid unless made in writing and in the presence of counsel.
The police line-up was not part of the custodial inquest, hence, petitioner
was not yet entitled, at such stage, to counsel.
When petitioner was identified by the complainant at the police line-up,
he had not been hold yet to answer for a criminal offense.
Thus, it was held that when the process had not yet shifted from the
INVESTIGATORY to the ACCUSATORY as when police investigation does
not elicit a confession the accused may not yet avail of the services of his
lawyer.
Since petitioner in the course of his identification in the police line-up had not
yet been held to answer for a criminal offense, he was, therefore, not deprived
of his right to be assisted by counsel because the accusatory process had not yet
set in. The police could not have violated petitioner's right to counsel and due
process as the confrontation between the State and him had not begun. In fact,
Updated by K Pascual. 2014. 4C. Bill of Rights Poli Digests. Atty. Jack Jimenez. 4C. 145


when he was identified in the police line-up by complainant he did not
give any statement to the police. He was, therefore, not interrogated at all
as he was not facing a criminal charge. Far from what he professes, the police
did not, at that stage, exact a confession to be used against him. For it was not
he but the complainant who was being investigated at that time. He was
ordered to sit down in front of the complainant while the latter was being
investigated" Petitioner's right to counsel had not accrued.
The 6
th
and 14
th
Am. to counsel attaches only at or after the time that adversary
judicial proceedings have been initiated against him. In our 1987 Constitution,
this was expanded, such that the right to counsel attaches at the start of
investigation against a respondent and, therefore, even before adversary
judicial proceedings against the accused have begun.
While the Court finds no real need to afford a suspect the services of
counsel during a police line-up, the moment there is a move or even an
urge of said investigators to elicit admissions or confessions or even plain
information which may appear innocent or innocuous at the time, from
said suspect, he should then and there be assisted by counsel, unless he
waives the right, but the waiver shall be made in writing and in the
presence of counsel.
YAP dissents: the investigatory proceeding has already started when the
accused was singled out and ordered to sit down with the complainant. Thus,
custodial investigation has begun.
98. ESCOBEDO VS. ILLINOIS
custodial investigation rights
Escobedo was convicted of murder in Illinois.
Escobedo apparently shot his brother-in-law. He was thereafter arrested
without a warrant.
o He made no statements at the police HQ, and was released
subsequently.
Benedict, the co-accused was the one in custody of the police.
o Benedict told the police that it was really Escobedo who fired
the shot.
o Escobedo was re-arrested.
He was handcuffed and taken to the HQ again.
o The detectives told him he might as well admit the crime,
because he was in custody and couldnt walk out the door.
His lawyer arrived later but was told that he could not see Escobedo
unless he first gets a writ of habeas corpus.
o Escobedo also requested for his lawyer but was denied.
Escobedo and his lawyer were afforded no opportunity to consult during
the course of the entire interrogation.
o It was also established that no one during the course of the
interrogation advised Escobedo of his rights.
Escobedo was eventually convicted.
ISSUE: WON Escobedos Rights in regard to custodial investigation were
violated? YES
SC:
The interrogation here was conducted before Escobedo was even formally
indicted.
When he requested but was denied an opportunity to consult his
lawyer, the investigation had ceased to be a general investigation of an
unsolved crime.
Petitioner had become the accused, and the purpose of the
interrogation was to get him to confess his guilt despite his
constitutional right not to do so.
o Without informing his of his absolute right to remain silent in the
face of this accusation, the police urged him to make a Statement.
o The result was to produce upon his mind the fear that if he
remained silent it would considered an admission of guilt, and
Updated by K Pascual. 2014. 4C. Bill of Rights Poli Digests. Atty. Jack Jimenez. 4C. 146


therefore render certain his being committed as the guilty
person.
He was a layman, he was undoubtedly unaware that under Illinois law, an
admission of mere complicity in the murder plot was legally as damaging as
admission of firing the fatal shots. A counsel should have been essential to
advise him of his right in such delicate situation. This was the stage when
the legal aid and advice were most critical to the accused.
The investigation here was no longer a general inquiry into a crime
but has begun to focus on a particular suspect, the suspect being taken
into police custody, and the police carrying out the process of
interrogations for the purpose of eliciting incriminating statements.
o He has requested and has been denied an opportunity to
consult with his lawyer, and the police have not effectively
warned him of his right to remain silent.
o When the process shifts from investigatory to accusatory,
when its focus is on the accused and its purpose is to elicit a
confession, the adversary proceeding begins to operate, and
the accused must be allowed to consult with his lawyer.
As such, no statement elicited by the police during the interrogation may be
used against him in a criminal trial.
99. MIRANDA VS. ARIZONA
rights of the accused
Syllabus

In each of these cases, the defendant, while in police custody, was questioned by
police officers, detectives, or a prosecuting attorney in a room in which he was
cut off from the outside world. None of the defendants was given a full and
effective warning of his rights at the outset of the interrogation process. In
all four cases, the questioning elicited oral admissions, and, in three of
them, signed statements as well, which were admitted at their trials. All
defendants were convicted, and all convictions, except in No. 584, were affirmed
on appeal.

Held:

1. The prosecution may not use statements, whether exculpatory or inculpatory,
stemming from questioning initiated by law enforcement officers after a person
has been taken into custody or otherwise deprived of his freedom of action in
any significant way, unless it demonstrates the use of procedural safeguards
effective to secure the Fifth Amendment's privilege against self-incrimination.
Pp. 444-491.

(a) The atmosphere and environment of incommunicado interrogation as it
exists today is inherently intimidating, and works to undermine the privilege
against self-incrimination. Unless adequate preventive measures are taken to
dispel the compulsion inherent in custodial surroundings, no statement
obtained from the defendant can truly be the product of his free choice. Pp. 445-
458.

(b) The privilege against self-incrimination, which has had a long and expansive
historical development, is the essential mainstay of our adversary system, and
guarantees to the individual the "right to remain silent unless he chooses to
speak in the unfettered exercise of his own will," during a period of custodial
interrogation [p437] as well as in the courts or during the course of other
official investigations. Pp. 458-465.

(c) The decision in Escobedo v. Illinois, 378 U.S. 478, stressed the need for
protective devices to make the process of police interrogation conform to the
dictates of the privilege. Pp. 465-466.

(d) In the absence of other effective measures, the following procedures to
safeguard the Fifth Amendment privilege must be observed: the person in
custody must, prior to interrogation, be clearly informed that he has the right to
remain silent, and that anything he says will be used against him in court; he
must be clearly informed that he has the right to consult with a lawyer and to
have the lawyer with him during interrogation, and that, if he is indigent, a
lawyer will be appointed to represent him. Pp. 467-473.

(e) If the individual indicates, prior to or during questioning, that he wishes to
remain silent, the interrogation must cease; if he states that he wants an
attorney, the questioning must cease until an attorney is present. Pp. 473-474.

(f) Where an interrogation is conducted without the presence of an attorney
and a statement is taken, a heavy burden rests on the Government to
Updated by K Pascual. 2014. 4C. Bill of Rights Poli Digests. Atty. Jack Jimenez. 4C. 147


demonstrate that the defendant knowingly and intelligently waived his right to
counsel. P. 475.

(g) Where the individual answers some questions during in-custody
interrogation, he has not waived his privilege, and may invoke his right to
remain silent thereafter. Pp. 475-476.

(h) The warnings required and the waiver needed are, in the absence of a fully
effective equivalent, prerequisites to the admissibility of any statement,
inculpatory or exculpatory, made by a defendant. Pp. 476-477.

2. The limitations on the interrogation process required for the protection of the
individual's constitutional rights should not cause an undue interference with a
proper system of law enforcement, as demonstrated by the procedures of the
FBI and the safeguards afforded in other jurisdictions. Pp. 479-491.

3. In each of these cases, the statements were obtained under circumstances
that did not meet constitutional standards for protection of the privilege against
self-incrimination. Pp. 491-499.

FACTS
On March 13, 1963, $8.00 in cash was stolen from a Phoenix, Arizona
bank worker.
o Police suspected and arrested Ernesto Miranda for committing the
theft.
During two-hours of questioning, Mr. Miranda, who was never offered
a lawyer, confessed not only to the $8.00 theft, but also to kidnapping
and raping an 18-year-old woman 11 days earlier.
Based largely on his confession, Miranda was convicted and sentenced
to 20 years in jail.
Miranda's attorneys appealed. First unsuccessfully to the Arizona Supreme
Court, and next to the U.S. Supreme Court.
Mirandas case was consolidated with 3 other cases (TOTAL 4 CASES)
o In each of these cases the defendant while in police custody
was questioned by police officers, detectives, or a prosecuting
attorney in a room in which he was cut off from the outside
world.
o None of the defendants was given a full and effective warning of his
rights at the outset of the interrogation process.
o In all four cases the questioning elicited oral admissions, and in
three of them signed statements as well, which were admitted at
their trials. All defendants were convicted and all convictions.

ISSUE: We decide in each of these cases is the admissibility of statements
obtained from a defendant questioned while in custody or otherwise
deprived of his freedom of action in any significant way.
o They all share salient features - incommunicado interrogation of
individuals in a police-dominated atmosphere, resulting in
self-incriminating statements without full warnings of
constitutional rights.
(the case is about the origin and rationale behind Sec 12)
SC:
The prosecution may not use statements, whether exculpatory or
inculpatory, stemming from custodial interrogation of the defendant
unless it demonstrates the use of procedural safeguards effective to
secure the privilege against self-incrimination.
By custodial interrogation, we mean questioning initiated by law
enforcement officers after a person has been taken into custody or
otherwise deprived of his freedom of action in any significant way.
o Prior to any questioning, the person must be warned that he has a
right to remain silent, that any statement he does make may be
used as evidence against him, and that he has a right to the
presence of an attorney, either retained or appointed.
o The defendant may waive effectuation of these rights, provided the
waiver is made voluntarily, knowingly and intelligently.
o If, however, he indicates in any manner and at any stage of the
process that he wishes to consult with an attorney before speaking
there can be no questioning. Likewise, if the individual is alone and
indicates in any manner that he does not wish to be interrogated,
the police may not question him.
History of Right to Remain Silent and Right to Counsel:
Updated by K Pascual. 2014. 4C. Bill of Rights Poli Digests. Atty. Jack Jimenez. 4C. 148


the famous Wickersham Report to Congress by a Presidential Commission,
where it was stated that police violence and the "third degree" flourished
at that time.
o The police resorted to physical brutality - beating, hanging,
whipping - and to sustained and protracted questioning
incommunicado in order to extort confessions.
In-custody interrogation is psychologically rather than physically
oriented.
o Coercion can be mental as well as physical, and that the blood of
the accused is not the only hallmark of an unconstitutional
inquisition." Interrogation still takes place in privacy.
There were MANUALS given to the police.
o The officers are told by the manuals that the "principal
psychological factor contributing to a successful interrogation
is privacy - being alone with the person under interrogation.
The manuals instruct the police to display an air of
confidence in the suspect's guilt and from outward
appearance to maintain only an interest in confirming
certain details.
The manuals suggest that the suspect be offered legal
excuses for his actions in order to obtain an initial
admission of guilt.
When the techniques described above prove unavailing,
the texts recommend they be alternated with a show of
some hostility.
One ploy often used has been termed the "friendly-unfriendly" or the "Mutt
and Jeff" act. Mutt, the relentless investigator, who knows the subject is
guilty and is not going to waste any time. Jeff, on the other hand, is
obviously a kindhearted man. The technique is applied by having both
investigators present while Mutt acts out his role. Jeff may stand by quietly
and demur at some of Mutt's tactics. When Jeff makes his plea for
cooperation, Mutt is not present in the room."
Then there is the Reverse line up. The accused is placed in a line-up, but
this time he is identified by several fictitious witnesses or victims who
associated him with different offenses. It is expected that the subject will
become desperate and confess to the offense under investigation in order to
escape from the false accusations."
There are but samples of interrogation techniques, the setting prescribed
by the manuals and observed in practice becomes clear.
o In essence, it is this: To be alone with the subject is essential to
prevent distraction and to deprive him of any outside support. The
aura of confidence in his guilt undermines his will to resist. He
merely confirms the preconceived story the police seek to have him
describe.
(then the court cites several cases.. blah blah blah..)
In each of the cases, the defendant was thrust into an unfamiliar
atmosphere and run through menacing police interrogation procedures.
o The potentiality for compulsion is forcefully apparent, for example,
in Miranda, where the indigent Mexican defendant was a
seriously disturbed individual with pronounced sexual
fantasies.
o The fact remains that in none of these cases did the officers
undertake to afford appropriate safeguards at the outset of the
interrogation to insure that the statements were truly the product
of free choice.
It is obvious that such an interrogation environment is created for no
purpose other than to subjugate the individual to the will of his examiner.
This atmosphere carries its own badge of intimidation. The current
practice of incommunicado interrogation is at odds with one of our
Nation's most cherished principles - that the individual may not be
compelled to incriminate himself.

History of the Right Against Self-Incrimination:
the trial of one John Lilburn, a vocal anti-Stuart Leveller, who was made to
take the Star Chamber Oath in 1637. The oath would have bound him to
answer to all questions posed to him on any subject. If he does not answer,
he will be killed. Later, the Parliament abolished the inquisitorial Court of
Star Chamber.
In sum, the privilege is fulfilled only when the person is guaranteed the
right "to remain silent unless he chooses to speak in the unfettered
exercise of his own will. (Note: The right to remain silent and the right to
counsel assures that the right to self-incrimination is protected Sec 17.
But Section 12 and Section 17 are separate and distinct constitutional rights
under the Philippine Constitution).
Updated by K Pascual. 2014. 4C. Bill of Rights Poli Digests. Atty. Jack Jimenez. 4C. 149


ISSUE: The question in these cases is whether the privilege TO REMAIN
SILENT is fully applicable during a period of custodial interrogation? YES
OF COURSE
SC:
There can be no doubt that the Fifth Amendment privilege is available
outside of criminal court proceedings and serves to protect persons in all
settings in which their freedom of action is curtailed in any significant way
from being compelled to incriminate themselves.
If a person in custody is to be subjected to interrogation, he must first
be informed in clear and unequivocal terms that he has the right to
remain silent.
o The warning of the right to remain silent must be
accompanied by the explanation that anything said can and
will be used against the individual in court.
o This warning is needed in order to make him aware not only of
the privilege, but also of the consequences of forgoing it.
o It is only through an awareness of these consequences that there
can be any assurance of real understanding and intelligent exercise
of the privilege.
The circumstances surrounding in-custody interrogation can operate very
quickly to overbear the will of one merely made aware of his privilege by
his interrogators. Therefore, the right to have counsel present at the
interrogation is indispensable to the protection of the Fifth Amendment
privilege under the system we delineate today. Our aim is to assure that the
individual's right to choose between silence and speech remains unfettered
throughout the interrogation process. A mere warning given by the
interrogators is not alone sufficient to accomplish that end. The need for
counsel to protect the Fifth Amendment privilege comprehends not merely
a right to consult with counsel prior to questioning, but also to have counsel
present during any questioning if the defendant so desires.
If the accused decides to talk to his interrogators, the assistance of
counsel can mitigate the dangers of untrustworthiness.
o With a lawyer present the likelihood that the police will practice
coercion is reduced, and if coercion is nevertheless exercised the
lawyer can testify to it in court.
o The presence of a lawyer can also help to guarantee that the
accused gives a fully accurate statement to the police and that the
statement is rightly reported by the prosecution at trial.
The failure to ask for a lawyer does not constitute a waiver. No effective
waiver of the right to counsel during interrogation can be recognized unless
specifically made after the warnings we here delineate have been given. The
defendant who does not ask for counsel is the very defendant who most
needs counsel. Where the assistance of counsel is a constitutional requisite,
the right to be furnished counsel does not depend on a request.
Accordingly we hold that an individual held for interrogation must be
clearly informed that he has the right to consult with a lawyer and to
have the lawyer with him during interrogation under the system for
protecting the privilege we delineate today. As with the warnings of the
right to remain silent and that anything stated can be used in evidence
against him, this warning is an absolute prerequisite to interrogation.
If an individual indicates that he wishes the assistance of counsel
before any interrogation occurs, the authorities cannot rationally
ignore or deny his request on the basis that the individual does not
have or cannot afford a retained attorney.
o The financial ability of the individual has no relationship to the
scope of the rights involved here. The privilege against self-
incrimination secured by the Constitution applies to all individuals.
The need for counsel in order to protect the privilege exists for
the indigent as well as the affluent.
In order fully to apprise a person interrogated of the extent of his rights
under this system then, it is necessary to warn him not only that he has
the right to consult with an attorney, but also that if he is indigent a
lawyer will be appointed to represent him.
o Without this additional warning, the admonition of the right to
consult with counsel would often be understood as meaning only
that he can consult with a lawyer if he has one or has the funds to
obtain one.
Once warnings have been given, the subsequent procedure is clear. If
the individual indicates in any manner, at any time prior to or during
questioning, that he wishes to remain silent, the interrogation must
cease. If the individual cannot obtain an attorney and he indicates that
he wants one before speaking to police, they must respect his decision
to remain silent.
If the interrogation continues without the presence of an attorney and
a statement is taken, a heavy burden rests on the government to
demonstrate that the defendant knowingly and intelligently waived
Updated by K Pascual. 2014. 4C. Bill of Rights Poli Digests. Atty. Jack Jimenez. 4C. 150


his privilege against self-incrimination and his right to retained or
appointed counsel.
o An express statement that the individual is willing to make a
statement and does not want an attorney followed closely by a
statement could constitute a waiver. But a valid waiver will not be
presumed simply from the silence of the accused after warnings
are given or simply from the fact that a confession was in fact
eventually obtained.
Where in-custody interrogation is involved, there is no room for the
contention that the privilege is waived if the individual answers some
questions or gives some information on his own prior to invoking his
right to remain silent when interrogated. Moreover, any evidence that
the accused was threatened, tricked, or cajoled into a waiver will, of course,
show that the defendant did not voluntarily waive his privilege.
The principles announced today deal with the protection which must be
given to the privilege against self-incrimination when the individual is first
subjected to police interrogation while in custody at the station or
otherwise deprived of his freedom of action in any significant way.
To summarize, we hold that when an individual is taken into custody
or otherwise deprived of his freedom by the authorities in any
significant way and is subjected to questioning, the privilege against
self-incrimination is jeopardized.
o Procedural safeguards must be employed to protect the privilege,
and unless other fully effective means are adopted to notify the
person of his right of silence and to assure that the exercise of the
right will be scrupulously honored, the following measures are
required.
o He must be warned prior to any questioning that he has the right to
remain silent, that anything he says can be used against him in a
court of law, that he has the right to the presence of an attorney,
and that if he cannot afford an attorney one will be appointed for
him prior to any questioning if he so desires.
o Opportunity to exercise these rights must be afforded to him
throughout the interrogation. After such warnings have been given,
and such opportunity afforded him, the individual may knowingly
and intelligently waive these rights and agree to answer questions
or make a statement. But unless and until such warnings and
waiver are demonstrated by the prosecution at trial, no evidence
obtained as a result of interrogation can be used against him.
Thus, the case is remanded. But the confession cannot be admitted.
Trivia: Ernesto Miranda was given a second trial at which his confession was
not presented. Based on the evidence, Miranda was again convicted of
kidnapping and rape. He was paroled from prison in 1972 having served 11
years. In 1976, Ernesto Miranda, age 34, was stabbed to death in a fight.
Summary: ASUC: ARREST SILENT USE AGAINST U - COUNSEL
1. You are under arrest.
2. You have the right to remain silent.
3. Anything you say can and will be used against you in a court of law.
4. You have the right to an attorney present now, or any future questioning.
100. PEOPLE VS. TAN
custodial investigation rights
Tan was charged with highway robbery and murder for stealing a
Honda motorcycle and killing its owner Saavedra.
He pleaded not guilty.
It was established that Tan and his co-accsued were invited by the
police officers in connection with two other robbery cases reported in
Lucena City.
o CONFESSION Tan allegedly gave an explicit account of what
really happened.
He allegedly told the police that they admitted
responsibility for the loss of the motorcycle and death of
Saavedra.
It was also averred that they sold the motorcycle to Teves.
The police testified that when they invited Tan to the HQ, there was no
warrant of arrest yet.
Updated by K Pascual. 2014. 4C. Bill of Rights Poli Digests. Atty. Jack Jimenez. 4C. 151


o The police merely informed Tan that he was a suspect for this
robbery of motorcycle, as well as the 2 other robbery cases in
Lucena.
o The police claims that they were merely conversing with Tan
inside the police station, that is why they did not bother to
inform Tan of his rights.
Tans defense was that his constitutional rights were violated, hence he
cannot be convicted.
ISSUE: May the confession of an accused, given before a police investigator
upon invitation and without the benefit of counsel, be admissible in
evidence against him? NO.
SC: ACQUITTED. The records of this case do not indicate that appellant was
assisted by counsel when he made such waiver.
The Constitution abhors an uncounselled confession or admission and
whatever information is derived therefrom shall be regarded as
inadmissible in evidence against the confessant.
R.A. No. 7438 reinforced the constitutional mandate protecting the
rights of persons under custodial investigation.
Under said law, custodial investigation' shall include the practice of
issuing an 'invitation' to a person who is investigated in connection
with an offense he is suspected to have committed, without prejudice
to the liability of the 'inviting' officer for any violation of law."
Custodial investigation involves any questioning initiated by law
enforcement authorities after a person is taken into custody or
otherwise deprived of his freedom of action in any significant manner.
The rules on custodial investigation begin to operate as soon as the
investigation ceases to be a general inquiry into an unsolved crime and
begins to focus a particular suspect, the suspect is taken into custody, and
the police carries out a process of interrogations that tends itself to eliciting
incriminating statements that the rule begins to operate.
Furthermore, not only does the fundamental law impose, as a requisite
function of the investigating officer, the duty to explain those rights to the
accused but also that there must correspondingly be a meaningful
communication to and understanding thereof by the accused. A mere
perfunctory reading by the constable of such rights to the accused
would thus not suffice.
A confession to be admissible must satisfy the following requirements:
VIEW Voluntary; Independent Counsel; Express; Writing.
o (1) it must be voluntary;
o (2) it must be made with the assistance of competent and
independent counsel;
o (3) it must be express; and
o (4) it must be in writing.
While the Constitution sanctions the waiver of the right to counsel, it must,
however, be "voluntary, knowing and intelligent, and must be made in the
presence and with the assistance of counsel." Even if the confession
contains a grain of truth, if it was made without the assistance of counsel, it
becomes inadmissible in evidence, regardless of the absence of coercion or
even if it had been voluntarily given.
(here, it was shown that the police invited Tan to the HQ merely out of
curiosity.) The constitutional rights of appellant, particularly the right to
remain silent and to counsel, are impregnable from the moment he is
investigated in connection with an offense he is suspected to have
committed, even if the same be initiated by mere invitation. "
101. PEOPLE VS. LABTAN
custodial investigation rights
Feliciano (co-accused) was convicted by RTC-CDO of highway robbery
and robbery with homicide, on the basis of a sworn statement which
he later repudiated during trial.
Feliciano was brought to the HQ where he executed a sworn statement.
o In that statement, he was assisted by a de officio lawyer, Atty
Pepito Chavez.
o The portions of the statement are as follows:
Q:Will you get a lawyer of your own to defend you in this
investigation?
A:No, sir. I can't (sic) pay the services of lawyer.
Updated by K Pascual. 2014. 4C. Bill of Rights Poli Digests. Atty. Jack Jimenez. 4C. 152


Q:Since you will not get your own lawyer, will you
agree that I'll (sic) give you Atty. Pepito Chavez as your
counsel de officio in this investigation?
A:Yes, sir. I agree that Atty. Pepito Chavez will be my
lawyer for the ascertainment of the truth.
Q: What have you done, if any?
A:On March 1993, I participated in a hold-up of a
certain driver Mr. Mercado who owned a jeep and we
got a car stereo including the jeep. We brought the
jeep to Buntong, Camaman-an and the driver,
however, we freed the driver later.
Feliciano during trial, repudiated his sworn statement contending that
he was questioned for over an hour by the police, who hit him in the fact, in
the chest, etc, and thereafter locked him up in jail. Later, he was forced by
the police to sign a piece of paper. Then he was brought to the office of Atty.
Chavez. He did [not] know what was happening. Atty. Chavez then signed
the papers also and did not even talk to him whatsoever.
ISSUE: Was confession in the sworn statement admissible in evidence? Did the
extrajudicial confession violate his right to counsel? Was Atty Chavez an
effective Counsel?
SC: It must be a right to an effective and vigilant counsel.
As the result of the changes in the patterns of police investigation, today's
accused confronts both expert adversaries and the judicial system well
before his trial begins. It is therefore appropriate to extend the counsel
guarantee to critical stages of prosecution even before the trial. The law
enforcement machinery at present involves critical confrontations of the
accused by the prosecution at pre-trial proceedings 'where the result might
well settle the accused's fate and reduce the trial itself to a mere formality.
We find that accused Feliciano had been denied of his right to have a
competent and independent counsel when he was questioned in the
Police Station.
o The police admitted that he started questioning Felciano
regarding his involvement in the killing of jeepney driver
Florentino Bolasito, notwithstanding the fact that he had
not been apprised of his right to counsel.
At that point, accused-appellant had been subjected to custodial
investigation without a counsel. A person is deemed under custodial
investigation where the police investigation is no longer a general inquiry
into an unsolved crime but has began to focus on a particular suspect who
had been taken into custody by the police who carry out a process of
interrogation that lends itself to elicit incriminating statements.
When SPO1 Cuarez investigated accused-appellant Feliciano, the latter
was already a suspect in the killing of jeepney driver Bolasito as shown
by the joint affidavit of SPO4 Johny Salcedo and SPO1 Florencio Bagaipo
who were the ones who arrested Feliciano.
The prosecution tried to establish that Atty. Pepito Chavez provided
effective and independent counselling to accused-appellant Feliciano
which cured the initial lack of counsel. However, this is belied by the
very testimony of Atty. Chavez showing he performed his duty in a
lackadaisical fashion.
o Atty. Chavez admitted that, he told the accused, that he will
just follow later because at that time when the accused came to
the office he (Atty Chavez) was working on some paper works.
o Atty. Chavez also admitted that, I started my investigation or
confrontation with Henry Feliciano informing him, appraising
him of his constitutional right to counsel, that he has a right to
remain silent and appraise him if it is his desire that I be his
lawyer because I told him if he has no desire that I will be his
lawyer, then he can look for another.

(see original for full transcript)
The right to counsel is a fundamental right and contemplates not a
mere presence of the lawyer beside the accused.
The term "EFFECTIVE and VIGILANT counsel" was explained thus:
o "necessarily and logically [requires] that the lawyer be present
and able to advise and assist his client from the time the
confessant answers the first question asked by the
investigating officer until the signing of the extrajudicial
confession.
o Moreover, the lawyer should ascertain that the confession
is made voluntarily and that the person under
Updated by K Pascual. 2014. 4C. Bill of Rights Poli Digests. Atty. Jack Jimenez. 4C. 153


investigation fully understands the nature and the
consequence of his extrajudicial confession in relation to
his constitutional rights.
o A contrary rule would undoubtedly be antagonistic to the
constitutional rights to remain silent, to counsel and to be
presumed innocent.
An effective counsel is one who can be made to act in protection of his
[accused's] rights, and not by merely going through the motions of
providing him with anyone who possesses a law degree.
Atty. Chavez did not provide the kind of counselling required by the
Constitution. He did not explain to accused- the consequences of his
action - that the sworn statement can be used against him and that it is
possible that he could be found guilty and sent to jail.
We also find that Atty. Chavez's independence as counsel is suspect -
he is regularly engaged by the Cagayan de Oro City Police as counsel
de officio for suspects who cannot avail the services of counsel. He
even received money from the police as payment for his services.
We also find that Atty. Chavez himself notarized the sworn statement
seriously compromised his independence. By doing so, he vouched for
the regularity of the circumstances surrounding the taking of the sworn
statement by the police. He cannot serve as counsel of the accused and
the police at the same time. There was a serious conflict of interest on
his part.
An independent counsel cannot be a special counsel, public or private
prosecutor, counsel of the police, or a municipal attorney whose interest is
admittedly adverse to the accused.
We have examined the three-page sworn statement allegedly executed by
Feliciano and we failed to see any badge of spontaneity and credibility
to it.
It was a stereotyped 'advice' appearing in practically all extrajudicial
confessions which are later repudiated has assumed the nature of
'legal form' or mode. Police investigators either automatically type it
together with the curt 'Opo' as the answer or ask the accused to sign it or
even copy it in their handwriting. Its tired, punctilious, fixed and artificially
stately style does not create an impression of voluntariness or even
understanding on the part of the accused. The showing of a spontaneous,
free and unconstrained giving up of a right is missing
Note: RA 7438 also provides for the rights of persons under custodial
investigation.

Sworn Statement Inadmissible. ACQUITTED.
SECTION 13. RIGHT TO BAIL
Section 13.
All persons,
o except those charged with offenses punishable by reclusion
perpetua when evidence of guilt is strong,
o shall, before conviction, be bailable by sufficient sureties, or be
released on recognizance as may be provided by law.
The right to bail shall not be impaired even when the privilege of the writ
of habeas corpus is suspended.
Excessive bail shall not be required.
102. CAMARA VS. ENAGE
right to bail
Camara, a municipal mayor in Misamis, was arrested for the alleged
participation in the killing of laborers.
o (killing of 14 and the wounding of 12 other laborers of the Tirador
Logging Co., at Nato, Esperanza, Agusan del Sur)
o He applied for BAIL.
The RTC judge Enage granted the bail citing that the prosecution failed to
prove that Camara would flee if he had the opportunity.
o However, the bail amount was P1,195,200.
The Secretary of Justice, Abad Santos, requested the judge to LOWER the
bond to P40,000, so as to make it reasonable. However, the Judge
remained adamant.
Updated by K Pascual. 2014. 4C. Bill of Rights Poli Digests. Atty. Jack Jimenez. 4C. 154


The Judge contended that the basis for the bond imposed was that
Camara had in the meantime, escaped from the provincial jail, and had
since remain at large. (thus making the case moot and academic).
ISSUE: Was the bail bond excessive?
SC: YES.
Before conviction, every person is bailable except if charged with
capital offenses when the evidence of guilt is strong.
o Such a right flows from the presumption of innocence in
favor of every accused who should not be subjected to the
loss of freedom, unless his guilt be proven beyond reasonable
doubt.
A bail is intended as a guarantee, a mode short of confinement, which
would with reasonable certainty, insure the attendance of the accused
for the subsequent trial.
o Nor is there anything unreasonable in denying this right to one
charged with a capital offense, when the evidence of guilt is
strong, as the likelihood is, rather than await the outcome of
the proceeding against him with a death sentence, an ever-
present threat, temptation to flee the jurisdiction would be too
great to be resisted. (Justice Fernando)
Where the right to bail exists, it should not be rendered nugatory by
requiring a sum that is excessive. If it would be excessive, then the right
to bail becomes meaningless. The sole function of the bail is to assure the
accuseds presence at trial, any amount or figure higher than an amount
reasonably calculated to fulfill this purpose is excessive.
Here, the P840,000 bail for multiple murder (14 counts), and P355,200 for
multiple frustrated murder (12 counts), is unconstitutional. It should be
P50,000 for the murder and P25,000 for the frustrated murder because
there are only 2 offenses charged. Even the DOJ recommended a total of
P40,000 bail for the 2 offenses.
The Guidelines for Bail Fixing:
o 1) ability of the accused
o 2) nature of offense
o 3) penalty for offense
o 4) character and reputation accused
o 5) health
o 6) character and strength evidence
o 7) probability of appearing
o 8) forfeiture of other bonds
o 9) whether fugitive
o 10) whether under other bond for other cases
There is reason to believe that any person in the position of petitioner
would under the circumstances be unable to resists thoughts of
escaping from confinement, reduced as he must have been to a stateof
desperation. In the same breath that he was told he could be bailed out,
the excessive amount required could only mean that provisional liberty
would bebeyond his reach.
103. OBOSA VS. CA
right to bail
Obosa, charged with 2 counts of murder for the ambush killing of Secretary
of ILG, Ferrer and his driver, Calderon.
He was convicted only of homicide. He applied for bail pending appeal.
Granted at P20,000 for each case. = P40,000
He was thus released from prison although at the time of the double
murder, HE WAS ALREADY SERVING TERM FOR ANOTHER CRIME,
ROBBERY.

ISSUE: Considering he has a pending case (on appeal) for murder, and a strong
evidence of guilt actually exists based on the CAs own determination, is Obosa
entitled to bail as a matter of right pending appeal of his conviction for
homicide?
The Solgen argues that the fact that an appeal throws the whole case open for
review, it reverts him back to his original situation as a person charged with a
capital offense of murder, and it was initially found by the RTC during the bail
hearings that a the evidence of guilt is strong. Hence, the Solgen argues that
now, pending appeal, he should not be given bail.
Updated by K Pascual. 2014. 4C. Bill of Rights Poli Digests. Atty. Jack Jimenez. 4C. 155


Obosa on the other hand, argues that although he was charged for murder, he
was convicted of a lesser offense of homicide, which is not punishable by death
or RP. Hence, he is still entitled to bail as a matter off right because the evidence
of guilt is NOT STRONG because he was convicted of a lesser offense.
SC:
The constitutional guarantee applies with equal force to one who has been
convicted of an offense not punishable by death or RP, even if originally charged
with a capital offense. In the ordinary course of things, there is a substantial
likelihood of his conviction being affirmed on appeal, or worse, that he can
instead be found guilty of the capital offense originally charged. In such an
instance, APPELLANT CANNOT BUT BE TEMPTED TO FLEE.
Here, bail cannot be granted as a matter of right even after an accused, who is
charged with a capital offense, appeals his conviction for a non-capital crime.
Courts must exercise utmost caution in deciding applications for bail
considering that the accused on appeal may still be convicted of the original
capital offense charged and that thus the risk attendant to jumping bail still
subsists. In fact, trial courts would be well advised to leave the matter of bail,
after conviction for a lesser crime than the capital offense originally charged, to
the appellate court's sound discretion.
The importance attached to conviction is due to the underlying principle that
bail should be granted only where it is uncertain whether the accused is guilty
or innocent, and therefore, where that uncertainty is removed by conviction it
would, generally speaking, be absurd to admit to bail. After a person has been
tried and convicted the presumption of innocence which may be relied upon in
prior applications is rebutted, and the burden is upon the accused to show error
in the conviction. From another point of view it may be properly argued that the
probability of ultimate punishment is so enhanced by the conviction that the
accused is much more likely to attempt to escape if liberated on bail than before
conviction. Justice Francisco
Note also under the Revised Rules on Criminal Procedure (which was
promulgated AFTER the bail was filed) - the presence of any of which could
preclude the grant of bail - are as follows:
That the accused is
a recidivist, quasi- recidivist, or habitual delinquent, or has committed the
crime aggravated by the circumstance of reiteration
accused is found to have previously escaped from legal confinement,
evaded sentence, or has violated the conditions of his bail without valid
justification;
accused committed the offense while on probation, parole, or under
conditional pardon-,
circumstances of the accused or his case indicate the probability of flight if
released on bail; or
undue risk that during the pendency of the appeal, the accused may commit
another crime."

It will be readily noted that, pursuant not only does the conviction of petitioner
for two counts of homicide disqualify him from being admitted to bail as a
matter of right and subject his bail application to the sound discretion of the
court.

The court found him to be disqualified because of
1) the conviction of 2 counts
2) he was a recidivist, escapee, risk of flight, undue risk to commit another
crime

Summary:
1) MATTER OF RIGHT at any stage, offense not punishable by death, RP,
2) MATTER OF DISCRETION upon conviction even if offense not punishable by
death, RP.
Updated by K Pascual. 2014. 4C. Bill of Rights Poli Digests. Atty. Jack Jimenez. 4C. 156


3) DENIED AUTOMATICALLY at any stage, if charged with a capital offense
AND the evidence of guilt strong
4) MATTER OF DISCRETION if charged with a capital offense BUT evidence of
guilt NOT strong
- if charged with capital offense BUT there are
grounds that his continued confinement would
endanger his life or health
SECTION 14 (1). CRIMINAL DUE PROCESS
104. SCOTYS DEPARTMENT STORE VS. MICALER
due process of law
Micaller is employed as a Salesgirl at the Scotys Department Store.
o She filed a ULP case against the employers, following her
dismissal because of her membership in the National Labor
Union.
Employers deny the charges.
o They claim that Micaller was dismissed because she committed
misconduct and serious disrespect to the management.
The CIR tried the case.
o The CIR imposed a fine against the manager of the store, YUKI
LAM. The CIR also fined other owners of the store, even if they are
not participating in its management.
These owners contend that RA875 is a penal law, and should therefore
be constructed in favor of the accused such that their guilt should have
been established not merely by presumptions but by clear and
convicting evidence.

ISSUE: Was the Court of Industrial Relations correct? Under RA 875, any person
who violates said law shall be punished by a fine in the discretion of the
Court. Does the word Court refer to the CIR? Can the CIR impose penalties?
SC: NO.
The power to impose the penalties provided for under RA875 is
lodged in ordinary courts, and not in the Court of Industrial Relations,
notwithstanding the definition of the word "Court" contained in section 2
(a) of said Act. Hence, the decision of the industrial court in so far as it
imposes a fine of P100 upon petitioners is illegal and should be
nullified.
No person shall be held to answer for a criminal offense without due
process of law" and that "In all criminal prosecutions the accused * * * shall
enjoy the right to be heard by himself and counsel, to be informed of the
nature and cause of the accusation against him, to have a speedy and public
trial, to meet the witnesses face to face, and to have compulsory process to
secure the attendance of witnesses in his behalf"
The procedure laid down by law to be observed by the CIR in dealing with
unfair labor practice cases negates those constitutional guarantees to the
accused.
o And this is so because, among other things, the law provides that
"the rules of evidence prevailing in courts of law or equity
shall not be controlling and it is the spirit and intention of this Act
that the CIR shall use every and all reasonable means to ascertain
the facts in each case speedily and objectively and without regard
to technicalities of law, or procedure."
o It is likewise enjoined that "the Court shall not be bound solely by
the evidence presented during the hearing but may avail itself of all
other means such as (but not limited to) ocular inspections and
questioning of well-informed persons which results must be made
a part of the record" [section 5 (b), Republic Act No. 875].
o All-this means that an accused may be tried without the right "to
meet the witnesses face to face" and may be convicted merely
on preponderance of evidence and not beyond reasonable
doubt. This is against the due process guaranteed by our
Constitution.
o It may be contended that this gap may be subserved by requiring
the Court of Industrial Relations to observe strictly the rules
applicable to criminal cases to meet the requirements of the
Constitution, but this would be tantamount to amending the law
which is not within the province of the judicial branch of our
Government.
105. ALMEDA V. VILLALUZ
Updated by K Pascual. 2014. 4C. Bill of Rights Poli Digests. Atty. Jack Jimenez. 4C. 157


FACTS:
Petitioner Leonardo Almeda (alias Nardong Paa) was charged, together
with five others, with the crime of qualified theft of a motor vehicle.
[Facts for side issue on bail bond:] The amount of the bond recommended
for the provisional release of Almeda was P15,000, and this was approved
by the respondent judge with a direction that it be posted entirely in cash.
o Almeda asked the trial court to allow him to post a surety bond in
lieu of the cash bond required of him.
At the same hearing, the respondent city fiscal, thru his assistant, reiterated
his oral motion made at a previous hearing for AMENDMENT of the
information so as to include allegations of RECIDIVISM and HABITUAL
DELINQUENCY in the particular case of Almeda.
o The trial court granted the respondent fiscal's motion in open
court. An oral motion for reconsideration was denied.
o Immediately thereafter, the assistant fiscal took hold of the
original information and, then and there, entered his
amendment by annotating the same on the back of the
document.
o The petitioner forthwith moved for the dismissal of the charge on
the ground of double jeopardy, but this motion and a motion for
reconsideration were denied in open court.

[IMPORTANT] ISSUE 2 (AMENDED INFO): Whether the amendment to the
information, after a plea of not guilty thereto, was properly allowed in
both substance and procedure. PROPERLY ALLOWED!

Anent the second issue posed by the petitioner, the amendment of the
information to include allegations of habitual delinquency and recidivism,
after a previous plea thereto by the accused, is valid and in no way violates
his right to be fully apprised before trial of the charges against him.
Under section 13 of Rule 110 of the Rules of Court, the trial court has
discretion to allow amendments to the information on all matters of
form after the defendant has pleaded and during the trial when the same
can be done without prejudice to the rights of the defendant.
o What are prohibited at this stage of the proceedings are
amendments in substance.
o And the substantial matter in a complaint or information
is the recital of facts constituting the offense charged and
determinative of the jurisdiction of the court.
o All other matters are merely of form.
Under our law, a person is considered a habitual delinquent "if within a
period of ten years from the date of his release or last conviction of the
crimes of serious or less serious physical injuries, robo, hurto, estafa or
falsification, he is found guilty of any of said crimes a third time or oftener."
The law imposes an additional penalty based on the criminal propensity of
the accused apart from that provided by law for the last crime of which he
is found guilty.
Habitual delinquency is not however, a crime in itself, it is only a
factor in determining a total penalty.
Article 62 of the Revised Penal Code which treats of habitual
delinquency does not establish a new crime, but only regulates the
"effect of the attendance of mitigating or aggravating circumstances
and of habitual delinquency." as its caption indicates.
In fact, the provision on habitual delinquency is found in a section of the
Code prescribing rules for the application of penalties, not in a section
defining offense.
A recidivist, upon the other hand, is one who, at the time of his trial for one
crime, shall have been previously convicted by final judgment of another
crime embraced in the same title of the Revised Penal Code.
Recidivism is likewise not a criminal offense; it is but one of the
aggravating circumstances enumerated by the said Code.
The additional allegations of habitual delinquency and recidivism do
not have the effect of charging another offense different or distinct
from the charge of qualified theft (of a motor vehicle) contained in the
information. Neither do they tend to correct any defect in the
jurisdiction of the trial court over the subject-matter of the case.
o The said new allegations relate only to the range of the
penalty that the court might impose in the event of
conviction.
o They do not alter the prosecution's theory of the case nor
possibly prejudice the form of defense the accused has or will
assume.
o Consequently, in authorizing the amendments, the
respondent judge acted with due consideration of the
petitioner's rights and did not abuse his discretion.
OTHER ISSUES
Updated by K Pascual. 2014. 4C. Bill of Rights Poli Digests. Atty. Jack Jimenez. 4C. 158


SKIP ISSUE 1 (BAIL): Whether or not the respondent judge has the authority
to require a strictly cash bond and disallow the petitioner's attempt to post a
surety bond for his provisional liberty.

SC:
In this jurisdiction, the accused, as of right, is entitled to bail prior to conviction
except when he is charged with a capital offense and the evidence of guilt is
strong. This right is guaranteed by the Constitution, and may not be denied even
where the accused has previously escaped detention, or by reason of his prior
absconding. In order to safeguard the right of an accused to bail, the
Constitution further provides that "EXCESSIVE BAIL SHALL NOT BE
REQUIRED." This is logical because the imposition of an unreasonable bail may
negate the very right itself. We have thus held that "where conditions imposed
upon a defendant seeking bail would amount to a refusal thereof and render
nugatory the constitutional right to bail, we would not hesitate to exercise our
supervisory powers to provide the required remedy."

The condition that the accused may have provisional liberty only upon his
posting of a cash bond is abhorrent to the nature of bail and transgresses
our law on the matter. The sole purpose of bail is to insure the attendance of
the accused when required by the court, and there should be no suggestion of
penalty on the part of the accused nor revenue on the part of the government.
The allowance of a cash bond in lieu of sureties is authorized in this jurisdiction
only because our rules expressly provide for it. Were this not the case, the
posting of bail by depositing cash with the court cannot be countenanced
because, strictly speaking, the very nature of bail presupposes the attendance of
sureties to whom the body of the prisoner can be delivered.

Thus, the trial court may not reject otherwise acceptable sureties and insist that
the accused obtain his provisional liberty only thru a cash bond.

But while we repudiate the particular measure adopted by the respondent
judge, we cannot fault the motive that caused him to demur to the petitioner's
offer of a surety bond. Based on the petitioner's past record. Fortunately, the
court is not without devices with which to meet the situation. First, it could
increase the amount of the bail bond to an appropriate level. Second, as part of
the power of the court over the person of the accused and for the purpose of
discouraging likely commission of other crimes by a notorious defendant while
on provisional liberty, the latter could be required, as one of the conditions of
his bail bond, to report in person periodically to the court and make an
accounting of his movements. Third, the accused might be warned, though this
warning is not essential to the requirements of due process, that under the
1973 Constitution. "Trial may proceed notwithstanding his absence provided
that he has been duly notified and his failure to appear is unjustified."

SKIP DOUBLE JEOPARDY ISSUE
It is clear that the petitioner Almeda has not yet been convicted nor
acquitted of the charge of qualified theft of a motor vehicle contained in the
original information. Neither has the case against him been dismissed or
otherwise terminated. The mere amendment of the information to include
allegations of habitual delinquency and recidivism does not have the effect
of a dismissal of the criminal action for qualified theft alleged in the original
information.
SKIP OTHER ISSUES
The procedure taken by the respondent fiscal and allowed by the
respondent judge in the amendment of the information does not, however,
merit our approbation. Under section 2 of Rule 15 of the Rules of Court, "all
motions shall be made in writing except motions for continuance made in
the presence of the adverse party, or those made in the course of a hearing
or trial." A motion to amend the information, after the accused has
pleaded thereto, is certainly one that should be placed in writing and
properly set for hearing. We are loath to give our imprimatur to the
kind of shortcut devised by the respondents, especially as it relates to
an alteration in the information. Considering, however, that the
petitioner was not deprived of his day in court and was in fact given
advance warning of the proposed amendment, although orally, we refrain
from disturbing the said amendment.
105.5. MATEO VS. VILLALUZ
due process of law
Mateo was charged with robbery and the killing of an American
serviceman in Sangley Point, Cavite.
An extrajudicial statement was later executed by Reyes, a co-accused,
implicating Mateo et. al.,. This statement was subscribed before
respondent Judge Villaluz.
Updated by K Pascual. 2014. 4C. Bill of Rights Poli Digests. Atty. Jack Jimenez. 4C. 159


When Reyes was called to testify for the prosecution, he however
impugned his written declaration stating that it was executed as a result
of a threat by a government agent.
Before deciding this issue, Mateo filed for the disqualification of the judge. It
is contended that such a repudiation would not sit well with respondent
Judge, who had thus placed himself in a position of being unable to pass on
such question with that degree of objectivity required by due process
(because it was before him that the statement was sworn to), although
admittedly, such a move did not fall squarely within one of the specific
grounds to inhibit judges.
The Judge denied the motion for his disqualification.
Mateo appealed on the ground that it was in violation of a constitutional
right not to be convicted of an offense without due process of law.
ISSUE:
Did the judge commit gadalej in not inhibiting?
Does the circumstance of a party having subscribed before respondent Judge an
extra-judicial statement purporting to describe the manner in which an offense
was committed, later on repudiated by him as the product of intimidation in the
course of his having been asked to testify against petitioners, would suffice to
negate a degree of objectivity?
SC: JUDGE WAS WRONG. HE SHOULD HAVE INHIBITED.
Due process cannot be satisfied in the absence of that degree of
objectivity on the part of a judge sufficient to reassure litigants of his
being fair and being just. Thereby there is the legitimate expectation that
the decision arrived at would be the application of the law to the facts as
found by a judge who does not play favorites. Due process of law requires a
hearing before an impartial and disinterested tribunal, and that every
litigant is entitled to nothing less than the cold neutrality of an impartial
judge. Elementary due process requires a hearing before an impartial and
disinterested tribunal. A judge has both the duty of rendering a just decision
and the duty of doing it in a manner completely free from suspicion as to its
fairness and as to his integrity.
A judge may not be legally prohibited from sitting in a litigation. But when
suggestion is made of record that he might be induced to act in favor of
one party or with bias or prejudice against a litigant arising out of
circumstance reasonably capable of inciting such a state of mind, he
should conduct a careful self-examination. He should exercise his
discretion in a way that the peoples faith in the courts of justice is not
impaired.
The invitation to judges to disqualify themselves is not always heeded. For
that matter, it is not always desirable that they should do so. It could
amount in certain cases to their being recreant to their trust. Justice
Perfecto's warning is not to be ignored; "to shirk the responsibility" entails
"the risk of being called upon to account for his dereliction." It could be an
instrument whereby a party could inhibit a judge in the hope of getting
another more amenable to his persuasive skill.
Here, the Judge could not be totally immune to what apparently was
asserted before him in such extrajudicial statement. Moreover, it is
unlikely that he was not in, the slightest bit offended by the affiant's
turnabout with his later declaration that there was intimidation by a
government agent exerted on him. His sense of fairness under the
circumstances could easily be blunted. The absence of the requisite due
process element is thus noticeable. There is this circumstance even more
telling.
It was he who attested to its due execution on wherein Rolando Reyes
admitted his participation in the crime and in addition implicated
petitioners. At that time, their motion for dismissal of the charges against
them was pending; its resolution was deferred by respondent Judge until
after the prosecution had presented and rested its evidence against affiant,
who was himself indicted and tried for the same offense, but in a separate
proceeding.
It cannot be doubted then that respondent Judge in effect ruled that
such extra-judicial statement was executed freely. With its repudiation
on the ground that it was not so at all, coercion having come into the
picture, there is apparent the situation of a judge having to pass on a
question that by implication had already been answered by him.
o Respondent Judge was called upon to review a matter on which
he had previously given his opinion. It is this inroad in one's
objectivity that is sought to be avoided by the law on
disqualification. The misgivings then as to the requirement of due
Updated by K Pascual. 2014. 4C. Bill of Rights Poli Digests. Atty. Jack Jimenez. 4C. 160


process for "the cold neutrality of an impartial judge" not being met
are more than justified.
*The court thus reminded lower court judges to limit themselves to
the task of adjudication and to leave to others the role of notarizing
declarations.
106. PEOPLE VS. TEEHANKEE
due process of law
This is the case of Maureen Hultman and Roland Chapman who were
murdered by CJ Teehankees son.
The relevant fact in this case is that the accused, on appeal, blames the
press and the media for his conviction, contended that the publicity
given to his case impaired his right to an impartial trial.
He cites that the RTC judge was pressured by high-ranking government
officials who avidly followed the case (VP Erap, DOJ Sec. Drilon, Pres
Cory even visited the victim while she was at the hospital)
He claims that the judge failed to protect him from PREJUDICIAL
PUBLICITY and the disruptive influences which attended the case.
o He claims there were placards, displayed during the hearing of the
cases, spectators inside the court room, converting the proceedings
into a carnival. He was also given the bad sign by several people
in the courtroom.
In short, the publicity given the case was massive, overwhelming, and
prejudicial as to effectively deprive the accused of right to impartial trial.
ISSUE: WON accuseds right to impartial trial was denied? NO
SC: RIGHT TO IMPARTIAL TRIAL WAS NOT DENIED.
It is true that the print and broadcast media gave the case at bar pervasive
publicity, just like all high profile and high stake criminal trials. But, the
right of an accused to a fair trial is not incompatible to a free press.
Responsible reporting in fact, enhances an accused's right to a fair trial. A
responsible press has always been regarded as the handmaiden of effective
judicial administration, especially in the criminal field.
The press does not simply publish information about trials but guards
against the miscarriage of justice by subjecting the police, prosecutors,
and Judicial processes to extensive public scrutiny and criticism."
Pervasive publicity is not per se prejudicial to the Right of an accused
to fair trial. The mere fact that the trial of appellant was given a day-to-day,
gavel-to-gavel coverage does not by itself prove that the publicity so
permeated the mind of the trial judge and impaired his impartiality.
For one, it is impossible to seal the minds of members of the bench from
pre-trial and other off-court publicity of sensational criminal cases. Also,
our idea of a fair and impartial Judge is not that of a hermit who is out of
touch with the world.
The records do not show that the trial judge developed actual bias
against appellant as a consequence of the extensive media coverage of the
pre-trial and trial of his case.
The totality of circumstances of the case does not prove that the trial
Judge acquired a fixed opinion as a result of prejudicial publicity which
is incapable of change even by evidence presented during the trial.
Appellant has the burden to prove this actual bias and he has not
discharged the burden.
As to the other accusations, the court found that when the placards were
raised, the trial judge immediately ordered the same to be hidden. Then
only did he start the arraignment. The judge also allowed the change of
venue noting that the courtroom was also too crowded. The judge also
limited the video coverage to 2 mins and only after that did he start with
trial. Reporters were also told to keep quiet.
Note also that, appellant should be the last person to complain against
the press for prejudicial coverage of his trial. The records reveal he
presented in court no less than seven (7) newspaper reporters and
relied heavily on selected portions of their reports for his defense. The
defense's documentary evidence consists mostly of newspaper clippings
relative to the investigation of the case at bar and which appeared to cast
doubt on his guilt. The press cannot be fair and unfair to appellant at the
same time.
Finally, it would not be amiss to stress the trial judge voluntarily inhibited
himself from further hearing the case to assuage appellant's suspicion of
bias and partiality. However, upon elevation of the trial Judge's voluntary
Updated by K Pascual. 2014. 4C. Bill of Rights Poli Digests. Atty. Jack Jimenez. 4C. 161


Order of Inhibition to this Court, we directed the trial judge to proceed with
the trial to speed up the administration of Justice. We found nothing in the
conduct of the proceedings to stir any suspicion of partiality against the
trial judge.
SECTION 14 (2). PRESUMPTION OF INNOCENCE
107. PEOPLE VS. MINGOA
presumption of innocence
Mingoa was prosecuted for the crime of malversation of public funds, after
he failed to produce the missing amount of P3,938, as OIC of the municipal
treasurer.
He explained to the examining officer that a few days before, he had, by
mistake, put the money in a large envelope, which he took with him to a
show and that he forgot it on his seat and it was not there anymore when he
returned.
Under RPC Art 217, there is a prima facie evidence of malversation when he
fails to produce the funds upon demand.
Mingoa claims that lacking direct evidence of actual misappropriation the
trial court convicted him on mere presumptions, that is, presumption of
criminal intent in losing the money under the circumstances alleged and
presumption of guilt from the mere fact that he failed, upon demand, to
produce the sum lacking.
ISSUE: Does this provision violate the constitutional right of an accused to be
presumed innocent until the contrary is proven?
SC: VALID PROVISION. NO VIOLATION OF CONSTI RIGHTS.
He did not overcome the presumption of guilt arising from his inability to
produce the fund which was found missing. If the money was really lost without
defendant's fault, the most natural thing for him to do would be to so inform his
superiors and apply for release from liability. But this he did not do. Instead, he
tried to borrow to cover the shortage. And on the flimsy excuse that he
preferred to do his own sleuthing, he even did not report the loss to the police.
Considering further, he had at first tried to avoid meeting the auditor who
wanted to examine his accounts, and that for sometime before the alleged loss
many teachers and other employees of the town had not been paid their
salaries, there is good ground to believe that defendant had really malversed the
fund in question and that his story about its loss was pure invention.
There is no constitutional objection to the passage of a law providing that the
presumption of innocence may be overcome by a contrary presumption
founded upon the experience of human conduct, and enacting what evidence
shall be sufficient to overcome such presumption of innocence." The legislature
may enact that when certain facts have been proved they shall be prima facie
evidence of the existence of the guilt of the accused and shift the burden of
proof provided there be a rational connection between the facts proved and
the ultimate fact presumed so that the inference of the one from proof of the
others is not unreasonable and arbitrary because of lack of connection between
the two in common experience.
In this case, the RPC provision creates a presumption of guilt once certain facts
are proved. The ultimate fact presumed is that the officer has malversed the
funds or property entrusted to his custody, and the presumption is made to
arise from proof that he has received them and yet he has failed to have them
forthcoming upon proper demand.
Clearly, the fact presumed is but a natural inference from the fact proved, so
that it cannot be said that there is no rational connection between the two.
Furthermore, the statute establishes only a prima facie presumption, thus giving
the accused an opportunity to present evidence to rebut it. The presumption is
reasonable and will stand the test of validity laid down in the above citations.
SECTION 14 (2). RIGHT TO COUNSEL
108. PEOPLE VS. FRISCO HOLGADO
right to counsel
Holgado charged with slight illegal detention for kidnapping and detaining
Fabreag in the house of Antero Holgado for 8 hours.
Updated by K Pascual. 2014. 4C. Bill of Rights Poli Digests. Atty. Jack Jimenez. 4C. 162


During arraignment, he appeared without counsel and he pleaded guilty. He
said, I PLEAD GUILTY BUT I WAS INSTRUCTED BY ONE MR. OCAMPO.
The fiscal investigated but found that this Mr. Ocampo had nothing to do
with the case.
He was later sentenced.
ISSUE: Was the right to counsel violated?
SC: Under the Rules of Court, Rule 112, (IDUT)
If the defendant appears without an attorney, he must be informed by the
court that of his right to have an attorney before being arraigned, and
must be asked if he desires the aid of the attorney.
If he desires but is unable to employ one, the court must assign an attorney
de officio.
A reasonable time must be allowed if he procures an attorney of his own.
NONE OF THESE WERE COMPLIED WITH BY THE RTC. The court did not inform
the accused of his right, neither was he asked if he desired the aid of one. The
court also did not inquire whether the accused was to employ an attorney, to
grant him reasonable time to procure one, or to assign an attorney de offico.
The question asked by the court was DO YOU HAVE AN ATTORNEY OR ARE
YOU GOING TO PLEAD GUILTY? This question was so framed that it could have
been construed by the accused as a suggestion from the court that he should
just plead guilty if he had no attorney. This is a denial of fair hearing.
There can be no fair hearing unless the accused be given the opportunity to be
heard by counsel. No person shall be held to answer for a criminal offense
without due process of law. He shall enjoy the right to be heard by himself and
by counsel. The right to be heard would be of little avail if it does not include the
right to be heard by counsel.
Even the most intelligent or educated man may have no skill in the science
of law, particularly in the rules of procedure, and without counsel, he may
be convicted not because he is guilty but because he does not know how to
establish his innocence.
This could happen more easily to persons who are ignorant or uneducated.
109. PEOPLE VS. SIM BEN
right to counsel
Sim Ben was convicted for exhibiting cinematographic films of indecent or
immoral scenes in his restaurant in Cebu.
He pleaded guilty but without aid of counsel.
It was found out that when the case was called for trial, the appellant was
informed by the Court of his right to have counsel and asked if he
desired the aid of one.
o He replied that he did not.
Then the Court asked if he was agreeable to have the information read
to him even without the assistance of counsel.
o His answer was in the affirmative.
The court interpreter translated the information to him in the local dialect
and after the translation he entered a plea of guilty.
He was asked whether he knew that because of the plea of guilty the
punishment as provided for by law would be imposed upon him and
he answered "Yes, sir."
The Court asked him if he insisted on his plea of guilty and he
answered "Yes, sir."
Thus, the Court sentenced him.
ISSUE: WON accuseds right to counsel was waived intelligently? YES,
INTELLIGENTLY WAIVED.
Updated by K Pascual. 2014. 4C. Bill of Rights Poli Digests. Atty. Jack Jimenez. 4C. 163


SC: RIGHT TO COUNSEL MAY BE WAIVED. IT NEED NOT BE IN WRITING.
WAIVER MAY BE ORAL. WAIVER MAY BE MADE EVEN WITHOUT THE
PRESENCE OF COUNSEL.
WAIVER MUST BE COMPETENTLY AND INTELLIGENTLY MADE.
His rights were fully protected and safeguarded.
The Court complied with its duty when it informed the appellant that it was
his right to have the aid of counsel. And before pronouncing the sentence
the Court took pains to ascertain whether he was aware of the
consequences of the plea he had entered.
Notwithstanding this precaution and warning, he waived his right to have
the aid of counsel and entered a plea of guilty to the information.
Appellant claims that he entered the plea of guilty because the fiscal
promised him that only a fine would be imposed.
The recommendation of the fiscal that only a fine be imposed upon the
appellant seems to bear out his claim; but such recommendation or one of
leniency does not mean that the appellant is not guilty of the crime charged
against him.
A promise to recommend a specific penalty such as fine does not
render the sentence void if the Court ignores the recommendation and
metes out to the defendant a penalty which is provided by law.
110. DELGADO VS. CA
right to counsel
Delgado was charged with estafa thru falsification of public documents. (in
connection with arranging travel documents of Erlinda).
Delgado pleaded NOT guilty.
o She was assisted by her counsel de parte, Atty. Yco.
On the date set for the continuation of the defense evidence, the lawyer
failed to appear despite prior notice. Instead, the lawyer sent a telegram
requesting for the postponement on the ground that he was allegedly SICK.
o No medical certificate however was submitted.
The fiscal objected, claiming it was dilatory.
The RTC ruled against Delgado and ordered the case to proceed.
o Delgado was deemed to have waived presentation of evidence, and
the case was submitted for decision.
o She was convicted. There was an entry of final judgment.
Delgado now filed a motion to set aside entry of judgment on the ground
that there was irregularity in the sending of notices and copy of the decision
to her lawyer.
This was denied. She was ordered arrested.
She further appealed, and filed a MR, on the ground of newly discovered
evidence, that she only knew then that Atty Yco was not in fact a member of
the Bar.
o She prayed for a new trial because she was deprived of the right to
be defended by a competent counsel.
ISSUE: WON Delgados right to competent counsel was violated? Yes.
SC: NEW TRIAL GRANTED.
An accused is entitled to be represented by a MEMBER OF THE PHILIPPINE
BAR in a criminal case filed against her.
Unless she is represented by a lawyer, there is a great danger that any
defense presented in her behalf would be inadequate considering the legal
pre-requisites and skills needed in court proceedings.
This is certainly a DENIAL OF DUE PROCESS.

111. PEOPLE VS. MALUNSING
right to counsel
Villegas was a co-accused for murder.
Updated by K Pascual. 2014. 4C. Bill of Rights Poli Digests. Atty. Jack Jimenez. 4C. 164


His Atty Pajarito explicitly manifested that his client Villegas had his own
lawyer.
Atty Pajarito appeared for Villegas in the preliminary investigation but only
because there was no other counsel.
However Villegas changed his mind and so the Court gave him a
lawyer, appointing Atty. Pajarito as counsel de officio. The court
ordered the case to proceed to trial
During the trial, Atty Pajarito was asked whether he would like to confer
with his client. He said, I think I know the case.
o However, no evidence was presented for and in behalf of Villegas.
In fact, all other defendants took the witness stand except for
Villegas.
Villegas was a very old man, ignorant, unlettered, during the entire
proceedings, he did not know what was going on.
The trial court never apprised him of his right to be assisted by a lawyer, it
never bothered to inquire why he was the only one who did not take the
witness stand.
The trial court proceeded with the case without knowing why Villegas did
not testify.
ISSUE: WON Right to Counsel of Villegas was violated? Yes
SC: RIGHT TO COUNSEL VIOLATED. NEW TRIAL ORDERED.
it is not enough that a counsel de oficio was appointed, where the
accused had indicated that he wanted a lawyer of his choice, a decision
prompted moreover by the fact that he had lost confidence in the member
of the bar thus designated.
Nor is it to manifest respect for this right if the counsel do oficio thus
named, instead of conferring with the accused, would just blithely inform
the judge that he was already fully prepared for his exacting responsibility.
Inasmuch as it is intended to assure a just and fair proceeding, he is entitled
at the most to a new trial where he can be duly represented either by a
counsel of his choice or by one appointed de oficio, one who would
discharge his task in a much more diligent and conscientious manner and
would not readily assume that he need not bother himself unduly with
familiarizing himself further with all aspects of the case. For only in such a
way may there be an intelligent defense.
In criminal cases there can be no fair hearing unless the accused be given
an opportunity to be heard by counsel. The right to be board would be of
little avail if it does not include the right to be heard by counsel.
o Even the most intelligent or educated man may have no skill in the
science of law, particularly in the rules of procedure, and, without
counsel, he may be convicted not bemuse he is guilty but because
he does not know how to establish his innocence. And this can
happen more easily to persons who are ignorant or uneducated.
It is not enough to ask him whether he desires the aid of an attorney,
but it is essential that the court should assign one de oficio for him if
he so desires and he is poor or grant him a reasonable time to procure
an attorney of his own.
What is more, it is one of the worthwhile innovations of the present
Constitution that even at the stage of custodial interrogation when the
police agencies are investigating a man's possible connection with a crime,
he is already entitled to counsel.

Note: Right to counsel is violated when the court appoints a counsel de
officio for an accused who manifested his desire to have his own counsel de
parte and thereafter ordered the case to proceed for trial.
The DENIAL OF EFFECTIVE REPRESENTATION applies to both counsel de offcio
and counsel of the accuseds choice.
SECTION 14 (2). RIGHT TO BE INFORMED
112. PEOPLE VS. REGALA
right to be informed of the nature and causes of accusation
Regala and Flores were both charged with the crime of MURDER WITH
ASSAULT upon an agent of a person in authority,
Updated by K Pascual. 2014. 4C. Bill of Rights Poli Digests. Atty. Jack Jimenez. 4C. 165


o for attacking and stabbing with a knife, a member of the Philippine
Constabulary who was performing his official duty.
The RTC found them guilty for the killing and qualified as murder under
the circumstances of treachery and evident premeditation.
o Thus, they were convicted of COMPLEX CRIME OF MURDER
WITH ASSAULT upon an agent of a person in authority.
They appealed the conviction contending that they cannot be
convicted of a complex crime because the information filed against
them did not allege the essential elements of assault, that the accused
then knew that the victim was an agent of a person in authority.
There was only a bare allegation of the fact that the accused
deliberately, and with intent to kill, with evident premed and treachery and
further taking advantage of nighttime, attacked and stabbed Desilos, while
he was performing his official duty.

ISSUE: Is the information sufficient to charge a complex crime?
SC:
NO. Such an allegation cannot be an adequate substitute for the essential
averment to justify a conviction for a complex crime.
There were NO ALLEGATION OF FACTS from which it can be implied that
the accused then knew that before or at the time of the assault, the
victim was an agent of a person in authority. There was no allegation of
the facts from which the elements may be implied.
Moreover, the fact that the crime of assault was established by evidence on
the part of the prosecution without objection on the part of the accused
cannot likewise cure the defective information because to do so would be
convicting the accused of a crime not properly alleged in the body of the
information, in violation of his constitutional right to be informed of the
nature and the cause of the accusation against him.
113. PEOPLE VS. ORTEGA
right to be informed of the nature and cause of accusation
Ortega and Garcia were found guilty of murder of Masangkay.
On appeal Garcia contends that he cannot be convicted of murder because
of the defective information.
The information accused him of attacking, assaulting, and stabbing
with a pointed weapon, the victim Masangkay. (however, later, it was
found that Masangkay died of drowning)
However, the prosecutions evidence itself shows that Garcia had
nothing to do with the stabbing, which was solely perpetrated by
Ortega.
o Garcias responsibility relates only to the attempted concealment of
the crime and the resulting drowning of Masangkay.
The RTC found him liable as co-conspirator.
SC:
The hornbook doctrine in our jurisdiction is that an accused cannot be
convicted of an offense, unless it is clearly charged in the complaint or
information. Constitutionally, he has the right to be informed of the nature
and cause of the accusation against him. To convict him of an offense other
than that charged in the compliant or information would be a violation of
this constitutional right.
Thus, Garcia cannot be convicted of homicide through drowning, when
the information charges murder through stabbing.
Garcia merely assisted in concealing the body of the victim.
At the autopsy, it was showed that the victim at that time was still alive, and
that he died only subsequently of drowning.
Drowning was the immediate cause of death, as medically demonstrated by
the muddy particles found in the victims lungs.
In assisting Ortega in carrying the body of Masangkay to the well, Garcia
was committing a felony, of concealing the body of the crime to prevent its
discovery.
o He is thus only an accessory to the crime of homicide.
Updated by K Pascual. 2014. 4C. Bill of Rights Poli Digests. Atty. Jack Jimenez. 4C. 166


SECTION 14 (2). RIGHT TO SPEEDY TRIAL
114. CONDE VS. RIVERA
right to speedy trial
Aurelia Conde, was a midwife who is an accused in 5 different informations
for various crimes and misdemeanors.
He has appeared with her witnesses and counsel at the hearing no less than
on 8 different occasions, only to see the cause postponed.
She has also 2x been required to come to the SC for aid.
Now, after the passage of more than 1 year from the time when the first
information was filed, there is yet no resolution of the cases because the
prosecutor has filed for postponement again.

SC: In all criminal prosecutions the accused shall enjoy the right to have a
speedy trial. Aurelia Conde, like all other accused persons, has a right to a
speedy trial in order that if innocent she may go free, and she has been deprived
of that right in defiance of law. Dismissed from her humble position, and
compelled to dance attendance on courts while investigations and trials are
arbitrarily postponed without her consent, is palpably and openly unjust to her
and a detriment to the public. By the use of reasonable diligence, the
prosecution could have settled upon the appropriate information, could have
attended to the formal preliminary examination, and could have prepared the
case for a trial free from vexatious, capricious, and oppressive delays.

On the one hand has been the petitioner, of humble station, without resources,
but fortunately assisted by a persistent lawyer, while on the other hand has
been the Government of the Philippine Islands which should be the last to set an
example of delay and oppression in the administration of justice. The Court is
thus under a moral and legal obligation to see that these proceedings come to
an end and that the accused is discharged from the custody of the law.

Where a prosecuting officer, without good cause, secures postponements of the
trial of a defendant against his protest beyond a reasonable period of time, as in
this instance for more than a year, the accused is entitled to relief by a
proceeding in mandamus to compel a dismissal of the information, or if he be
restrained of his liberty, by habeas corpus to obtain his freedom.

The writ prayed for shall issue and the Provincial Fiscal of Tayabas shall abstain
from further attempts to prosecute the accused pursuant to informations
growing out of the facts set forth in previous informations, and the charges now
pending before the justice of the peace of Lucena, Tayabas, are ordered
dismissed, with costs against the respondent fiscal.
SECTION 14 (2). RIGHT TO PUBLIC TRIAL
115. GARCIA VS. DOMINGO
right to public trial

Several informations were filed against Calo and Carbonell for slight
physical injuries.
The trial of the cases in question was held, with the conformity of the
accused and their counsel, in the chambers of Judge Garcia."
Later, they filed an injunction for the deferment of the promulgation of the
judgment citing that their constitutional and statutory rights had been
violated, adversely affecting their - right to a free and impartial trial, '
[noting] that the trial of these cases lasting several weeks were held
exclusively in chambers and not in the court room open to the public.

ISSUE: Was there a public trial?
Updated by K Pascual. 2014. 4C. Bill of Rights Poli Digests. Atty. Jack Jimenez. 4C. 167


SC: YES. (justice Fernando)
Trial should be public in order to offset any danger of conducting it in an illegal
and unjust manner.
The crucial question of the meaning to be attached this provision remains. The
Constitution guarantees an accused the right to a public trial. What does it
signify? Offhand it does seem fairly obvious that here is an instance where
language is to be given a literal application. The trial must be public. It
possesses that character when anyone interested in observing the manner a
judge conducts the proceedings in his courtroom may do so. There is to be no
ban on such attendance. His being a stranger to the litigants is of no moment. No
relationship to the parties need be shown. The thought that lies behind this
safeguard is the belief that thereby the accused is afforded further protection,
that his trial is likely to be conducted with regularity and not tainted with any
impropriety. It is thus understandable why such a right is deemed embraced in
procedural due process. Where a trial takes place, as is quite usual, in the
courtroom and a calendar of what cases arc to be heard is posted, no problem
arises. It is the usual course of events that individuals desirous of being present
are free to do so. There is the well recognized exception though that warrants
the exclusion of the public where the evidence may be characterized as
"offensive to decency or public morals.

What did occasion difficulty in this suit was that for the convenience of the
parties, and of the city court Judge, it was in the latter's air-conditioned
chambers that the trial was held. Did that suffice to vitiate the proceedings as
violative of this right? The answer must be in the negative. There is no showing
that the public was thereby excluded. It is to be admitted that the size of the
room allotted the Judge would reduce the number of those who could be
present. Such a fact though is not indicative of any transgression of this right.
Courtrooms are not of uniform dimensions. Some are smaller than others.
Moreover, as admitted by Justice Black in his masterly In re Oliver opinion, it
suffices to satisfy the requirement of a trial being public if the accused could
"have his friends, relatives and counsel present no matter with what offense he
may be charged."

The Supreme Court held that as it affirmatively appears on the record that the
accused offered no objection to the trial of his case in the place where it was
held, his right is deemed waived."

An objective appraisal of conditions in municipal or city courts shows a
crowded daily calendar, the nature of the cases handled, civil as well as criminal,
the relaxed attitude on procedural rules not being strictly adhered to all make
for a less tense atmosphere. As a result the attendance of the general public is
much more in evidence; nor is its presence unwelcome.

Summary: 1. Right was waived. 2. Public means, anyone interested in observing
the trial may do so, even a stranger. 3. Relationship to litigants immaterial. 4.
Size of courtroom not indicative of violation of right.
Exceptions (When there can be no public trial: a) sensitive rape cases, b)
military secrets, national security issues.
SECTION 14 (2). RIGHT TO CONFRONTATION OF WITNESS

116. PEOPLE VS. ORTIZ-MIYAKE
right to compulsory process confrontation

Ortiz was charged with large scale illegal recruitment as well as estafa by
means of false pretenses.
She allegedly promised Marasigan a job as a factory worker in Taiwan. After
paying however, she was never issued a visa. She found out later that she
was also not booked by Ortiz for the flight to Taiwan, and also that none of
the staff in the recruitment agency knew Ortiz.
Updated by K Pascual. 2014. 4C. Bill of Rights Poli Digests. Atty. Jack Jimenez. 4C. 168


There were 2 other victims, however it was only Marasigan who testified at
the trial, since the other 2 were abroad.
The prosecution insisted that even if 2 out of 3 complainants in the illegal
recruitment case were unable to testify because they were abroad, Ortiz
should still be guilty of the offense against all 3 complainants, and should be
convicted for large scale illegal recruitment.
On the other hand, Ortiz claims that she is not guilty of large scale illegal
recruitment because she did not recruit the 3 victims, but merely purchased
for their plane tickets. She also argues that in the prosecution for large scale
illegal recruitment, there must be at least 3 complainants who should
appear as witnesses in the trial. Since it was only Marasigan who testified,
the conviction is groundless.
The RTC of Makati convicted her of large scale illegal recruitment, by
adopting the decision of the Paranaque court (in another case) to show that
illegal recruitment was also committed by Ortiz against the 2 other
complainants.

SC: Illegal recruitment is deemed large scale if committed against 3 or more
persons individually or as a group. It is evident in illegal recruitment cases, the
number of persons victimized is determinative. Where illegal recruitment is
committed against a lone victim, the accused maybe convicted only of simple
illegal recruitment, which has a lower penalty.

It was argued by the prosecution that there was an earlier conviction in the MTC
of Paranaque where Ortiz was found guilty of estafa against the 2 other
complainants.
SC: The court cannot validly adopt the facts embodied in the Paranaque case to
show that illegal recruitment was also committed against the 2 other
complainants.
It is erroneous to base the conviction for large scale illegal recruitment on the
facts of a previous decision by the Paranaque court. It is a violation of the right
of Ortiz to confront the witnesses, that is the 2 other complainants, during the
trial before it.
Here, illegal recruitment was proven tohave been committed against only one
person, Marasigan. Thus, Ortiz may be liable only for simple illegal recruitment.

The accused in a criminal case is guaranteed the right to confrontation. It has 2
purposes:
1) to secure the opportunity for cross-exam
2) to allow the judge to observe the deportment/conduct of the witness while
testifying

This right however is not absolute as it is sometimes impossible to produce a
witness who has already testified in a previous proceeding. In such case, his
previous testimony is made admissible as a distinct piece of evidence, by way of
exception to the hearsay rule. The previous testimony is made admissible
because it makes the administration of justice orderly and more expeditious.

The adoption by the Makati RTC of the facts in the Paranaque court decision
does not fall under the exception to the right of confrontation. The law covers
only the use of testimonies of absent witnesses made in previous
proceedings, and does not include the utilization of the previous decisions or
judgments.

In this case, the prosecution never offered the prior testimonies of the 2 other
complainants in the Paranaque court case. Instead, what the prosecution
offered was the decision itself, as the basis for large scale illegal recruitment.

This is wrong. A previous decision or judgment, while admissible in evidence,
may only prove that an accused was previously convicted of a crime. It may not
Updated by K Pascual. 2014. 4C. Bill of Rights Poli Digests. Atty. Jack Jimenez. 4C. 169


be used to prove that the accused is guilty of a crime charged in a subsequent
case, in lieu of the requisite evidence proving the commission of the crime, as
said previous decision is hearsay. To sanction its being used as a basis for
conviction in a subsequent case would constitute a violation of the right of the
accused to confront the witnesses against him.

The Makati court's utilization of and reliance on the previous decision of the
Paraaque court must be rejected. Every conviction must be based on the
findings of fact made by a trial court according to its appreciation of the
evidence before it. A conviction may not be based merely on the findings of fact
of another court.

In illegal recruitment in large scale, while the law does not require that at least
three victims testify at the trial, it is necessary that there is sufficient evidence
proving that the offense was committed against three or more persons. This
Court agrees with the trial court that the evidence presented sufficiently proves
that illegal recruitment was committed by appellant against Marasigan, but the
same conclusion cannot be made as regards Generillo and Del Rosario as well.

117. PEOPLE VS. SENERIS
right to compulsory process confrontation

The wife, Pilar Angeles, induces and offered a sum of money to co-accused
Nemenio, to kill her husband.
Nemenio pleaded guilty and sentenced accordingly. He later testified
against the wife, Pilar Angeles, as prosecutions witness.
When the direct exam was completed, the defense moved to hold the cross-
exam in abeyance for April 19.
Come April 19, Nemenio did not appear because he was not served a
subpoena. It was reset to June 7 where cross-exam began but was not
finished.
However on June 21, Nemenio was shot dead by the police for allegedly
escaping prison.
This made cross examination incomplete.
The judge then ordered that the whole testimony inadmissible since the
cross-exam was incomplete.

ISSUE: Should the testimony be rendered inadmissible?
SC: YES. INADMISSIBLE.

The constitutional right of confrontation, which guarantees to the accused the
right to cross-examine the witnesses for the prosecution, is one of the most
basic rights of an accused person under our system of justice. It is a
fundamental right which is part of due process not only in criminal proceedings
but also in civil proceedings as well as in Proceedings in administrative
tribunals with quasi-judicial Powers.

The right to confrontation has 2 purposes:
1) to secure the opportunity for cross-exam
2) to allow the judge to observe the deportment/conduct of the witness while
testifying
It ensures that the witness will give his testimony in open court under oath,
thus deterring lying b the threat of perjury. If forces the witness to submit to
cross-exam, a valuable instrument in exposing falsehood and bringing out the
truth.

Updated by K Pascual. 2014. 4C. Bill of Rights Poli Digests. Atty. Jack Jimenez. 4C. 170


While the right to confrontation and to cross-exam are fundamental rights, the
same can be waived expressly or impliedly by conducting amounting to
renunciation of the right. There is waiver when the party was given opportunity
to confront and cross-examine an opposing witness, but failed to take advantage
of it for reasons attributable to him alone.

Until such cross-exam has been finished, the testimony of the witness cannot be
considered as complete. In short, where the right to cross-exam is lost wholly or
in part through the fault of the cross-examiner, then the testimony of direct due
to exam may be recognized, but when the cross-exam is not and cannot be done
or completed due to causes attributable to the party offering the witness then
the uncompleted testimony becomes incompetent and inadmissible.

The direct testimony of the witness who dies before the completion of the
cross-exam can thus be stricken only insofar as not covered by the cross.
Where the death or illness prevents cross-examination under such
circumstances that no responsibility of any sort can be attributed to either
the witness of his party, it seems harsh measure to strike out all that has
been obtained on the direct examination.

The Lufthansa case were it was held that when cross-examination is not and
cannot be done or completed due to causes attributable to the party offering the
witness, the uncompleted testimony is thereby rendered incompetent As clear
as day, the Lufthansa ruling therefore applies only if there is a finding that the
cause for non-completion of the cross-examination of a witness was attributable
to the very party offering the said witness.

That is not applicable in this case because the cause of the non-completion of
the cross-exam was A FORTUITOIUS EVENT. (death)

ISSUE: Was there a wavier of right to cross-exam when the defense failed to
avail of cross-examine Nemenio right after there was direct exam? Was there a
waiver because the defense moved for abeyance?
SC: NO WAIVER.
It will be noted that the defense moved for abeyance because in order to have
the opportunity to make preparations, for an effective exercise thereof
considering the nature of the case a capital one and the length of the direct
examination. The deferment of the cross exam requested by the defense was
approved by the judge without any objection on the part of the prosecution.
And on the date for the cross-examination of the witness Nemenio, defense
failure to cross-examine the said witness not of his own design but because said
witness failed to appear on that date for the reason that due to the oversight of
the court a personnel the subpoena for said witness was not served on him.

No fault can be imputed to the defense for the length of time that elapsed before
her counsel was able to commence his cross-examination of the witness.

BUT in this case, the cross-examination made by the defense of the deceased
witness Nemenio was extensive and already covered the subject matter of his
direct testimony as state witness relating to the essential elements of the crime
of parricide, and what remained for further cross-examination is the matter of
price or reward allegedly paid by Angeles for the commission of the crime,
which is merely an aggravating circumstance and does not affect the existence
of the offense charged, the respondent judge gravely abused his discretion in
declaring as entirely inadmissible the testimony of the state witness who died
through no fault of any of the parties before his cross-examination could be
finished.
118. PEOPLE VS. NARCA
right to compulsory process confrontation

Updated by K Pascual. 2014. 4C. Bill of Rights Poli Digests. Atty. Jack Jimenez. 4C. 171


Narca was charged with murder for killing and hacking with the use of a
bolo, of Reglos.
On bail hearings, the victims wife, Elizabeth Reglos, testified on direct
examination.
The defense however moved that the cross-exam be conducted on the next
hearing day.
However, Elizabeth was never cross-examined by the defense because she
and her son were bludgeoned to death.
Narca was still convicted.
On appeal, Narca claims that the testimony of the wife in the bail hearings
should not be given weight since she was not cross-examined.

ISSUE: Is Elizabeths testimony admissible?
SC: YES.
The defense's failure to cross-examine Elizabeth Reglos was occasioned by her
supervening death. Lack of cross-examination due to the death of the witness
does not necessarily render the deceased's previous testimony expungible.
Where death prevents cross-examination under such circumstances that no
responsibility of any sort can be ascribed to the plaintiff or the witness, it seems
a harsh measure to strike out all that has obtained in the direct examination."

The wholesale exclusion of testimonies was too inflexible a solution to the
procedural impasse because it prejudiced the party whose only fault was to die
before he could be cross-examined. The prudent alternative should have been to
admit the direct examination so far as the loss of cross-examination could have
been shown to be not in that instance a material loss. And more compelling so in
the instant case where it has become evident that the adverse party was
afforded a reasonable chance for cross-examination but through his own fault
failed to cross-examine the witness.

ISSUE: Was there waiver when the defense sought postponement of the cross-
examination for the next hearing day?
SC: YES. WAIVER.
Besides, mere opportunity and not actual cross-examination is the essence of
the right to cross-examine. Appellants lost such opportunity when they sought
the deferment of their cross-examination of Elizabeth, and they only have
themselves to blame in forever losing that right by reason of Elizabeth's demise.
This Court held that the right to cross-examination "is a personal one which
may be waived expressly or impliedly by conduct amounting to a renunciation
of the right of cross-examination. Thus, where a party has had the opportunity
to Gross-examine a witness but failed to avail himself of it, he necessarily
forfeits the right to crossexamine and the testimony given on direct
examination of the witness will be received or allowed to remain in the record.

ISSUE: Can the testimony given in bail hearings be recognized at the trial
proper?
SC: YES.
Evidence presented during the bail hearings,like the testimony of deceased
witness Elizabeth, are "considered automatically reproduced at the trial"
subject only to the possible recall of the "witness for additional examination
unless the witness is dead, outside the Philippines or otherwise unable to
testify."
Rule 115 provides that "either party may utilize as part of its evidence the
testimony of a witness who is deceased given in another case or proceeding,"
SECTION. 16
119. TATAD VS. SB
speedy disposition of cases
Updated by K Pascual. 2014. 4C. Bill of Rights Poli Digests. Atty. Jack Jimenez. 4C. 172



1) Tatad was charged with violation of the AGCP (bribery, giving benefits
to relatives, see below) when he was still secretary and head of the dept
of public information.
2) 2 months after Tatads resignation, Antonio delos Reyes filed a
complaint with the Tanodbayan. This was referred to the Criminal
Investigation Service.
3) Tatad moved to dismiss the case claiming immunity from prosecution.
DENIED.
4) So the case was filed before the Sandiganbayan (SB).
5) Tatad now filed a motion to quash the information on the ground that
he was deprived of due process, and the right to speedy disposition of
cases, amounting to loss of jurisdiction to file the information. DENIED.
6) Later, an amended information was filed against him, changing the date
of the commission of the offense.
7) Tatad filed a motion to quash again. DENIED AGAIN.
8) He now assails the denial of the motion to quash.

ISSUE: Whether the prosecutions long delay in filing the cases with the SB had
deprived Tatad of the right to due process and speedy disposition of cases.

Tatad claims that the Tanodbayan violated the constitutional mandate of "due
process" and "speedy disposition of cases" in unduly prolonging the termination
of the preliminary investigation and in filing the informations only after more
than a decade from the commission of the offenses, which amounted to loss of
jurisdiction and authority to file the informations.

SC: These were the findings of fact by the SC.
1974 - Reyes originally filed a report with the PSG concerning the charges
against Tatad.
1979 - This report was made to sleep in the office of the PSG until 1979. In
1979, the 1974 complaint was resurrected by a formal
complaint with the Tanodbayan because Tatad resigned
from the Marcos cabinet.
1980 - Tanodbayan acted on the complaint only in 1980.
1985 Tanodbayan approved the resolution, recommending the filing of
the information with the SB. 5 informations were filed.

SC: THERE WAS DENIAL OF DUE PROCESS AND SPEEDY DISPOSITION.
Firstly, the complaint came to life, only after Tatad had a falling out with
President Marcos. Secondly, the Tanodbayan referred the complaint to the
Presidential Security Command for fact-finding investigation and report, instead
of the established procedures prescribed by law for preliminary investigation,
which require the submission of affidavits and counter-affidavits by the
complainant and the respondent and their witnesses. The prosecution was
politically motivated.

Also, the long delay in resolving the case under preliminary investigation can
not be justified on the basis of the facts on record. The law (P.D. No. 911)
prescribes a ten-day period for the prosecutor to resolve a case under
preliminary investigation by him from its termination. While this period fixed
by law is merely "directory," yet, on the other hand, it can not be disregarded or
ignored completely, with absolute impunity.

We find the long delay in the termination of the preliminary investigation by the
Tanodbayan in the instant case to be violative of the constitutional right of the
accused to due process.

Updated by K Pascual. 2014. 4C. Bill of Rights Poli Digests. Atty. Jack Jimenez. 4C. 173


A delay of close to three (3) years can not be deemed reasonable or justifiable in
the light of the circumstance obtaining in the case at bar. We are not impressed
by the attempt of the Sandiganbayan to sanitize the long delay by indulging in
the speculative assumption that "the delay may be due to a painstaking and
gruelling scrutiny by the Tanodbayan as to whether the evidence presented
during the preliminary investigation merited prosecution of a former high-
ranking government official." In the first place, such a statement suggests a
double standard of treatment, which must be emphatically rejected. Secondly,
three out of the five charges against the petitioner were for his alleged failure to
file his sworn statement of assets and liabilities required by RA 3019, which
certainly did not involve complicated legal and factual issues necessitating such
"painstaking and gruelling scrutiny" as would justify a delay of almost three
years in terminating the preliminary investigation. The other two charges
relating to alleged bribery and alleged giving of unwarranted benefits to a
relative, while presenting more substantial legal and factual issues, certainly do
not warrant or justify the period of three years, which it took the Tanodbayan to
resolve the case.

It has been suggested that the long delay in terminating the preliminary
investigation should not be deemed fatal, for even the complete absence of a
preliminary investigation does not warrant dismissal of the information. True-
but the absence of a preliminary investigation can be corrected by giving the
accused such investigation. But an undue delay in the conduct of a preliminary
investigation can not be corrected, for until now, man has not yet invented a
device for setting back time.

CASES AGAINST TATAD DISMISSED.























120. GONZALES VS. SB
speedy disposition of cases

Updated by K Pascual. 2014. 4C. Bill of Rights Poli Digests. Atty. Jack Jimenez. 4C. 174


Gonzales was a former director of the Bureau of Fisheries and Aquatic
Resources. He entered into an agreement with Palanca for the use of Otoshi-
Ami Net, or Lambaklad, for an experimental test fishing.
Later, a complaint was filed before the Fiscal of Manila for malversation of
public funds.
Another complaint was filed by with the Tanodbayan for illegal use of
vessel and loss of the Lumbaklad. Gonzales filed his counter-affidavit
answer.
However, The TB ruled that a PI should be conducted for the loss of the
Lumbaklad only.
Still later however, the TB also ordered that a PI be conducted for the
charge of illegal use of government vessels (which was previously
dismissed).
There was protracted litigation, until in the case reached the SB.
Gonzales argues that when he filed his counter-affidavit at the TB, as of that
date, the case should have been considered submitted for resolution. He
claims that there was long delay and that the case should be dismissed
already.
The Solgen contends that there was no delay since from the filing of the
complaint with the TB up to the filing of the information with the SB, there
had been numerous incidents in between, which required resolution during
the PI. The Solgen claims the long period was not due to inaction, but
precisely due to investigations conducted.

SC: NO DELAY. While there may seem to have been a delay in the termination of
the preliminary investigation involved in the case at bar, the same cannot be
imputed solely to the prosecution.

The right to a speedy disposition of a case, like the right to speedy trial, is
deemed violated only when the proceeding is attended by vexatious, capricious,
and oppressive delays; or when unjustified postponements of the trial are asked
for and secured, or when without cause or justifiable motive a long period of
time is allowed to elapse without the party having his case tried. The balancing
test used to determine whether a defendant has been denied his right to a
speedy trial, or a speedy disposition of a case for that matter, is measured by
factors such as
1) length of the delay,
2) reason for the delay,
3) the defendant's assertion or non-assertion of his right, and
4) prejudice to the defendant resulting from the delay.

In the present case, it will be noted that it was only on August 22, 1988 when
the complaint was filed. The original information was filed on May 6, 1989.
After the filing of said information, Gonzales himself filed several motions
seeking reconsideration and re-evaluation of the case and praying for the
suspension of the proceedings during the pendency of the resolution of these
motions. Thereafter, the Sandiganbayan ordered the conduct of a preliminary
investigation and, subsequently, the amended information was filed. It is,
therefore, apparent and irremissible that the delay is equally chargeable to
Gonzales. Hence, he cannot now seek the protection of the law to cover up for
his own actuations or benefit from what he now considers the adverse effects of
his own conduct in the case.

CHAMP Page 174 9/16/2014

121. US VS. NAVARRO
right against self-incrimination

Updated by K Pascual. 2014. 4C. Bill of Rights Poli Digests. Atty. Jack Jimenez. 4C. 175


1) Article 481 of the Penal Code provides that a private person who shall lock
up or detain another, or in any way deprive him of his liberty shall be
punished with the penalty of prision mayor.
2) The second paragraph of article 483 provides that one who illegally detains
another and fails to give information concerning his whereabouts, or does
not prove that he set him at liberty, shall be punished with cadena temporal
in its maximum degree to life imprisonment.
3) The punishment for the crime mentioned in, article 483 of the Penal Code is
the penalty of cadena temporal in its maximum degree to cadena perpetua,
or in other words one convicted of simply depriving a person of his liberty
may be imprisoned for a term of from six to twelve years and one convicted
of depriving a person of his liberty and who shall not state his whereabouts
or prove that he had set said person at liberty may be punished by
imprisonment for a term of seventeen years four months and one day, to
life, as in this case. In other words, for failure on the part of the defendant to
testify regarding the whereabouts of the person deprived of his liberty, or
to prove that he was set at liberty, the punishment may be increased from
imprisonment for a term of six years to life imprisonment.
4) This provision of the law has the effect of forcing a defendant to become a
witness in his own behalf or to take a much severer punishment. The
burden is put upon him of giving evidence if he desires to lessen the
penalty, or, in other words, of criminating himself, for the very statement of
the whereabouts of the victim or the proof that the defendant set him at
liberty amounts to a confession that the defendant unlawfully detained the
person.
5) So the evidence necessary to clear the defendant, under article 483 of the
Penal Code, would have the effect of convicting him under article 481.
6) Navarro, the accused claims that the provisions are illegal since no person
shall be compelled to be a witness against himself.

SC:
The evidence shows that some one has been taken away from home and has not
been heard of again, and the facts point to the prisoner as the presumptive
criminal. He is told to state what he knows of the matter. If he does so, and
proves that the person detained was liberated by him, or that such person is
living in such and such a place, then the prosecuting attorney will know that he
must draw a charge under the first or following sections of article 481,
according to whether the facts elicited by the preliminary or summary
investigation show only a detention in general, or for the specific periods of
time indicated in the latter part of the section. But if the prisoner fails to prove
the whereabouts of the person whom he is accused of making away with, or that
he liberated him, then the prosecuting attorney has a case falling within the last
paragraph of article 483.
It follows, therefore, from an examination of the old law that no prosecution
under this article would have ever been possible without a concomitant
provision of the procedural law which made it the duty of the accused to testify
and permitted the prosecution to draw an unfavorable deduction from his
refusal to do so.

Origins of the right against self-incrimination:
It was established on the grounds, of public policy and humanity-of policy,
because if the party were required to testify, it would place the witness under
the strongest temptation to commit the crime of perjury, and of humanity,
because it would prevent the extorting of confessions by duress. The very
object of adopting this provision of law was to wipe out such practices as
formerly prevailed in these Islands of requiring accused persons to submit to
judicial examinations, and to give testimony regarding the offenses with which
they were charged.

In Emery's case it was said that the principle applies equally to any compulsory
disclosure of the guilt of the offender himself, whether sought directly as the
object of the inquiry, or indirectly and incidentally for the purpose of
establishing facts involved in an issue between the parties.

Updated by K Pascual. 2014. 4C. Bill of Rights Poli Digests. Atty. Jack Jimenez. 4C. 176


If the disclosure thus made would be capable of being used against him as a
confession of crime, or an admission of facts tending to prove the commission of
an offense, such disclosure would be an accusation against himself.
In the present case, if the defendant, as said before, disclosed the whereabouts
of the person taken, or shows that he was given his liberty, this disclosure may
be used to obtain a conviction under article 481 of the Penal Code.

In this case, if the defendant does not do certain things, if he does not make
certain statements or proofs, he is severely punished.

If it be urged that the defendant is not compelled to testify, that he may remain
mute, the answer is that, the illegal detention only being proved by the
prosecution, if he does not make certain proof, if he remains mute, then not only
the presumption but the fact of guilt follows as a consequence of his silence, and
such a conclusion is not permitted under American law.

A law which, while permitting a person accused of a crime to be a witness s in
his own behalf, should at the same time authorize a presumption of guilt from
his omission to testify would be a law adjudging guilt without evidence, and
while it might not be obnoxious to the constitutional provision against
compelling a party in a criminal case to give evidence against himself, would be
a law reversing the presumption of innocence, and would violate the
fundamental principles binding alike upon the legislature and the courts."

It is the duty of the prosecution, in order to convict one of a crime, to produce
evidence showing guilt beyond a reasonable doubt; and the accused can not be
called upon, either by express words or acts to assist in the production of such
evidence; nor should his silence be taken as proof against him. He has a right to
rely on the presumption of innocence until the prosecution proves him guilty of
every element of the crime with which he is charged.

Summary: The provision has the effect of forcing the defendant to punishment. It
places the burden on him to give evidence if he wants a lesser penalty. The
information charged amounts to a confession that he voluntarily detained the
victim!! This would convict him under Art 481.







122. CABAL VS. KAPUNAN
right against self incrimination

Maristella of the Phil. Army filed a complaint with the DND charging Chief of
Staff Cabal, with graft and corruption, unexplained wealth, dictatorial
tendencies.
The President created a committee to investigate.
The Committee ordered Cabal to take the witness stand.
Cabal objected, invoking his right against self incrimination.
The Committee insisted that he take the wtiness stand and be sworn to,
subject only to his right to refuse to answer any question that may be
incriminatory.
The Committee claimed however that the investigation is merely
administrative and not criminal in nature. They also claimed that under the
Updated by K Pascual. 2014. 4C. Bill of Rights Poli Digests. Atty. Jack Jimenez. 4C. 177


constitution, what can be refused in merely to answer an incriminatory
question, but cannot refuse to take the witness stand altogether.

ISSUE: At the outset, it is not disputed that the accused in a criminal case may
refuse, not only to answer incriminatory questions, but, also, to take the witness
stand. Hence, the issue before us boils down to whether or not the proceedings
before the aforementioned Committee is civil or criminal in character.

SC: Although said Committee was created to investigate the administrative
charge of unexplained wealth, there seems to be no question that Col. Maristela
does not seek the removal of petitioner herein as Chief of Staff of the AFP. As a
matter of fact he no longer holds such office. The real purpose of the charge
against petitioner is to apply the Anti-Graft Law, which authorizes the forfeiture
to the State of property of a public officer or employee which is manifestly out
of proportion to his salary as such public officer or employee and his other
lawful income and the income from legitimately acquired property. Such
forfeiture has been held, however, to partake of the nature of a penalty.

A forfeiture, as thus defined, is imposed by way of punishment not by the mere
convention of the parties, but by the lawmaking power, to insure a prescribed
course of conduct. The effect of such a forfeiture is to transfer the title to the
specific thing from the owner to the sovereign power. Thus, proceedings for
forfeiture of property are deemed criminal or penal, and, hence, the
exemption of defendants in criminal case from the obligation to be
witnesses against themselves are applicable thereto. Such proceedings are
criminal in nature to the extent that where the person using the res illegally is
the owner of rightful possessor of it the forfeiture proceeding is in the nature of
a punishment.

A witness or party called as a witness cannot be made to testify against himself
as to matters which would subject his property to forfeiture. No person could be
compelled to testify against himself or to answer any question which would
have had a tendency to expose his property to a forfeiture, or to form a link in a
chain of evidence for that purpose, as well as to incriminate him.

The rule protecting a person from being compelled to furnish evidence
which would incriminate him exists not only when he is liable criminally
to prosecution and punishment, but also when his answer would tend to
expose him to a forfeiture.

No person shall be compelled in any criminal case to be a witness against
himself. This prohibition against compelling a person to take the stand as
a witness against himself applies only to criminal, quasi-criminal, and
penal proceedings, including a proceeding civil in form for forfeiture of
property by reason of the commission of an offense, but not a proceeding
in which the penalty recoverable is civil or remedial in nature.

The privilege of a witness not to incriminate himself is not infringed by merely
asking the witness a question which he refuses to answer. The privilege is
simply an option of refusal, and not a prohibition of inquiry. However, where
the position of the witness is virtually that of an accused on trial, it would
appear that he may invoke the privilege in support of a blanket refusal to
answer any and all questions."

In short, in this case, Cabal even in an administrative proceeding under the Anti
Graft Law, CANNOT BE REQUIRED TO TAKE THE WITNESS STAND. A person
may not be compelled to testify in an action against him for a penalty or to
answer any question as a witness which would subject him to a penalty or
forfeiture, where the penalty or forfeiture is imposed as a vindication of the
public justice of the state.

CHAMP Page 177 9/16/2014
Updated by K Pascual. 2014. 4C. Bill of Rights Poli Digests. Atty. Jack Jimenez. 4C. 178



Right Against Self-Incrimination = available:
1) criminal cases
2) civil cases
3) administrative cases
as long as the penalty is penal in nature.





























123. PASCUAL VS. MEDICAL BOARD
right against self incrimination

This is the case of Doctorney Pascual ng Formed.
There was an administrative case before the Board of Medical Examiners,
for alleged medical malpractice against him.
The complainants wanted to present Doctorney Pascual himself as their
first witness.
Updated by K Pascual. 2014. 4C. Bill of Rights Poli Digests. Atty. Jack Jimenez. 4C. 179


Doctorney objected, citing that he was the respondent in the malpractice
case. He claims that if he testified for the complainants, it would violate his
constitutional right to be exempt from being a witness against himself.
The Board however ruled against him. He was ordered to testify.
He filed an action for prohibition/ injnunction.
He argued that in compelling him to take the witness stand, the Board was
guilty, of grave abuse of discretion for failure to respect the constitutional
right against self-incrimination, the administrative proceeding against him,
which could result in forfeiture or loss of a privilege, being quasi-criminal in
character.
The Board on the other hand contended that, the right against self-
incrimination being available only when a question calling for an
incriminating answer is asked of a witness. They likewise alleged that the
right against self- incrimination cannot be availed of in an administrative
hearing.

ISSUE: Can he be ordered to testify?
SC: NO. DOCTORNEY PASCUAL WINS.

We conclude that the right against self incrimination is not available only to
lawyers, but such a principle is equally applicable to a proceeding that could
possibly result in the loss of the privilege to practice the medical profession.

The proceeding for forfeiture while administrative in character thus possesses a
criminal or penal aspect. In this case, petitioner would be similarly
disadvantaged. He could suffer not the forfeiture of property but the revocation
of his license as medical practitioner. (deprivation of livelihood)

The right is not limited to just refusing to answer incriminating questions.
The constitutional guarantee protects as well the right to silence. The
accused has a perfect right to remain silent and his silence cannot be used as
a presumption of his guilt." It is the right of a defendant "to forego testimony,
to remain silent, unless he chooses to take the witness stand-with undiluted,
unfettered exercise of his own free genuine will."

The principle of humanity on which this right is predicated, precludes all resort
to force or compulsion, whether physical or mental. Current judicial opinion
places equal emphasis on its identification with the right to privacy. The Fifth
Amendment enables the citizen to create a zone of privacy which government
may not force to surrender to his detriment."

We hold that in an administrative hearing against a medical practitioner for
alleged malpractice, respondent Board of Medical Examiners cannot,
consistently with the self-incrimination clause, compel the person proceeded
against to take the witness stand without his consent.


124. ALMONTE VS. VASQUEZ
right against self-incrimination

Almonte was the Commissioner of the EIIB, while Perez was the Budget
Chief of the EIIB.
There was an anonymous letter alleging illegal disbursements of funds
from unfilled positions in the plantilla of EIIB. The letter mentioned a
syndicate headed by Perez who is manipulating funds for ghost agents.
Updated by K Pascual. 2014. 4C. Bill of Rights Poli Digests. Atty. Jack Jimenez. 4C. 180


The OMB issued a subpoena duces tecum requiring the Chief
Accountant of EIIB, Mr. Rogado, to produce all documents relating to
the Funds of the EIIB, such as salary vouchers.
Almonte and Perez moved to quash the subpoena. DENIED because the
subpoena was not directed to them but to the Chief Accountant Rogado.
Still Almonte and Perez insisted on the quashal arguing that the Chief
Accountant is a person under their supervision, and that the OMB was
doing indirectly what he could not do directly (ie, compelling them
Almonte and Perez, to produce evidence against themselves).

ISSUE: Can the OMB subpoena the documents?
SC: YES. ALMONTE AND PEREZ CANNOT CLAIM RIGHT AGAINST SELF-
INCRIMINATION.

It is enough to state that the documents being required to be produced are
PUBLIC RECORDS and those to whom the subpoena duces tecum is directed are
GOVERNMENT OFFICIALS who are in possession and custody of the documents.

Moreover, since they claim that the disbursements have already been audited
and cleared by COA, then there is no reason why they should object to the
examination of the documents.

Court directed that the inspection of the subpoenaed documents be made
personally in camera by the OMB. (that is, the public excluded).





















125. PEOPLE VS. MALIMIT
right against self-incrimination

Updated by K Pascual. 2014. 4C. Bill of Rights Poli Digests. Atty. Jack Jimenez. 4C. 181


Accused killed the owner of the store. He took the wallet of the victim
containing his residence certificate, ID, and a bunch of keys. These items
were recovered from the accused.
He was then charged with robbery with homicide.
He argued the admissibility of the wallet and its contents as evidence,
claiming it violates the right against self-incrimination.

SC: We are not persuaded. The night against self-incrimination guaranteed
under our fundamental law finds no application in this case.
The right against self incrimination is a prohibition of the use of physical or
moral compulsion, to extort communications from him. It is simply a
prohibition against legal process to extract from the [accused]'s own lips,
against his will, admission of his guilt. It does not apply to the instant case
where the evidence sought to be excluded is not an incriminating
statement but an object evidence.

"If, in other words (the rule) created inviolability not only for his [physical
control of his] own vocal utterances, but also for his physical control in
whatever form exercise, then, it would be possible for a guilty person to shut
himself up in his house, with all the tools and indicia of his crime, and defy the
authority of the law to employ in evidence anything that might be obtained by
forcibly overthrowing his possession and compelling the surrender of the
evidential articles a clear reduction ad absurdum. In other words, it is not
merely compulsion that is the kernel of the privilege, but testimonial
compulsion."

Wallet and its contents admissible.


(there was an issue on Sec 12 rights in custodial investigation The court said
that the violation of the Miranda rights only makes inadmissible only
extrajudicial confession or admission made during custodial investigation. It
does not affect the admissibility of OTHER KINDS OF EVIDENCE. The
admissibility of other evidence, provided they are relevant to the issue and is
not otherwise excluded by law or rules, is not affected even if obtained or taken
in the course of custodial investigation. This supports the conclusion that the
wallet and its contents are admissible. )

The admissibility of Malaki's wallet, identification card, residence certificate and
keys are for the purpose of establishing other facts relevant to the crime. The
wallet is admissible to establish the fact that it was the very wallet taken from
Malaki on the night of the roberry. The identification card, residence certificate
and keys found inside the wallet, on the other hand, are admissible to prove that
the wallet really belongs to Malaki.

CHAMP Page 181 9/16/2014







126. US. VS. TAN TENG
right against self-incrimination

Updated by K Pascual. 2014. 4C. Bill of Rights Poli Digests. Atty. Jack Jimenez. 4C. 182


1. Tan Teng was accused of raping a 7 year old girl.
2. He was arrested and taken to the police station and stripped of his clothing
and examined. The policeman who examined the defendant swore that his
body bore every sign that he was suffering from the venereal disease
known as gonorrhea. The policeman took a portion of the substance
emitting from his body and turned it over to the Bureau of Science for the
purpose of having a scientific analysis made of the same. The result of the
examination showed that the he was suffering from gonorrhea.
3. The prosecution relied on the fact that the victim contracted the same
disease.
4. Defendant contended that the result of the scientific examination made by
the Bureau of Science of the substance taken from his body, at or about the
time he was arrested, was not admissible in evidence as proof of the fact
that he was suffering from gonorrhea. The defendant objected to the
admissibility of such evidence upon the ground that it was requiring him to
give testimony against himself.
5. The objection was overruled upon the ground that "the accused was not
compelled to make any admission or answer any questions, and the mere
fact that an object found upon his person was examined seems no more to
infringe the rule invoked, than would the introduction of stolen property
taken from the person of a thief."

SC: ADMISSIBLE.
The prohibition of compelling a man in a criminal court to be a witness against
himself, is a prohibition of the use of physical or moral compulsion, to extort
communications from him, not an exclusion, of his body as evidence, when it
may be material. The prohibition is simply a prohibition against legal process to
extract from the defendant's own lips, against his will, an admission of his guilt.

Such an inspection of the bodily features by the court or by witnesses, can not
violate the privilege granted under the Philippine Bill, because it does not call
upon the accused as a witness-it does not call upon the defendant for his
testimonial responsibility. Mr. Wigmore says that evidence obtained in this way
from the accused, is not testimony by his body but his body itself.

Also, the substance was taken from the body of the defendant without his
objection. The examination of the substance was made by competent medical
authority and the result showed that the defendant was suffering from said
disease. Such evidence was clearly admissible.


(see original for other examples of bodily inspections)
observing wound on the hands of the accused.
Comparing the hand of the accused with the bloody prints of a hand upon a
wall
Causing the prints of the shoes to be made in the sand for comparison.







127. BELTRAN VS. SAMSON
right against self-incrimination

Updated by K Pascual. 2014. 4C. Bill of Rights Poli Digests. Atty. Jack Jimenez. 4C. 183


The fiscal filed a petition before the court for the purpose of comparing the
petitioner's handwriting and determining whether or not it is he who wrote
certain documents supposed to be falsified.
Petitioner complains that the respondent judge ordered him to appear
before the provincial fiscal to take dictation in his own handwriting from
the latter.

ISSUE: whether the writing from the fiscal's dictation by the petitioner for the
purpose of comparing the latter's handwriting and determining whether he
wrote certain documents supposed to be falsified, constitutes evidence against
himself within the scope and meaning of the constitutional right against self-
incrimination.

SC: NOT ALLOWED.
Note in this case, there is no information filed yet, much less a trial. This is only
an investigation prior to the information and with a view to filing it.

While it is true that Professor Wigmore,(of Evidence), says that:
"Measuring or photographing the party is not within the privilege. Nor is the
removal or replacement of his garments or shoes. Nor is the requirement that
the Party move his body to enable the foregoing things to be done. Requiring
him to make specimens of handwriting is no more than requiring him to move
his body,
In the case before us, writing is something more than moving the body, or
the hand, or the fingers; writing is not a purely mechanical act, because it
requires the application of intelligence and attention; and in the case at bar
writing means that the petitioner herein is to furnish a means to
determine whether or not he is the falsifier, as the petition of the respondent
fiscal clearly states.

We say that, for the purposes of the constitutional privilege, there is a similarity
between one who is compelled to produce a document, and one who is
compelled to furnish a specimen of his handwriting, for in both cases, the
witness is required to furnish evidence against himself.

Furthermore, the present ease is more serious than that of compelling the
production of documents or chattels, because here the witness is compelled to
write and create, by means of the act of writing, evidence which does not
exist, and which may identify him as the falsifier.

For though the disclosure thus sought be not oral in form, and though the
documents or chattels be already in existence and not desired to be first written
and created by a testimonial act or utterance of the person in response to the
process, still no line can be drawn short of any process which treats him as a
witness; because in virtue of it he would be at any time liable to make oath to
the identity or authenticity or origin of the articles produced."

Considering the circumstance that the petitioner is a municipal treasurer, it
should not be a difficult matter for the fiscal to obtain OTHER genuine
specimens of his handwriting.

(see original)

128. PEOPLE VS. ECHEGARAY
right against cruel and unusual punishment

Updated by K Pascual. 2014. 4C. Bill of Rights Poli Digests. Atty. Jack Jimenez. 4C. 184


Echegaray charged and convicted with raping his 10 year old daughter. He
was sentenced to death pursuant to RA7659, which was the law at the time
of commission of crime.
In appealing the conviction, it raised the constitutionality of the Death
Penalty Law as being severe and excessive, cruel and unusual in violation of
the constitution.
He invokes the ruling in Furman vs. Georgia wherein the US Supreme
Court categorically ruled that death penalty is cruel and degrading.
He also argues that death is an excessive and cruel punishment for a crime
of rape because there is no taking of life in rape. He invokes the ruling in
Coker vs. Georgia which said that while rape deserves serious punishment,
it should not involve the taking of human life. In rape, life is not over for the
victim. Death penalty should only be imposed where the crime was murder.

ISSUE: Is the death penalty cruel and unusual punishment?
SC: NO. VALID LAW.

The penalty is neither cruel, unjust nor excessive. In the US case of Kemmler, it
was held that punishments are cruel when they involve torture or a lingering
death. It implies there something inhuman, barbarous, something more than the
extinguishment of life. It is degrading if it involves public humiliation. The
severity is not sufficient, but must be disproportionate to the crime committed.
Excessiveness is measured by 1) seriousness of the crime, 2) policy of the
legislative, 3) perversity of the accused.

The issue in Furman vs. Georgia is not so much the death penalty itself, but the
arbitrariness pervading the procedures by which the death penalty was
imposed by the jury. The Furman case did not outlaw death penalty because it
was cruel and unusual per se. It was nullified because the discretion in which
the statute vested in trial judges and sentencing juries was uncontrolled and
without any parameters, guidelines, or standards. (There was apparently a
discrimination against the accused who was black.)

With regard to the case of Coker vs. Georgia, the SC held that this case has no
bearing on Philippine experience and culture. Such a premise is in fact an
ennobling of the biblical notion of retributive justice of "an eye for an eye, a
tooth for a tooth". But, the forfeiture of life simply because life was taken, never
was a defining essence of the death penalty in the context of our legal history
and cultural experience; rather, the death penalty is imposed in heinous crimes
because the perpetrators thereof have committed unforgivably execrable acts
that have so deeply dehumanized a person or criminal acts with severely
destructive effects, and because they have so caused irreparable and substantial
injury to both their victim and the society and a repetition of their acts would
pose actual threat to the safety of individuals and the survival of government,
they must be permanently prevented from doing so.

The court also emphasized that under the Constitution, the Congress has the
power to reimpose the death penalty for compelling reasons, involving heinous
crimes. Congress can define or describe what is meant by the word heinous
crimes, and can specify which crimes would qualify as heinous. Thus, the court
should not be the venue for debates regarding the morality or propriety of the
death sentence because the law itself already provided for specific and well
defined criminal acts.

RA 7659 already sufficiently defined what are heinous crimes crimes
punished with death are those that are grievous, odious, and hateful by reason
of inherent viciousness, atrocity and perversity, those that are repugnant and
outrageous to common standards of norms and decency and morality in a just,
civilized and ordered society. They also include crimes which are despicable
because life is callously taken, or the victim is treated as an animal or
dehumanized.

Updated by K Pascual. 2014. 4C. Bill of Rights Poli Digests. Atty. Jack Jimenez. 4C. 185


There is also another concept of the degree of punishment: that you may
NOT punish a person for what he is, but punish only what he has done. (ex. drug
addicts)


See original and bernas primer.






































Updated by K Pascual. 2014. 4C. Bill of Rights Poli Digests. Atty. Jack Jimenez. 4C. 186


129. LOZANO VS. MARTINEZ
non-imprisonment for debt / non-payment of poll tax (cedula)

The petitioners, being charged with BP 22, assail the statutes
constitutionality.
BP 22 punishes a person "who makes or draws and issues any check on
account or for value, knowing at the time of issue that he does not have
sufficient funds in or credit with the drawee bank for the payment of said
check in full upon presentment, The penalty prescribed is imprisonment of
not less than 30 days nor more than one year or a fine or not less than the
amount of the check nor more than double said amount, but in no case to
exceed P200,000.00, or both such fine and imprisonment at the discretion
of the court.
Petitioners insist that since the offense under BP 22 is consummated only
upon the dishonor or non-payment of the check when it is presented to the
drawee bank, the statute is really a "bad debt law" rather than a "bad check
law." What it punishes is the non-payment of the check, not the act of
issuing it. The statute, it is claimed, is nothing more than a veiled device to
coerce payment of a debt under the threat of penal sanction.

SC: LAW VALID.
The constitutional prohibition against imprisonment for debt is a safeguard that
evolved gradually during the early part of the nineteenth century which
permitted creditors to cause the incarceration of debtors who could not pay
their debts.

The gravamen of the offense punished by BP 22 is the act of making and issuing
a worthless check or a check that is dishonored upon its presentation for
payment. It is not the non-payment of an obligation which the law
punishes. The law is not intended or designed to coerce a debtor to pay his
debt. The thrust of the law is to prohibit, under pain of penal sanctions, the
making of worthless checks and putting them in circulation. Because of its
deleterious effects on the public interest, the practice is proscribed by the law.
The law punishes the act not as an offense against property, but an offense
against public order.

It may be unconstitutional for the legislature to penalize a person for non-
payment of a debt ex contractu. But certainly it is within the prerogative of the
lawmaking body to proscribe certain acts deemed pernicious and inimical to
public welfare. BP 22 is a declaration by the legislature that, as a matter of
public policy, the making and issuance of a worthless check is deemed a public
nuisance to be abated by the imposition of penal sanctions. (valid exercise of
police power) It had been reported that the approximate value of bouncing
checks per day was close to 200 million pesos, and thereafter when overdrafts
were banned by the Central Bank, it averaged between 50 million to 80 million
pesos a day. 26

Checks have the element of certainty or assurance that the instrument will be
paid upon presentation. For this reason, checks have become widely accepted as
a medium of payment in trade and commerce. Although not legal tender, checks
have come to be perceived as convenient substitutes for currency in commercial
and financial transactions. The basis or foundation of such perception is
confidence. If such confidence is haken, the usefulness of checks as currency
substitutes would be greatly diminished or may become nil. Any practice
therefore tending to destroy that confidence should be deterred, for the
proliferation of worthless checks can only create havoc in trade circles and the
banking community.


130. CUISON VS. CA
dj

Updated by K Pascual. 2014. 4C. Bill of Rights Poli Digests. Atty. Jack Jimenez. 4C. 187


The trial court convicted the accused of double murder and ordered him to
pay P30,000 to the heirs of the victim.
On appeal, the CA affirmed the conviction, but increased the indemnity to
P50,000.
The case was remanded to the trial court for the promulgation of the CA
decision. The trial court however promulgated the decision only with
respect to the civil liability. Its dispositive portion only mentioned the civil
liability but not the criminal liability.
The CA ordered the RTC to promulgate the decision anew to include the
affirmance of the conviction.
The accused now invoked DJ that the first promulgation of the CA decision
already terminated the criminal cases.

ISSUE: Can the accused invoke DJ as a defense?
SC: NO.

The promulgation of the decision is not merely incomplete, but also VOID. This
is because the RTC promulgated the decision only with respect to the civil
liability. Thus, it did not effectively terminate the criminal case against the
accused.

To claim DJ:
first jeopardy must have attached prior to the second
o upon a valid indictment
o before a competent court
o after arraignment
o after valid plea
o case was dismissed or otherwise terminated without the express
consent of the accused.
first jeopardy must have been validly terminated
second jeopardy must be for the same offense, or the it includes or is
necessarily included in the first offense charged, or an attempt of
frustration thereof

The rule is that a criminal prosecution includes a civil action for the recovery of
indemnity. Thus, a decision in such cases should dispose of both the criminal as
well as the civil liabilities of the accused. In this case however, the RTC
promulgated only the civil aspect of the case, but not the criminal.

Since criminal cases have not been terminated, the first jeopardy has not yet
attached. Hence, DJ cannot propser as a defense.

The promulgation of only one part of the decision (civil liability) is not a bar to the
subsequent promulgation of the other part, the imposition of criminal liability.
Double jeopardy is not violated.

CHAMP Page 187 9/16/2014





Updated by K Pascual. 2014. 4C. Bill of Rights Poli Digests. Atty. Jack Jimenez. 4C. 188


131. PEOPLE VS. OBSANIA
DJ - termination

Accused was indicted for rape with robbery.
Later. the Asst. Provincial Fiscal filed an information for RAPE with
additional averment that the offense was committed WITH LEWD DESIGNS
Accused pleaded not guilty and filed a MTD contending that the complaint
was fatally defective for failure to allege lewd designs in the original
complaint. He claims that the failure of the complaint filed by the
complainant to allege that the acts committed were with lewd designs did
not give the court jurisdiction to try the case. The subsequent information
which contained the averment did not cure this jurisdictional infirmity.
The court granted the MTD and dismissed the action.
The Fiscal now appealed.

In a complaint for rape it is not necessary to allege "lewd design" or "unchaste
motive," for to require such averment is to demand a patent superfluity. It is
clear that the complaint here satisfies the requirements of legal sufficiency of an
indictment for rape as it unmistakably alleges that the accused had carnal
knowledge of the complainant by means of violence and intimidation. As such
the court had already acquired jurisdiction over the case, on the first
information filed.

The complaint filed with the municipal court in the case at bar was valid; the
court a quo was a competent tribunal with jurisdiction to hear the case; the
record shows that the accused pleaded not guilty upon arraignment.

ISSUE: whether the dismissal of the case was without the express consent of the
accused. ISSUE: Does the fiscals appeal place the accused in DJ?
SC: NO DJ.

The accused admits that the dismissal was ordered by the trial judge upon his
motion to dismiss. However, he vehemently contends that an erroneous
dismissal of a criminal action, even upon the instigation of the accused in a
motion to quash or dismiss, does not bar him from pleading the defense of
double jeopardy in a subsequent appeal by the Government or in a new
prosecution for the same offense. The accused suggests that the above-
enumerated cases have abandoned the previous ruling of this Court to the effect
that when a case is dismissed, other than on the merits, upon motion of the
accused personally or through counsel, such dismissal is to be regarded as with
the express consent of' the accused and consequently he is deemed to have
waived his right to plead double jeopardy and/or he is estopped5 from claiming
Such defense on appeal by the Government or in another indictment for the
same offense.

The Doctrine of Waiver of DJ provides that when the case is dismissed with the
express consent of the accused, the dismissal will NOT be a bar to another
prosecution for the same offense because his action in having the case
dismissed CONSTITUTES A WAIVER OF HIS RIGHT, for the reason that he
thereby prevents the court from proceeding to the trial on the merits and
rendering a judgment against him. [salico doctrine]

Had the dismissal of the case been anchored on a Motion To Dismiss, the
accused will not be entitled o the protection against double jeopardy.

There is also a Doctrine of Estoppel which held that when the trial court
dismisses the case on a disclaimer of jurisdiction upon the instigation of the
accused, he is estopped on appeal from asserting the jurisdiction of the lower
court in invoking second jeopardy. The doctrine of estoppel is the same as the
Updated by K Pascual. 2014. 4C. Bill of Rights Poli Digests. Atty. Jack Jimenez. 4C. 189


doctrine of waiver: the thrust of both is that A DISMISSAL, OTHER THAN ON
THE MERITS, SOUGHT BY THE ACCUSED IN A MTD, IS DEEMED TO BE WITH
HIS EXPRESS CONSENT AND BARS HIM FROM SUBSEQUENTLY INTERPOSING
THE DEFENSE OF DJ ON APPEAL OR IN A NEW PROSECUTION FOR THE SAME
OFFENSE.

There was a lengthy discussion on the different cases of Bangalao, Ferrer and
Labatete which the accused argued abandoned the Salico Doctrine. [see
original] but the point is that the dismissals in those cases cited by the accused
were different from his case since they were all considered acquittals because
they were predicated on the right to speedy trial and the failure of the
government to prosecute. Thus, even if they motion to dismiss was sought by
the accused, there is still no waiver or estoppel. In those cases, the dismissal
amounted to an acquittal because of the failure of government to prosecute the
accused, who is presumed innocent.

Thus, the Salico Doctrine remains valid and it presupposes a dismissal not
amounting to an acquittal. Here, the dismissal was predicated on the erroneous
contention of the accused that the complaint was defective by failure of
averments, and such infirmity affected the jurisdiction of the trial court. But as
stated earlier, the dismissal in this case did not terminate the case on the merits.

2 doctrines of Waiver and Estoppel requires 2 conditions:
1) Dismissal must be sought by the accused personally or through his counsel
2) Such dismissal must not be on the merits and must not amount to an
acquittal.

























Updated by K Pascual. 2014. 4C. Bill of Rights Poli Digests. Atty. Jack Jimenez. 4C. 190



132. CUDIA VS. CA
DJ- attachment

Cudia was arrested in Mabalacat, Pampanga for illegal possession of
unlicensed revolver
The City prosecutor of Angeles City filed in the RTC of Angeles an
information for illegal possession committed in Angeles City.
He pleaded not guilty.
During trial it was shown that the crime was really committed in Mabalacat,
not in Angeles.
Thus the case was re-assigned to another RTC branch (because there was
an internal arrangement among the judges that crimes committed outside
Angeles City can be tried only by certain branches of the RTC).
The Provincial prosecutor now filed a new information charging Cudia with
illegal possession and file a motion to withdraw the first information. Thus,
there were 2 separate informations for the same offense filed against Cudia.
Cudia filed a motion to quash the new information on the ground of DJ.

SC:To claim DJ:
first jeopardy must have attached prior to the second
o before a competent court
o upon a valid indictment
o after arraignment
o after valid plea
o case was dismissed or otherwise terminated without the express
consent of the accused.
first jeopardy must have been validly terminated
second jeopardy must be for the same offense, or the it includes or is
necessarily included in the first offense charged, or an attempt of
frustration thereof

ISSUE: Was there a court of competent jurisdiction?
SC: YES. The RTC of Angeles covers the municipality of Angeles City and
Mabalacat.
It is necessary that there be a court of competent jurisdiction, for jurisdiction to
try the case is essential to place an accused in jeopardy. Clearly, Branches 56 to
62 had jurisdiction over the respective territories as apportioned. Consequently,
notwithstanding the internal arrangement of the judges of the Angeles City
RTCs, Branch 60 indubitably had jurisdiction over instant case. Writ large in law
books is the doctrine that jurisdiction is conferred by law and not by mere
administrative policy of any trial court.

ISSUE: Was there a valid complaint or indictment?
SC: NO. It is plainly apparent that the City Prosecutor of Angeles City had no
authority to file the first information, the offense having been committed in the
Municipality of Mabalacat, which is beyond his jurisdiction. It is thus the
Provincial Prosecutor of Pampanga, not the City Prosecutor, who should
prepare informations for offenses committed within Pampanga but outside of
Angeles City. An information, when required to be filed by a public prosecuting
officer, cannot be filed by another. It must be exhibited or presented by the
prosecuting attorney or someone authorized by law. If not, the court does not
acquire jurisdiction. An infirmity in the information, such as lack of authority
of the officer signing it, cannot be cured by silence, acquiescence, or even by
express consent.
Updated by K Pascual. 2014. 4C. Bill of Rights Poli Digests. Atty. Jack Jimenez. 4C. 191



Therefore, the first jeopardy has not attached yet because of a defective
complaint. He cannot claim DJ.

In fine, there must have been a valid and sufficient complaint or information in
the former prosecution. If, therefore, the complaint or information was
insufficient because it was so defective in form or substance that the conviction
upon it could not have been sustained, its dismissal without the consent of the
accused cannot be pleaded. As the fiscal had no authority to file the information,
the dismissal of the first information would not be a bar to petitioner's
subsequent prosecution. Jeopardy does not attach where a defendant pleads
guilty to a defective indictment that is voluntarily dismissed by the prosecution.

The dismissal of the first defective information does not bar a subsequent
prosecution.

ISSUE: The accused claims that it was the error of the City prosecutor, and this
should not be used to prejudice him.
SC: No. The State is not bound by the mistakes of its officials.

CHAMP Page 191 9/16/2014

Note: If the dismissal occurred during the preliminary investigation, then there is
NO DOUBLE JEOPARDY.

























Updated by K Pascual. 2014. 4C. Bill of Rights Poli Digests. Atty. Jack Jimenez. 4C. 192









133. GUERRERO VS. CA
DJ

Guerrero was charged with triple homicide through reckless imprudence
Accused was a pilot flying a non-commercial plane which crashed in Nueva
Ecija
When the case was submitted for decision, it was raffled to a new judge.
(judge Aquino)
Judge Aquino ordered the parties to follow-up and complete the transcript
of stenographic notes within 30 days considering that the same was found
to be incomplete. since the parties were not able to complete the transcript
of stenographic notes, the court ordered the retaking of the testimonies of
the witnesses.
petitioner claims that he is entitled to a dismissal of the criminal case
equivalent to an acquittal on the merits based on the violation of his right to
speedy trial resulting from the failure to render a prompt disposition of
judgment.
He also claims, that through no fault of his, seven of the ten witnesses who
testified for the accused will no longer be able to testify anew. So too, three
witnesses for the prosecution have died and thus would not be able to
appear during the re-hearing. And even if all witnesses would be able to
testify again, "the passage of a long period of time spanning more than two
decades since the incident complained of will tend to confuse or hinder
than aid the accurate recall of the facts and circumstances of the case,
Finally he contends that that the re-hearing would place him in double
jeopardy

SC: NO DJ.
In this case, there has been no termination of the criminal prosecution -
i.e. of that "first jeopardy."

In the present case, there has not even been a first jeopardy, since the fourth
element - dismissal or termination of the case without the express consent of
the accused - is not present. (The case was merely re-raffled). Moreover,
measured against the aforequoted standard, the retaking of testimonies cannot
in any wise be deemed a second jeopardy. Hence, it is beyond dispute that
petitioner's claim of double jeopardy is utterly without basis

To claim DJ:
first jeopardy must have attached prior to the second
o before a competent court
o upon a valid indictment
o after arraignment
o after valid plea
o case was dismissed or otherwise terminated without the express
consent of the accused.
Updated by K Pascual. 2014. 4C. Bill of Rights Poli Digests. Atty. Jack Jimenez. 4C. 193


first jeopardy must have been validly terminated
second jeopardy must be for the same offense, or the it includes or is
necessarily included in the first offense charged, or an attempt of
frustration thereof






134. TUPAS VS ULEP.
DJ - termination

2 identical informations were filed against Tupaz in the RTC for
nonpayment of deficiency income tax in 1997
The cases were assigned to different branches. Tupaz was arraigned in the
branch where the second case was filed. He pleaded not guilty there.
The information was amended to change the date of commission. But she
was not re-arraigned on this amended information.
She thus filed a motion for reinvestigation.
Erroneously thinking that the 2
nd
information was for non-payment of
deficiency contractors tax, the prosecution filed a motion to withdraw the
case (since the accused is exempted from paying contractors tax). This
motion was granted and the case was dismissed (they thought that the 2
informations were identical).
Later, realizing the mistake, the prosecution filed a motion to reinstate the
case.
The accused invoked DJ. She contends that by reinstating the information,
the trial court exposed her to double jeopardy. Neither the prosecution nor
the trial court obtained her permission before the case was dismissed. She
was placed in jeopardy for the first time after she pleaded to a valid
complaint filed before a competent court and the case was dismissed
without her express consent.
The Solgen however, argues that reinstating the information does not
violate petitioner's right against double jeopardy. He asserts that petitioner
induced the dismissal of the complaint when she sought the reinvestigation
of her tax liabilities. By such inducement, petitioner waived or was
estopped from claiming her right against double jeopardy. Solicitor General
further contends that, assuming arguendo that the case was dismissed
without petitioner's consent, there was no valid dismissal of the case since
Prosecutor Agcaoili was under a mistaken assumption that it was a charge
of nonpayment of contractor's tax.

SC: DOUBLE JEOPARDY!!
The reinstatement of the information would expose her to double jeopardy. An
accused is placed in double jeopardy if he is again tried for an offense for which
he has been convicted, acquitted or in another manner in which the indictment
against him was dismissed without his consent. In the instant case, there was a
valid complaint filed against petitioner to which she pleaded not guilty. The
court dismissed the case at the instance of the prosecution, without asking for
accused-petitioner's consent. This consent cannot be implied or presumed.
Such consent must be expressed as to have no doubt as to the accused's
conformity. As petitioner's consent was not expressly given, the dismissal of the
case must be regarded as final and with prejudice to the re-filing of the case.

Davide dissents:
Not having been re-arraigned on the amended information, which validly
supplanted the original information, the erroneous withdrawal of the
Updated by K Pascual. 2014. 4C. Bill of Rights Poli Digests. Atty. Jack Jimenez. 4C. 194


information and its subsequent reinstatement cannot place the petitioner in
double jeopardy. Firstly, the withdrawal had no legal effect since the
information was amended. Secondly, petitioner was not arraigned on the
amended information. And, thirdly, petitioner is estopped on the matter since
she had asked for a reinvestigation on the basis of the amended information.

CHAMP Page 194 9/16/2014




135. PEOPLE VS. VELASCO
DJ

Mayor Honorato Galvez was charged with murder of Vinculado.
He was acquitted due to insufficiency of evidence.
The State now wants the reversal of the acquittal, claiming that the
exculpation of the accused from all criminal liability by Judge Velasco
constitutes gadalej. It is claimed that the Judge deliberately and wrongfully
disregarded certain facts, and exercised gross judicial indiscretion and
arbitrariness.
The State claims that there would be no double jeopardy since under US
cases, the double jeopardy clause permits a review of acquittals where
there would be no retrial required should the judgment be overturned.

SC: NOT ALLOWED. (there was a lengthy discussion on the origins of the right
and the different US cases)
This petition for certiorari actually seeks the review of the judgment of
acquittal. In the requisites for DJ, the rules do not distinguish whether it occurs
at the level of the trial court or on appeal. This establishes the FINALITY OF
ACQUITTAL RULE. An acquittal is final and unappealable on the ground of DJ,
whether it happens at the trial court or before the appellate court.

The remand to a trial court a judgment of acquittal brought before the Supreme
Court on certiorari cannot be allowed unless there is a finding of mistrial
[GALMAN CASE].
In the Galman case, DJ cannot be invoked when the SC sets aside the
judgment of acquittal because there was a denial of due process. There was
a sham trial where the President ordered the courts to rig the trial and
predetermine the final outcome of acquittal. A dictated coerced and
scripted verdict of acquittal is a void judgment.

On the basis of humanity, fairness and justice, an acquitted defendant is entitled
to the right of repose as a direct consequence of the finality of his acquittal. The
philosophy underlying this rule establishing the absolute nature of acquittals if
part of the paramount importance criminal justice system attaches to the
protection of the innocent against wrongful conviction. The finality of acquittal
rule is a need for repose, a desire to know the exact extent of ones liability.

Also, errors of judgment are not to be confused with errors of jurisdiction. The
judge really considered the evidence received at trial. (testimonies of the
relative positions of victims, trajectory, location of gunshot wounds, etc). While
the appreciation thereof may have resulted in possible lapses in evidence
evaluation, it nevertheless does not detract from the fact that the evidence was
considered and passed upon. Thus, there was no error of jurisdiction. No
gadalej. Certiorari does not lie for mere error in judgment.

Updated by K Pascual. 2014. 4C. Bill of Rights Poli Digests. Atty. Jack Jimenez. 4C. 195


Panganiban, separate opinion:
While certiorari may be used to correct an abusive acquittal, it must be shown
in that extraordinary proceeding that the lower court blatantly abused its
authority to a point so grave as to deprive it of its very power to dispense
justice. But if the petition merely calls for an ordinary review of the findings of
the lower court, then the right against DJ would be violated. Such a recourse is
tantamount to converting the petition for certiorari into an appeal.

136. GALMAN VS. SB
DJ

This is the case of the Ninoy Aquino assassination. The accused were
charged with the killing of Ninoy and Galman. Among the accused were AFP
Chief of Staff General Fabian Ver.
They were acquitted by the Sandiganbayan.
The mother and son of Galman filed this petition alleging that there was
mistrial because the SB committed serious irregularities during the trial,
resulting in the miscarriage of justice. They assert that the Tanodbayan did
not represent the interest of the people, because he failed to exert genuine
efforts to present vital and important evidence for the prosecution.
They also claim that the SB justices were biased, prejudiced and partial in
favor of the accused.
The accused however argue that a re-trial would be placing them double
jeopardy.

SC: It was established, in the Vazquez Commission, that President Marcos (code
name Olympus), stage-managed in and from Malacanang, a scripted and pre-
determined manner of handling and disposing of the Aquino-Galman murder
case, and that the prosecution of the case and the Justices who tried and decided
the same, acted under the compulsion of some pressure which proved to be
beyond their capacity to resist, and which not only prevented the prosecution to
fully ventilate its position, and to offer all evidence it could have presented, but
also predetermined the final outcome of the case of total absolution of the
26 respondents of all criminal and civil liability.

President Marcos from beginning to end, misused the overwhelming resources
of the government and his authoritarian powers to corrupt and make a mockery
of the judicial process in the case. There was a secret Malacanang conference at
which the President called the Presiding Justice of the SB, and the entire
prosecution panel and told them how to handle and rig the trial and the close
monitoring of the entire proceedings to assure the pre-determined ignominous
final outcome are without parallel and precedent in our annals and
jurisprudence. This is the evil of one man rule at its very worst.

These cast illegality to the entire trial from the very beginning. No court whose
Presiding Justice received orders or suggestions from a President whose decree
made it possible to refer a case to his court can be an impartial court.

Consequently, DJ does not attach where a criminal trial was a sham. 1) the
proceedings were closely monitored by the President, 2) evidence was
suppressed, 3) witnesses were threatened, to secure recantation of testimony 4)
trial was finished only in 6 mos pursuant to a scripted scenario, 5) Presiding
Justices hostile attitude against the prosecution.

A dictated and coerced and scripted verdict of acquittal is a void judgment. It is
no judgment at all. It neither binds nor bars anyone. It is a lawless thing which
can be treated as an outlaw. Thus the first jeopardy was never terminated.

See original
Updated by K Pascual. 2014. 4C. Bill of Rights Poli Digests. Atty. Jack Jimenez. 4C. 196




137. MELO VS. PEOPLE
rule on supervening fact DJ

1. Melo was charged with frustrated homicide, for injuring Obillo with a
kitchen knife. The accused suffered serious wounds requiring medical
attention for more than 30 days.
2. He pleaded not guilty.
3. Subsequently, the victim died from the wounds.
4. An amended information was filed charging him with consummated
homicide.
5. Melo filed a MTD alleging DJ.

ISSUE: Is the defense of DJ proper?
SC: NO.
The amended information should stand. It was proper for the court to dismiss
the first information and order the filing of a new one for the reason that the
proper offense was not charged in the first and that the second did not place the
accused in second jeopardy for the same or identical offense.

When a person is charged with an offense and the case is terminated either by
acquittal or conviction, or in any other matter without the consent of the
accused, he cannot again be charged with the same or identical offense. It must
be noticd that the protection for this constitutional prohibition is against a
SECOND JEOPARDY FOR THE SAME OFFENSE, the only exception being, when
the act is punished by a law and an ordinance, conviction or acquittal under
either shall constitute a pbar to another prosecution for the SAME ACT. The
phrase SAME OFFENSE means not only that the second offense is exactly the
same as the one alleged in the first information, but also that the 2 offenses are
identical. There is identity between the two offenses when the evidence to
support a victim for one offense would be sufficient to warrant a conviction for
the other. THIS IS THE SAME-EVIDENCE TEST.

Under the Rules, there is identity between 2 offense not only when the second
offense is exactly the same as the first, but also when the second offense is an
attempt to commit the first, or frustration thereof, or when it necessarily
includes or is necessarily included in the offense charged in the first
information. An offense is said to be necessarily included in another when some
of the essential ingredients of the former as alleged in the information
constitute the latter, and vice versa. Thus, one who has been charged with an
offense cannot again charged with the same or identical offense through the
latter be lesser or greater than the former.

BUT, this rule of identity DOES NOT APPLY when the second offense was not in
existence at the time of the first prosecution. This is because in such case there
is no possibility for the accused, during the first prosecution, to be convicted for
an offense that was then inexistent. Thus, where the accused was charged with
physical injuries and after conviction the injured person dies, the charge for
homicide against the same accused does not put him twice in jeopardy. The rule
is that "where after the first prosecution a new fact supervenes for which the
defendant is responsible, which changes the character of the offense and,
together with the facts existing at the time, constitutes a new and distinct
offense" the accused cannot be said to be in second jeopardy if indicted for the
new offense .

Accordingly, an offense may be said to necessarily include or to be necessarily
included in another offense, for the purpose of determining the existence of
double jeopardy, when both offenses were in existence during the pendency of
the first prosecution, for otherwise, if the second offense was then inexistent, no
Updated by K Pascual. 2014. 4C. Bill of Rights Poli Digests. Atty. Jack Jimenez. 4C. 197


jeopardy could attach therefor during the first prosecution, and consequently a
subsequent charge for the same cannot constitute second jeopardy.

Note also, when a person who has already suffered his penalty for an offense, is
charged with a new and greater offense under the Diaz doctrine herein
reiterated, said penalty may be credited to him in case of conviction for the
second offense.

138. PEOPLE VS. CITY COURT MANILA
same offense ordinance and statutes

1. Gonzales was charged with violation of Art 201 of the RPC (exhibition
of indecent and immoral motion pictures).
2. Later, he was charged with violation of RA3060 (exhibition of motion
pictures not duly passed/ approved by the Board of Censors).
3. Pleaded not guilty to both. But he later withdrew plea on the RPC case.
4. He filed a MTQ both informations in the 2 cases on the ground of DJ. He
claims that there is a pending criminal case (RA 3060) where the
information allegedly contains the same allegations as the information
in the criminal case (RPC).
5. The City Court dismissed the RPC case, stating that considering that the
basis of the allegations in the informations were identical, he will be
exposed to DJ because the allegations are not only similar but identical.
6. The prosecution appealed, arguing that he accused could not invoke the
constitutional guarantee against double jeopardy, when there had been
no conviction, acquittal, dismissal or termination of criminal
proceedings in another case for the same offense.
7. Gonzales however, argues that conviction or acquittal in, or dismissal
or termination of a first case is not necessary, so long as he had been
put in jeopardy of being convicted or acquitted in the first case of the
same offense.

SC: NO DJ.
The two (2) informations with which the accused was charged, do not make out
only one offense, contrary to private respondent's allegations. In other words,
the offense defined in section 7 of Rep. Act No. 3060 punishing the exhibition of
motion pictures not duly passed by the Board of Censors for Motion Pictures
does not include or is not included in the offense defined in Article 201 (3) of
the Revised Penal Code punishing the exhibition of indecent and immoral
motion pictures. The two (2) offenses do not constitute a jeopardy to each
other. A scrutiny of the two (2) laws involved would show that the two (2)
offenses are different and distinct from each other.

The crime punished in RA. 3060 is a malum prohibitum in which criminal intent
need not be proved because it is presumed, while the offense punished in
Article 201 (3) of the Revised Penal Code is malum in se, in which criminal
intent is an indispensable ingredient. The gravamen of the offense defined in RA
3060 is the public exhibition of any motion picture which has not been
previously passed by the Board of Censors for Motion Pictures. The motion
picture may not be indecent or immoral, but if it has not been previously
approved by the Board, its public showing constitutes a criminal offense. On the
other hand, the offense punished in Article 201 (3) of the RPC is the public
showing of indecent or immoral plays, scenes, acts, or shows, not just motion
pictures.Considering these differences in elements and nature, there is no
identity of the offenses here involved for which legal jeopardy in one may be
invoked in the other. Evidence required to prove one offense is not the same
evidence required to prove the other. The defense of double jeopardy cannot
prosper.

It is a cardinal rule that the protection against double jeopardy may be invoked
only for the same offense or identical offenses. A single act may offend against
Updated by K Pascual. 2014. 4C. Bill of Rights Poli Digests. Atty. Jack Jimenez. 4C. 198


two (or more) entirely distinct and unrelated provisions of law, and if one
provision requires proof of an additional fact or element which the other does
not, an acquittal or conviction or a dismissal of the information under one does
not bar prosecution under the other. Where two different laws (or articles of
the same code) define two crimes, prior jeopardy as to one of them is no
obstacle to a prosecution of the other, although both offenses arise from the
same facts, if each crime involves some important act which is not an essential
element of the other.

CHAMP Page 198 9/16/2014

139. PEOPLE VS. RELOVA
2 kinds of DJ

1. For installing in his iceplant, electrical devices to reduce the reading of
electric current consumption, the accused Opulencia was charged with
violation of Ordinance #1, of Batangas City.
2. The ordinance penalizes unauthorized installation of devices to
decrease consumption of electricity.
3. Opulencia pleaded not guilty.
4. The case was dismissed on the ground of prescription because light
felonies prescribe in 2 months.
5. A new case was thereafter filed for Theft of Electricity under Art 308
RPC, filed 14 days after the dismissal of the previous case.
6. He filed a MTQ, alleging that he had been previously acquitted of the
offense charged in the second information and that the filing thereof
was violative of his constitutional right against double jeopardy.

7. The first information filed was one for unlawful or unauthorized
installation of electrical wiring and devices, acts which were in
violation of an ordinance. The principal purpose for (sic) such a
provision is to ensure that electrical installations on residences or
buildings be done by persons duly authorized or adept in the matter, to
avoid fires and accidents due to faulty electrical wirings. it is primarily
a regulatory measure and not intended to punish or curb theft of
electric fluid which is already covered by the Revised Penal Code."
8. The unauthorized installation punished by the ordinance [of Batangas
City] is not the same as theft of electricity [under the Revised Penal
Code]; that the second offense is not an attempt to commit the first or a
frustration thereof and that the second offense is not necessarily
included in the offense charged in the first information."

SC:
The rule is that, the constitutional protection against double jeopardy is
available although the prior offense charged under an ordinance be different
from the offense charged subsequently under a national statute such as the RPC,
provided that both offenses spring from the some act or set of acts.

There are two (2) kinds of double jeopardy. The first sentence prohibits double
jeopardy of punishment for the same offense, whereas the second contemplates
double jeopardy of punishment for the same act. Under the first sentence, one
may be twice put in jeopardy of punishment of the same act, provided that he is
charged with different offenses, or the offense charged in one case is not
included in, or does not include, the crime charged in the other case. The second
sentence applies, even if the offenses charged are not the same, owing to the fact
that one constitutes a violation of an ordinance and the other a violation of a
statute. If the two charges are based on one and the same act, conviction or
acquittal under either the law or the ordinance shall bar a prosecution under
the other. Incidentally, such conviction or acquittal is not indispensable to
sustain the plea of double jeopardy of punishment for the same offense. So long
as jeopardy has attached under one of the informations charging said offense,
the defense may be availed of in the other case involving the same offense, even
if there has been neither conviction nor acquittal in either case.
Updated by K Pascual. 2014. 4C. Bill of Rights Poli Digests. Atty. Jack Jimenez. 4C. 199



Where one offense is charged under a municipal ordinance while the other is
penalized by a statute, the critical inquiry is to the identity of the acts which
the accused is said to have committed. and which are alleged to have given rise
to the two offenses. The constitutional protection against DJ is available so long
as the acts which constitute or have given rise to the first offense under a
municipal ordinance are the same acts which constitute or have given rise to the
offense charged under a statute.

IN THIS CASE, the accused conceded that he effected or permitted such
unauthorized installation for the very purpose of reducing his electric power
bill. This corrupt intent was thus present from the very moment that such
unauthorized installation began. The immediate physical effect of the
unauthorized installation was the inward flow of electric current into
Opulencia's ice plant without the corresponding recording thereof in his electric
meter. In other words, the "taking" of electric current was integral with the
unauthorized installation of electric wiring and devices.

The identity of offenses that must be shown need not be absolute identity: the
first and second offenses may be regarded as the "same offense" where the
second offense necessarily includes the first offense or is necessarily included in
such first offense or where the second offense is an attempt to commit the first
or a frustration thereof. Thus, for the constitutional plea of double jeopardy to
be available, not all the technical elements constituting the first offense need be
present in the technical definition of the second offense. The law here seeks to
prevent harrassment of an accused person by multiple prosecutions for offenses
which though different from one another are nonetheless each constituted by a
common set or overlapping sets of technical elements.

When the acts of a person which physically occur on the same occasion and are
infused by a common intent or design or negligence and therefore form a moral
unity, it should not be segmented and sliced, to produce as many different acts
as there are offenses under municipal ordinances or statutes that an
enterprising prosecutor can find.


CHAMP Page 199 9/16/2014

















140. PEOPLE VS. JABINAL
Updated by K Pascual. 2014. 4C. Bill of Rights Poli Digests. Atty. Jack Jimenez. 4C. 200


ex post facto law

1. Jabinal was a secret agent who was found guilty of illegal possession of
firearms (unlicensed revolver).
2. On appeal he relied on the case of Macarandang, which cleared Secret
Agents from liability because at the time he was found to possess a
certain firearm without a license, he had nevertheless been appointed
from the Governor as Secret Agent, to assist in the peace and order,
with authority to carry said firearm. In this Macarandang case, the
court ruled that the Revised Admin Code exempts peace officers from
the requirement of license to possess firearms. Because Macarandang
was then an appointed secret agent, he was also deemed as a peace
officer.
3. However, in the case of Mapa, this doctrine was abandoned. There, it
was stated that there is no provision for secret agents, hence he is not
exempt.
4. 1959 Macarandang Case.
5. 1962 Jabinal was appointed as secret agent
6. 1967 Mapa Case.
7. 1968 time when Jabinals case was decided (convicted)
8. Jabinal argues that he should be acquitted because was should be
applied is the Macarandang case. He claims that the RTC erred when it
retroactively applied the Mapa case to his case.

SC: ACQUITTED.
The doctrine laid down in Lucero and Macarandang was part of the
jurisprudence, hence, of the law, of the land, at the time appellant was found in
possession of the firearm in question and when he was arraigned by the trial
court. It is true that the doctrine was overruled in the Mapa case in 1967, but
when a doctrine of this Court is overruled and a different view is adopted, the
new doctrine should be applied prospectively, and should not apply to parties
who had relied on the old doctrine and acted on the faith thereof. This is
especially true in the construction and application of criminal laws, where it is
necessary that the punishability of an act be reasonably foreseen for the
guidance of society.

Considering that appellant was conferred his appointments as Secret Agent and
Confidential Agent and authorized to possess a firearm pursuant to the
prevailing doctrine enunciated in Macarandang and Lucero, under which no
criminal liability would attach to his possession of said firearm in spite of the
absence of a license and permit therefor, appellant must be absolved. Certainly,
appellant may not be punished for an act which at the time it was done was held
not to be punishable.


Note: New doctrines should be applied prospectively and should not apply to
parties who relied on the old doctrine in good faith.
CHAMP Page 200 9/16/2014








Updated by K Pascual. 2014. 4C. Bill of Rights Poli Digests. Atty. Jack Jimenez. 4C. 201


ARTI CLE I V AND V CI TI ZENSHIP AND SUFFRAGE

141. TECSON VS. COMLEC
citizenship

1. This is the FPJ case. FPJ filed his certificate of candidacy for President.
In his certificate of candidacy, FPJ, representing himself to be a natural-
born citizen of the Philippines, stated his name to be "Fernando Jr.," or
"Ronald Allan" Poe, his date of birth to be 20 August 1939 and his place
of birth to be Manila.
2. Atty Fornier, filed a disqualification case against FPJ before the
Comelec, claiming that FPJ made a material misrepresentation in his
certificate of candidacy by claiming to be a natural-born Filipino citizen
when in truth, his parents were foreigners; his mother, Bessie Kelley
Poe, was an American, and his father, Allan Poe, was a Spanish national,
being the son of Lorenzo Pou, a Spanish subject.
3. Atty Fornier, also said that granting, Allan F. Poe Sr. was a Filipino
citizen, he could not have transmitted his Filipino citizenship to FPJ, the
latter being an illegitimate child of an alien mother. He claims FPJ is
illegitimate because, first, Allan F. Poe contracted a prior marriage to a
certain Paulita Gomez before his marriage to Bessie Kelley, making his
subsequent marriage to Bessie Kelley bigamous and FPJ an illegitimate
child. and, second, even if no such prior marriage had existed, Allan F.
Poe, married Bessie Kelly only a year after the birth of respondent.
Birth to unmarried parents would make FPJ an illegitimate child.
Petitioner contended that as an illegitimate child, FPJ so followed the
citizenship of his mother, Bessie Kelley, an American citizen.
4. Defense: FPJ presented a certification issued by the National Archives
that there appeared to be no available information regarding the birth
of Allan F. Poe in the registry of births, a certification that no available
information about the marriage of Allan F. Poe and Paulita Gomez could
be found, a certificate of birth of Ronald Allan Poe, a copy of the
purported marriage contract between Fernando Pou and Bessie Kelley,
among many other documents.

SC: Under the Consti, a President can be elected only if he is a natural-born
citizen of the Philippines. The term "natural-born citizens," is defined to include
"those who are citizens of the Philippines from birth without having to perform
any act to acquire or perfect their Philippine citizenship."
The only conclusions that could be drawn with some degree of certainty from
the documents would be that -
1. The parents of FPJ were Allan F. Poe and Bessie Kelley;
2. FPJ was born to them on 20 August 1939;
3. Allan F. Poe and Bessie Kelley were married to each other on 16
September, 1940;
4. The father of Allan F. Poe was Lorenzo Poe; and
5. At the time of his death on 11 September 1954, Lorenzo Poe was 84
years old.
6. Allan Poe was a Filipino because his father, Lorenzo Poe, albeit a
Spanish subject, was not shown to have declared his allegiance to Spain
by virtue of the Treaty of Paris and the Philippine Bill of 1902.

Where jurisprudence regarded an illegitimate child as taking after the
citizenship of its mother, it did so for the benefit the child. It was to ensure a
Filipino nationality for the illegitimate child of an alien father in line with the
assumption that the mother had custody, would exercise parental authority and
had the duty to support her illegitimate child. It was to help the child, not to
prejudice or discriminate against him.
The fact of the matter perhaps the most significant consideration is that
the 1935 Constitution, the fundamental law prevailing on the day, month and
year of birth of respondent FPJ, can never be more explicit than it is. Providing
neither conditions nor distinctions, the Constitution states that among the
Updated by K Pascual. 2014. 4C. Bill of Rights Poli Digests. Atty. Jack Jimenez. 4C. 202


citizens of the Philippines are those whose fathers are citizens of the
Philippines. There utterly is no cogent justification to prescribe conditions or
distinctions where there clearly are none provided.
The issue of whether or not respondent FPJ is a natural-born citizen, in
turn, depended on whether or not the father of respondent, Allan F. Poe,
would have himself been a Filipino citizen and, in the affirmative, whether or
not alleged illegitimacy of respondent prevents him from taking after the
Filipino citizenship of his putative father. Any conclusion on the Filipino
citizenship of Lorenzo Pou could only be drawn from the presumption that
having died in 1954 at 84 years old, Lorenzo would have been born sometime in
the year 1870, when the Philippines was under Spanish rule, and that San
Carlos, Pangasinan, his place of residence upon his death in 1954, in the absence
of any other evidence, could have well been his place of residence before death,
such that Lorenzo Pou would have benefited from the en masse
Filipinization that the Philippine bill had effected in 1902. That
citizenship (of Lorenzo Pou), if acquired, would thereby extend to his son,
Allan F. Poe, father of respondent FPJ. The 1935 Constitution, during
which regime respondent FPJ has seen first light, confers citizenship to all
persons whose fathers are Filipino citizens regardless of whether such
children are legitimate or illegitimate.

But while the totality of the evidence may not establish conclusively
that respondent FPJ is a natural-born citizen of the Philippines, the evidence on
hand still would preponderate in his favor enough to hold that he cannot be
held guilty of having made a material misrepresentation in his certificate of
candidacy in violation of Section 78, in relation to Section 74, of the Omnibus
Election Code.
FPJ ALLOWED TO RUN.
CHAMP Page 202 9/16/2014
DAVIDE, CONCURRING.
For purposes of the citizenship of an illegitimate child whose father is a Filipino
and whose mother is an alien, proof of paternity or filiation is enough for the
child to follow the citizenship of his putative father, pursuant to paragraph 3 of
Section 1 of Article IV of the 1935 Constitution, which reads:
Section 1. The following are citizens of the Philippines:
(3) Those whose fathers are citizens of the Philippines.
I agree with the amici curiae that this provision makes no distinction between
legitimate and illegitimate children of Filipino fathers. It is enough that filiation
is established or that the child is acknowledged or recognized by the father.

OTHERS, CONCURRING:
(They all agree with the amici curiae the illegitimacy of respondent Poe
is inconsequential in determining whether he is a natural born Filipino citizen.)

For there is really no difference in principle between, on the one hand, the
illegitimate child of a Filipino mother and an alien father, and, on the other
hand, the illegitimate child of a Filipino father and an alien mother. As long as
the childs filiation to his supposed father is established, it does not matter
whether he is legitimate or an illegitimate child.

Fr. B says, when the Constitution says: The following are citizens of the
Philippines: Those whose fathers are citizens of the Philippines, the
Constitution means just that without invidious distinction. Ubi lex non
distinguit nec nos distinguere debemus, especially if the distinction has no
textual foundation in the Constitution, serves no state interest, and even
imposes an injustice on an innocent child. What flow from legitimacy are civil
rights; citizenship is a political right which flows not from legitimacy but from
paternity. And paternity begins when the ovum is fertilized nine months before
birth and not upon marriage or legitimation.

Updated by K Pascual. 2014. 4C. Bill of Rights Poli Digests. Atty. Jack Jimenez. 4C. 203


(Then there are still those who said that the petition is premature since FPJ has
not yet won the election. The disqualification case should be filed only after the
elections.)


CARPIO, DISSENTING.
FPJ is not a natural-born Philippine citizen since there is no showing that
his alleged Filipino father Allan F. Poe acknowledged him at birth. The
Constitution defines a natural-born citizen as a Philippine citizen from birth
without having to perform any act to acquire or perfect his Philippine
citizenship. Private respondent Fernando Poe, Jr. does not meet this citizenship
qualification.

CARPIO-MORALES, DISSENTING: (she gave the longest dissent of 117-pages)
I am adopting the rule that an illegitimate child of an alien-mother who
claims to be an offspring of a Filipino father may be considered a natural-born
citizen if he was duly acknowledged by the latter at birth, thus leaving the
illegitimate child with nothing more to do to acquire or perfect his citizenship.

(I think that there was still an issue on the citizenship of FPJs father. Parang hindi
yata sya Filipino. See original)

Assuming arguendo, therefore, that Allan F. Poe, the putative father of
FPJ, was indeed a Filipino citizen at the time of his birth, no evidence has been
submitted to show that Allan F. Poe did indeed acknowledge FPJ as his own son
at birth. In fact, as emphasized by petitioner Fornier, in the course of the
proceedings before the COMELEC, both parties verified that there was no such
acknowledgment by Allan F. Poe on the dorsal portion of FPJs Birth Certificate.
Since FPJ then was born out of wedlock and was not acknowledged
by his father, the only possible Filipino parent, at the time of his birth, the
inescapable conclusion is that he is not a natural-born Philippine citizen.






142. MOY YAO VS. COMMISSION ON IMMIGRATION
citizenship

1. Yao = husband, aka Edilberto Lim. Yeung = wife.
2. Earlier, Yeung applied for visa to enter the Philippines as non-
immigrant. She was Chinese resident from Kowloon HKG.
3. When she was here in the RP, he contracted a marriage with Yao, an
alleged Filipino Citizen.
4. Because she was overstaying (expired temporary visa), she was
ordered arrested and deported. She filed an injunction against the
Immigration to prevent her arrest.
5. She claims, under the Revised Naturalization Law, Any woman who is
now or may hereafter be married to a citizen of the Philippines, and
who might herself be lawfully naturalized shall be deemed a citizen of the
Philippines." Thus she claims that she is a Filipino citizen by virtue of
her marriage to Yao alias Edilberto Lim under the Naturalization Laws
of the Philippines."
Updated by K Pascual. 2014. 4C. Bill of Rights Poli Digests. Atty. Jack Jimenez. 4C. 204


6. The CID argued that the provision 'who might herself be lawfully
naturalized' incontestably implies that an alien woman may be deemed
a citizen of the Philippines by virtue of her marriage to a Filipino citizen
only, if she possesses all the qualifications and none of the
disqualifications specified in the law, because these are the explicit
requisites provided by law for an alien to be naturalized. Yeung while
claiming not to be disqualified, does not and cannot allege that she
possesses all the qualifications to be naturalized, naturally because,
having been admitted as a temporary visitor, it is obvious at once that
she lacks, at least, the requisite length of residence in the Philippines.
Also, her authorized stay in the RP was to expire just a little over one
month before the expiry date of her stay, it is evident that said marriage
was effected merely for convenience to defeat or avoid her then
impending compulsory departure, not to say deportation. This cannot
be permitted.

SC:
Under the Commonwealth Act 473, an alien woman marrying a Filipino, native
or naturalized, becomes ipso facto a FILIPINA, provided she is not disqualified
to be a citizen of the Philippines under Sec 4 of that law. Likewise, an alien
woman married to an alien who is subsequently naturalized here follows the
Philippine citizenship of her husband the moment he takes his oath as a Filipino
citizen, provided that SHE does not suffer from any of the disqualifications
under Sec 4.

The Constitution itself recognizes as Philippine citizens "Those who are
naturalized in accordance with law". Citizens by naturalization, under this
provision, include not only those whose are naturalized in accordance with legal
proceedings for the acquisition of citizenship, but also those who acquire
citizenship by "derivative naturalization" or by operation of law, as, for
example, the "naturalization" of an alien wife through the naturalization of her
husband, or by marriage of an alien woman to a citizen.

The leading idea or purpose of Section 15 was to confer Philippine citizenship
by operation of law upon certain classes of aliens as a legal consequence of their
relationship, by blood or by affinity, to persons who are already citizens of the
Philippines. Whenever the fact of relationship of the persons enumerated in the
provisions concurs with the fact of citizenship of the person to who they are
related, the effect is for said person to become ipso facto citizens of the
Philippines.

The legislature could not have intended that an alien wife should not be deemed
a Philippine citizen unless and until she proves that she might herself be
lawfully naturalized. Far from it, the law states in plain terms that she shall be
deemed a citizen of the Philippines if she is one "who might herself be lawfully
naturalized." The proviso that she must be one "who might herself be lawfully
naturalized" is not a condition precedent to the vesting or acquisition of
citizenship; it is only a condition or a state of fact necessary to establish her
citizenship as a factum probandun, i.e., as a fact established and proved in
evidence. The word "might," as used in that phrase, precisely implies that at the
time of her marriage to Philippine citizen, the alien woman "had (the) power" to
become such a citizen herself under the laws then in force.

JBL Reyes dissents:
Our naturalization law separates qualifications from disqualifications; the
positive quatifications under Section 3 thereof express a policy of restriction as
to candidates for naturalization as much as the disqualifications under Section
4. (JBL Reyes seems to agree with the Commissioner on Immigration here.)


Please see original.


Updated by K Pascual. 2014. 4C. Bill of Rights Poli Digests. Atty. Jack Jimenez. 4C. 205

































143. CO VS. HRET
citizenship NBC

1. Ong was proclaimed as the duly elected congressman of Samar (2d).
2. Co, the losing candidate, filed an election protests on that ground that
Ong is not a natural born citizen of the Philippines.
3. Ongs grandfather was from China. He was able to get a certificate of
residence during the Spanish times. Ongs father was also born in China
but was brought to Samar by the grandfather.
4. The father grew up and met a natural born Filipina, Lao (mother). They
got married, and Ong was one of the offspring.

SC:
Updated by K Pascual. 2014. 4C. Bill of Rights Poli Digests. Atty. Jack Jimenez. 4C. 206


Under the Philippine Bill of 1902, those who were inhabitants of the Philippines
who were Spanish subjects in 1899, and were residing in the Philippines and
their children born subsequently were conferred the status of Filipino citizen.

Here, the grandfather became a permanent resident of the Philippines in 1895
because certificate of residence was issued to him. His grandfather qualified as
a Filipino. His father applied for naturalization and was also declared a Filipino
citizen, only that when he (the father) took his oath of allegiance, Ong (the
Congressman) was only a minor then. His mother was also a natural born
citizen.

Thus, it would be unnatural to expect Ong (the Congressman) to still formally
elect Philippine Citizenship when he became of age, since he was already a
Filipino citizen because of the naturalization of his father when he (Ong, the
Congressman) was still a minor. Election of citizenship presupposes that one is
an alien. Any election of Philippine citizenship by Ong would not only be
superfluous but also absurd considering that the law itself had already elected
Philippine citizenship for him.

The filing of a sworn statement or formal declaration is a requirement only for
those who still have to elect citizenship. But for those like Ong who are
ALREADY FILIPINOS when the time to elect came up, there are acts of
deliberate choice which cannot be less binding. These acts are:, Ong previously
passed the CPA exams, which profession requires Philippine citizenship. He also
worked as examiner in the Central Bank. He is a registered voter, voting during
elections. He has considered himself as a Filipino. These are formal
manifestations of his choice of Philippine citizenship.

He could not have foreseen that the 1987 Constitution would still require him to
file a sworn statement electing Philippine Citizenship. Sec 1, Par 3, should be
applied only to those who elected Philippine citizenship before February 2,
1987. It is curative in character. Its purpose is to remedy the inequitable
situation that would arise if those born of Filipino fathers and alien mothers
would be considered natural-born citizens and those born of Filipino mothers
and alien fathers would not be considered the same.







144. IN RE CHING.
Citizenship

1. Ching was the legitimate son of a Chinese and a Filipino. Ching was
born in the Philippines and has resided here since his birth.
2. He took up law at SLU, Baguio City, and in 1998, took the Bar.
3. As proof of his citizenship, he presented the certification from the PRC
that he was already a CPA. He also presented a voters certification, and
a certification that he was previously elected as Board member.
4. Sometime in 1999, he made a formal Affidavit of Election.
5. The Solgen argued that being a child of a Chinese Father and a Filipino
Mother born under the 1935 Constitution, he was still a Chinese Citizen
unless upon reaching the age of majority, he elected Philippine
citizenship. Solgen argued further that his Philippine citizenship can be
perfected only upon election upon reaching the age of majority. Thus,
the argument is that Ching has not yet formally elected Philippine
Citizenship and that assuming he already did, it was too late.
Updated by K Pascual. 2014. 4C. Bill of Rights Poli Digests. Atty. Jack Jimenez. 4C. 207



ISSUE: Was the election of Philippine citizenship taken within reasonable time?
SC: NO.

The opinion of the DOJ interpreted the reasonable time for election as 3 years
from reaching the age of majority, or at 21 years old. Here, he made the election
more than 14 years after reaching the age of majority beyond the allowable
period.

The claim that he has continuously stayed in the Philippines and his being a
CPA, a registered voter, a former public official, CANNOT VEST IN HIM
PHILIPPINE CITIZENSHIP as the law specifically lays down the requirements for
acquisition of Philippine citizenship by election.

Under CA 625, the procedure for valid election of Philippine citizenship:
1) statement signed and sworn to by the party, filed with the LCR
2) oath of allegiance
The limit of 3 years can be deduced from the fact that the CA 625 and the 1935
Constitution did not provide for the time within which to elect. It only provided
that election should be made upon reaching the age of majority. At that time, the
age of majority was 21. This is the basis of the DOJ opinion.
















145. BENGSON VS. HRET
citizenship

1. Cruz was a natural-born citizen. However later, he enlisted in the US
Marine Corps USMC and without consent of the RP, took an oath of
allegiance to the US. He thus lost his Filipino citizenship because under
CA43, he loses Filipino citizenship by rendering service to or accepting
commission to the armed forces of another country.
2. Subsequently, he was naturalized as US citizen.
3. Later, he re-acquired his Philippine citizenship through repatriation
under RA2630. He ran and won as Congressman of Pangasinan (2d).
4. His opponent Bengson filed this disqualification case against Cruz.
Bengson claims that Cruz is no longer a NBC, because he has since lost
Updated by K Pascual. 2014. 4C. Bill of Rights Poli Digests. Atty. Jack Jimenez. 4C. 208


his Philippine citizenship when he swore allegiance to the US. NBC are
those who are citizens from birth without having to perform any act of
acquire or perfect such citizenship.
5. Cruz on the other hand, claims that he re-acquired his NBC status when
he was repatriated, since the phrase from birth refers to the innate,
inherent and inborn characteristic of being a NBC.

ISSUE: Can Cruz, a natural born citizen, who later became an American citizen,
can still be considered a natural born citizen, after his reacquisition of
Philippine citizenship? YES.

SC: There are 2 ways of acquiring citizenship:
1) by birth a natural born citizen a citizen of the country at the time of his
birth
2) by naturalization a naturalized citizen

NBC are those citizens of the Philippines from birth without having to perform
any act to acquire or perfect his citizenship. Naturalized citizens are those who
have become Filipino citizens through naturalization, under the Naturalization
Act. To be naturalized he has to prove that he has all the qualifications and none
of the disqualifications provided for by law.

Filipinos who have lost their citizenship may re-acquire the same by:
1) by naturalization
2) by repatriation
3) by direct act of Congress

So, naturalization is both a mode of acquisition and re-acquisition of Philippine
citizenship. Under our law, a former Filipino citizen who wants to re-acquire his
Philippine citizenship must also possess all the qualifications and none of the
disqualifications provided for by law.

Repatriation simply consists in taking an oath of allegiance, as distinguished
from the lengthy process of naturalization. Repatriation results in the recovery
of the original nationality. This means that if he was originally a natural born
citizen before he lost his Philippine citizenship, he will be restored to his
former status as a natural born citizen.

Here, Cruz lost citizenship when he rendered service to the USMC. He however
re-acquired citizenship under the repatriation law. Having thus taken the
required oath of allegiance, Cruz is deemed to have recovered his original
status as a natural born citizen, a status which he acquired at birth. THE
ACT OF REPATRIATION ALLOWS HIM TO RECOVER OR RETURN TO HIS
ORIGINAL STATUS BEFORE HE LOST HIS PHILIPPINE CITIZENSHIP.



146. MERCADO VS. MANZANO
citizenship

1. Edu Manzano won as Vice Mayor of Makati. His proclamation was
suspended because a disqualification case was filed against him
alleging that he is not a Filipino citizen but a US citizen.
2. Edu admitted that his is registered as a foreigner with the BID under
and ACR. He also admits that he was born in SFO and thus considered
an American citizen. However, he also admits that his parents were
Updated by K Pascual. 2014. 4C. Bill of Rights Poli Digests. Atty. Jack Jimenez. 4C. 209


both Filipinos. Thus, notwithstanding his registration as an American
citizen, he did not lose his Filipino citizenship. (dual citizen)
3. Under the Local Govt Code, dual citizens are disqualified from running.

ISSUE: is dual citizenship a ground for disqualification?
SC: NO. IT MUST BE UNDERSTOOD AS REFERRING TO DUAL ALLEGIANCE.
Dual citizenship is different from dual allegiance. The first arises when as a
result of the concurrent application of the different laws of two or more states, a
person is simultaneously considered a national by the said states. The second
however, refers to the situation where a person simultaneously owes, by some
positive act, loyalty to two or more states. Dual allegiance is the result of an
individuals volition, and is not involuntary.

Clearly, what the constitution refers to is not dual citizens per se, but with
naturalized citizens who maintain their allegiance to their countries of origin
even after their naturalization. Thus, persons with mere dual citizenship do
not fall under the disqualification under the LGC. As Fr. B pointed out, dual
citizenship is just a reality imposed on us because we have no control of the
laws of citizenship of other countries.

Note also that by filing a certificate of candidacy when he ran for vice mayor,
Edu elected Philippine citizenship and effectively renounced his American
citizenship, thus removing any disqualification he might have as a dual
citizen. The oath of allegiance contained in the certificate of candidacy is
sufficient to constitute renunciation of American citizenship.

How about the fact that he is registered with the BID with an ACR and that he
holds a blue passport (US)? There is no merit in this. Before filing his certificate
of candidacy he really had dual citizenship. These are merely assertions of his
American nationality before the termination of his American citizenship.

Note also that there is no law requiring the election of Philippine citizenship to
be made upon majority age. Considering also the fact that he has spent his youth
and adulthood, got his education, practiced his profession as an artist, and voted
in the past elections here, it leaves no doubt of his election of Philippine
citizenship.

CHAMP Page 209 9/16/2014







147. MAKALINTAL VS. COMELEC
suffrage

1. Atty.Makalintal assails the constitutionality of the Overseas Absentee
Voting Act of 2003. RA 9189.
2. Section 5(d) provides:
Sec. 5. Disqualifications. The following shall be disqualified
from voting under this Act: d) An immigrant or a permanent resident
who is recognized as such in the host country, unless he/she
executes, upon registration, an affidavit prepared for the purpose by
Updated by K Pascual. 2014. 4C. Bill of Rights Poli Digests. Atty. Jack Jimenez. 4C. 210


the Commission declaring that he/she shall resume actual physical
permanent residence in the Philippines not later than three (3) years
from approval of his/her registration under this Act. Such affidavit
shall also state that he/she has not applied for citizenship in another
country. Failure to return shall be cause for the removal of the name
of the immigrant or permanent resident from the National Registry of
Absentee Voters and his/her permanent disqualification to vote in
absentia.
3. It was argued that Section 5(d) is unconstitutional because it violates
Section 1, Article V of the 1987 Constitution which requires that the
voter must be a resident in the Philippines for at least one year and in
the place where he proposes to vote for at least six months immediately
preceding an election.
4. Also, he argues that Section 1, does not allow provisional registration or
a promise by a voter to perform a condition to be qualified to vote in a
political exercise; that the legislature should not be allowed to
circumvent the requirement of the Constitution on the right of suffrage
by providing a condition thereon which in effect amends or alters the
aforesaid residence requirement to qualify a Filipino abroad to vote.
He claims that the right of suffrage should not be granted to anyone
who, on the date of the election, does not possess the qualifications
5. Section 1, Article V of the Constitution specifically provides that
suffrage may be exercised by (1) all citizens of the Philippines, (2) not
otherwise disqualified by law, (3) at least eighteen years of age, (4) who
are residents in the Philippines for at least one year and in the place
where they propose to vote for at least six months immediately
preceding the election.
6. Petitioner questions the rightness of the mere act of execution of an
affidavit to qualify the Filipinos abroad who are immigrants or
permanent residents, to vote.

ISSUE: Does Section 5(d) of Rep. Act No. 9189 allowing the registration of voters
who are immigrants or permanent residents in other countries by their mere
act of executing an affidavit expressing their intention to return to the
Philippines, violate the residency requirement in Section 1 of Article V of the
Constitution?

SC:LAW VALID.
The essence of R.A. No. 9189 is to enfranchise overseas qualified Filipinos. The
right of absentee and disabled voters to cast their ballots at an election is purely
statutory.

Ordinarily, an absentee is not a resident and vice versa; a person cannot be at
the same time, both a resident and an absentee. However, under our election
laws and the countless pronouncements of the Court pertaining to elections, an
absentee remains attached to his residence in the Philippines as residence
is considered synonymous with domicile. For political purposes the concepts
of residence and domicile are dictated by the peculiar criteria of political laws.
As these concepts have evolved in our election law, what has clearly and
unequivocally emerged is the fact that residence for election purposes is
used synonymously with domicile.
A citizen may leave the place of his birth to look for
greener pastures, , to improve his lot and that, of course,
includes study in other places, practice of his vocation,
reengaging in business. When an election is to be held, the
citizen who left his birthplace to improve his lot may decide
to return to his native town, to cast his ballot, but for
professional or business reasons, or for any other reason, he
may not absent himself from the place of his professional or
business activities.
So, they are here registered as voters as he has the
qualifications to be one, and is not willing to give up or lose
the opportunity to choose the officials who are to run the
government especially in national elections. Despite such
registration, the animus revertendi to his home, to his
domicile or residence of origin has not forsaken him.
This may be the explanation why the registration of a voter in a
Updated by K Pascual. 2014. 4C. Bill of Rights Poli Digests. Atty. Jack Jimenez. 4C. 211


place other than his residence of origin has not been deemed
sufficient to consider abandonment or loss of such residence of origin.
In other words, residence in this provision refers to two
residence qualifications: residence in the Philippines and
residence in the place where he will vote. As far as residence in the
Philippines is concerned, the word residence means domicile, but as
far as residence in the place where he will actually cast his ballot is
concerned, the meaning seems to be different. He could have a
domicile somewhere else and yet he is a resident of a place for six
months and he is allowed to vote there. So that there may be serious
constitutional obstacles to absentee voting, unless the vote of the
person who is absent is a vote which will be considered as cast in
the place of his domicile.
The Constitutional Commission recognized the fact that while millions of
Filipinos reside abroad principally for economic reasons and hence they
contribute in no small measure to the economic uplift of this country, their
voices are marginal insofar as the choice of this countrys leaders is concerned.
It is clear from these that they intended to enfranchise as much as possible all
Filipino citizens abroad who have not abandoned their domicile of origin.

The fact that a Filipino may have been physically absent from the
Philippines and may be physically a resident of the United States, for
example, but has a clear intent to return to the Philippines, will make him
qualified as a resident of the Philippines under this law.
The execution of the affidavit itself is not the enabling or enfranchising act.
The affidavit required in Section 5(d) is not only proof of the intention of the
immigrant or permanent resident to go back and resume residency in the
Philippines, but more significantly, it serves as an explicit expression that he
had not in fact abandoned his domicile of origin. To repeat, the affidavit is
required of immigrants and permanent residents abroad because by their status
in their host countries, they are presumed to have relinquished their intent to
return to this country; thus, without the affidavit, the presumption of
abandonment of Philippine domicile shall remain.

The rationale for this, is that we want to be expansive and all-inclusive in
this law. That as long as he is a Filipino, no matter whether he is a green-
card holder in the U.S. or not, he will be authorized to vote. But if he is
already a green-card holder, that means he has acquired permanent
residency in the United States, then he must indicate an intention to
return. This is what makes for the definition of domicile.

Section 5(d) does not only require an affidavit or a promise to resume actual
physical permanent residence in the Philippines not later than three years from
approval of his/her registration, the Filipinos abroad must also declare that
they have not applied for citizenship in another country. Thus, they must return
to the Philippines; otherwise, their failure to return shall be cause for the
removal of their names from the National Registry of Absentee Voters and
his/her permanent disqualification to vote in absentia.

He is presumed not to have lost his domicile by his physical absence from this
country. His having become an immigrant or permanent resident of his host
country does not necessarily imply an abandonment of his intention to return to
his domicile of origin, the Philippines. Therefore, under the law, he must be
given the opportunity to express that he has not actually abandoned his
domicile in the Philippines by executing the affidavit required by Sections 5(d)
and 8(c) of the law.

ISSUE: What happens to the votes cast by the qualified voters abroad who were
not able to return within three years as promised? What is the effect on the
votes cast by the non-returnees in favor of the winning candidates?
SC: The votes cast by qualified Filipinos abroad who failed to return within
three years shall not be invalidated because they were qualified to vote on the
date of the elections, but their failure to return shall be cause for the removal of
the names of the immigrants or permanent residents from the National Registry
of Absentee Voters and their permanent disqualification to vote in absentia.

Updated by K Pascual. 2014. 4C. Bill of Rights Poli Digests. Atty. Jack Jimenez. 4C. 212


See original.

You might also like